Вы находитесь на странице: 1из 537

2007 PREP SA on CD-ROM Question: 1

You are precepting a resident who has just evaluated a 4-year-old incompletely immunized immigrant boy who has classic varicella lesions and a history that is consistent with this diagnosis. Of the following, the MOST accurate statement is that

A. lesions of both varicella and smallpox follow a 7- to 10-day course from eruption to resolution B. lesions of both varicella and smallpox frequently produce deep, pitted scars C. varicella lesions appear in stages or crops; smallpox lesions are uniformly in the same stage
of development

D. varicella lesions are concentrated on the face; smallpox lesions are concentrated over bony
prominences

E. varicella lesions are transient vesicles; smallpox lesions are persistent pustules until resolution
of the illness

Copyright 2007 by the American Academy of Pediatrics

page 1

2007 PREP SA on CD-ROM Critique: 1 Preferred Response: C

Historically, the disease most often confused with severe varicella was smallpox. Smallpox, caused by the variola virus, a member of the orthopoxvirus family, has an incubation period of 7 to 17 days. During the incubation period, virus replicates in the upper respiratory tract. A primary viremia ensues, during which the liver and spleen are seeded. A secondary viremia follows, the skin is seeded, and the classic eruption appears. Initial symptoms of smallpox may include fever as high as 104F (40C), generalized malaise, severe headache, vomiting, and backache. Characteristic skin findings appear 1 to 2 days after the onset of fever. The rash begins on the face and then spreads to involve the extremities and trunk. Initially, lesions are erythematous macules; they evolve into papules, vesicles, and firm pustules (Item C1A). Crusts form at 8 or 9 days and persist for 3 to 4 weeks. As crusts separate, patients often are left with significant scars or depigmentation. Aspects of smallpox that differentiate it from severe varicella are that the majority of lesions are observed on the face and extremities (with lesser numbers on the trunk) and that all lesions are in a similar stage of development. In contrast, the lesions of varicella erupt initially on the trunk and later appear on the face and extremities. Lesions are erythematous papules that evolve to form superficial vesicles, pustules, and crusts. In varicella (unlike smallpox), lesions are observed in varying stages of development (ie, as older lesions crust, new lesions appear) (Item C1B). By 7 to 10 days after infection, all lesions have crusted. Permanent scars are rare, occurring only when lesions have been secondarily infected with bacteria. References: American Academy of Pediatrics. Smallpox (variola). In: Pickering LK, ed. Red Book: 2006 Report of the Committee on Infectious Diseases. 27th ed. Elk Grove Village, Ill: American Academy of Pediatrics; 2006: 591-595 Cieslak J, Henretig FM. Biologic and chemical terrorism. In: Behrman RE, Kliegman RM, Jenson HB, eds. Nelson Textbook of Pediatrics. 17th ed. Philadelphia, Pa: WB Saunders Co; 2004:23782385 Myers MG, Stanberry LR, Sevard JF. Varicella-zoster virus. In: Behrman RE, Kliegman RM, Jenson HB, eds. Nelson Textbook of Pediatrics. 17th ed. Philadelphia, Pa: WB Saunders Co; 2004:1057-1062 Paller AS, Mancini AJ. Viral diseases of the skin. In: Hurwitz Clinical Pediatric Dermatology. 3rd ed. Philadelphia, Pa: Elsevier Inc; 2006: 397-423

Copyright 2007 by the American Academy of Pediatrics

page 2

2007 PREP SA on CD-ROM Question: 2

A 15-year-old boy presents with melena and anemia. Endoscopy demonstrates a nodular gastritis of the antrum (Item Q2A) and an ulcer. Biopsies of the antrum demonstrate spiralshaped organisms consistent with Helicobacter pylori (Item Q2B). You prescribe amoxicillin, clarithromycin, and lansoprazole for 2 weeks. At a follow-up visit, the family asks whether the treatment has been successful in eradicating the organism. Of the following, the PREFERRED noninvasive test to evaluate whether the pathogen has been eradicated is

A. fecal Campylobacter-like organisms (CLO) test B. fecal H pylori antigen C. salivary H pylori antibody concentrations D. serum H pylori immunoglobulin G serology E. serum H pylori urease concentrations

Copyright 2007 by the American Academy of Pediatrics

page 3

2007 PREP SA on CD-ROM Critique: 2 Preferred Response: B

Helicobacter pylori infection is a known risk factor for gastritis and duodenal ulcers in children and adults. Rarely, and primarily in older adulthood, H pylori also is associated with a gastric lymphoma of the mucosal-associated lymphoid tissue (MALToma). The gold standard for the diagnosis of H pylori infection of the stomach is endoscopy with biopsy. Endoscopy may show a nodular gastritis of the antrum (Item C2A), and histology of the gastric mucosa demonstrates the characteristic curved organisms (Item C2B) in the gastric glands. Because endoscopy is invasive, other surrogate markers of infection have been identified. Of the options offered, the H pylori fecal antigen is the best to document eradication in a previously treated host. Patients colonized with H pylori have detectable antigen in their stool that disappears upon eradication of the organism. H pylori immunoglobulin G serology (serum antibody) is a useful marker for epidemiologic studies of past or current infection, but its sensitivity and positive predictive value in children is suboptimal. The same is true for salivary antibody. Accordingly, a positive antibody screen should be confirmed by a second test (either fecal antigen, urea breath test, or endoscopy). The Campylobacter-like organisms (CLO) test is performed on a duodenal biopsy. In the CLO test, the duodenal biopsy specimen is placed in a test tube containing chemical reagents. The H pylori bacteria convert urea to ammonia and carbon dioxide via their urease enzyme, and the alkalinity of the ammonia can be detected using an indicator dye. The CLO test cannot be performed on feces. Serum urease concentrations are not helpful in identifying H pylori, which is a mucosal bacterium. Diagnosis, treatment, and eradication of H pylori are well summarized in the American Academy of Pediatrics Red Book and in the North American Society for Pediatric Gastroenterology practice guideline. Patients who have documented ulcers should be tested for H pylori and the organism eradicated if found, but it is unclear if asymptomatic children colonized with H pylori need to be treated. Therapy is given for 14 days and should include a proton pump inhibitor (eg, omeprazole, lansoprazole, pantoprazole) and two antibiotics (eg, tetracycline + clarithromycin, amoxicillin + metronidazole, amoxicillin + clarithromycin) (Item C2C). Treatment failures are common, either because of resistant bacteria or because of poor compliance with the regimen. Therefore, testing for eradication of the organism (either by fecal antigen, urease breath test, or endoscopy) should be performed more than 1 month after therapy has been completed. References: American Academy of Pediatrics. Helicobacter pylori infections. In: Pickering LK, ed. Red Book: 2006 Report of the Committee on Infectious Diseases. 27th ed. Elk Grove Village, Ill: American Academy of Pediatrics; 2006:321-322 Gold BD, Colletti RB, Abbott M, et al. Helicobacter pylori infection in children: recommendations for diagnosis and treatment. J Pediatr Gastroenterol Nutr. 2000;31:490-497. Available at: http://www.jpgn.org/pt/re/jpgn/fulltext.00005176-200011000-00007.htm

Copyright 2007 by the American Academy of Pediatrics

page 4

2007 PREP SA on CD-ROM Question: 3

A term newborn is delivered by emergent cesarean section because of intrauterine growth restriction, oligohydramnios, and nonreassuring fetal heart rate monitoring in labor. Delivery room resuscitation includes endotracheal intubation and assisted ventilation with 100% oxygen, chest compressions, intravenous epinephrine, and volume expansion. Apgar scores are 1, 2, and 3 at 1, 5, and 10 minutes, respectively. An umbilical cord arterial blood gas measurement documents a pH of 6.9 and a base deficit of 20 mmol/L. At 12 hours of age, the infant demonstrates tonic-clonic convulsive activity of the arms and legs with a concomitant decrease in heart rate and bedside pulse oximetry saturation. Of the following, the MOST likely cause for this infant's seizure is

A. hypercalcemia B. hypercarbia C. hyperglycemia D. hypomagnesemia E. hypoxia

Copyright 2007 by the American Academy of Pediatrics

page 5

2007 PREP SA on CD-ROM Critique: 3 Preferred Response: E

Seizures are the most frequent sign of central nervous system injury in the newborn. When seizures occur in a newborn who has depressed neuromotor tone, reflexes, and cardiopulmonary function at birth that requires assisted ventilation, perinatal asphyxia is likely. In this event, Apgar scores typically are depressed to less than 3 at 5 or more minutes after birth, and there is a severely acidotic umbilical cord arterial pH (<7.0), with evidence of metabolic acidemia. Poor tolerance of labor and asphyxia are more common in fetuses that have experienced intrauterine growth restriction. Because the infant in the vignette has the previously described features, hypoxic-ischemic encephalopathy (HIE) must be considered as a cause for the seizures. HIE is the most common cause of seizures occurring in the first 24 hours of postnatal life and accounts for up to 67% of early neonatal seizures. Other causes of neonatal seizure include intracranial hemorrhage, cerebrovascular accidents (stroke), or hemorrhagic infarction (10% to 15%); intracranial malformation (<10%); transient hypoglycemia or hypocalcemia (<10%); drug withdrawal (<5%); and inborn errors of metabolism (<5%). When seizures occur beyond the first 24 hours after birth, especially in the absence of any history of fetal or neonatal asphyxia, the evaluation should focus on potential causes other than HIE. An additional cause for later seizures is infection (meningitis, encephalitis). Asphyxia may result in hypocalcemia and hypoglycemia; hyperglycemia and hypercalcemia are not associated with HIE and do not typically cause seizures. Hypomagnesemia may accompany hypocalcemia in the infant of a diabetic mother, but it is not common following asphyxia and is not associated with neonatal seizures. Hypercarbia may occur in the depressed newborn who has inadequate ventilation, but it is not associated with seizures unless there is corresponding hypoxia. References: Allan WC. The clinical spectrum and prediction of outcome in hypoxic-ischemic encephalopathy. NeoReviews. 2002;3:e108-e115. Available at: http://neoreviews.aappublications.org/cgi/content/full/3/6/e108 Hahn JS, Olson DM. Etiology of neonatal seizures. NeoReviews. 2004;5:e327-e335. Available at: http://neoreviews.aappublications.org/cgi/content/full/5/8/e327 Riviello JJ, Jr. Pharmacology review: drug therapy for neonatal seizures: part 2. NeoReviews. 2004;5:e262-e268. Available at: http://neoreviews.aappublications.org/cgi/content/full/5/6/e262 Thureen PJ, Anderson MS, Hay WW, Jr. The small-for-gestational age infant. NeoReviews. 2001;2:e139-e149. Available at: http://neoreviews.aappublications.org/cgi/content/full/2/6/e139 Wu YW, Backstrand KH, Zhao S, Fullerton HJ, Johnston SC. Declining diagnosis of birth asphyxia in California: 1991-2000. Pediatrics. 2004;114:1584-1590. Available at: http://pediatrics.aappublications.org/cgi/content/full/114/6/1584

Copyright 2007 by the American Academy of Pediatrics

page 6

2007 PREP SA on CD-ROM Question: 4

A 2-year-old boy presents with a 3-day history of diarrhea and vomiting. He has been able to tolerate small amounts of fluids. He is moderately dehydrated, with dry mucous membranes and a heart rate of 145 beats/min. Of the following, the BEST management for this patient's fluid status is

A. hospitalization with intravenous fluids and a restrictive bland diet B. hospitalization with intravenous fluids and gut rest for 24 hours C. oral rehydration therapy at home followed by a clear liquid diet for 24 hours D. oral rehydration therapy at home followed by a diet of fruits, vegetables, and meats E. oral rehydration therapy at home followed by a restrictive bland diet

Copyright 2007 by the American Academy of Pediatrics

page 7

2007 PREP SA on CD-ROM Critique: 4 Preferred Response: D

Dehydration results from a total body loss of water and sodium. Acute infectious gastroenteritis is among the most common causes of dehydration in infants and young children. Both mild and moderate dehydration may be managed at home with oral rehydration therapy, even if the child continues to have intermittent vomiting. Commercial oral rehydration solutions (ORS) are widely available and should be used for this purpose. All ORS are designed to replace lost electrolytes (sodium, chloride, potassium, and bicarbonate) glucose, and water. Young children who have mild dehydration have an estimated water loss of 50 mL/kg, and this amount of ORS can be given via a spoon or syringe in small amounts over 2 to 4 hours. Those who have moderate dehydration should receive 100 mL/kg over 2 to 4 hours. Care should be taken to monitor ongoing losses from stool and emesis, and intravenous or nasogastric rehydration therapy should be considered if losses are excessive or if dehydration worsens or does not improve. Oral rehydration therapy should not be used for children who have severe dehydration, shock, suspected intestinal obstruction, obtundation, or ileus. Once adequate hydration has been assured or rehydration is complete, a normal diet should be given to the child to ensure adequate caloric and nutrient intake. A period without gastrointestinal intake is unnecessary and may delay nutritional recovery. Clear liquid and bland diets also should not be used because they do not provide adequate nutrition. Infants should be given human milk or their usual formula at full strength because diluted formula or human milk (eg, one-half or one-quarter strength formula) will not meet the childs caloric requirements and may worsen electrolyte abnormalities. Lactose-free formulas are generally unnecessary because most children do not develop lactase deficiency. Older children should be given a regular diet of complex carbohydrates, fruits, vegetables, and meats. High sugar-containing liquids should be avoided because the osmotic load of these liquids may worsen diarrhea. References: Finberg L. Dehydration in infancy and childhood. Pediatr Rev. 2002;23:277-282. Available at: http://pedsinreview.aappublications.org/cgi/content/full/23/8/277 King CK, Glass R, Bresee JS, Duggan C, Centers for Disease Control and Prevention. Managing acute gastroenteritis among children: oral rehydration, maintenance, and nutritional therapy. MMWR Recomm Rep. 2003;52(RR-16):1-16. Available at: http://www.cdc.gov/mmwr/preview/mmwrhtml/rr5216a1.htm

Copyright 2007 by the American Academy of Pediatrics

page 8

2007 PREP SA on CD-ROM Question: 5

A worried grandmother brings her 2-year-old grandchild to the emergency department immediately upon finding the boy with an open bottle of 81-mg chewable aspirin (which is used by the grandfather for coronary artery disease prophylaxis). She is unsure of the number of tablets in the bottle prior to ingestion, but the original number was 30, and there are now three remaining. The child has vomited once and is fussy and lethargic. Physical evaluation reveals a 12-kg child who has tachypnea and tachycardia. Laboratory results include a pH of 7.45, carbon dioxide of 25 mEq/L (25 mmol/L), and bicarbonate of 18 mEq/L (18 mmol/L). A salicylate measurement result is pending. Of the following, the next BEST step in the management of this child is to

A. administer activated charcoal B. administer sodium bicarbonate intravenously C. administer syrup of ipecac D. observe the child clinically in the emergency department E. remeasure the salicylate level in 6 hours

Copyright 2007 by the American Academy of Pediatrics

page 9

2007 PREP SA on CD-ROM Critique: 5 Preferred Response: A

Salicylates are common in the homes of many children because many older adults (who may be parents or grandparents) use salicylates for prophylaxis against cardiovascular disease and for arthritis pain. Methylsalicylate present in liniments is especially concentrated. The toddler may ingest such preparations accidentally, as described for the boy in the vignette, and the adolescent may use salicylates in suicide gestures or attempts. Toxic effects of salicylates can include gastritis, anticoagulant effects, increased metabolism, hyperventilation and respiratory alkalosis, and hepatitis. Reye syndrome, which is characterized by hepatitis and encephalopathy, may occur if aspirin is given during certain viral infections. Signs and symptoms include lethargy or coma, vomiting, tachypnea, and tachycardia. The next best step in the management of the child described in the vignette is the administration of activated charcoal. Multiple doses of activated charcoal adsorb salicylates from both the intestinal tract and the systemic circulation. For a child who has ingested a potentially toxic dose of salicylates, serum salicylate concentrations should be measured 2 to 6 hours after the ingestion. However, administration of activated charcoal should not be delayed until the salicylate concentration has been measured if a toxic dose has been ingested. The child in the vignette may have ingested 27 81-mg tablets for a total dose of more than 2,100 mg or 182 mg/kg. This dose can be expected to cause mild-tomoderate toxicity. A serum salicylate value higher than 30 mg/dL is considered toxic, concentrations higher than 70 mg/dL reflect severe toxicity, and those greater than 100 mg/dL are life-threatening. Because gastric emptying time is prolonged with salicylate ingestion, gastrointestinal decontamination by lavage may be effective up to 6 hours after ingestion. However, the child described in the vignette is fussy and lethargic. Therefore, syrup of ipecac is not indicated. Administration of sodium bicarbonate to alkalinize the urine as well as correction of acidosis, hyperkalemia, and hypocalcemia are important adjuncts to gastric decontamination. Cautious administration of intravenous sodium bicarbonate may be indicated clinically in severe acidosis because acidosis enhances brain toxicity, but the patient must be monitored for worsening hypernatremia and hypokalemia as a response to alkalinization. Although the child in the vignette should be observed and repeat salicylate concentrations obtained, gastrointestinal decontamination with activated charcoal is therapeutic and, thus, the next best step. References: Mariscalco MM. Salicylism. In: McMillin JA, DeAngelis CD, Feigin RD, Warshaw JB, eds. Oskis Pediatrics: Principles and Practice. 3rd ed. Philadelphia, Pa: Lippincott, Williams & Wilkins; 1999:623-625 Woolf AD. Poisoning in children and adolescents. Pediatr Rev. 1993;14:411-422

Copyright 2007 by the American Academy of Pediatrics

page 10

2007 PREP SA on CD-ROM Question: 6

A mother brings her 10-month-old son to the emergency department because he has been vomiting for the past 10 days. The child has not experienced any diarrhea. On physical examination, he is lethargic and has dry mucous membranes, reduced tears, a full anterior fontanelle, and 2-second capillary refill. After a second intravenous bolus of 20 mL/kg of normal saline, the boy extends his arms and legs forcefully for 10 seconds. Of the following, the MOST appropriate next step in the management of this child is administration of

A. additional intravenous normal saline bolus of 20 mL/kg B. intravenous dexamethasone of 1 mg/kg C. intravenous fosphenytoin bolus at 20 mg/kg phenytoin equivalents over 10 minutes D. intravenous prochlorperazine of 5 mg E. rapid intravenous lorazepam of 0.05 mg/kg

Copyright 2007 by the American Academy of Pediatrics

page 11

2007 PREP SA on CD-ROM Critique: 6 Preferred Response: B

Brain tumors occur rarely in the pediatric population; the incidence is just under 1 in 25,000 children annually. Despite its rarity, the ominous nature of such a diagnosis requires that practitioners be able to recognize readily the clinical manifestations of a brain tumor. Clinical prodromes may include features of increased intracranial pressure (ICP) (Item C6A), findings of a localizing nature, or symptoms and signs without a localizing quality. Elevated ICP often is insidious and nonspecific initially. Among school-age children, declining academic performance, fatigue, behavioral changes, and vague intermittent headaches are common. Over time, morning headaches, especially pain at the occipital or frontal region, along with vomiting and lethargy ensue. Horizontal diplopia from abducens nerve paresis is common. Papilledema may develop if the pressure is longstanding. Among infants, irritability, anorexia, failure to thrive, and even developmental regression can be early signs of increased ICP. Macrocephaly, splitting of the cranial sutures, or a bulging anterior fontanelle can follow. The setting sun sign, a downward deviation of the eyes, may be seen with attendant hydrocephalus. Localizing findings depend on the region of the brain involved. Supratentorial (cerebrum, basal ganglia, thalamus, hypothalamus, and optic chiasm) tumors can produce hemiparesis, hemisensory loss, hyperreflexia, seizures, or visual complaints. Infratentorial (cerebellum and brainstem) tumors lead to ataxia, hemiparesis, hyperreflexia, or cranial neuropathies, but not seizures. Nonlocalizing symptoms and signs are perhaps the most subtle manifestations of a brain tumor. The child may display changes in affect, energy level, behavior, or weight. Sexual precocity or delayed puberty, growth failure, and somnolence can suggest hypothalamic or pituitary dysfunction. Vomiting can occur with direct irritation of the area postrema in the floor of the fourth ventricle or from a generalized increase in ICP. The vomiting without diarrhea displayed by the 10-month-old in the vignette, along with a full anterior fontanelle in the face of mild dehydration, points to an intracranial process. In addition, the child is exhibiting posturing, not a seizure, from continued overhydration and worsening intracranial pressure following the saline administration. Accordingly, administration of intravenous dexamethasone, along with other measures to treat increased ICP, is most appropriate. Treatment with prochlorperazine will only mask the childs vomiting. Administration of fosphenytoin or lorazepam is not indicated in the absence of seizures. Continued aggressive hydration will worsen this childs ICP. References: Kuttesch JF Jr, Alter JL. Brain tumors in childhood. In: Behrman RE, Kliegman RM, Jenson HB, eds. Nelson Textbook of Pediatrics. 17th ed. Philadelphia, Pa: WB Saunders; 2004:1702-1708 Strother DR, Pollock IF, Fisher PG, et al. Tumors of the central nervous system. In: Pizzo PA, Poplack DG, eds. Principles and Practice of Pediatric Oncology. 4th ed. Philadelphia, Pa: Lippincott, Williams & Wilkins; 2002:751-824

Copyright 2007 by the American Academy of Pediatrics

page 12

2007 PREP SA on CD-ROM Question: 7

An 18-month-old child has been brought to your urgent care clinic for evaluation. He and his mother are in town visiting his grandmother. His mother tells you that she found him playing with an open bottle of his grandmother's medication. On physical examination, he is sleepy but arousable, pale, mildly diaphoretic, and afebrile. His respiratory rate is 20 breaths/min, heart rate is 60 beats/min, and blood pressure is 65/40 mm Hg. His lungs are clear, there are no murmurs, and his pulses are weak. Of the following, the MOST likely cause for this patient's presentation is ingestion of

A. beta blocker B. captopril C. hydralazine D. pseudoephedrine E. tricyclic antidepressant

Copyright 2007 by the American Academy of Pediatrics

page 13

2007 PREP SA on CD-ROM Critique: 7 Preferred Response: A

Accidental ingestion of medications continues to be a significant cause of pediatric morbidity and potential mortality. The problem is greatest in toddlers and is exacerbated when medications are left within reach of children or not appropriately secured in a locked cabinet. The reaction to ingestion of a given medication depends on the class and dose. Beta blockers can have multiple and varying effects on the heart and other organ systems. In the heart, they typically exhibit some degree of negative chronotropic (slowing of the heart rate), dromotropic (slowing of the conduction through the AV node), and inotropic (decrease in the ventricular force of contraction) effects. As a result, this class of medications generally is used to control certain types of arrhythmia and hypertension and to reduce myocardial work and oxygen demand. The depressed sensorium, bradycardia for age, and hypotension with diminished pulses described for the child in the vignette are most consistent with beta blocker ingestion. Captopril is an angiotensin-converting enzyme (ACE) inhibitor. ACE activity converts angiotensin I to angiotensin II, which is a potent vasoconstrictor. Therefore, ACE inhibitors lower the systemic vascular resistance and the systemic blood pressure, decreasing the afterload (work) of the left ventricle. Hydralazine is an arteriolar dilator that may have some positive inotropic effect. It often is used as antihypertensive therapy and typically increases the heart rate. Pseudoephedrine is an alpha agonist that is used commonly for relief of symptoms caused by nasal congestion. Its effects on the cardiovascular system are to cause systemic vasoconstriction that may increase blood pressure, and it also may cause tachycardia. Tricyclic antidepressants are a diverse group of medications that can be used for a variety of noncardiac indications, including depression. When taken in high doses, they may affect the cardiovascular system by causing vasodilation, flushing, hypotension, and tachyarrhythmias. Although captopril, hydralazine, and tricyclic antidepressants may be associated with hypotension, they all would be expected to cause (or be associated with) a corresponding tachycardia. References: Dobson JV, Webb SA. Life-threatening pediatric poisonings. J S C Med Assoc. 2004;100:327332. Abstract available at: http://www.ncbi.nlm.nih.gov/entrez/query.fcgi?cmd=Retrieve&db=pubmed&dopt=Abstract&list_ui ds=15835193&query_hl=11&itool=pubmed_docsum Opie L, Gersh B. Drugs for the Heart. 6th ed. Philadelphia, Pa: WB Saunders Co; 2004 Riordan M, Rylance G, Berry K. Poisoning in children 1: general management. Arch Dis Child. 2002;87:392-396. Available at: http://adc.bmjjournals.com/cgi/content/full/87/5/392

Copyright 2007 by the American Academy of Pediatrics

page 14

2007 PREP SA on CD-ROM Question: 8

The parents of a child who has Down syndrome and a 47,XX+21 karyotype come to you for counseling about future pregnancies. Of the following, their risk for giving birth to another child who has trisomy is CLOSEST to

A. no greater than the general population at risk B. 1% added to the mother's age-related risk C. 5% added to the mother's age-related risk D. 10% added to the mother's age-related risk E. 25% added to the mother's age-related risk

Copyright 2007 by the American Academy of Pediatrics

page 15

2007 PREP SA on CD-ROM Critique: 8 Preferred Response: B

Approximately 94% of individuals who have Down syndrome have trisomy of chromosome 21. Studies of the origins of trisomy 21 (47 chromosomes with trisomy for chromosome 21) show that 90% to 95% of cases are due to a maternal meiotic error, with 75% of these occurring in meiosis I. Approximately 3% to 5% are due to paternal meiotic errors, and the remainder are due to mitotic nondisjunction. Recurrence risk estimates for trisomy are based on empiric data. The overall recurrence risk for having a live-born child who has any trisomy is approximately 1% added to the mothers age-related risk for having a child who has a trisomy, which increases over time. Risk estimates vary slightly from time of amniocentesis to time of term delivery because there is natural loss of some trisomic fetuses between the two events. As a womans age increases, her 1% recurrence risk becomes relatively less significant compared with her age-related risk. For example, at the age of 40 years, a womans risk for having a child who has Down syndrome is 1 in 90 compared with a 1 in 1,500 risk for a 22-year-old woman. Thus, a 22-year-old woman who has a child affected with trisomy 21 has a 1/100 plus a 1/1,500 risk (approximately 1.1%) risk for having another affected child compared with a 0.07% risk for other 22-year-old women who have not had children affected with trisomy. In contrast, a 40-year-old woman who has a child with trisomy 21 has a 1/100 plus a 1/90 (approximately 2.1%) risk for having another affected child, which is similar to the 1.1% risk for 40-year-old women who have not had children with trisomy. References: Hall JG. Chromosomal clinical abnormalities. In: Behrman RE, Kliegman RM, Jenson HB, eds. Nelson Textbook of Pediatrics. 17th ed. Philadelphia Pa: Saunders; 2004:382-391 Nussbaum RL, McInnes RR, Willard HF. Clinical cytogenetics: disorders of the autosomes and the sex chromosomes. In: Thompson & Thompson Genetics in Medicine. 6th ed, revised reprint. Philadelphia, Pa: Saunders; 2004:157-179 Roizen NJ, Stark AR. Epidemiology and genetics of Down syndrome. UpToDate. 2006:14.1. Available at: http://www.utdol.com/utd/content/topic.do?topicKey=dis_chld/2412&type=P&selectedTitle=27~54

Copyright 2007 by the American Academy of Pediatrics

page 16

2007 PREP SA on CD-ROM Question: 9

A 15-month-old infant has been breastfed since birth. He eats finger foods (eg, peas, carrots) and occasionally some cereal. His mother adheres to a vegan diet and plans the same for her child. A complete blood count documents anemia. Of the following, the MOST likely cause of this infant's anemia is a deficiency of

A. folic acid B. niacin C. riboflavin D. thiamine E. vitamin B12

Copyright 2007 by the American Academy of Pediatrics

page 17

2007 PREP SA on CD-ROM Critique: 9 Preferred Response: E

The infant described in the vignette has been exclusively breastfed by a strict vegan mother and is at risk for anemia due to vitamin B12 deficiency. The appearance of hypersegmented neutrophils in the peripheral blood precedes the development of classic megaloblastic (macrocytic) anemia. The infants deficiency may be caused by reduced placental transfer of vitamin B12 in utero and lower vitamin content in the mothers milk. In addition, the infant is being offered other foods that are consistent with a vegan diet and contain little vitamin B12. Deficiency of vitamin B12, including dietary deficiency, is unusual in infants and children in the United States. Other causes of deficiency are pernicious anemia; impaired gastric and small bowel absorption, including resections; and rare inherited metabolic disorders such as methylmalonic aciduria and intrinsic factor deficiency. Vitamin B12 deficiency is diagnosed by recognition of risk factors, documentation of low serum B12 concentrations in the infant, and a subsequent response to treatment. Demonstration of low serum vitamin B12 values in the mother supports the diagnosis. Folic acid deficiency is an unlikely cause of the anemia for the infant described in the vignette. Folate is present in many foods consumed by vegans, and human milk provides adequate folate for the breastfed infant. Folate deficiency rarely is present in newborns because the fetus extracts adequate folate from the mother, even in the presence of maternal deficiency. Defects of folate metabolism are rare, although infants exclusively fed goat milk are at risk for folate deficiency and possible megaloblastic anemia. A diagnosis of folate deficiency should be confirmed, including the exclusion of vitamin B12 deficiency, before initiating folate replacement therapy. Treatment with therapeutic doses of folate may delay the diagnosis of vitamin B12 deficiency and increase the risk of neurologic complications. Erythrocyte folic acid concentration is a better measure of folate sufficiency than serum folate concentrations. Deficiencies of thiamine (vitamin B1), riboflavin (vitamin B2), and niacin (vitamin B3) are rare and not associated with anemia. Thiamine deficiency is associated with beriberi (cardiomyopathy, peripheral neuropathy, and encephalopathy). Riboflavin deficiency is associated with cheilosis and glossitis. Pellagra (dementia, diarrhea, and dermatitis) is associated with niacin deficiency. References: Glader B. Megaloblastic anemias. In: Behrman RE, Kliegman RM, Jenson HB, eds. Nelson Textbook of Pediatrics. Philadelphia, Pa: Saunders; 2004:1611-1613 Nutritional aspects of vegetarian diets. In: Kleinman RE, ed. Pediatric Nutrition Handbook. 5th ed. Elk Grove Village, Ill: American Academy of Pediatrics; 2004:191-208 Vitamins. In: Kleinman RE, ed. Pediatric Nutrition Handbook. 5th ed. Elk Grove Village, Ill: American Academy of Pediatrics; 2004:339-365

Copyright 2007 by the American Academy of Pediatrics

page 18

2007 PREP SA on CD-ROM Question: 10

A 15-year-old boy comes to your office for a health supervision visit. He expresses concern that he is only 5 ft, 2 in tall and is not competitive in track. On physical examination, he appears healthy, has a height of 62 in, and weighs 96 lb. His testes are 8 mL in volume bilaterally, there is slight pubertal phallic enlargement, and he has Sexual Maturity Rating 3 pubic hair. He has a small amount of subareolar breast tissue. His last health supervision visit was 2 years ago. He did not have pubic hair at the last visit, and his testes were described as "prepubertal" in size. Of the following, the MOST likely cause of his short stature is

A. constitutional delayed puberty B. exercise-induced growth delay C. Klinefelter syndrome D. prolactinoma E. undernutrition

Copyright 2007 by the American Academy of Pediatrics

page 19

2007 PREP SA on CD-ROM Critique: 10 Preferred Response: A

Puberty is considered delayed in boys if there is no testicular enlargement by age 14 years. Unless other features are present on physical examination or history, it is very difficult to separate boys who have delayed puberty clinically from those who have true hypogonadotropic hypogonadism until failure of pubertal progression persists for several years. After puberty commences, adult testicular size usually is achieved by 3.21.8 years. The boy described in the vignette is of normal height for age and is progressing into puberty, based on review of his previous visit and his present appearance. Most likely, he has only mild constitutional delayed puberty. Most boys who have constitutional delayed puberty are underweight and do not have other signs of chronic illness. Exercise and weight loss rarely induce the type of marked growth delay in boys that is seen in girls who are competitive gymnasts or active in ballet. Although Klinefelter syndrome may lead to failure to progress through puberty, the increasing testicular size of the boy in the vignette makes this diagnosis unlikely. Prolactinomas can inhibit pubertal growth, but are not common. Underweight caused by undernutrition must be severe before it inhibits growth and puberty. Only close follow-up is required for a boy who seems to be progressing into puberty. However, if there is evidence of failure to progress into puberty, a full evaluation to determine if the boy has an underlying chronic illness such as celiac disease or inflammatory bowel disease, hypergonadotropic hypogonadism as seen in Klinefelter syndrome, or other endocrine disorder such as a prolactinoma, is indicated. References: Cannavo S, Venturino M, Curto L, et al. Clinical presentation and outcome of pituitary adenomas in teenagers. Clin Endocrinol (0xf). 2003;58:519-527. Abstract available at: http://www.ncbi.nlm.nih.gov/entrez/query.fcgi?db=pubmed&cmd=Retrieve&dopt=Abstract&list_ui ds=12641637&query_hl=5&itool=pubmed_docsum Israel EJ, Levitsky LL, Anupindi SA, Pitman MB. Case records of the Massachusetts General Hospital. Weekly clinicopathological exercises. Case 3-2005. A 14-year-old boy with recent slowing of growth and delayed puberty. N Engl J Med. 2005;352:393-403 Lanfranco F, Kamischke A, Zitzmann M, Nieschlag E. Klinefelters syndrome. Lancet. 2004;364:273-283. Abstract available at: http://www.ncbi.nlm.nih.gov/entrez/query.fcgi?db=pubmed&cmd=Retrieve&dopt=Abstract&list_ui ds=15262106&query_hl=3&itool=pubmed_docsum Misra M, Park-Bennett S. Disorders of puberty. In: Burg FD, Ingelfinger JR, Polin RA, Gershon AA, eds. Gellis & Kagans Current Pediatric Therapy. Philadelphia, Pa: WB Saunders; 2002:706710 Rogol AD, Clark PA, Roemmich JN. Growth and pubertal development in children and adolescents: effects of diet and physical activity. Am J Clin Nutr. 2000;72(suppl):521S-528S Available at: http://www.ajcn.org/cgi/content/full/72/2/521S Sedlmeyer IL, Palmert MR. Delayed puberty: analysis of a large case series from an academic center. J Clin Endocrinol Metab. 2002;87:1613-1620. Available at: http://jcem.endojournals.org/cgi/content/full/87/4/1613

Copyright 2007 by the American Academy of Pediatrics

page 20

2007 PREP SA on CD-ROM Question: 11

You are examining a 2-year-old girl who has a 6-month history of developmental regression. During her first postnatal year, she met all motor, language, and social milestones. Her head circumference, which currently is at the 3rd percentile, was at the 75th percentile at birth. On physical examination, she makes poor eye contact and repetitively wrings her hands. Of the following, the MOST appropriate diagnostic test is

A. arylsulfatase A B. fragile X C. hexosaminidase A D. MECP2 gene testing E. urine N-acetyl-aspartic acid

Copyright 2007 by the American Academy of Pediatrics

page 21

2007 PREP SA on CD-ROM Critique: 11 Preferred Response: D

The developmental regression, acquired microcephaly, and hand-wringing movements described for the girl in the vignette are typical findings in the X-linked condition Rett syndrome. Rett syndrome is a neurodegenerative disorder that is associated with developmental regression as well as generalized tonic-clonic seizures, poor feeding, constipation, sleep disorder, breath-holding spells, scoliosis, muscle wasting, cardiac arrhythmias, and death in late adolescence. The diagnosis can be made by testing for the MECP2 gene (chromosome X q28), a gene for a transcription factor that binds to methylated CpG island and silences transcription. Deficiency of arylsulfatase A causes the degenerative lysosomal disorder metachromatic leukodystrophy. Fragile X testing is performed primarily in boys who have cognitive impairment and dysmorphic features. Hexosaminidase A deficiency causes Tay-Sachs disease. Accumulation of N-acetyl-aspartic acid in the urine is associated with the leukodystrophy Canavan disease. References: Johnston MV. Neurodegenerative disorders of childhood. In: Behrman RE, Kliegman RM, Jenson HB, eds. Nelson Textbook of Pediatrics. 17th ed. Philadelphia, Pa: WB Saunders Co; 2004:2029-2034 Rich J. In brief: degenerative central nervous system (CNS) disease. Pediatr Rev. 2001;22:175176. Available at: http://pedsinreview.aappublications.org/cgi/content/full/22/5/175

Copyright 2007 by the American Academy of Pediatrics

page 22

2007 PREP SA on CD-ROM Question: 12

A 7-year-old girl presents to your clinic with a 2- to 3-day history of a nonproductive cough, malaise, and temperature to 101F (38.3C). On physical examination, you note that the girl does not appear ill and are surprised to hear widespread crackles in the lungs bilaterally. Chest radiography demonstrates bilateral diffuse infiltrates (Item Q12A). Of the following, the MOST appropriate antimicrobial agent to treat this girl's infection is

A. amoxicillin B. azithromycin C. doxycycline D. levofloxacin E. trimethoprim-sulfamethoxazole

Copyright 2007 by the American Academy of Pediatrics

page 23

2007 PREP SA on CD-ROM Critique: 12 Preferred Response: B

School-age children are at risk for lower respiratory tract infections (LRTIs) from Mycoplasma pneumoniae. The highest incidence of LRTI from M pneumoniae occurs in children 5 to 14 years of age. Frequent symptoms associated with pneumonia due to Mycoplasma include fever, cough, and malaise. The lack of coryza can be useful in differentiating illness due to Mycoplasma from other common viral agents. The radiographic pattern associated with this pathogen varies, but bilateral, diffuse infiltrates (Item C12A), as reported for the girl in the vignette, are common. Macrolides (eg, erythromycin, clarithromycin, azithromycin) are the preferred antimicrobial agents to treat infections due to Mycoplasma. Because azithromycin can be administered once daily and has few adverse effects compared with the other macrolides, it has become the preferred agent. These agents also are effective against common respiratory flora such as Moraxella catarrhalis, Haemophilus influenzae, Streptococcus pyogenes, and viridans streptococci. Other atypical pathogens, such as Chlamydia sp, Bordetella pertussis, and Legionella pneumophila, also can be treated with the macrolides. Clarithromycin and azithromycin have activity against many nontuberculous mycobacterial species (eg, M avium complex, M kansasii), as well. Doxycycline is effective against Mycoplasma sp, but should not be used in children younger than 8 years of age due to potential bone and teeth staining. Similarly, levofloxacin is effective against many atypical agents, including Mycoplasma sp, but should not be used routinely for children younger than 18 years of age because of the interference with cartilage growth demonstrated in puppies treated with the fluoroquinolones. Amoxicillin and trimethoprimsulfamethoxazole are not effective against Mycoplasma. References: Gaston B. Pneumonia. Pediatr Rev. 2002;23:132-140. Available at: http://pedsinreview.aappublications.org/cgi/content/full/23/4/132 James LP, Rahman SM, Farrar HC, Jacobs RF. Antimicrobial agents. In: Long SS, Pickering LK, Prober CG, eds. Principles and Practice of Pediatric Infectious Diseases. 2nd ed. Philadelphia, Pa: Churchill Livingstone; 2003:1458-1510

Copyright 2007 by the American Academy of Pediatrics

page 24

2007 PREP SA on CD-ROM Question: 13

A 7-year-old boy comes to your office with complaints of daily bedwetting for 2 months. He was completely toilet trained by 4 years of age and had been dry at night except for occasional (about once per month) minor bedwetting until recently. He denies daytime enuresis, dysuria, frequency, urgency, fever, abdominal pain, or constipation. He has no history of urinary tract infections. His physical examination reveals weight and height at the 75th percentiles and no abnormalities. Of the following, the MOST important next step in this child's evaluation is to obtain

A. abdominal radiography B. renal ultrasonography C. serum electrolyte measurement D. urinalysis E. voiding cystourethrography

Copyright 2007 by the American Academy of Pediatrics

page 25

2007 PREP SA on CD-ROM Critique: 13 Preferred Response: D

After achieving daytime and nighttime dryness, most children experience occasional episodes of nocturnal enuresis until the age of 5 to 6 years. Boys generally achieve dryness later than girls, and some boys continue to experience occasional bedwetting episodes to the age of 7 to 8 years. Indeed, about 15% of 7-year-old boys experience occasional bedwetting, although the symptoms resolve spontaneously in most. History, physical examination, and urinalysis can identify the cause of new-onset nocturnal enuresis for most children who have secondary nocturnal enuresis (ie, achieved general nighttime dryness once toilet trained). A simple review of recent voiding patterns and general lifestyle issues may disclose the cause of the bedwetting in many patients. It is vital to consider the common causes of new-onset bedwetting, including psychosocial stress, urinary tract infection, and new disease (eg, diabetes mellitus or insipidus). For children who have normal findings on history, physical examination, and urinalysis, imaging studies such as abdominal radiography, renal ultrasonography, and voiding cystourethrography are unlikely to aid in determining the cause of bedwetting. Measurement of serum electrolytes rarely is warranted at the outset if the history, physical examination, and urinalysis results are normal. If a specific cause is identified (eg, urinary tract infection), treatment of the primary condition often results in resolution of the bedwetting. However, behavioral or pharmacotherapy may be required for many children in whom a primary cause cannot be identified and who experience unabated episodes of enuresis. Therapeutic options include counseling, hypnosis, urine alarm, imipramine, no treatment, or desmopressin. Counseling may be helpful if the childs bedwetting is due to psychosocial stress, but it has mixed results. Behavioral therapy such as hypnosis also induces resolution in some patients but success is very dependent on the expertise of the therapist. Results also have been mixed with urine alarms. Success requires strict compliance, but if the child and parents are motivated, many children experience at least a moderate reduction in bedwetting episodes. Anticholinergic medications (eg, oxybutynin) or medications that have anticholinergic properties (eg, imipramine) that alter bladder physiology in favor of prolonged filling and decreased contractions are most successful in children who have underlying bladder dyssynergy. Common adverse effects of imipramine include dry mouth and constipation. Rarely, it may cause blood pressure changes or arrhythmias. Common adverse effects of oxybutynin include gastrointestinal symptoms and urinary retention. Finally, intranasal desmopressin may improve water reabsorption during the nighttime, but the ability to concentrate the urine is not impaired in most children who experience bedwetting, thus limiting the overall effectiveness of this medication. Common adverse effects of desmopressin include gastrointestinal symptoms, headache, flushing, and rarely, hyponatremia. References: Bower WF, Yip SK, Yeung CK. Dysfunctional elimination symptoms in childhood and adulthood. J Urol. 2005;174:1623-1627. Abstract available at: http://www.ncbi.nlm.nih.gov/entrez/query.fcgi?cmd=Retrieve&db=pubmed&dopt=Abstract&list_ui ds=16148668&query_hl=21&itool=pubmed_docsum Lee T, Suh HJ, Lee HJ, Lee JE. Comparison of effects of treatment of primary nocturnal enuresis with oxybutynin plus desmopressin, desmopressin alone or imipramine alone: a randomized controlled clinical trial. J Urol. 2005;174:1084-1087. Abstract available at: http://www.ncbi.nlm.nih.gov/entrez/query.fcgi?cmd=Retrieve&db=pubmed&dopt=Abstract&list_ui ds=16094064&query_hl=18&itool=pubmed_docsum Yagci S, Kibar Y, Akay O, et al. The effect of biofeedback treatment on voiding and urodynamic parameters in children with voiding dysfunction. J Urol. 2005;174:1994-1997. Abstract available at: http://www.ncbi.nlm.nih.gov/entrez/query.fcgi?db=pubmed&cmd=Retrieve&dopt=AbstractPlus&li st_uids=16217376

Copyright 2007 by the American Academy of Pediatrics

page 26

2007 PREP SA on CD-ROM Question: 14

You are speaking to the mother of a child who attends a junior high school where one of the students was diagnosed with meningococcal disease 24 hours ago. Her child does not have any classes with the index patient and, except for passing him in the hall during lunch 3 days ago, has had no other contact with the patient. The child's mother is frantic because the school sent home a notice asking parents to bring their children to the public health department or their private physician to receive antibiotic prophylaxis. Of the following, the MOST appropriate advice for this parent is that her child

A. does not require antibiotic prophylaxis and does not need to be seen B. does not require antibiotic prophylaxis but needs to be evaluated to determine if she is
developing symptoms of meningococcal disease

C. needs to be seen to obtain nasopharyngeal cultures for meningococcal organisms and if the
cultures are positive, may require antibiotic prophylaxis

D. requires antibiotic prophylaxis and should be seen immediately E. should be seen immediately to determine if she needs to be hospitalized and treated for
possible meningococcal disease

Copyright 2007 by the American Academy of Pediatrics

page 27

2007 PREP SA on CD-ROM Critique: 14 Preferred Response: A

Close contacts of all persons who have invasive meningococcal disease, whether sporadic or in an outbreak, are at high risk for infection and should receive chemoprophylaxis within 24 hours of diagnosis of the primary case, regardless of vaccination status. Close contacts include all household contacts, child care and nursery school contacts during the previous 7 days, persons who have had direct contact with the patients oral secretions, and persons who frequently eat or sleep in the same dwelling as the index patient. However, classroom contacts of students who have meningococcal disease, such as the child described in the vignette, are considered casual contacts (no history of direct exposure to the index patients oral secretions), and the use of prophylactic antibiotics is not recommended. In view of this and the fact that the contact is asymptomatic, she does not require medical evaluation at this time, and nasopharyngeal cultures are not indicated. Because secondary cases of meningococcal disease can occur several weeks after the onset of disease in the index case, the use of meningococcal vaccine is a possible adjunct to chemoprophylaxis if the serogroup is contained in the vaccine. Other infections can spread easily in the household setting and may require the use of postexposure prophylactic immunoglobulin, antibiotics, or vaccines to prevent development of disease in individuals exposed to the index case. Selected diseases and prophylactic interventions are summarized in Item C14A. For detailed advice regarding management of specific cases, pediatricians should consult the most recent Report of the Committee on Infectious Diseases (Red Book), the Centers for Disease Control and Prevention, or a pediatric infectious disease specialist. References: American Academy of Pediatrics. Haemophilus influenzae infections. In: Pickering LK, ed. Red Book: 2006 Report of the Committee on Infectious Diseases. 27th ed. Elk Grove Village, Ill: American Academy of Pediatrics; 2006:310-318 American Academy of Pediatrics. Hepatitis A. In: Pickering LK, ed. Red Book: 2006 Report of the Committee on Infectious Diseases. 27th ed. Elk Grove Village, Ill: American Academy of Pediatrics; 2006:326-335 American Academy of Pediatrics. Meningococcal infections. In: Pickering LK, ed. Red Book: 2006 Report of the Committee on Infectious Diseases. 27th ed. Elk Grove Village, Ill: American Academy of Pediatrics; 2006:452-460 American Academy of Pediatrics. Pertussis. In: Pickering LK, ed. Red Book: 2006 Report of the Committee on Infectious Diseases. 27th ed. Elk Grove Village, Ill: American Academy of Pediatrics; 2006:498-520 American Academy of Pediatrics. Varicella-zoster infections. In: Pickering LK, ed. Red Book: 2006 Report of the Committee on Infectious Diseases. 27th ed. Elk Grove Village, Ill: American Academy of Pediatrics; 2006:711-725 Robinson J. Infectious diseases in schools and child care facilities. Pediatr Rev. 2001;22:39-46. Available at: http://pedsinreview.aappublications.org/cgi/content/full/22/2/39

Copyright 2007 by the American Academy of Pediatrics

page 28

2007 PREP SA on CD-ROM Question: 15

An 18-year-old boy presents to the emergency department 30 minutes after eating at a seafood restaurant. He states that approximately 10 minutes into his meal he developed generalized hives, pruritus, and difficulty breathing. He has a history of shellfish food allergy, although he had ordered steak and denies eating any crab, lobster, or shrimp. On physical examination, the patient appears to have labored breathing, audible wheezing, and diffuse raised erythematous lesions (Item Q15A) on his trunk and extremities. His vital signs include a temperature of 98.5F (37C), heart rate of 100 beats/min, respiratory rate of 22 breaths/min, blood pressure of 110/60 mm Hg, and pulse oximetry of 92% on room air. Of the following, the MOST appropriate immediate action is

A. administration of 100% oxygen B. administration of 1 L intravenous normal saline C. administration of intramuscular epinephrine D. administration of beta-2 agonist nebulization E. observation

Copyright 2007 by the American Academy of Pediatrics

page 29

2007 PREP SA on CD-ROM Critique: 15 Preferred Response: C

The adolescent described in the vignette most likely is experiencing an adverse food reaction, specifically anaphylaxis to shellfish. Patients who have known food allergies, such as this boy, must remain vigilant regarding food choices and reading of food labels because foods may contain hidden ingredients or may be contaminated with allergenic proteins. Most likely, the nonallergenic food (steak) in this case was contaminated with an allergenic food (shellfish) while being prepared. For highly allergic individuals, only a small amount of antigen is required to cause a reaction. Because of the risk of cross-contamination, many allergists recommend strict avoidance of the allergenic food within the home. The rapid (<30 min) onset of urticaria and wheezing in a shellfish-allergic patient who is eating in a seafood restaurant is likely anaphylaxis and warrants prompt administration of intramuscular epinephrine. Other supportive measures, such as administration of oxygen, intravenous fluids, and beta-2 agonists, may improve some clinical symptoms, but they do not reverse anaphylaxis. Simple observation clearly is not indicated in a person who has food allergy and is experiencing anaphylaxis with respiratory symptoms. Other causes that should be considered during an acute food reaction include food poisoning, particularly scombroid fish poisoning. Members of the Scomberesocidae or Scombridae families (eg, albacore, mackerel, tuna, kingfish) that have spoiled can have bacterial overgrowth. The bacterial overgrowth is responsible for converting histidine, found in high concentrations in the flesh of scombroid fish, to histamine. Reactions may be indistinguishable from anaphylaxis, with affected individuals developing rapid-onset flushing, tachycardia, and a rash, but food poisoning is more likely to involve many restaurant patrons. Observation may be all that is needed in these cases, although more severe symptoms of hypotension and wheezing can be managed with epinephrine, intravenous fluids, antihistamines, and steroids. References: Chegini S, Metcalfe DD. Contemporary issues in food allergy: seafood toxin-induced disease in the differential diagnosis of allergic reactions. Allergy Asthma Proc. 2005;26:183-190. Abstract available at: http://www.ncbi.nlm.nih.gov/entrez/query.fcgi?db=pubmed&cmd=Retrieve&dopt=AbstractPlus&li st_uids=16119031 Sampson HA, Leung DYM. Adverse reactions to foods. In: Behrman RE, Kliegman RM, Jenson HB, eds. Nelson Textbook of Pediatrics. 17th ed. Philadelpha, Pa: WB Saunders Co; 2004:789792

Copyright 2007 by the American Academy of Pediatrics

page 30

2007 PREP SA on CD-ROM Question: 16

A 3-year-old child is rushed to the emergency department after the mother found her with an open and empty bottle of acetaminophen. The mother has no idea how many tablets were in the bottle. She estimates that no more than 1 hour has passed since the child ingested the tablets. The child began to vomit during the trip to the emergency department, and has vomited three times more since her arrival. The child is awake and alert but clearly unhappy, crying even in her mother's arms. She appears pale and diaphoretic. Her heart rate is 110 beats/min, respiratory rate is 26 breaths/min, temperature is 98.6F (37C), and blood pressure is 90/60 mm Hg. Of the following, the MOST appropriate statement about acetaminophen toxicity is that

A. an antidote is available, but its use can be deferred until further information is gathered B. given the short duration since the ingestion, it will be helpful to administer syrup of ipecac C. multiple episodes of vomiting indicate that irreversible liver damage already has occurred D. the administration of activated charcoal is contraindicated in acetaminophen toxicity E. the contents of one bottle of acetaminophen are not sufficient to cause life-threatening toxicity
in a child

Copyright 2007 by the American Academy of Pediatrics

page 31

2007 PREP SA on CD-ROM Critique: 16 Preferred Response: A

N-acetyl cysteine is a highly effective antidote to acetaminophen (APAP) toxicity if administered within 10 hours of ingestion, but there is time to evaluate the level of APAP exposure before resorting to this management option for the patient described in the vignette. The level of APAP exposure is determined by measuring serum APAP concentrations 4 or more hours after ingestion and plotting the result on a nomogram (Item C16A). Serum APAP concentrations below the lower line are not likely to result in toxicity, levels between the two lines represent possible toxicity, and levels above the upper line indicate a significant risk of toxicity. If it can be established definitively that a child has ingested more than the minimal toxic dose of 140 mg/kg from the history alone, immediate administration of the antidote is appropriate, especially if serum APAP concentrations are not immediately available. The primary toxicity of APAP is severe hepatic damage due to the binding of toxic metabolites to the hepatocytes. Normally, the metabolites conjugate with glutathione, rendering them harmless, but in severe overdose, glutathione is depleted. The benefit of N-acetylcysteine stems from its ability to act as a glutathione precursor. The first stage of APAP toxicity occurs within several hours of ingestion and includes anorexia, nausea, and vomiting. These symptoms generally resolve within 12 to 24 hours. In the case of significant toxicity, a latent phase develops and lasts 1 to 4 days, during which time liver enzyme concentrations may begin to rise. In severe toxicity, jaundice and liver tenderness develop at the end of the latent phase. About 2% to 4% of patients who develop toxic plasma concentrations and do not receive antidotal treatment progress to hepatic failure and death. Since 2003, the American Academy of Pediatrics has recommended against the use of ipecac in toxic ingestions, partly because the prolonged vomiting that can result interferes with other management options. Although toxicologists advised against the administration of activated charcoal in the recent past due to concerns about interference with the absorption of Nacetylcysteine, recent studies have shown clinically insignificant reductions in N-acetylcysteine absorption following charcoal administration. Charcoal now is recommended after most cases of APAP overdose. Vomiting is a nonspecific response to APAP ingestion and does not predict hepatic toxicity. Toxicity is confirmed by an increase in serum transaminases and a prolonged prothrombin time. There are many different APAP-containing products on the market, with a wide range of total medication per bottle. The potential for toxicity depends on the weight of the child and the contents of the bottle. In the absence of that information, it is not possible to state that the child in the vignette has received a subtoxic dose. References: American Academy of Pediatrics Committee on Drugs. Acetaminophen toxicity in children. Pediatrics. 2001;108:1030-1024. Available at: http://pediatrics.aappublications.org/cgi/content/full/108/4/1020 American Academy of Pediatrics Committee on Injury, Violence, and Poison Prevention. Poison treatment in the home. Pediatrics. 2003;112:1182-1185 Available at: http://pediatrics.aappublications.org/cgi/content/full/112/5/1182 Bond GR. Home syrup of ipecac use does not reduce emergency department use or improve outcome. Pediatrics. 2003;112:1061-1064. Available at: http://pediatrics.aappublications.org/cgi/content/full/112/5/1061 McGuigan ME: Poisoning potpourri. Pediatr Rev. 2001:22:295-302. Available at: http://pedsinreview.aappublications.org/cgi/content/full/22/9/295 Osterhoudt KC, Ewald MB, Shannon M, Henretig FM. Toxicologic emergencies. In: Fleisher GR, Ludwig S, Henretig FM, eds. Textbook of Pediatric Emergency Medicine. 5th ed. Philadelphia, Pa: Lippicott Williams & Wilkins; 2006:951-1008

Copyright 2007 by the American Academy of Pediatrics

page 32

2007 PREP SA on CD-ROM Question: 17

A sexually active adolescent male presents with the primary complaint of pubic and perianal pruritus. Careful examination reveals pubic or "crab" lice infestation (Item Q17A). Of the following, the MOST characteristic feature of this infestation is that the lice

A. are difficult to detect because of their rapid movement B. lay viable eggs that may hatch up to 7 days after being attached to a hair shaft C. may survive for 36 hours without a blood meal D. only infest pubic and perianal regions E. rarely infest African-American patients

Copyright 2007 by the American Academy of Pediatrics

page 33

2007 PREP SA on CD-ROM Critique: 17 Preferred Response: C

Pediculosis, the infestation of humans by lice, has been documented for millennia. Three species of lice infest humans: Pediculus humanus humanus, the body louse; Pediculus humanus capitis, the head louse; and Pthirus pubis, the crab louse. The hallmark of louse infestation is pruritus at the site of bites. Lice are more active at night, frequently disrupting sleep of the host, which is the derivation of the term feeling lousy. Adult crab lice can survive without a blood meal for 36 hours. Unlike head lice, which may travel up to 23 cm/min, pubic lice are sluggish, traveling a maximum of 10 cm/d. Viable eggs on pubic hairs may hatch up to 10 days later. Crab louse infestation is localized most frequently to the pubic and perianal regions but may spread to the mustache, beard, axillae, eyelashes, or scalp hair. Infestation usually is acquired through sexual contact, and the finding of pubic lice in children (often limited to the eyelashes) should raise concern for possible sexual abuse. Maculae caeruleae (Item C17A), blue-gray macules observed on the abdomen and thighs at sites where lice have fed, is a useful, although less common finding. Crab lice affect all races and ethnic groups. This is in contrast to head lice, which rarely infest African-Americans, perhaps because the oval cross-sectional shape of their scalp hair does not permit lice to grasp the hair effectively. The preferred treatments for pubic lice infestation are permethrin 1% or pyrethrin with piperonyl butoxide. Other options include permethrin 5% or malathion (although the latter agent is expensive, potentially flammable, and may be irritating when applied to the groin). Lindane is effective, but concerns about toxicity if used improperly or ingested inadvertently limit its use. It is considered a second-line therapy, is contraindicated for use in neonates and pregnant women, and should be used with caution in those weighing less than 110 lb. If the eyelashes are affected, petrolatum may be applied to them three to five times daily for 10 days. References: Meinking T, Taplin D. Infestations: lice. In: Schachner LA, Hansen RC, eds. Pediatric Dermatology. 3rd ed. St. Louis, Mo: Mosby; 2003:1141-1160 Paller AS, Mancini AJ. Bites and infestations. In: Hurwitz Clinical Pediatric Dermatology. 3rd ed. Philadelphia, Pa: Elsevier Inc; 2006:479-502

Copyright 2007 by the American Academy of Pediatrics

page 34

2007 PREP SA on CD-ROM Question: 18

A 6-year-old girl presents with a 1-year history of periumbilical, nonradiating abdominal pain. The pain occurs at least three times per week and lasts up to 30 minutes. There is no history of heartburn, constipation, or diarrhea. Physical examination, complete blood count, erythrocyte sedimentation rate, and urinalysis yield normal results. A Helicobacter pylori serology (immunoglobulin G antibody) is positive. Of the following, a TRUE statement regarding this patient is that

A. empiric therapy with omeprazole and trimethoprim-sulfamethoxazole should be instituted B. the H pylori antibody test is more sensitive in younger children than older children C. the positive serology should be confirmed by another diagnostic test D. the prevalence of H pylori increases with higher socioeconomic status E. this patient most likely has a gastric ulcer

Copyright 2007 by the American Academy of Pediatrics

page 35

2007 PREP SA on CD-ROM Critique: 18 Preferred Response: C

There is no clear association between Helicobacter pylori and chronic recurrent abdominal pain of childhood. Chronic recurrent abdominal pain affects approximately 10% to 15% of school-age children. No structural or inflammatory cause of the pain is identified in most cases. Affected children usually have functional bowel disease. Functional bowel disease in children and teens can be categorized as: nonulcer dyspepsia (epigastric discomfort, early satiety, and bloating), irritable bowel syndrome (abdominal cramps associated with diarrhea or constipation), and classic functional pain of childhood (periumbilical, crampy, and nonradiating). A subset of children who have chronic recurrent abdominal pain also have concurrent H pylori infection. Commonly, this infection is identified on routine serologic screening by their primary care physicians, as described for the girl in the vignette. In most such children, especially in those who have no epigastric symptoms, the H pylori probably represents asymptomatic colonization rather than the cause of the pain. The only firm indications for eradicating H pylori in adults are duodenal ulcer and gastric lymphoma (MALToma). Randomized, controlled trials in adults who have nonulcer dyspepsia suggest that eradication of H pylori does not resolve dyspeptic symptoms. Similar large-scale trials have not been conducted in children. Therefore, it remains controversial whether children who have chronic abdominal pain and H pylori infection should receive therapy for their colonization. Nevertheless, some open-label studies in children do suggest that eradicating H pylori may alleviate pain. In addition, eradicating H pylori may prevent the development of subsequent peptic ulcer disease or (far less commonly) gastric lymphoma. However, the serology has a poor predictive value in a low-prevalence population. Therefore, one approach is to confirm a positive serology with a second diagnostic test (fecal antigen, urea breath test, or endoscopy). If results of the second test are positive, therapy should be considered. Although endoscopy is the gold standard diagnostic test for H pylori and can identify other causes of abdominal pain (eg, esophagitis, gastritis, ulcers, celiac disease), it is also the most invasive test. Thus, the physician must determine benefits, risks, and cost of diagnostic testing and treatment in individual patients. Omeprazole and trimethoprim/sulfisoxazole do not eradicate H pylori. The H pylori antibody test is less sensitive in younger children. The prevalence of H pylori decreases with increasing socioeconomic status. Gastric ulcers are uncommon in children, and a child who has periumbilical abdominal pain most likely has functional abdominal pain without peptic ulcer disease. References: AAP Subcommittee on Chronic Abdominal Pain. Chronic abdominal pain in children. Pediatrics. 2005;115:e370-e381. Available at: http://pediatrics.aappublications.org/cgi/content/full/115/3/e370 Gold BD, Colletti RB, Abbott M, et al. Helicobacter pylori infection in children: recommendations for diagnosis and treatment. J Pediatr Gastroenterol Nutr. 2000;31:490-497 Available at: http://www.jpgn.org/pt/re/jpgn/fulltext.00005176-200011000-00007.htm McNamara DA, Buckley M, OMorain CA. Nonulcer dyspepsia. Current concepts and management. Gastroenterol Clin North Am. 2000;29:807-818 Abstract available at: http://www.ncbi.nlm.nih.gov/entrez/query.fcgi?cmd=Retrieve&db=pubmed&dopt=Abstract&list_ui ds=11190065&query_hl=11&itool=pubmed_docsum

Copyright 2007 by the American Academy of Pediatrics

page 36

2007 PREP SA on CD-ROM Question: 19

You are conducting rounds in the newborn nursery with a group of residents. You describe the choices for infant nutrition that might optimize growth and development. Of the following, you are MOST likely to state that

A. preterm and term infants both require 100 to 120 kcal/kg per day of energy to grow B. preterm infants require less caloric intake per kilogram to grow than do term infants C. term infants require 60 to 80 kcal/kg per day of energy to grow D. term infants require 30 to 50 mL/kg per day of fluid intake E. term infants whose birthweights are greater than 2,500 g require more energy per kilogram to
grow than those whose birthweights are less than 2,500 g

Copyright 2007 by the American Academy of Pediatrics

page 37

2007 PREP SA on CD-ROM Critique: 19 Preferred Response: A

The energy (caloric) requirement for newborns to grow and develop varies with gestational age, presence of illness, a history of surgery and need for wound healing, and the neonatal environment. Energy use is partitioned into that required for basic metabolic function (basal metabolic rate), thermic expenditure of enteral feeding and digestion, physical activity, and the synthesis of new tissue. For term and preterm infants, these categories have the following requirements: Categorykcal/kg per day Resting metabolic rate50 to 60 Activity0 to 10 Temperature regulation0 to 10 Growth of new tissue10 to 15 Storage of energy (mostly fat)20 to 30 Energy excreted (urine and stool)10 to 15 TOTAL90 to 140

Special considerations for preterm infants and low-birthweight (LBW) term infants (<2,500 g) warrant administration of the higher end of this caloric range. The preterm and LBW infant have lower fat stores and cannot conduct thermoregulation well, resulting in greater expenditure in heat production than at term. Similarly, such infants must accomplish a greater proportion of organ system development and new tissue synthesis (as well as production of storage fat) than the term infant, and this uses more energy. All of these considerations are greater still in the very low-birthweight (<1,500 g) or extremely low-birthweight (<1,000 g) infant. Further, conditions such as heart or lung disease, surgery, or infection increase energy expenditure. Because digestive or absorptive problems also may increase the excretion of unused energy, attention must be paid to the source of energy (carbohydrate, fat, and protein composition and content) in the milk provided to infants. The correct fluid volume for term newborns is 60 to 100 mL/kg per day. References: Adamkin D. Feeding the preterm infant. In: Bhatia J, ed. Perinatal Nutrition: Optimizing Infant Health and Development. New York, NY: Marcel Dekker; 2005:165-190 Denne SC, Poindexter BB, Leitch CA, Ernst JA, Lemons PK, Lemons JA. Nutrition and metabolism in the high-risk neonate: enteral nutrition. In: Martin RJ, Fanaroff AA, Walsh MC, eds. Fanaroff and Martins Neonatal-Perinatal Medicine: Diseases of the Fetus and Infant. 8th ed. Philadelphia, Pa: Mosby-Elsevier; 2006:661-678 Kleinman RE. Nutritional needs of the preterm infant. In: AAP Nutrition Handbook. 5th ed. Elk Grove Village, Ill: American Academy of Pediatrics; 2004:23-46 Kuzma-OReilly B, Duenas ML, Greecher C, et al. Evaluation, development, and implementation of potentially better practices in neonatal intensive care nutrition. Pediatrics. 2003;111: e461e470. Available at: http://pediatrics.aappublications.org/cgi/content/full/111/4/SE1/e461 Schanler RJ. The low birth weight infant. In: Walker WA, Watkins JB, Duggan C, eds. Nutrition in Pediatrics: Basic Science and Clinical Applications. 3rd ed. Hamilton, Ontario, Canada: BC Decker, Inc; 2003:491-514

Copyright 2007 by the American Academy of Pediatrics

page 38

2007 PREP SA on CD-ROM Question: 20

A 3-year-old child who has a history of recurrent otitis media with effusion (OME) in infancy is brought to the clinic. His mother is afraid that he has a hearing loss because he does not talk as much as his brother did at the same age. He speaks in three-word sentences, and you can understand fewer than 50% of his words. Results of his physical examination, including the ears, are normal. Of the following, the MOST appropriate statement regarding this child's condition is that

A. even mild conductive hearing loss could affect his later school performance without frank
speech delay

B. OME does not cause conductive hearing loss severe enough to cause speech delay C. performing hearing screening solely in response to parental concern is not recommended D. testing air and bone conduction thresholds in the office will help you rule out hearing loss E. the absence of middle ear fluid rules out conductive hearing loss

Copyright 2007 by the American Academy of Pediatrics

page 39

2007 PREP SA on CD-ROM Critique: 20 Preferred Response: A

Some children who have hearing loss may be diagnosed during neonatal screening, but many develop hearing loss later in infancy or early childhood and usually are brought to medical attention because of a delay in language development. The 3-year-old child described in the vignette has language that is less than 50% understood by the practitioner, which is abnormal and necessitates an audiologic evaluation. In addition, a parents concern about a childs speech should prompt the pediatrician to investigate the childs hearing abilities and should not be ignored. A history of poor school performance and inattention at school are other reasons to consider a hearing evaluation. Hearing impairment can be divided into three primary categories: sensorineural, conductive, and mixed. Causes of sensorineural losses include genetic diseases, infections, ototoxic medications, and anatomic anomalies. Conductive hearing loss is more common than sensorineural hearing loss in childhood. It results from interrupted sound transmission in the external or middle ear, and middle ear effusion is the most common cause. The hearing loss can vary between 20 and 60 dB, which may be substantial enough to cause significant speech delay. It has been shown that even mild speech delay can impair school performance. Children who have borderline hearing loss (16 to 25 dB) may miss 10% of speech in an environment with noise (eg, a noisy classroom), and interaction with peers may be affected. Children who have mild hearing loss (26 to 40 dB) may miss up to 50% of speech in a noisy classroom and may develop poor self-esteem because of being labeled as a problem student. Moderate hearing loss (41 to 55 dB) can cause significant problems with communication. Early diagnosis, with a prompt, complete speech and language evaluation, is the key to maximizing speech improvement. The earlier the cause of conductive hearing loss is discovered and corrected (eg, through tympanostomy tube placement), the more likely the child will develop normal language. Testing air and bone conduction thresholds with a tuning fork may help distinguish between conductive and sensorineural hearing loss, but it should not be used to diagnose hearing loss, and results may be unreliable if the hearing loss is bilateral. The absence of middle ear fluid at the time of the examination does not rule out hearing loss. Therefore, audiometry testing should be performed if hearing loss is suspected. References: Gregg RB, Wiorek LS, Arvedson JC. Pediatric audiology: a review. Pediatr Rev. 2004;25:224234. Available at: http://pedsinreview.aappublications.org/cgi/content/full/25/7/224 Haddad J Jr. Hearing loss. In: Behrman RE, Kliegman RM, Jenson HB, eds. Nelson Textbook of Pediatrics. 17th ed. Philadelphia, Pa: WB Saunders Co; 2004:2129-2135 Jarvelin MR, Mati-Torkko E, Sorri MJ, Rantakallio PT. Effect of hearing impairment on educational outcomes and employment up to the age of 25 years in northern Finland. Br J Audiol. 1997;31:165-175. Abstract available at: http://www.ncbi.nlm.nih.gov/entrez/query.fcgi?cmd=Retrieve&db=pubmed&dopt=Abstract&list_ui ds=9276099&query_hl=10&itool=pubmed_docsum

Copyright 2007 by the American Academy of Pediatrics

page 40

2007 PREP SA on CD-ROM Question: 21

The pediatric resident at the emergency department notifies you that one of your patients, an 18month-old boy, has been admitted to the hospital for observation after being found with an open bottle of lemon-scented furniture polish. On arrival at the emergency department, the child had lemon-scented polish on his clothing, and his breath smelled of the substance. His mother stated that he had vomited once, but there has been no choking, gagging, or coughing since the ingestion. Of the following, the MOST appropriate management includes

A. administration of activated charcoal B. consultation for urgent bronchoscopy C. gastric lavage D. observation for 6 hours for signs of distress E. serial chest radiographs

Copyright 2007 by the American Academy of Pediatrics

page 41

2007 PREP SA on CD-ROM Critique: 21 Preferred Response: D

Ingestion of a hydrocarbon can cause irritation of the orogastric mucosa and the respiratory epithelium. Because hydrocarbons often are volatile, simple inhalation of vapor may cause choking and gagging, even if no liquid is ingested. Initial physical examination may reveal the smell of the hydrocarbon substance on the patients breath, as described for the boy in the vignette; other symptoms of inhalation or ingestion may include wheezing, laryngospasm, and other signs of respiratory distress. Ingestion may result in coughing and vomiting, which can cause further aspiration or lung irritation from the gastric contents. Absence of tachypnea is a good prognostic sign. Fulminant chemical pneumonitis associated with hypoxia may occur and reaches a peak within 3 days of ingestion. Fever occurs early (usually within 8 hours) and is due to direct chemical inflammation, not infection. Management of asymptomatic patients, such as the boy described in the vignette, may include only observation for 4 to 6 hours. Gastric lavage or administration of either ipecac or activated charcoal is contraindicated (unless the hydrocarbon is mixed with certain high-risk toxins or the volume ingested is large) because of the risk of vomiting and additional aspiration. Symptomatic patients should have respiratory distress managed with oxygen, a trial of bronchodilators, and supportive care. There is no evidence-based support for empiric treatment with antibiotics or steroids. Chest radiographs initially appear normal, especially in asymptomatic children, although they may reveal signs of pneumonitis (Item C21A) by 6 hours after ingestion. Radiography is not necessary in asymptomatic children who have normal findings on physical examination. Bronchoscopy rarely is indicated or helpful. For most children, the outcome following hydrocarbon ingestion is good. In rare cases, bronchospasm may continue or pneumatoceles may form. References: Fortenberry JD, Mariscalco MM. General principles of poisoning. In: McMIllan JA, DeAngelis CD, Feigin RD, Warshaw JB, eds. Oskis Pediatrics: Principles and Practice. 3rd ed. Philadelphia, Pa: Lippincott, Williams & Wilkins; 1999:621-622

Copyright 2007 by the American Academy of Pediatrics

page 42

2007 PREP SA on CD-ROM Question: 22

A 17-year-old girl complains of clumsiness over the past 3 days. She has had moderate headaches for 1 month and neck discomfort for 3 days. Physical examination reveals rightsided dysmetria and left upper and lower extremity numbness to pinprick and weakness, graded as 4/5. The remainder of her examination results are normal. Of the following, the MOST appropriate evaluation to establish this patient's diagnosis is

A. computed tomography scan of the brain B. lumbar puncture C. measurement of nerve conduction velocities D. measurement of somatosensory evoked potentials E. urine toxicology screen

Copyright 2007 by the American Academy of Pediatrics

page 43

2007 PREP SA on CD-ROM Critique: 22 Preferred Response: A

Headache is a frequent primary or secondary complaint in pediatric practice. The clinician must be able to distinguish primary, benign headaches (ie, migraines or tension headaches) from secondary, pathologic headaches, which can stem from a tumor, abscess, or other causes of increased intracranial pressure. Secondary or ominous headaches are characterized by occipital or frontal pain. The headache frequently is worse in the morning, after the child has been recumbent at night and intracranial pressure rises with less effect from gravity. The child may wake in the early morning with a complaint of head pain. Vomiting may occur at that time, too. Headaches that are associated with persistent vomiting are worrisome, especially when diarrhea is absent. Secondary headache also may be precipitated by the Valsalva maneuver. New-onset neurologic findings in the first 2 to 6 months after onset of pain are especially concerning for a secondary headache. Specifically, the finding of papilledema, strabismus, unilateral weakness, or ataxia should alert the practitioner to the likelihood of a structural lesion. Neurologic deficits such as weakness, strabismus, or ataxia also can be seen rarely with migraine variants, such as hemiplegic migraine, ophthalmoplegic migraine, or basilar migraine, respectively, but such atypical headache syndromes are diagnoses of exclusion. Urgent neuroimaging should be obtained for children who have headache and neurologic deficits. Computed tomography scan of the head without contrast can be obtained quickly without sedation to identify a structual lesion causing headache from mass effect. Alernatively, magnetic resonance imaging of the head can be obtained if the child is stable and monitored closely during any sedation that may be required. The headaches exibited by the 17-year-old girl in the vignette, combined with her ataxia, left hemisensory loss, right dyscoordination, and neck pain, point to a posterior fossa mass. Head computed tomography scan is the most appropriate next step. Lumbar puncture is contraindicated when there is suspicion of a mass. Nerve conduction velocities are useful only to evaluate a peripheral neuropathy, and somatosensory evoked potentials are used to evaluate demyelination in the peripheral or central nervous system. A toxin would not produce this constellation of symptoms and signs. References: Fisher PG. Help for headaches: a strategy for your busy practice. Contemp Pediatr. 2005;22:3441 Haslam RHA. Headaches. In: Behrman RE, Kliegman RM, Jenson HB, eds. Nelson Textbook of Pediatrics. 17th ed. Philadelphia, Pa: WB Saunders; 2004:2012-2015 Honig PJ, Charney EB. Children with brain tumor headaches. Distinguishing features. Am J Dis Child. 1982;136:121-124. Abstract available at: http://www.ncbi.nlm.nih.gov/entrez/query.fcgi?cmd=Retrieve&db=pubmed&dopt=Abstract&list_ui ds=7064925&query_hl=7&itool=pubmed_DocSum Lewis DW, Ashwal S, Dahl G, et al, Quality Standards Subcommittee of the American Academy of Neurology; Practice Committee of the Child Neurology Society. Practice parameter: evaluation of children and adolescents with recurrent headaches: report of the Quality Standards Subcommittee of the American Academy of Neurology and the Practice Committee of the Child Neurology Society. Neurology. 2002;59:490-498. Available at: http://www.neurology.org/cgi/content/full/59/4/490

Copyright 2007 by the American Academy of Pediatrics

page 44

2007 PREP SA on CD-ROM Question: 23

You are supervising a pediatric resident in her continuity clinic. She is evaluating a 4-week-old male infant who has had projectile vomiting after feeding for the past week. After reviewing the patient's electrolyte levels, she obtains an electrocardiogram (ECG) and asks you to help interpret it (Item Q23A). The ECG reveals flat T waves. Of the following, the MOST likely electrolyte abnormality suggested by the electrocardiographic findings is

A. hypercalcemia B. hyperkalemia C. hypernatremia D. hypocalcemia E. hypokalemia

Copyright 2007 by the American Academy of Pediatrics

page 45

2007 PREP SA on CD-ROM Critique: 23 Preferred Response: E

Hypokalemia generally is defined as a serum potassium concentration of less than 3.5 mEq/L (3.5 mmol/L) in infants and children. Potassium is an intracellular cation that is vital in maintaining the normal transmembrane charge in cells. Hypokalemia may be precipitated by gastrointestinal loss from vomiting or diarrhea, especially when there is inadequate oral intake. Other primary causes include aldosterone excess or renal tubular disorders. Abnormalities of potassium homeostasis can affect cardiac conduction and often are observed with defined changes in surface electrocardiography. The typical electrocardiographic findings seen in a child who has hypokalemia include ST segment depression, flattening of the T waves, and the appearance of a U wave (Item C23A), as seen for the infant in the vignette. Generally, there are broadened T and U waves, and rarely the QT interval can be prolonged, which places the patient at greater risk for the development of ventricular dysrhythmias. However, arrhythmias with hypokalemia are unusual unless the patient is receiving digoxin, whose arrhythmogenic effects are potentiated by hypokalemia. Hyperkalemia most commonly is due to renal failure, adrenal insufficiency, or an iatrogenic overdose. As levels exceed 5.5 mEq/L (5.5 mmol/L), the T waves on the electrocardiogram may become tall and peaked (Item C23B). Prolonged conduction delay also may develop as potassium concentrations increase. Hypercalcemia may be due to a variety of causes and typically shortens the QT interval by shortening the ST segment. Hypercalcemia also may affect the sinus node, causing sinus slowing or sinus arrest. Hypocalcemia, in contrast, prolongs the ST segment, thereby prolonging the QT interval. Hypernatremia is not associated with electrocardiographic abnormalities. References: Garson A Jr. Electrocardiography. In: Garson A Jr, Bricker JT, Fisher DJ, Neish SR, eds. The Science and Practice of Pediatric Cardiology. 2nd ed. Baltimore, Md: Williams & Wilkins; 1998:735-788 Schaefer TJ, Wolford RW. Disorders of potassium. Emerg Med Clin North Am. 2005;23:723-74. Abstract available at: http://www.ncbi.nlm.nih.gov/entrez/query.fcgi?orig_db=PubMed&db=PubMed&cmd=Search&ter m=%22Emergency+medicine+clinics+of+North+America%22[Jour]+AND+2005[pdat]+AND+Sch aefer+TJ[author]

Copyright 2007 by the American Academy of Pediatrics

page 46

2007 PREP SA on CD-ROM Question: 24

A female infant is born with unilateral cleft lip and palate. Findings on the remainder of the physical examination are normal, and you inform the parents that this apparently is an isolated birth defect. During counseling of the family about their risk for having another similarly affected child, the statement that you are MOST likely to include is that

A. because the defect is isolated, their risk is no greater than that of any other couple B. cleft lip with/without cleft palate is a multifactorial trait that has a 4% risk of recurrence C. the recurrence risk can be estimated only after a chromosome analysis has been obtained D. the risk is increased for future females, but not males E. the risk is increased only if one of the parents had cleft lip with/without cleft palate

Copyright 2007 by the American Academy of Pediatrics

page 47

2007 PREP SA on CD-ROM Critique: 24 Preferred Response: B

Isolated cleft lip with/without cleft palate is inherited as a multifactorial trait that is due to the interaction of genetic and environmental factors. Multifactorial inheritance commonly refers to traits that occur with greater frequency among individuals in a particular family than they do in the general population, but at a lower rate than would be expected for single-gene traits. Recurrence risks for multifactorial traits have been derived by analysis of many families that express the trait. For cleft lip with/without cleft palate, the recurrence risk for a family in which neither parent is affected and there is one affected child is approximately 4% in future pregnancies. In general, disorders that are inherited as multifactorial traits include most of the common birth defects (Item C24A) as well as many adult-onset diseases. For most multifactorial birth defects, the recurrence risk for parents who have had one affected child ranges from 2% to 5%, although some traits are associated with higher risks (eg, 10% for hypospadias). Risks may be modified based on other characteristics, such as the severity of the defect, the sex of the affected individual, and the number of affected individuals in the family. For example, unilateral cleft lip has a recurrence risk of 4%, while bilateral cleft lip has a recurrence risk of 6%. Similarly, some defects have a sex predilection, including Hirschsprung disease and pyloric stenosis, which appear more commonly in males, and hip dislocation, which is more common among females. In families in which one of these defects has occurred, the risk to future offspring is greater if the original affected child is of the less commonly affected sex. For example, the recurrence risk for subsequent siblings of a child who has pyloric stenosis is greater if the affected child is a girl rather than a boy. Estimation of recurrence risks also requires that information be obtained about the extended family to determine if there are other similarly affected individuals, particularly first-degree relatives. The risk is lowest in families in whom a single individual is affected; families that include multiple affected individuals have a higher risk. For example, the recurrence risk for subsequent siblings of a child who has clubfoot is 3%, but if one parent also had clubfoot, the risk increases to 10%. To provide the most accurate information about recurrence risk to families, all children who have birth defects should be examined carefully for the presence of other abnormalities. The possibility that the most obvious defect actually is part of a genetic syndrome, which may carry a higher risk, must be considered in all cases. For example, cleft lip is the most obvious feature of van der Woude syndrome, which also includes lip pits and the possible absence of central incisors. This disorder is transmitted as an autosomal dominant trait, with a 50% recurrence risk. Thus, families that have this genetic syndrome are at much higher risk for recurrence, and the parents should be examined if this disorder is suspected. Alternatively, some birth defects can occur following exposure to a teratogen, in which case the recurrence risk in future pregnancies is not increased if the teratogen is avoided (eg, cleft lip with/without cleft palate in fetuses exposed to phenytoin). In general, chromosome analysis should be obtained in infants and children who have two or more major abnormalities (eg, clefting and congenital heart disease) or one major anomaly and two or three minor defects (eg, epicanthal folds, single palmar crease). Accordingly, chromosome analysis is not indicated for the infant described in the vignette, who has an isolated cleft lip and palate. References: Bacino CA. Approach to congenital malformations. UpToDate. 2006:14.1. Available at: http://www.utdol.com/utd/content/topic.do?topicKey=dis_chld/7840&type=P&selectedTitle=86~15 2 Jones KL. Dysmorphology. In: Behrman RE, Kliegman RM, Jenson HB, eds. Nelson Textbook of Pediatrics. 17th ed. Philadelphia Pa: Saunders; 2004:616-623 Stal S, Hollier LH Jr, Edwards M. Facial clefts and holoprosencephaly. UpToDate. 2006;14.1. Available at:

Copyright 2007 by the American Academy of Pediatrics

page 48

2007 PREP SA on CD-ROM

http://www.utdol.com/utd/content/topic.do?topicKey=neonatol/13931&type=P&selectedTitle=3~5

Copyright 2007 by the American Academy of Pediatrics

page 49

2007 PREP SA on CD-ROM Question: 25

A 15-year-old postmenarcheal girl presents with persistent right lower quadrant discomfort of 6 weeks' duration and a feeling of abdominal fullness. Her last menstrual period was 3 weeks ago. Results of a urine pregnancy test are negative, and urinalysis results are normal. Ultrasonography reveals a well-defined right ovarian cyst (Item Q25A) filled with multiple echoes. Of the following, the BEST next step for the management of this patient is

A. laparoscopic cyst aspiration B. measurement of serum tumor markers C. oophorectomy D. repeat ultrasonography in 4 to 8 weeks E. therapy with combined oral contraceptives

Copyright 2007 by the American Academy of Pediatrics

page 50

2007 PREP SA on CD-ROM Critique: 25 Preferred Response: B

Ovarian masses in adolescent girls may result from functional cysts and benign or malignant neoplasms. Ovarian cysts are very common in adolescents. Although many ovarian cysts are asymptomatic, some patients present with pain, irregular menses, urinary frequency, constipation, or pelvic heaviness. In general, a simple, fluid-filled cyst less than 6 cm in size documented on ultrasonography may be managed with conservative observation that usually includes follow-up examinations or ultrasonography. Many of these cysts resolve in 4 to 8 weeks. However, solid masses and cysts that have ultrasonographic findings suspicious for malignancy require further evaluation. Suspicious cysts have complex features, such as septations, are multiloculated, and have increased echoes or calcifications, as described for the cyst in the vignette. Such cysts subsequently are imaged with computed tomography scan or magnetic resonance imaging to define more clearly the characteristics of a solid mass or suspicious cyst and identify liver or lung metastases. Surgical intervention is based on the radiologic images and the identification of tumor markers. Tumor markers (eg, CA 125, alphafetoprotein, human chorionic gonadotropin, carcinoembryonic antigen, F9 embryoglycan) assist in the diagnosis of an ovarian neoplasm and reflect clinical response or recurrences. Accordingly, serum tumor markers should be measured in the patient in the vignette. Patients should be counseled about the risks and benefits of surgical approaches, adjunctive chemotherapy, emerging genetic tests, and preservation of reproductive potential. Treatment of a patient who has an ovarian neoplasm should involve collaborative participation of the surgeon, pathologist, geneticist, and oncologist to develop an individualized plan of care. Laparoscopic cyst aspiration is not an appropriate intervention for a cyst that has complex features on ultrasonography. The aspiration of large, simple ovarian cysts has been associated with a high rate of recurrence. Many large, asymptomatic cysts have been followed with repeat ultrasonography and spontaneously resolved. Oophorectomy should not be the presumptive surgical intervention. Management plans should be based on tumor markers and pathologic tissue sections. Repeat ultrasonography in 4 to 6 months is inadequate to assess an ovarian mass that has complex ultrasonographic features. In 4 to 8 weeks, most simple, fluid-filled ovarian cysts resolve or decrease in size, although corpus luteum cysts may be slower to regress, usually showing evidence of regression by 3 months. Therapy with combined estrogen and progestin oral contraceptives that suppress the hypothalamic ovarian axis has been offered to patients who have simple, fluid-filled ovarian cysts, particularly those who have von Willebrand disease or who are recipients of anticoagulation therapy. Such therapy is an attempt to decrease the likelihood of future cyst formation and the risk of hemorrhage. References: Laufer MR, Goldstein DP. Benign and malignant ovarian masses. In: Emans SJH, Laufer MR, Goldstein DP, eds. Pediatric and Adolescent Gynecology. 5th ed. Philadelphia, Pa: Lippincott, Williams & Wilkins; 2005:685-728 Nelson AL, Neinstein LS. Pelvic masses. In: Neinstein LS, ed. Adolescent Health Care A Practical Guide. 4th ed. Philadelphia, Pa: Lippincott, Williams & Wilkins; 2002:994-1000 Stepanian M, Cohn DE. Gynecologic malignancies in adolescents. Adoles Med Clin. 2004;15:549568

Copyright 2007 by the American Academy of Pediatrics

page 51

2007 PREP SA on CD-ROM Question: 26

A 16-year-old boy is brought to your office by his parents because he wants medicine to help him grow and develop muscles. He has been working out for 1 year without much effect. On physical examination, his height is 63 in (adjusted mid-parental height, 73 in), he weighs 106 lb, and he has testes that are 8 mL in volume. He appears healthy but very young. His father says that he was very small as a boy but was given three injections of growth hormone when he was 16 and grew to his present height of 71 in during late high school and college. Laboratory test results for the boy are normal, including a complete blood count, erythrocyte sedimentation rate, electrolytes, blood urea nitrogen, creatinine, prolactin, insulin-like growth factor-1, tissue transglutaminase antibody, quantitative immunoglobulin A, free thyroxine, and thyroid-stimulating hormone. A bone age radiograph is reported to resemble that of a 13-year-old. Of the following, your BEST approach is to

A. counsel that this is delayed puberty and that he will grow over the next few years B. obtain brain magnetic resonance imaging and refer the boy to an endocrinologist for growth
hormone studies

C. offer an endocrinologic referral for a 1- to 2-year course of growth hormone therapy D. offer an endocrinologic referral for a 1- to 2-year course of testosterone therapy E. suggest that high-calorie food supplements be taken daily to enhance growth

Copyright 2007 by the American Academy of Pediatrics

page 52

2007 PREP SA on CD-ROM Critique: 26 Preferred Response: A

The normal screening laboratory results and delayed bone age described for the boy in the vignette suggest the diagnosis of constitutional delayed puberty. Because he already has entered puberty, only counseling that his growth spurt is soon to come is necessary. A normal insulin-like growth factor-1 concentration does not entirely rule out growth hormone deficiency, but the boys regular growth rate, reasonable height, strong family history for pubertal delay, and signs of puberty make growth hormone deficiency very unlikely. Therefore, referral to an endocrinologist and magnetic resonance imaging are unnecessary at this point. It is likely that the boys father received testosterone rather than growth hormone injections. Daily injections of growth hormone induce, on average, additional height of approximately 1/3 inch for each year of treatment. However, if treatment is stopped before adult height or target height is reached, growth usually slows, and the growth hormone-induced additional height prediction is lost. Lowdose testosterone injections sometimes are administered to induce some external signs of puberty (a slight increase in growth, increase in phallic size and in pubic hair) in a boy who has pubertal delay. Treatment for 3 to 6 months appears not to compromise adult height. Such therapy can be offered to the boy and his family, but is never required. In girls, estrogen therapy also induces puberty. However, this treatment rarely is used in constitutional delay because exogenous estrogen advances bone age and decreases adult height. There is no evidence that high-calorie food supplements enhance growth or height significantly in otherwise normal pubertal children. References: Brook CG. Treatment of late puberty. Horm Res. 1999;51(suppl 3):101-103 Abstract available at: http://www.ncbi.nlm.nih.gov/entrez/query.fcgi?cmd=Retrieve&db=pubmed&dopt=Abstract&list_ui ds=10592451&query_hl=50&itool=pubmed_docsum Cuttler L. Safety and efficacy of growth hormone treatment for idiopathic short stature [editorial]. J Clin Endocrinol Metab. 2005;90:5502-5504 Misra M, Park-Bennett S. Disorders of puberty. In: Burg FD, Ingelfinger JR, Polin RA, Gershon AA, eds. Gellis & Kagans Current Pediatric Therapy. Philadelphia, Pa: WB Saunders; 2002:706710 Nathan BM, Palmert MR. Regulation and disorders of pubertal timing. Endocrinol Metab Clin North Am. 2005;34:617-641. Abstract available at: http://www.ncbi.nlm.nih.gov/entrez/query.fcgi?orig_db=PubMed&db=PubMed&cmd=Search&ter m=%22Endocrinology+and+metabolism+clinics+of+North+America%22[Jour]+AND+617[page]+ AND+2005[pdat]

Copyright 2007 by the American Academy of Pediatrics

page 53

2007 PREP SA on CD-ROM Question: 27

You are seeing for the first time a 15-month-old boy who was born at 28 weeks' gestation. He had an afebrile seizure at 12 months, but takes no medications. He uses both a cup and a bottle and takes most solid foods without choking. He wakes frequently at night. He can sit alone, but does not crawl or walk. He uses three words other than "mama" or "dada." His growth parameters are at the 50th percentile for length and head circumference and at the 10th percentile for weight. On physical examination, you note increased tone in his lower extremities and trunk. Of the following, the MOST likely other information that you would expect is a history of

A. constipation B. developmental regression C. recurrent rashes D. tachycardia E. tendon releases

Copyright 2007 by the American Academy of Pediatrics

page 54

2007 PREP SA on CD-ROM Critique: 27 Preferred Response: A

Cerebral palsy is an umbrella term covering a group of nonprogressive, but often changing, motor impairment syndromes due to lesions or anomalies of the brain that arise in the early stages of its development. Some 2 to 3 per 1,000 children have cerebral palsy in the United States. Although it may be difficult to determine the exact cause of the motor impairment for an individual child, several clinical situations are associated with increased risk of cerebral palsy, including brain malformation, chromosomal abnormalities, intrauterine growth restriction, prematurity, birth hypoxia, and postnatal events such as traumatic injury and meningitis. The motor manifestations of cerebral palsy are treated best by an interdisciplinary team of clinicians who can address medical and surgical management of tone and joint contractures as well as therapists who can address therapeutic issues such as range of motion and equipment needs. As described for the boy in the vignette, children who have cerebral palsy frequently have other coexisting conditions, such as cognitive disability or mental retardation, skeletal malformations, visual impairment, communication disorders, seizures, behavioral concerns, abnormal oral-motor function, and poor growth and nutrition. Screening for and treatment of associated conditions can support his motor, social-adaptive, and cognitive development. He most likely will have difficulty with feeding and constipation, and it is helpful to provide anticipatory guidance for the family to ensure that he receives adequate nutrition. Cerebral palsy is a nonprogressive condition; it is not characterized by developmental regression. Cerebral palsy also is not associated with a higher risk of skin or cardiac conditions. The boy may require tendon releases in the future, but no contractures are noted on this physical examination, and he is very young to develop this complication of cerebral palsy. References: Cooley WC; American Academy of Pediatrics Committee on Children With Disabilities. Providing a primary care medical home for children and youth with cerebral palsy. Pediatrics. 2004;114:1106-1113. Available at: http://pediatrics.aappublications.org/cgi/content/full/114/4/1106 Green L, Greenberg GM, Hurwitz E. Primary care of children with cerebral palsy. Clin Family Pract. 2003;5:467-491 Johnston MV. Cerebral palsy. In: Behrman RE, Kliegman RM, Jenson HB, eds. Nelson Textbook of Pediatrics. 17th ed. Philadelphia, Pa: WB Saunders Co; 2004:2024-2025

Copyright 2007 by the American Academy of Pediatrics

page 55

2007 PREP SA on CD-ROM Question: 28

You are working in a refugee camp when a mother brings in her 8-day-old boy. The mother states he started becoming irritable 2 days ago, and now any loud noise appears to cause him pain, as evidenced by muscle tightening and back arching causing his head to nearly touch his feet. Physical examination reveals only a dried packing on his umbilical cord, as is the local custom. He appears normal until he is stimulated by touch or a loud noise, and then he begins to cry, stiffens, and arches his back. The stiffness continues until he calms down. Of the following, the MOST likely diagnosis is

A. bacterial meningitis B. botulism C. generalized seizure D. tetanus E. viral encephalitis

Copyright 2007 by the American Academy of Pediatrics

page 56

2007 PREP SA on CD-ROM Critique: 28 Preferred Response: D

Tetanus is caused by a neurotoxin from the anaerobic bacterium Clostridium tetani and is characterized by muscle spasms (Item C28A) that often are worsened by noise or other external stimuli, as described for the infant in the vignette. The muscle spasms can progress from mild to severe over a 1-week period. Severe spasms subside over a period of weeks if the patient survives. The clinical manifestations of tetanus include localized tetanus (local muscle spasm in areas contiguous with the infected wound), cephalic tetanus (dysfunction of the cranial nerves in association with a wound on the head or neck), and generalized tetanus (full body involvement). Neonatal tetanus remains prevalent in areas of the world where mothers have not been vaccinated and local umbilical cord care is poor. The diagnosis is made clinically, and treatment includes the use of human tetanus immune globulin (TIG), oral antimicrobial agents (metronidazole or penicillin) for 10 to 14 days, and other supportive measures (eg, ventilatory support, decreased external stimuli such as loud noises). In addition, all infected wounds should be cleaned properly and debrided. Tetanus is prevented best through immunization with tetanus toxoid. Tetanus toxoid is available: 1) in combination with diphtheria toxoid and acellular pertussis vaccine (DTaP) to provide basic immunity against tetanus, diphtheria, and pertussis; 2) as part of a double antigen (DT) for children up to 6 years of age who cannot receive the pertussis component of the DTaP; and 3) as a single antigen (tetanus toxoid) (TT) to immunize pregnant women and women of childbearing age to prevent tetanus in their newborns. DT must be replaced with Td or dTaP, which has a reduced diphtheria toxoid content, for patients older than 7 years of age to lessen the adverse effects associated with the use of the higher-dose diphtheria toxin. For most indications, TT is administered in combination with a diphtheria toxoid-containing preparation. Patients who have bacterial meningitis and viral encephalitis usually demonstrate illness continuously, not just when stimulated by loud noises. A generalized seizure typically includes loss of consciousness and a postictal state. Infection with Clostridium botulinum (botulism) is a neuroparalytic disorder that is not manifested by muscle spasms. References: American Academy of Pediatrics. Tetanus. In: Pickering LK, ed. Red Book: 2006 Report of the Committee on Infectious Diseases. 27th ed. Elk Grove Village, Ill: American Academy of Pediatrics; 2006:648-654 Arnon SS. Tetanus (Clostridium tetani). In: Behrman RE, Kliegman RM, Jenson HB, eds. Nelson Textbook of Pediatrics. 17th ed. Philadelphia, Pa: WB Saunders Co; 2004:951-953

Copyright 2007 by the American Academy of Pediatrics

page 57

2007 PREP SA on CD-ROM Question: 29

A 9-year-old girl develops daily daytime enuresis for 3 weeks. You notice her sitting in a chair in your office waiting room with her legs crossed and squirming. About 20 minutes later, upon entering the examination room, she runs to the bathroom to urinate. You are able to obtain a urine sample, and results of analysis are normal. Her vital signs and findings on history and physical examination are all unremarkable. Of the following, the MOST appropriate next step in her management is to

A. consult a urologist B. design a voiding routine C. limit nighttime fluids D. obtain renal ultrasonography E. prescribe oxybutynin

Copyright 2007 by the American Academy of Pediatrics

page 58

2007 PREP SA on CD-ROM Critique: 29 Preferred Response: B

Daytime enuresis is a common disorder for children. It is usually a transient phenomenon, and the cause is often elusive. The symptoms of daytime enuresis generally resolve regardless of the type of intervention. The girl described in the vignette, who has developed daytime enuresis, sits with her legs crossed and squirming in the waiting room and runs to the bathroom to urinate. These are classic symptoms of daytime enuresis and withholding urine in resistant children. She also has normal findings on urinalysis. Establishing a fixed voiding routine (eg, voiding at assigned times throughout the day) likely will resolve her symptoms. It is very unlikely that the girl has a congenital urinary tract malformation because she did not develop enuresis until the age of 9 years. Thus, consultation with a urologist or renal ultrasonography is unnecessary. Limiting fluids at night would be unhelpful because she has daytime, not nighttime, enuresis and because limiting fluids at night is not efficacious for bedwetting. Finally, she has no symptoms of urgency incontinence (frequency, dribbling, recurrent urinary tract infections) and, therefore, therapy such as oxybutynin, which reduces the symptoms of voiding dysfunction in many patients, is not warranted. Children who present with enuresis and an underlying behavioral condition (eg, environmental stress) may benefit from counseling or hypnosis, although the efficacy varies according to the expertise of the clinician. Bladder training exercises can help some children who do not empty completely with each voiding. References: Hellerstein S, Linebarger JS. Voiding dysfunction in pediatric patients. Clin Pediatr (Phila). 2003;42:43-49. Abstract available at: http://www.ncbi.nlm.nih.gov/entrez/query.fcgi?cmd=Retrieve&db=pubmed&dopt=Abstract&list_ui ds=12635981&query_hl=15&itool=pubmed_docsum Kodman-Jones C, Hawkins L, Schulman SL. Behavioral characteristics of children with daytime wetting. J Urol. 2001;166:2392-2395. Abstract available at: http://www.ncbi.nlm.nih.gov/entrez/query.fcgi?db=pubmed&cmd=Retrieve&dopt=AbstractPlus&li st_uids=11696795 Robson WLM, Leung AKC, Van Howe R. Primary and secondary nocturnal enuresis: similarities in presentation. Pediatrics. 2005;115:956-959. Available at: http://pediatrics.aappublications.org/cgi/content/full/115/4/956

Copyright 2007 by the American Academy of Pediatrics

page 59

2007 PREP SA on CD-ROM Question: 30

A 7-month-old male infant adopted from a Somalian orphanage has been in the United States for 3 weeks. Ten days after arrival, the infant is hospitalized with a fever to 104F (40C), increased irritability, and decreased oral intake and activity. Results of a blood culture are positive for Haemophilus influenzae type b. Since arriving in the United States, he has been attending the home child care that his adoptive mother operates, which includes six other children between 12 and 18 months of age, each of whom has been fully immunized. Of the following, the MOST appropriate recommendation for chemoprophylaxis for child care attendees and staff is

A. no prophylaxis for attendees and supervisory personnel B. prophylaxis for all attendees C. prophylaxis for all attendees and supervisory personnel D. prophylaxis only for attendees between 12 and 15 months of age E. prophylaxis only for supervisory personnel

Copyright 2007 by the American Academy of Pediatrics

page 60

2007 PREP SA on CD-ROM Critique: 30 Preferred Response: A

Haemophilus influenzae type b (Hib) is an encapsulated pleomorphic gram-negative coccobacillus that is transmitted from person to person by inhalation of respiratory droplets or by direct contact with respiratory secretions. Careful observation of exposed unimmunized or incompletely immunized household, child care, or nursery contacts is important. The risk of invasive Hib disease is increased among unimmunized household contacts younger than 4 years of age. Nursery and child care center contacts also may be at increased risk of secondary disease, although the magnitude of this risk is unclear. The risk of secondary disease in children attending child care centers seems to be lower than that seen for agesusceptible household contacts and is rare when all contacts are older than 2 years of age. When two or more cases of invasive Hib disease have occurred within 60 days in a child care center or nursery school where unimmunized or incompletely immunized children attend, rifampin prophylaxis for all attendees and supervisory personnel, regardless of age, should be considered. When a single case has occurred, as in the vignette, the use of rifampin prophylaxis in exposed child care groups that include unimmunized or incompletely immunized children is controversial; many experts recommend no prophylaxis for attendees or supervisory personnel. In addition to the recommendations for chemoprophylaxis, unimmunized or incompletely immunized children should receive a dose of conjugate Hib vaccine and should complete the recommended age-specific immunization schedule. References: American Academy of Pediatrics. Haemophilus influenzae infections. In: Pickering LK, ed. Red Book: 2006 Report of the Committee on Infectious Diseases. 27th ed. Elk Grove Village, Ill: American Academy of Pediatrics; 2006:310-318 Centers for Disease Control and Prevention. Progress toward elimination of Haemophilus influenzae type b disease among infants and childrenUnited States, 1998-2000. MMWR Morb Mortal Wkly Rep. 2002;51:234-237. Available at: http://www.cdc.gov/mmwr/preview/mmwrhtml/mm5111a4.htm

Copyright 2007 by the American Academy of Pediatrics

page 61

2007 PREP SA on CD-ROM Question: 31

You are performing a presedation physical examination in a 3-year-old female who is scheduled to have a repeat computed tomography (CT) scan with contrast. The mother mentions that her daughter experienced diffuse hives and facial swelling 10 minutes after the contrast administration during her first CT. Of the following, the BEST way to prevent future contrast reactions is to:

A. administer 1 L intravenous normal saline prior to the procedure B. perform desensitization to contrast C. provide pretreatment with oral antihistamines and steroids D. use a high-osmolar contrast agent E. use a contrast agent with low iodine content

Copyright 2007 by the American Academy of Pediatrics

page 62

2007 PREP SA on CD-ROM Critique: 31 Preferred Response: C

Reactions to radiocontrast media (RCM) are estimated to occur in 1% to 12% of all individuals receiving standard RCM. RCM reactions are termed anaphylactoid, pseudoallergic, or nonimmunoglobulin (Ig)E-mediated. The mechanism for these reactions differs from that for anaphylaxis, which is an IgE-mediated reaction, but the clinical symptoms are indistinguishable because both result in mast cell degranulation and histamine release. Initially, RCM reactions were believed to be due to the iodine content, but studies have demonstrated that iodine is not the cause. It now generally is agreed that the osmolarity of the solution is the primary cause. Compared with low-osmolar RCM, high-osmolar RCM is associated with higher rates of adverse reactions during both initial and subsequent exposures. Thus, switching to a high-osmolar agent does not lessen the likelihood for a reaction. Switching to a low-osmolar agent (eg, iohexol, ioversol, iopamidol) and pretreating with corticosteroids and a combination of H1- and H2-antihistamines lowers the risk of reaction to less than 1% in patients who have experienced a previous reaction to a high-osmolar RCM. Desensitization is a procedure used to allow safe administration of medications that cause IgE-mediated reactions. It does not have a role in the treatment of RCM anaphylactoid reactions and generally is not recommended for other non-IgE-mediated reactions, such as idiosyncratic reactions, serum sickness, or erythema multiforme. Appropriate hydration is important prior to any procedure using contrast to limit the risk of nephrotoxicity, but there is no specific recommendation regarding administration of intravenous fluid to decrease the risk for an anaphylactoid reaction. Further, a 1-L intravenous fluid bolus in a 3-year-old child who is hemodynamically stable is excessive. References: Boguniewicz M. Adverse reactions to drugs. In: Behrman RE, Kliegman RM, Jenson HB, eds. Nelson Textbook of Pediatrics. 17th ed. Philadelphia, Pa: WB Saunders Co 2004:783-786 Hagan JB. Anaphylactoid and adverse reactions to radiocontrast agents. Immunol Allergy Clin North Am. 2004;24:507-519. Abstract available at: http://www.ncbi.nlm.nih.gov/entrez/query.fcgi?orig_db=PubMed&db=PubMed&cmd=Search&ter m=Immunol+Allergy+Clin+North+Am[Jour]+AND+507[page]+AND+2004[pdat]

Copyright 2007 by the American Academy of Pediatrics

page 63

2007 PREP SA on CD-ROM Question: 32

A 16-year-old girl is brought to the emergency department by ambulance after her mother found a suicide note on her bed. The girl claims that she took "20 or 30" ibuprofen tablets 4 hours ago and nothing else. Although she is tearful, her physical examination findings are normal. Of the following, the MOST appropriate next step in the management of this patient is to

A. administer sodium bicarbonate B. administer syrup of ipecac C. obtain a blood level of acetaminophen D. obtain a blood level of ibuprofen E. perform gastric lavage

Copyright 2007 by the American Academy of Pediatrics

page 64

2007 PREP SA on CD-ROM Critique: 32 Preferred Response: C

The patient described in the vignette has attempted suicide by medication overdose. Although she has stated that she took only ibuprofen, coingestants should be assumed until proven otherwise. Dangerous coingestants that can be detected with laboratory tests include acetaminophen or aspirin and illicit drugs of abuse such as narcotics, cocaine, or ethanol. It would be prudent to measure serum acetaminophen and aspirin concentrations and obtain a urine toxicology screen. No currently available assays measure serum ibuprofen levels in the clinical setting. Because other toxic substances may have been ingested, administration of activated charcoal is appropriate for the girl described in the vignette. Ibuprofen overdose is a common presenting complaint in the emergency department, but significant toxicity is exceedingly rare. In most cases, supportive care leads to an uneventful recovery. In a recent report, two thirds of patients were asymptomatic, and 20% experienced mild or moderate symptoms. There have been fewer than 10 case reports of fatal overdose with ibuprofen, and in each case, there were complicating factors such as coingestion of another toxic substance or other disease processes. Other rare reported complications associated with ibuprofen overdose include duodenal perforation, renal toxicity, metabolic acidosis, somnolence, coma, and subfulminant hepatitis requiring liver transplantation. The American Academy of Pediatrics does not recommend syrup of ipecac for the management of medication overdose because it is inefficient compared with gastric lavage and because the prolonged vomiting it can induce may interfere with therapy. Gastric lavage is only marginally effective and is suggested only in the setting of ingestion of a highly toxic substance in the previous 1 to 2 hours. Gastric lavage more than 2 hours after ingestion is unlikely to provide benefit. The administration of sodium bicarbonate for metabolic acidosis rarely is indicated, except for cases of severe acidosis in which the pH is lower than 7.0. There is no role for bicarbonate administration in enhancing elimination of ibuprofen. References: Clarke SF, Arepalli N, Armstrong C, Dargan PI. Duodenal perforation after ibuprofen overdose. J Toxicol Clin Toxicol. 2004;427:983-985 Easley RB, Altemeier WA 3rd. Central nervous system manifestations of an ibuprofen overdose reversed by naloxone. Pediatr Emerg Care. 2000;16:39-41 Available at: http://www.ncbi.nlm.nih.gov/entrez/query.fcgi?orig_db=PubMed&db=PubMed&cmd=Search&ter m=%22Pediatric+emergency+care%22[Jour]+AND+39[page]+AND+2000[pdat] Laurent S, Rahier J, Geubel AP, Lerut J, Horsmans Y. Subfulminant hepatitis requiring liver transplantation following ibuprofen overdose. Liver. 2000;20:93-94 Volans G, Monaghan J, Colbridge M. Ibuprofen overdose. Int J Clin Pract Suppl. 2003;135:5460. Available at: http://www.ncbi.nlm.nih.gov/entrez/query.fcgi?orig_db=PubMed&db=PubMed&cmd=Search&ter m=%22International+journal+of+clinical+practice.+Supplement%22[Jour]+AND+54[page]+AND+2 003[pdat]

Copyright 2007 by the American Academy of Pediatrics

page 65

2007 PREP SA on CD-ROM Question: 33

A nurse asks you to examine a 26-hour-old infant who recently developed a rash. On physical examination, you note erythematous macules (Item Q33A) over the trunk, face, and proximal extremities. Most of the macules have tiny central pustules. The infant is breastfeeding well, and the remainder of the physical examination findings are normal. No lesions were present at birth. Of the following, analysis of the pustular contents is MOST likely to reveal

A. eosinophils B. gram-positive cocci C. multinucleated giant cells D. polymorphonuclear leukocytes E. pseudohyphae and budding yeast

Copyright 2007 by the American Academy of Pediatrics

page 66

2007 PREP SA on CD-ROM Critique: 33 Preferred Response: A

Erythema toxicum neonatorum presents as blotchy, 2- to 3-cm erythematous macules (Item C33A), each of which has a single 1- to 4-mm central papule, vesicle, or pustule. Lesions are not present at birth but appear in the first 24 to 48 hours thereafter. The condition occurs in about 50% of healthy term infants; preterm infants are affected less often. Individual lesions resolve in 4 to 5 days, with new lesions appearing for up to 10 days. A Wright-stained preparation of the pustule contents reveals numerous eosinophils (Item C33B). If obtained, a complete blood count may demonstrate eosinophilia. The condition is benign and self-limited, and no treatment is necessary. A number of neonatal skin conditions are characterized by vesicles and pustules. They may be distinguished from erythema toxicum clinically and by microscopic examination of lesional contents. The pustules caused by staphylococcal folliculitis (Item C33C) contain gram-positive cocci, the vesicles of herpes simplex virus infection (Item C33D) contain multinucleated giant cells (Item C33E), the pustules of transient neonatal pustular melanosis (Item C33F) contain polymorphonuclear leukocytes, and the pustules of congenital candidiasis (Item C33G) contain pseudohyphae and budding yeast (Item C33H). References: Eichenfield L, Larralde M. Neonatal skin and skin disorders: congenital erosive and vesicular dermatosis, vesiculopustular and bullous diseases of the newborn. In: Schachner LA, Hansen RC, eds. Pediatric Dermatology. 3rd ed. St. Louis, Mo: Mosby; 2003:232-239 Marchini G, Nelson A, Edner J, Lonne-Rahm S, Stavrus-Evers A, Hultenby K. Erythema toxicum neonatorum is an innate immune response to commensal microbes penetrated into the skin of the newborn infant. Pediatr Res. 2005;58:613-616. Available at: http://www.pedresearch.org/cgi/content/full/58/3/613 Weston WL, Lane AT, Morelli JG. Skin diseases in newborns: transient skin disease. In: Color Textbook of Pediatric Dermatology. 3rd ed. St. Louis, Mo: Mosby; 2002:302-303

Copyright 2007 by the American Academy of Pediatrics

page 67

2007 PREP SA on CD-ROM Question: 34

A 16-year-old Caucasian girl from the northeastern United States presents for a health supervision visit. She drinks three to four diet sodas a day and describes herself as "a couch potato." She has a history of asthma and had been hospitalized for administration of intravenous methylprednisolone twice in the past year. Physical examination findings are normal. She is at Sexual Maturity Rating 4 and has had normal menses for 3 years. A year ago, she fell while walking and fractured her ulna. You are considering therapy for suspected osteopenia. Of the following, the MOST appropriate therapy for this patient at this time is

A. intranasal calcitonin B. oral alendronate C. oral calcium and vitamin D supplement D. oral estrogen E. oral phosphorus supplement

Copyright 2007 by the American Academy of Pediatrics

page 68

2007 PREP SA on CD-ROM Critique: 34 Preferred Response: C

The adolescent described in the vignette has risk factors for osteopenia (low bone density). These include living in the northeastern United States, not being active, drinking diet soda (which contains phosphoric acid), having received intravenous steroids, and already having had one fracture. The initial evaluation of a teen in whom osteopenia is suspected should include determination of calcium, phosphorus, 25-hydroxyvitamin D, and parathyroid hormone concentrations as well as bone mineral density. Age-adjusted normal values for bone mineral density are now available. If osteopenia or hypovitaminosis D is demonstrated, the initial therapy involves a calcium supplement (600 to 1,200 mg/d) and vitamin D replacement (usually 800 units of vitamin D). Other therapies, such as alendronate, calcitonin, estrogen, and phosphorus, usually are not necessary in otherwise healthy adolescent girls. In a patient who has suspected osteopenia, bone density of the lumbar spine and other sites can be assessed using dual-energy x-ray absorptiometry (DEXA). The patient's bone density is compared with age- and sex-adjusted normal values. A Z score of less than 1.0 indicates the patient is below the 20th percentile for bone density, which most investigators define as "osteopenia." Although osteopenia correlates with hip fractures in the elderly, the clinical significance of osteopenia in an adolescent is still unclear and a topic of ongoing research. Therefore, routine bone density measurements are not recommended as a screening test in children. However, this test should be considered in children who receive long-term steroids, have a history of recurrent fractures, or have a chronic illness (eg, rheumatoid arthritis, Crohn disease). In addition to bone density, measures of serum calcium, phosphorus, 25hydroxyvitamin D, and parathyroid hormone are performed in the assessment of the child who has osteopenia. Many factors contribute to bone health in children, including intake of calcium and phosphorus, renal excretion of calcium, physical activity, and vitamin D status. According to one recent study from the northeastern United States, the prevalence of vitamin D deficiency in otherwise healthy adolescents is approximately 25%, and vitamin D levels are lower in the winter (when there is less exposure to sunlight). In this study, vitamin D deficiency also was associated with African-American race, decreased milk consumption, and increased soft drink consumption. Adolescents all should be educated regarding the need to take in the recommended dietary allowance of calcium and vitamin D. References: Gordon CM, DePeter KC, Feldman HA, Grace E, Emans SJ. Prevalence of vitamin D deficiency among healthy adolescents. Arch Pediatr Adolesc Med. 2004;158:531-537. Abstract available at: http://www.ncbi.nlm.nih.gov/entrez/query.fcgi?cmd=Retrieve&db=pubmed&dopt=Abstract&list_ui ds=15184215&query_hl=13&itool=pubmed_docsum Semeao E, Jawad AF, Stouffer NO, Zemel BS Piccoli DA, Stallings VA. Risk factors for low bone mineral density in children and young adults with Crohn's disease. J Pediatr. 1999;135:593-600. Abstract available at: http://www.ncbi.nlm.nih.gov/entrez/query.fcgi?cmd=Retrieve&db=pubmed&dopt=Abstract&list_ui ds=10547248&query_hl=15&itool=pubmed_docsum

Copyright 2007 by the American Academy of Pediatrics

page 69

2007 PREP SA on CD-ROM Question: 35

A term male infant is born to a woman who has known multiple drug abuse problems. Her urine drug screen was positive for barbiturates, benzodiazepines, and opioids upon admission to the labor and delivery unit. The infant is delivered vaginally without complications. Apgar scores are 8 and 9 at 1 and 5 minutes, respectively. His birthweight is 3,500 g. You are asked to evaluate the infant for early discharge at 23 hours of age. Findings on physical examination are normal, with the exception of jitteriness, and the infant is not breastfeeding well. Of the following, the BEST reason to keep this infant in the hospital is that

A. a negative urine drug screen for the infant is required B. breastfeeding failure requires a lactation consultation C. foster care placement must be arranged D. jitteriness is most likely due to hypoglycemia that requires treatment E. neonatal abstinence syndrome may not be fully evident for 5 days or more

Copyright 2007 by the American Academy of Pediatrics

page 70

2007 PREP SA on CD-ROM Critique: 35 Preferred Response: E

In the United States, 5% to 15% of all pregnancies may be complicated by perinatal drug exposure, and it is not uncommon for a pregnant woman to use more than one illicit substance. The most commonly used legal substances are tobacco and alcohol. Either, or both, may complicate the use and effects of other substances, such as opioids, cocaine, amphetamines, or marijuana. The newborn may experience withdrawal from drug exposure following delivery. Such withdrawal effects must be distinguished from drug effects. The direct effects of some drugs of abuse include sedation or somnolence, depression of respiratory drive, depressed neuromotor tone, bradycardia, thermoregulatory problems, and pinpoint pupils (methadone, morphine, heroin, codeine, oxycodone, alcohol, barbiturates, benzodiazepines) or increased sensory stimulation with agitation, tachycardia, hypertension, and dilated pupils (amphetamines, cocaine). Early discharge is not an option for any infant in whom perinatal drug exposure is a real concern, such as the infant described in the vignette, because the onset of symptoms of abstinence syndrome following delivery and postnatal cessation of drug exposure varies with the substance to which the fetus was exposed. It is also important to consider the timing of last known exposure (maternal use). No true withdrawal or abstinence syndrome for cocaine exists. Similarly, amphetamine effects dissipate after a few postnatal days, and no real abstinence syndrome exists for this class of substances. However, neither drug is considered safe or free of prenatal damage to the developing brain; both may have direct effects on neurotransmitters and affect brain circulation. Effects of cocaine and amphetamine on the maternal and uteroplacental circulation also may be related to the reduced fetal body and head growth described in some exposed fetuses. Alcohol and barbiturate withdrawal syndromes share many common physical findings, including hyperactivity, irritability, crying, hyperphagia but poorly coordinated sucking and oral feeding, altered sleep-wake periods, tremors, diaphoresis, and seizures. Alcohol withdrawal also may present with hypoglycemia and excessive glucocorticoid release. In addition to a controlled, thermoneutral environment, with minimal stimulation, phenobarbital provides the best pharmacologic management. Such treatment should be used in conjunction with an objective scoring system for physical findings to titrate the dose to effect. A schedule for tapering phenobarbital over 4 to 6 weeks is recommended. An alternative, though less well substantiated, management approach is the use of a dilute tincture of opium and diazepam. Opioid withdrawal is best recognized by an abstinence syndrome that is characterized by numerous behavioral and physiologic stigmata, including hyperirritability, tremors, jitteriness, hypertonia, gastrointestinal distress (loose stools, emesis, and feeding problems), and vague autonomic symptoms such as yawning, lip-smacking, persistent sucking, mottling, and fever. Seizures may occur. Methadone or oral morphine must be used to manage the physical findings, which may not become manifest for 5 days (longer if the mother was treated with methadone). Again, the use of a neonatal abstinence scoring system is essential in managing affected newborns. A urine drug screen for the infant described in the vignette may be obtained within 24 hours and would be most concerning if positive for any of the substances identified on his mothers urine drug screen. However, the more time that passes after birth and before collecting the newborns urine, the less likely that the results will be positive because most drugs and their metabolites are cleared by 96 hours after birth. At fewer than 24 hours of postnatal age, the infant may be demonstrating simple breastfeeding problems or the problems may reflect oralmotor coordination issues related to drug exposure. Jitteriness suggests neurologic irritability associated with maternal drug use; hypoglycemia is a less likely cause in an appropriately grown term infant. Although social issues such as placement of an infant in a safe environment must be addressed, foster care placement may take a long time and is not the best reason for delaying discharge. References: American Academy of Pediatrics Committee on Drugs. Neonatal drug withdrawal. Pediatrics.

Copyright 2007 by the American Academy of Pediatrics

page 71

2007 PREP SA on CD-ROM

1998;101:1079-1088. Available at: http://pediatrics.aappublications.org/cgi/content/full/101/6/1079 Godding V, Bonnier C, Fiasse L, et al. Does in utero exposure to heavy maternal smoking induce nicotine withdrawal symptoms in neonates? Pediatr Res. 2004;55:645-651. Available at: http://www.pedresearch.org/cgi/content/full/55/4/645 Johnson K, Gerada C, Greenough A. Treatment of neonatal abstinence syndrome. Arch Dis Child Fetal Neonatal Ed. 2003;8:F2-F5. Available at: http://fn.bmjjournals.com/cgi/content/full/88/1/F2 Law KL, Stroud LR, LaGasse LL, Niaura R, Liu J, Lester BM. Smoking during pregnancy and newborn neurobehavior. Pediatrics. 2003;111:1318-1323 Available at: http://pediatrics.aappublications.org/cgi/content/full/111/6/1318 Philipp BL, Merewood A, O'Brien S. Methadone and breastfeeding: new horizons [commentary]. Pediatrics. 2003;111:1429-1430. Available at: http://pediatrics.aappublications.org/cgi/content/full/111/6/1429 Wang M. Perinatal drug abuse and neonatal drug withdrawal. eMedicine Specialties: Pediatrics: Neonatology. Available at: http://www.emedicine.com/ped/topic2631.htm

Copyright 2007 by the American Academy of Pediatrics

page 72

2007 PREP SA on CD-ROM Question: 36

A 15-month-old boy presents to the emergency department following the acute onset of nausea, vomiting, and abdominal pain. He appears agitated. His heart rate is 160 beats/min, but other vital signs are normal. His mother, who has asthma, reports finding her theophylline bottle lying empty on the floor in her bedroom. His capillary blood gas measurement is normal, and his blood glucose concentration is 190 mg/dL (10.5 mmol/L). Of the following, the MOST appropriate management of this patient is

A. administration of activated charcoal B. administration of beta-adrenergic receptor antagonists C. administration of ipecac D. gastric lavage E. whole-bowel irrigation

Copyright 2007 by the American Academy of Pediatrics

page 73

2007 PREP SA on CD-ROM Critique: 36 Preferred Response: A

Although theophylline is prescribed infrequently in children, pediatricians should be aware of the clinical manifestations and management of theophylline toxicity because some adults still use the drug for the treatment of asthma and chronic obstructive pulmonary diseases. Patients who have acute theophylline toxicity often complain of nausea, vomiting, and abdominal pain and may exhibit symptoms of restlessness, tremors, and seizures. Tachycardia and tachypnea are common. Laboratory evaluation may reveal hypokalemia, hyperglycemia, and high anion gap metabolic acidosis. Electrocardiographic findings include sinus and supraventricular tachycardias and ventricular dysrhythmias. Chronic theophylline toxicity may occur in older patients who are taking medications that are metabolized by the cytochrome P450 system and have hepatic impairment or renal impairment. Symptoms in these patients may be more subtle and milder. As with other poisonings, the acute management of theophylline toxicity begins with the ABCs (airway, breathing, and circulation), followed by intravenous catheter placement and fluid administration. The mainstay of treatment for both acute and chronic toxicity is the administration of multiple doses of activated charcoal, which should be continued until symptoms resolve and theophylline levels are within therapeutic range. Electrolytes should be monitored closely, and antiemetic therapy may be used to aid with charcoal administration if vomiting is persistent. If seizures occur, benzodiazepines and barbiturates are effective, and cardiac dysrhythmias should be treated aggressively as appropriate for the type of dysrhythmia. Beta-adrenergic receptor antagonists have been shown to be effective in treating some dysrhythmias associated with theophylline toxicity, but hypotension may be exacerbated by these agents, and their use should be reserved for refractory cases. Administration of ipecac no longer is recommended for any toxic ingestion. Gastric lavage may be advantageous if given within 1 hour of large ingestions, but it may delay the administration of charcoal, which is the definitive treatment. Whole-bowel irrigation should be reserved for cases in which the theophylline values continue to rise despite adequate charcoal administration. References: American Academy of Pediatrics Committee on Injury, Violence, and Poison Prevention. Policy statement: poison treatment in the home. Pediatrics. 2003;112:1182-1185 Available at: http://pediatrics.aappublications.org/cgi/content/full/112/5/1182 Stork CM, Howland MA. Theophylline. In: Ford MD, Delaney KA, Ling LJ, Erickson T, eds. Clinical Toxicology. Philadelphia, Pa: WB Saunders Co; 2001:412-416 Woolf AD. Poisoning by unknown agents. Pediatr Rev. 1999;20:166-170 Available at: http://pedsinreview.aappublications.org/cgi/content/full/20/5/166

Copyright 2007 by the American Academy of Pediatrics

page 74

2007 PREP SA on CD-ROM Question: 37

A 2-year-old girl, who has been healthy, is scheduled for a health supervision visit today. When you last saw her at her 15-month-old visit, there were no problems. You have learned that the family has just moved from the suburbs, purchased a home in an area of town where houses date back to the early 20th century, and are in the midst of a major remodeling project. You decide to discuss with the parents the risks associated with remodeling an older home. Of the following, it is MOST important to discuss with these parents the risks associated with their child's exposure to:

A. arsenic B. asbestos C. lead D. mercury E. radon

Copyright 2007 by the American Academy of Pediatrics

page 75

2007 PREP SA on CD-ROM Critique: 37 Preferred Response: C

Children are exposed to numerous potentially toxic environmental substances at home, at school, and at play. Buildings slated for remodeling or repair in which children live, learn, or play should be inspected for possible toxic exposures prior to such construction to minimize exposures. For the child described in the vignette, the single most important potential exposure is environmental lead. Lead poisoning adversely affects central nervous system development and cognition in the growing child. Although potential exposure to lead may be difficult to document, serum blood concentrations are inexpensive and easy to obtain, and they correlate well with the risk for developmental disability. Furthermore, lead poisoning is potentially treatable. Assessment of the risk of children and families may be determined, in part, by questionnaire. Widespread blood lead screening of children living in high-risk areas is mandated by the Medicaid program at ages 1 and 2 years. A blood lead level of greater than 10 mcg/dL (<0.48 mcmol/L) deserves further evaluation of the childs environment. Lead abatement in the environment has decreased widespread exposure dramatically, but children who are exposed to remodeling of older homes, impoverished children, and African-American children continue to be at increased risk. Exposure to asbestos in an older school building or home is possible, but asbestos is not a high-risk exposure unless the building materials containing the substance are deteriorating, are within reach of children who might touch or play with them, or are being removed as part of building renovation. Asbestosis is a pulmonary disease that has a high rate of associated malignancy if inhalation exposure is prolonged. There is no good screening tool to measure asbestos exposure or risk. Both children and adults should be removed from schools or homes undergoing abatement of asbestos-containing tiles, insulation, and other materials. Because the risk for lung cancer associated with asbestos exposure is exponentially higher in persons who smoke or are exposed to cigarette smoke and because smoking is a far greater risk factor for cancer, discussion of smoke avoidance and smoking cessation should accompany discussions about environmental risk factors. Radon gas results from radioactive decay of radium, which is found in rocks and soil and enters homes through cracks and pores in concrete and granite floors and walls, especially basements. There is no clinical test for radon exposure in humans, but test kits are available to assess the radon content of homes. Long-term exposure has been associated with increased cancer risk, as with exposure to other sources of radiation. Although ingestion of fish containing high levels of mercury has been associated with chronic neurologic impairments, it is not a significant problem associated with home renovations. Recent withdrawal of elemental mercury-containing thermometers from the market and removal of mercury sphygmomanometers represent an effort to decrease the presence of environmental mercury. Arsenic has been removed as a component of most modern pesticides, but it may persist in ground water, especially around areas previously used for smelting and mining. It is also a potential hazard in childrens playground equipment as a component of pressure-treated lumber (copper chromium arsenate). Arsenic may contaminate the skin by simply rubbing the hand over a treated surface. Burning of such pressure-treated lumber has been reported to cause acute arsenic toxicity, with gastrointestinal and pulmonary disturbances. Chronic exposure may cause fatigue, malaise, and paresthesias. References: American Academy of Pediatrics Committee on Environmental Health. Arsenic. In: Etzel RA, ed. Pediatric Environmental Health. 2nd ed. Elk Grove Village, Ill: American Academy of Pediatrics; 2003:87-98 American Academy of Pediatrics Committee on Environmental Health. Asbestos. In: Etzel RA, ed. Pediatric Environmental Health. 2nd ed. Elk Grove Village, Ill: American Academy of Pediatrics; 2003:99-112

Copyright 2007 by the American Academy of Pediatrics

page 76

2007 PREP SA on CD-ROM

American Academy of Pediatrics Committee on Environmental Health. Ionizing radiation (including radon). In: Etzel RA, ed. Pediatric Environmental Health. 2nd ed. Elk Grove Village, Ill: American Academy of Pediatrics; 2003:215-234 American Academy of Pediatrics Committee on Environmental Health. Lead. In: Etzel RA, ed. Pediatric Environmental Health. 2nd ed. Elk Grove Village, Ill: American Academy of Pediatrics; 2003:249-266 American Academy of Pediatrics Committee on Environmental Health. Mercury. In: Etzel RA, ed. Pediatric Environmental Health. 2nd ed. Elk Grove Village, Ill: American Academy of Pediatrics; 2003:267-282 American Academy of Pediatrics Committee on Environmental Health. Pesticides. In: Etzel RA, ed. Pediatric Environmental Health. 2nd ed. Elk Grove Village, Ill: American Academy of Pediatrics; 2003:323-360 American Academy of Pediatrics Committee on Environmental Health. Water pollutants. In: Etzel RA, ed. Pediatric Environmental Health. 2nd ed. Elk Grove Village, Ill: American Academy of Pediatrics; 2003:393-416 Markowitz M. Lead poisoning. Pediatr Rev. 2000;21:327-335. Available at: http://pedsinreview.aappublications.org/cgi/content/full/21/10/327

Copyright 2007 by the American Academy of Pediatrics

page 77

2007 PREP SA on CD-ROM Question: 38

A 12-year-old girl presents with her third headache in the last 2 months. She describes the pain as pounding, sharp, and severe. The pain is bifrontal and has been present for 1 hour. Past history is notable for motion sickness at age 4 years. Physical examination results are normal, but the girl draws a picture of dots when asked if she sees anything prior to pain. Of the following, the MOST appropriate treatment for this child is

A. intramuscular meperidine B. intranasal butorphanol C. oral ibuprofen D. oral zolmitriptan E. subcutaneous sumatriptan

Copyright 2007 by the American Academy of Pediatrics

page 78

2007 PREP SA on CD-ROM Critique: 38 Preferred Response: C

The 12-year-old girl described in the vignette has acute recurrent headaches that are consistent with migraine with aura, ie, classic migraine. Her headaches are short and pounding and have long pain-free intervals. The dots she sees during episodes are likely scintillating scotomas. The girl does not display the features of ominous secondary (ie, pathologic) headaches, namely, papilledema, strabismus, weakness, or ataxia. Migraines tend to be familial and can be triggered by fatigue, glare, caffeine, fasting, lack of sleep, menstruation, stress, or other factors. Mood changes can trigger migraine, although depression tends to produce chronic daily headaches with less migrainous characteristics. Management of migraines requires both pharmacologic and nonpharmacologic techniques. After the child and family are reassured about the benign nature of the headache, triggers should be sought and removed. Daily exercise, routine meals, good hydration, and sufficient sleep are essential. Other nonpharmacologic measures should be considered, such as relaxation techniques or biofeedback. These nonpharmacologic measures also are effective in headaches associated with stress, particularly when chronic. Abortive medications also can be useful to stop pain, although placebo alone can be associated with cessation of pain in perhaps one third of children. A recent practice parameter, endorsed by the American Academy of Pediatrics, reviewed drug treatments for acute headache and found convincing data to support the use of acetaminophen (15 mg/kg orally), ibuprofen (10 mg/kg orally), or nasal sumatriptan (20 mg if body mass is 30 kg or more; 5 mg if less than 30 kg). Daily prophylactic medications, such as amitriptyline, topiramate, cyproheptadine, or valproic acid, are indicated only for the child who has several disabling migraines every month. Evidence to support the use of subcutaneous sumatriptan or oral zolmitriptan in children is limited at this time. Intramuscular meperidine and intranasal butorphanol are both narcotics that have addictive potential and are not indicated for treatment of childhood migraine. References: Fisher PG. Help for headaches: a strategy for your busy practice. Contemp Pediatr. 2005;22:3441 Forsyth R, Farrell K. Headache in childhood. Pediatr Rev. 1999;20:39-45 Available at: http://pedsinreview.aappublications.org/cgi/content/full/20/2/39 Haslam RHA. Headaches. In: Behrman RE, Kliegman RM, Jenson HB, eds. Nelson Textbook of Pediatrics. 17th ed. Philadelphia, Pa: WB Saunders; 2004:2012-2015 Lewis D, Ashwal S, Hershey A, et al; American Academy of Neurology Quality Standards Subcommittee; Practice Committee of the Child Neurology Society. Practice parameter: pharmacological treatment of migraine headache in children: report of the American Academy of Neurology Quality Standards Committee and the Practice Committee of the Child Neurology Society. Neurology. 2004;63:2215-2224. Available at: http://www.neurology.org/cgi/content/full/63/12/2215

Copyright 2007 by the American Academy of Pediatrics

page 79

2007 PREP SA on CD-ROM Question: 39

You are seeing a 6-year-old boy for a health supervision visit. He was born with trisomy 21 and an atrioventricular septal defect for which he underwent complete surgical repair at 2 months of age with an excellent result. His mother reports that he has not been sleeping well, that he snores loudly, and that she believes that he fatigues easily with play. On physical examination, he appears in no distress, his respiratory rate is 24 breaths/min, his heart rate is 100 beats/min, and his blood pressure is 110/70 mm Hg. His precordial impulse is prominent, and there is a loud second heart sound and a 2/6 holosystolic murmur at the cardiac apex with radiation to the left axilla. There is mild hepatomegaly and jugular venous congestion. Of the following, the MOST likely cause of this patient's findings is

A. dehiscence of the ventricular septal patch B. dilated cardiomyopathy C. residual atrial septal defect D. right heart failure E. supraventricular tachycardia

Copyright 2007 by the American Academy of Pediatrics

page 80

2007 PREP SA on CD-ROM Critique: 39 Preferred Response: D

Obstructive sleep apnea syndrome (OSAS) refers to a condition in which the upper airway is obstructed completely or partially during sleep and is associated with hypoxemia, hypercapnia, and disrupted normal ventilation and sleep patterns. OSAS has a prevalence of 1% to 2% in children. Several risk factors are associated with OSAS, including adenotonsillar hypertrophy, craniofacial abnormalities, nasal obstruction, neuromuscular disorders, and obesity. The pathophysiology results from mechanical obstruction of the upper airway due to tonsillar hypertrophy or decreased muscular tone of the airway. The clinical manifestations are snoring during sleep, mouth breathing, restless sleep patterns, mood swings, inattentiveness, learning difficulties, and failure to thrive. The diagnosis is confirmed by polysomnography, which measures the precise number of apneic and hypopneic episodes. OSAS can have widespread complicating effects on the respiratory and metabolic systems as well as a significant impact on the psychosocial well-being of the patient. Obese patients who have OSAS have a higher incidence of systemic hypertension and stroke. Right heart failure may develop in patients who have severe OSAS, likely due to hypoxemia and hypercapnia. These factors subsequently have an effect as pulmonary vasoconstrictors. With increasing pulmonary vasoconstriction, increased right ventricular and pulmonary artery pressure is generated to deliver blood into the lungs. Over time, the increased afterload on the right heart may lead to hypertrophy, dilation, and failure of the right ventricle. The trisomy 21 coupled with the sleep patterns and behaviors that are consistent with OSAS described for the patient in the vignette have led to right heart failure. Dehiscence of a ventricular septal patch placed 6 years earlier is extremely unlikely, and if present, likely would lead to pronounced signs and symptoms consistent with pulmonary overcirculation. Residual atrial septal defect would not be expected to cause significant symptoms in a 6-year-old child and would not explain the findings during sleep. Findings of a dilated cardiomyopathy would be referable to the systemic circulation, and tachycardia probably would develop as a compensatory mechanism for the poor cardiac output. Supraventricular tachycardia is not present; the patients heart rate is normal for his age. References: Bandla P, Brooks LJ, Trimarchi T, Helfaer M. Obstructive sleep apnea syndrome in children. Anesthesiol Clin North Am. 2005;23:535-549. Abstract available at: http://www.ncbi.nlm.nih.gov/entrez/query.fcgi?orig_db=PubMed&db=PubMed&cmd=Search&ter m=%22Anesthesiology+clinics+of+North+America%22[Jour]+AND+2005[pdat]+AND+Bandla+P[ author] Iber C. Sleep-related breathing disorders. Neurol Clin. 2005;23:1045-1057 Abstract available at: http://www.ncbi.nlm.nih.gov/entrez/query.fcgi?cmd=Retrieve&db=pubmed&dopt=Abstract&list_ui ds=16243615&query_hl=18&itool=pubmed_docsum Rosen CL. Obstructive sleep apnea syndrome in children: controversies in diagnosis and treatment. Pediatr Clin North Am. 2004;51:153-167. Abstract available at: http://www.ncbi.nlm.nih.gov/entrez/query.fcgi?orig_db=PubMed&db=PubMed&cmd=Search&ter m=%22Pediatric+clinics+of+North+America%22[Jour]+AND+2004[pdat]+AND+Rosen+C[author]

Copyright 2007 by the American Academy of Pediatrics

page 81

2007 PREP SA on CD-ROM Question: 40

You are notified by the newborn screening program that a 2-week-old infant in your practice has an elevated phenylalanine level, which is confirmed by repeat testing. The mother reports that the baby is healthy and breastfeeding well. Of the following, the MOST important first step in management of this infant is to

A. admit the baby to the hospital for further evaluation B. consult with a metabolic geneticist or nutritionist C. instruct the mother that she no longer should breastfeed D. place the baby immediately on phenylalanine-free formula E. send urine for organic acid analysis

Copyright 2007 by the American Academy of Pediatrics

page 82

2007 PREP SA on CD-ROM Critique: 40 Preferred Response: B

Phenylketonuria (PKU) is an autosomal recessive inborn error of metabolism that has an incidence of approximately 1 in 10,000 to 1 in 25,000 individuals. The gene that causes PKU is well described and is mapped to chromosome 12 on the long arm. More than 240 mutations causing PKU have been defined, and carrier detection and prenatal diagnosis are available using molecular genetic testing. Newborn screening programs for PKU initially were established based on the discovery that the early diagnosis and treatment of the disease virtually eliminate morbidity and mortality. An elevated blood phenylalanine (PHE) concentration detected by newborn screening performed after the initiation of feeding, as described for the infant in the vignette, suggests the need for a diagnostic evaluation. However, it is important to realize that most infants in whom elevated PHE concentrations are identified through screening programs do not have PKU; they simply have delayed maturation of the metabolizing enzymes. Individuals who have classic PKU and continue to feed normally experience a rapid rise in serum PHE concentration to levels greater than 30 mg/dL (1,815 mcmol/L)(normal, 1 mg/dL [60.5 mcmol/L]). Some individuals have a variant termed hyperphenylalaninemia, which also results from defective phenylalanine hydroxylase. Yet another group of individuals has biopterin deficiency; biopterin serves as a cofactor (in the form of tetrahydrobiopterin) for the formation of tyrosine from PHE. Following the identification of an infant who has a positive newborn screening result for PKU, the most efficient next step is either to repeat the screening test or to send blood for amino acid quantitation. Because such test results are seen infrequently in general pediatric practices, it is important to consult with a metabolic geneticist. The results of the amino acid analysis allow the geneticist to distinguish among true PKU, in which the PHE level is very elevated and the tyrosine level is negligible; hyperphenylalaninemia, in which the level is elevated in the intermediate range (10 to 20 mg/dL [605 to 1,200 mcmol/L]) and there may be some tyrosine; and newborn immaturity, in which the PHE level is elevated, but there is sufficient tyrosine. It is not necessary to admit the baby described in the vignette to the hospital at this point because he is stable and is not expected to decompensate. If the mother discontinues breastfeeding and the baby does not have PKU, the opportunity to breastfeed may be lost. Phenylalanine-free formula is expensive, and 2 to 3 days of therapy while awaiting test results has no clear advantage. Finally, urine organic acid analysis is not helpful. Once the diagnosis of PKU is confirmed, it is critical to confer with a metabolic geneticist/nutritionist and, if at all possible, to have the patient seen at a metabolic clinic. Therapy involves restriction of dietary PHE to the amount tolerated by the patient and regular measurement of plasma PHE concentrations. Medical foods that contain amino acid mixtures or protein hydrolysates that lack phenylalanine are substituted for approximately 75% of the protein required by the infant. It also is important to monitor plasma tyrosine concentrations because tyrosine becomes an essential amino acid in affected children. When carefully managed and followed, individuals who have PKU have a very bright prognosis, both for health and for cognitive function. It is important to note that in the past, affected children were taken off their special diets at about the age of 6 years. However, it now appears best to keep affected individuals on their diets indefinitely because of measurable decreases in the intelligence quotient and school performance of individuals who came off the diet. Additionally, because of the adverse effects of hyperphenylalaninemia on the fetus, women who have PKU should be on special diets and monitored closely during pregnancy. References: American Academy of Pediatrics Committee on Genetics. Maternal phenylketonuria. Pediatrics. 2001;107:427-428. Available at: http://pediatrics.aappublications.org/cgi/content/full/107/2/427 Mitchell JJ, Scriver CR. Phenylalanine hydroxylase deficiency. GeneReviews. 2005. Available at: http://www.geneclinics.org/servlet/access?db=geneclinics&site=gt&id=8888891&key=NOg1Avz eZtDD1&gry=&fcn=y&fw=pvPE&filename=/profiles/pku/index.html

Copyright 2007 by the American Academy of Pediatrics

page 83

2007 PREP SA on CD-ROM

Rezvani I. Phenylalanine. In: Behrman RE, Kliegman RM, Jenson HB, eds. Nelson Textbook of Pediatrics. 17th ed. Philadelphia Pa: Saunders; 2004:398-402 Wilcox WR, Cederbaum SD. Amino acid metabolism. In: Rimoin DL, Connor JM, Pyeritz RE, Korf BR, eds. Emery and Rimoins Principles and Practice of Medical Genetics. 4th ed. New York, NY: Churchill Livingstone; 2002:2405-2440

Copyright 2007 by the American Academy of Pediatrics

page 84

2007 PREP SA on CD-ROM Question: 41

A 16-year-old girl complains of dysuria of 1 week's duration. She was seen 4 days ago at an urgent care facility and was prescribed trimethoprim-sulfamethoxazole because of abnormal urinalysis findings. Although she has taken the antibiotic as directed, she has had no relief from the dysuria. She has no nausea, vomiting, flank pain, vaginal discharge, or fever. Of the following, the MOST appropriate course of action is to

A. add doxycycline to the treatment B. change the antibiotic to ciprofloxacin C. inspect her genitalia D. order ultrasonography of the kidneys E. treat with fluconazole

Copyright 2007 by the American Academy of Pediatrics

page 85

2007 PREP SA on CD-ROM Critique: 41 Preferred Response: C

Dysuria is a nonspecific symptom that may be caused by a variety of conditions, including urinary tract infections (UTIs), sexually transmitted infections (STIs), vulvitis, vaginitis, urethritis, dermatitis, urolithiasis, hypercalciuria, pinworms, chemical irritation, dysfunctional voiding patterns, and trauma. Factors that can guide appropriate laboratory tests and treatment include patient age (eg, pinworms in a young child) and sex (UTI is very uncommon in adolescent males), family history (eg, renal disease or stones), patient sexual history (abuse, symptoms in partner[s]), and findings on the physical examination. History alone does not reliably differentiate the dysuria caused by a UTI from that associated with an STI. Diagnostic clues often are found during inspection of the genitalia. Adolescent females who have dysuria, pyuria, and normal findings on genitalia inspection and have not been sexually active may be treated presumptively for a UTI with or without a urine culture. However, some clinicians prefer to obtain a pretreatment urine culture to confirm a UTI as a cause of the dysuria. A positive test for leukocyte esterase reflecting pyuria or the presence of white blood cells on microscopy is suggestive, but not specific, for a UTI. Other causes of pyuria include vaginitis, cervicitis, and urethritis. Pyuria and a positive urine test for nitrite indicate bacteriuria, but a concurrent STI may be present. Many adolescent females who have UTIs (eg, caused by Staphylococcus saprophyticus) do not have a positive urine nitrite test result. An adolescent male should not be treated empirically for a UTI; the presence of pyuria in a male most likely is due to an STI. However, males who have pyuria and a positive urine nitrite test should have a urine culture and further evaluation for associated conditions (eg, anatomic abnormalities, other foci of infection). An STI must be considered in patients who present with dysuria and have been sexually active, sexually abused, or have suspicious findings during genital inspection. Nucleic acid amplification (eg, polymerase chain reaction) testing on a urine specimen is useful to diagnose infection with Chlamydia trachomatis and Neisseria gonorrhoeae. A bimanual examination also may be necessary to evaluate adolescent females who have lower abdominal pain or abnormal vaginal discharge that might indicate pelvic inflammatory disease. A urine culture is required for the evaluation of dysuria accompanied by symptoms suggestive of acute pyelonephritis (eg, flank pain, fever, vomiting). Inspection of the genitalia of the girl in the vignette may reveal clues to the cause of the dysuria. The examiner may find dermatitis, evidence of trauma, vaginal discharge, or lesions associated with sexually transmitted diseases (eg, herpetic vesicles, Bartholin gland cyst). Although the patient may have persistent dysuria due to a chlamydial infection or UTI unresponsive to the antibiotic, the empiric addition or change of antibiotic (eg, to doxycycline, ciprofloxacin) is not appropriate without a more detailed history, other laboratory tests, and inspection of the genitalia. Ultrasonography of the kidneys is not warranted because the patient has no symptoms suggestive of upper tract disease. Dysuria frequently is associated with a monilial vaginitis, but empiric treatment with fluconazole without additional assessment is not appropriate. References: Bonny AE, Brouhard BH. Urinary tract infections among adolescents. Adolesc Med Clin. 2005;16:149-161 DAngelo LJ, Neinstein LS. Genitourinary tract infections. In: Neinstein LS, ed. Adolescent Health Care A Practical Guide. 4th ed. Philadelphia, Pa: Lippincott, Williams & Wilkins; 2002:551-564 Peters CA. Pediatric urology in the pediatric and adolescent girl. In: Emans SJH, Laufer MR, Goldstein DP, eds. Pediatric and Adolescent Gynecology. 5th ed. Philadelphia, Pa: Lippincott Williams & Wilkins; 2005:494-524 Woods ER, Emans SJ. Vulvovaginal complaints in the adolescent. In: Emans SJH, Laufer MR,

Copyright 2007 by the American Academy of Pediatrics

page 86

2007 PREP SA on CD-ROM

Goldstein DP, eds. Pediatric and Adolescent Gynecology. 5th ed. Philadelphia, Pa: Lippincott, Williams & Wilkins; 2005:525-559

Copyright 2007 by the American Academy of Pediatrics

page 87

2007 PREP SA on CD-ROM Question: 42

A 16-year-old girl is being seen in your clinic because of headaches for the past month that sometimes awaken her at night. At her last visit 2 years ago, she was well, 5 ft 2 in tall, and weighed 105 lb, with Sexual Maturity Rating (SMR) 4 breast development and SMR 4 pubic hair, but she had not reached menarche. On examination at this visit, she is 5 ft 2 in tall, weighs 110 lb, and still has not begun menstruating. On evaluation, you note bitemporal visual field deficits, perhaps worse on the left. In addition to magnetic resonance imaging and an ophthalmologic evaluation, the laboratory test that is MOST likely to be most diagnostically revealing is

A. adrenocorticotropic hormone B. insulin-like growth factor-1 C. luteinizing hormone D. prolactin E. thyroid-stimulating hormone

Copyright 2007 by the American Academy of Pediatrics

page 88

2007 PREP SA on CD-ROM Critique: 42 Preferred Response: D

The 16-year-old girl described in the vignette has had arrest of pubertal development with failure of menarche for at least 2 years. The bitemporal visual field deficits imply the presence of a large suprasellar mass. The two most common causes for such a mass lesion are pituitary macroadenomas and craniopharyngiomas, and the most common macroadenoma is a prolactinoma. Elevations of prolactin values could be due to increased prolactin secretion from a prolactinoma or to compression of the pituitary stalk by another pituitary tumor, a craniopharyngioma, or other space-occupying lesion. Because stalk compression interferes with the dopaminergic inhibitory influences on pituitary release of prolactin, prolactin concentrations of up to 200 mcg/L may be due solely to stalk compression. Prolactin concentrations in that range interfere with normal menstrual cycling and suppress normal puberty. A large tumor also might interfere with pubertal and other pituitary hormones because of its mass. Accordingly, measuring prolactin will be most helpful in determining whether the girl has a prolactinoma. Prolactinomas usually respond to treatment with medication such as cabergoline; surgery rarely is required. Elevated adrenocorticotropic hormone (ACTH) values might be seen in Cushing disease, but the girl in the vignette exhibits no signs or symptoms of Cushing disease, such as weight gain, skin striae, hypertension, cushingoid facies, and muscle weakness. Further, the pituitary tumors in affected individuals rarely are macroadenomas because the symptoms lead to an earlier diagnosis. A low concentration of insulin-like growth factor-1 might suggest growth hormone deficiency, and a high concentration suggests acromegaly. No signs or symptoms of acromegaly, such as rapid growth, coarsening of facial features, sweating, or thickening of skin, are described for the girl. Measurement of thyroid-stimulating hormone (TSH) might identify a TSH-producing tumor or severe hypothyroidism that is primary because of thyroid gland failure. A TSH-producing tumor, which is very rare, produces hyperthyroidism, and no symptoms of hyperthyroidism are described for the girl in the vignette. Sometimes severe primary hypothyroidism can lead to the development of an enlarged pituitary composed of thyrotropes (TSH-secreting cells), but no symptoms or signs of severe hypothyroidism (eg, pallor, lethargy, dry skin, coarse facies, slow heart rate, myxedema) are described. Measurement of luteinizing hormone would be useful if the clinician suspected ovarian failure. However, follicle-stimulating hormone is a better measure of ovarian failure because it rises higher than luteinizing hormone if there is ovarian dysfunction. Rare pituitary tumors produce luteinizing hormone, but these tumors do not produce neuroendocrine symptoms in females unless they also produce folliclestimulating hormone. References: Cannavo S, Venturino M, Curto L, et al. Clinical presentation and outcome of pituitary adenomas in teenagers. Clin Endocrinol (0xf). 2003;58:519-527 Abstract available at: http://www.ncbi.nlm.nih.gov/entrez/query.fcgi?cmd=Retrieve&db=pubmed&dopt=Abstract&list_ui ds=12641637&query_hl=43&itool=pubmed_docsum Karavitaki N, Brufani C, Warner TJ, et al. Craniopharyngiomas in children and adults: systematic analysis of 121 cases with long-term follow-up. Clin Endocrinol (Oxf). 2005;62:397-409 Abstract available at: http://www.ncbi.nlm.nih.gov/entrez/query.fcgi?orig_db=PubMed&db=PubMed&cmd=Search&ter m=%22Clinical+endocrinology%22[Jour]+AND+397[page]+AND+2005[pdat] Molitch ME. Disorders of prolactin secretion. Endocrinol Metab Clin North Am. 2001;30:585-610

Copyright 2007 by the American Academy of Pediatrics

page 89

2007 PREP SA on CD-ROM Question: 43

An 8-year-old boy is inattentive at home and school, has difficulty completing his homework, and is failing reading. Physical examination findings are normal, he has friends at school, and the family has been living in their newly built home for the past 3 years. You begin to discuss a diagnosis of attention-deficit/hyperactivity disorder, and his mother asks you what tests you will perform to try to determine the cause of the problem. Of the following, your BEST response is that you will order

A. a lead level B. an electroencephalogram C. computed tomography scan of the brain D. no tests at this time E. thyroid studies

Copyright 2007 by the American Academy of Pediatrics

page 90

2007 PREP SA on CD-ROM Critique: 43 Preferred Response: D

Attention-deficit/hyperactivity disorder (ADHD) is a common, yet heterogeneous disorder that has a complex etiology; genetic, environmental, and biologic factors all play roles. For example, ADHD is associated with both genetic syndromes such as fragile X and intrauterine toxic exposures such as fetal alcohol syndrome. It also has been shown in family and twin studies that ADHD is more common in close family members of those diagnosed with the disorder. Research continues into the neurobiologic basis of ADHD. Evidence is increasing that alterations in the frontal lobe and frontal subcortical connections play a significant role in the disorder. Affected children also have difficulties with executive functioning, such as organization, impulse control, and inattention, that are common in other disorders involving impaired frontal lobe function. Although not necessary for all children, a full psychoeducational evaluation likely is necessary before ADHD can be diagnosed in the boy in the vignette. The boy also may have a learning disability or an environmental cause of his academic difficulty, such as a chaotic home environment or significant social stresses. ADHD is diagnosed clinically using American Psychiatric Association Diagnostic and Statistical Manual of Mental Disorders-4th ed (DSM-IV) criteria; routine medical testing is not necessary. Lead intoxication is associated with cognitive dysfunction, including inattention and hyperactivity, but it is not a significant cause of ADHD. A lead level should be obtained if any concerns for lead exposure are established. Thyroid dysfunction also may cause cognitive changes, but other information from the history or physical examination should prompt the request of these studies. Electroencephalography or computed tomography scan of the brain is unlikely to provide clinically useful information in a child who has no history of seizure, significant brain injury, or neurologic findings on examination. References: American Academy of Pediatrics Committee on Quality Improvement, Subcommittee on Attention-Deficit/Hyperactivity Disorder. Clinical practice guideline: diagnosis and evaluation of the child with attention-deficit/hyperactivity disorder. Pediatrics. 2000;105:1158-1170. Available at: http://pediatrics.aappublications.org/cgi/content/full/105/5/1158 Reiff MI, Tippins S, LeTourneau AA. ADHD: A Complete and Authoritative Guide. Elk Grove Village, Ill: American Academy of Pediatrics; 2004 Sims MD. Attention-deficit/hyperactivity disorder. In: Behrman RE, Kliegman RM, Jenson HB, eds. Nelson Textbook of Pediatrics. 17th ed. Philadelphia, Pa: WB Saunders Co; 2004: 107-110

Copyright 2007 by the American Academy of Pediatrics

page 91

2007 PREP SA on CD-ROM Question: 44

Upon entering your examination room, you find a father who is visibly upset. He has brought his 1-month-old and 2-year-old daughters to see you today because his wife was recently diagnosed with tuberculosis. As you question him further, he tells you that his wife's physician told him that his wife's chest radiograph was abnormal and she was "smear-positive." He hands you results from the health department stating that he and his daughters all had negative skin tests and chest radiographs. His wife is at the health department today to start medicines against tuberculosis, and he wants to know if there is anything that should be done for the children. Of the following, the MOST appropriate treatment is to

A. administer the bacille Calmette-Gurin vaccine B. begin isoniazid therapy C. provide reassurance D. remove the children from the home for 3 months E. repeat the skin test in 1 month

Copyright 2007 by the American Academy of Pediatrics

page 92

2007 PREP SA on CD-ROM Critique: 44 Preferred Response: B

Cases of suspected or proven tuberculosis should be reported to the local health department immediately. If a woman has pulmonary tuberculosis, it is unlikely that she will infect any fetus, but she may infect the newborn after delivery. In such cases, all contacts should have physical examinations, tuberculin skin tests, and chest radiographs. Besides these tests, a lumbar puncture is indicated for any newborn who possibly may have congenital tuberculosis. The mother and infant also should be screened for the human immunodeficiency virus. If results of the evaluation for active disease in the contacts of a person who has tuberculosis are negative, as reported for the children in the vignette, all children younger than 4 years of age should begin isoniazid therapy, with follow-up skin testing in 12 weeks. If there is no sign of tuberculous disease upon follow-up skin testing and the source case has demonstrated a good clinical response and has been adherent to the treatment program, the isoniazid can be stopped. If the tuberculin skin test is positive at follow-up, the child should be reassessed with a chest radiograph for tuberculous disease. If active disease is not present, he or she should complete a 9-month (total) course of isoniazid therapy. The children in the vignette should not receive bacille Calmette-Gurin vaccine or be removed from their home unless the mother has risk factors for or is infected with a multidrugresistant isolate. Reassurance and repeating the skin testing in 1 month is not appropriate in this case because the mother is smear-positive and both children are at high risk for disseminated disease because of their ages. It is important to remember that most children who have active tuberculosis do not produce sputum and, therefore, are not contagious and do not require special precautions. Exceptions to this rule are children who have cavitary pulmonary lesions, positive sputum smears, laryngeal disease, extensive pulmonary disease, or suspected congenital tuberculosis. If patients have positive sputum smears, isolation should be continued until effective therapy has been initiated, sputum smears are negative, and the cough has abated. Children who have no cough or negative sputum smears can be hospitalized safely in an open ward. References: American Academy of Pediatrics. Tuberculosis. In: Pickering LK, ed. Red Book: 2006 Report of the Committee on Infectious Diseases. 27th ed. Elk Grove Village, Ill: American Academy of Pediatrics; 2006:678-698 Pediatric Tuberculosis Collaborative Group. Targeted tuberculin skin testing and treatment of latent tuberculosis infection in children and adolescents. Pediatrics. 2004;114:1175-1201. Available at: http://pediatrics.aappublications.org/cgi/content/full/114/4/S2/1175

Copyright 2007 by the American Academy of Pediatrics

page 93

2007 PREP SA on CD-ROM Question: 45

You are examining a 5-year-old girl who always has had significant daytime wetting and a history of recurrent urinary tract infections. Findings on physical examination are normal except for the presence of a sacral dimple above the gluteal cleft. Her urinalysis reveals a specific gravity of 1.005, pH of 5.5, no blood, no protein, and no white or red blood cells. Magnetic resonance imaging of the spine reveals spinal dysraphism. Of the following, the MOST important next step to determine the cause of this child's primary enuresis is to obtain

A. abdominal computed tomography scan B. abdominal radiography C. abdominal ultrasonography D. renal biopsy E. urine culture

Copyright 2007 by the American Academy of Pediatrics

page 94

2007 PREP SA on CD-ROM Critique: 45 Preferred Response: C

Primary enuresis is defined as the absence of achievement of dryness during the daytime, nighttime, or both. In contrast to secondary enuresis, in which the child experiences a period of dryness after completing toilet training, primary enuresis generally has an anatomic cause. The daytime wetting, history of recurrent urinary tract infections (UTIs), sacral dimple above the gluteal cleft, specific gravity of 1.005 and no other abnormalities on urinalysis, and spinal dysraphism on magnetic resonance imaging reported for the girl in the vignette are consistent with probable spina bifida occulta. The spina bifida may be associated with bladder dysfunction because innervation of the bladder originates from the sacral spine, although some investigators believe that the association may be only an incidental finding. Children who have bladder dysfunction of any cause exhibit dribbling, frequency, and recurrent UTIs due to bladder dyssynergy. This may cause some degree of urinary tract obstruction because bladder emptying is interrupted. Such obstruction can be detected easily by abdominal ultrasonography, which is far more accurate for documenting urinary tract anomalies than abdominal radiography. Abdominal computed tomography scan may be more precise than ultrasonography for some conditions in the urinary tract, but it is far more expensive, and the precision for detecting urinary tract obstruction is generally similar. A renal biopsy is indicated for children who have findings suggestive of primary glomerular disease (eg, proteinuria), but would not be helpful to assess urinary tract anomalies. Finally, although children who have bladder malformation may develop UTIs, the child in the vignette has no symptoms or findings on urinalysis suggestive of a UTI. Moreover, a positive urine culture result will not aid in ascertaining the cause of her primary enuresis. References: Kumar P, Aneja S, Kumar R, Taluja V. Spina bifida occulta in functional enuresis. Indian J Pediatr. 2005;72:223-225. Available at: http://www.ijppediatricsindia.org/article.asp?issn=00195456;year=2005;volume=72;issue=3;spage=223;epage=225;aulast=Kumar Samuel M, Boddy SA. Is spina bifida occulta associated with lower urinary tract dysfunction in children? J Urol. 2004;171:2664-2666. Abstract available at: http://www.ncbi.nlm.nih.gov/entrez/query.fcgi?cmd=Retrieve&db=pubmed&dopt=Abstract&list_ui ds=15118447&query_hl=22&itool=pubmed_docsum

Copyright 2007 by the American Academy of Pediatrics

page 95

2007 PREP SA on CD-ROM Question: 46

A 2,700-g male infant born at 36 weeks' gestation is being treated for suspected neonatal sepsis following the development of respiratory distress shortly after birth. His mother had a fever to 102F (38.9C) during labor and delivery but reports that she had no illnesses during pregnancy. Of the following, the MOST appropriate antibiotic regimen for this infant is

A. ampicillin and an aminoglycoside B. clindamycin and a third-generation cephalosporin C. meropenem and an aminoglycoside D. piperacillin and an aminoglycoside E. vancomycin and a third-generation cephalosporin

Copyright 2007 by the American Academy of Pediatrics

page 96

2007 PREP SA on CD-ROM Critique: 46 Preferred Response: A

Neonatal sepsis is a clinical syndrome characterized by systemic signs of infection accompanied by bacteremia occurring during the first month after birth. Neonatal sepsis occurs in 1 to 5 per 1,000 live births, with mortality rates ranging from 5% to 15%. It is categorized as early-onset (infections occurring before 7 days of age) and late-onset (infections occurring between 7 and 30 days of age). The most common bacteria causing early-onset bacterial infections are group B streptococci (Streptococcus agalactiae), Escherichia coli, enterococci, other enteric bacilli, alpha-hemolytic streptococci, Listeria monocytogenes, Streptococcus pneumoniae, Haemophilus influenzae, and other maternal genital flora. Empiric antibiotic therapy for neonates suspected of having early-onset sepsis, such as the infant described in the vignette, is a combination regimen that provides coverage for the most common organisms, penetrates into the central nervous system, and has minimal toxicity. The recommended empiric therapy, which has remained unchanged for 3 decades, is ampicillin plus gentamicin. Although some experts advocate substitution of cefotaxime or another third-generation cephalosporin for gentamicin, these agents have no better efficacy than ampicillin and gentamicin, and the routine use of these agents for empiric therapy of sepsis in neonatal intensive care units is associated with outbreaks of sepsis due to multiple-drug-resistant enteric organisms, especially Enterobacter and Serratia sp. Clindamycin and a third-generation cephalosporin would not provide coverage for Enterococcus and Listeria sp. Meropenem plus an aminoglycoside, piperacillin plus an aminoglycoside, and vancomycin plus a third-generation cephalosporin provide much broader coverage than needed and increase the risk for the development of antibiotic-resistant strains. References: Baltimore RS. Neonatal sepsis: epidemiology and management. Paediatr Drugs. 2003;5:723-740 Abstract available at: http://www.ncbi.nlm.nih.gov/entrez/query.fcgi?orig_db=PubMed&db=PubMed&cmd=Search&ter m=%22Paediatric+drugs%22[Jour]+AND+723[page]+AND+2003[pdat] de Man P, Verhoeven BAN, Verbrugh HA, et al. An antibiotic policy to prevent emergence of resistant bacilli. Lancet. 2000;355:973-978 Abstract available at: http://www.ncbi.nlm.nih.gov/entrez/query.fcgi?orig_db=PubMed&db=PubMed&cmd=Search&ter m=%22Lancet%22[Jour]+AND+973[page]+AND+2000[pdat] Edwards MS, Baker CJ. Bacterial infections in the neonate. In: Long SS, Pickering LK, Prober CG, eds. Principles and Practice of Pediatric Infectious Diseases. 2nd ed. New York, NY: Churchill Livingstone; 2003:536-542 Fanos V, DallAgnola A. Antibiotics in neonatal infections: a review. Drugs. 1999;58:405-427 Abstract available at: http://www.ncbi.nlm.nih.gov/entrez/query.fcgi?orig_db=PubMed&db=PubMed&cmd=Search&ter m=%22Drugs%22[Jour]+AND+405[page]+AND+1999[pdat] Gerdes JS. Diagnosis and management of bacterial infections in the neonate. Pediatr Clin North Am 2004;51:939-959. Abstract available at: http://www.ncbi.nlm.nih.gov/entrez/query.fcgi?orig_db=PubMed&db=PubMed&cmd=Search&ter m=%22Pediatric+clinics+of+North+America%22[Jour]+AND+939[page]+AND+2004[pdat]

Copyright 2007 by the American Academy of Pediatrics

page 97

2007 PREP SA on CD-ROM Question: 47

A 10-year-old boy presents to the clinic for evaluation of an insect sting reaction. He was playing in his yard yesterday when he was stung on the right forearm by a wasp. His parents immediately placed ice over the area, but they are worried because when the child awoke today, the redness that initially was localized to the sting site had extended to involve his entire right forearm. The boy denies fever, chills, nausea, vomiting, or difficulty breathing. His vital signs on physical examination are normal. His right forearm is diffusely erythematous and warm to palpation but not tender. Of the following, this child's insect sting reaction is BEST characterized as

A. cellulitis B. large local reaction C. normal reaction D. systemic anaphylaxis E. toxic reaction

Copyright 2007 by the American Academy of Pediatrics

page 98

2007 PREP SA on CD-ROM Critique: 47 Preferred Response: B

The reaction described for the child in the vignette is consistent with a large local reaction. When evaluating a child who experiences an insect sting reaction, the history should categorize the reaction type to help determine a management plan. Insect sting reactions are classified as local or normal reactions, large local reactions, or systemic reactions. Local or normal reactions are most common and generally are of little medical consequence. Typical symptoms include erythema, warmth, pain, and swelling confined to the sting site. Oral antihistamines, cold compresses, and sometimes topical corticosteroids can be used to alleviate symptoms. Large local reactions generally begin with mild erythema that increases in size over the next 12 to 24 hours, often exceeding 20 cm and occasionally involving the entire extremity. Such reactions are immunoglobulin (Ig)E-mediated and may increase the risk for anaphylaxis with subsequent stings to 5% to 10%, although long-term studies and current guidelines do not recommend antibiotics, skin testing, or immunotherapy for individuals who only experience large local reactions. Cellulitis can occur after any insect sting, but is rare compared with large local reactions. Fever, chills, tenderness, and progression of swelling beyond 48 hours are common with cellulitis but typically absent with large local reactions. Systemic reactions include anaphylaxis and toxic reactions. Toxic reactions occur from the direct effect of venom following multiple (usually >50) stings and have been reported to cause renal failure, rhabdomyolysis, diffuse intravascular coagulation, and even seizures. Rapid onset of cutaneous (generalized hives, angioedema, flushing), respiratory (throat tightness, chest tightness, dyspnea, wheezing), or circulatory signs (dizziness, hypotension, syncope) after only a few stings usually represents anaphylaxis, an IgE-mediated reaction. Any individual older than 16 years of age who experiences anaphylaxis after an insect sting should be counseled about the increased risk (30% to 60%) for similar reactions with subsequent insect stings, prescribed self-injectable epinephrine, and considered for allergy consultation. Individuals younger than 16 years of age who experience cutaneous symptoms only (urticaria or angioedema) do not appear to have the same risk with future stings and may be managed conservatively. References: Freeman TM. Clinical practice. Hypersensitivity to hymenoptera stings. N Engl J Med. 2004;351:1978-1984 Sicherer SH, Leung DYM. Insect allergy. In: Behrman RE, Kliegman RM, Jenson HB, eds. Nelson Textbook of Pediatrics. 17th ed. Philadelphia, Pa: WB Saunders Co 2004:786-788

Copyright 2007 by the American Academy of Pediatrics

page 99

2007 PREP SA on CD-ROM Question: 48

A 10-year-old child is brought to your office for evaluation of a 1-day history of fever, vomiting, diarrhea, and abdominal pain. His mother states that he has vomited five times, and the emesis has been clear. He has had four episodes of nonbloody diarrhea. He describes his abdominal pain as crampy but cannot localize it to any specific part of his abdomen. He denies any symptoms of dysuria. On physical examination, the child is in no acute distress, his temperature is 99.2F (37.3C), heart rate is 102 beats/min, respiratory rate is 26 breaths/min, and blood pressure is 105/70 mm Hg. Results of examination of the head, neck, chest, and heart are normal. His abdomen is soft, and there is no guarding. There is no rebound tenderness. He complains of mild discomfort on deep palpation of his entire abdomen. He has hyperactive bowel sounds on auscultation, and he has no flank tenderness. Of the following, the MOST appropriate next step in the management of this patient is to

A. administer intravenous fluids B. obtain blood for a complete blood count C. obtain serum for electrolyte analysis D. order frontal supine and upright abdomen radiographs E. send the patient home with instructions for supportive care

Copyright 2007 by the American Academy of Pediatrics

page 100

2007 PREP SA on CD-ROM Critique: 48 Preferred Response: E

The patient described in the vignette has signs and symptoms consistent with viral gastroenteritis. This is a clinical diagnosis that requires no further evaluation in the absence of toxicity, rebound tenderness, distention, or evidence for dehydration, as in this child. Neither a complete blood count nor serum electrolyte determinations are likely to alter management. The boy should be sent home with instructions for supportive care. Most children who have vomiting due to gastroenteritis can maintain sufficient levels of hydration with glucose- and electrolyte-containing solutions. Intravenous hydration is not required unless fluid loss exceeds 10% of body weight or the patient is experiencing more moderate 5% to 10% dehydration in conjunction with persistent vomiting and an inability or unwillingness to take oral fluids. Abdominal radiographs provide limited information for the evaluation of a child who has vomiting and diarrhea, and even in suspected appendicitis, rarely are helpful. Further evaluation for abdominal pain is indicated if information gained from the history and physical examination suggest potential surgical emergencies such as appendicitis (tenderness localized to the right lower quadrant with or without rebound), small bowel obstruction (absent bowel sounds, abdominal distention), perforated viscus (rigid abdomen, distention, fever, toxicity), malrotation (diffuse tenderness, abdominal distention), intussusception (severe crampy abdominal pain alternating regularly with periods of pain relief), or peritonitis (fever, toxicity, diffuse abdominal tenderness). References: Bellemare S, Hartling L, Wiebe N, et al. Oral rehydration versus intravenous therapy for treating dehydration due to gastroenteritis in children: a meta-analysis of randomised controlled trials. BMC Med. 2004;2:11 Available at: http://www.biomedcentral.com/1741-7015/2/11 Fleisher GR. Infectious disease emergencies. In: Fleisher GR, Ludwig S, Henretig FM, eds. Textbook of Pediatric Emergency Medicine. 5th ed. Philadelphia, Pa: Lippincott Williams & Wilkins; 2006:783-852 Kwok MY, Kim MK, Gorelick MH. Evidence-based approach to the diagnosis of appendicitis in children. Pediatr Emerg Care. 2004;20:690-698 Rothrock SG, Grenn SM, Harding M, et al. Plain abdominal radiography in the detection of acute medical and surgical disease in children: a retrospective analysis. Pediatr Emerg Care. 1991;7:281-285 Abstract available at: http://www.ncbi.nlm.nih.gov/entrez/query.fcgi?orig_db=PubMed&db=PubMed&cmd=Search&ter m=Pediatr+Emerg+Care[Jour]+AND+281[page]+AND+1991[pdat] Rothrock SG, Pagane J. Acute appendicitis in children: emergency department diagnosis and management. Ann Emerg Med. 2000;36:39-51 Abstract available at: http://www.ncbi.nlm.nih.gov/entrez/query.fcgi?orig_db=PubMed&db=PubMed&cmd=Search&ter m=Ann+Emerg+Med[Jour]+AND+39[page]+AND+2000[pdat]

Copyright 2007 by the American Academy of Pediatrics

page 101

2007 PREP SA on CD-ROM Question: 49

A 1-month-old infant presents with frecklelike macules (Item Q49A) over his face and extremities. The hospital record reveals that he had multiple papules and pustules distributed over his entire body, including palms and soles, at birth. The infant appears to be very healthy and thriving. Of the following, analysis of the pustular contents in the newborn period MOST likely would have revealed

A. eosinophils B. gram-positive cocci C. multinucleated giant cells D. polymorphonuclear leukocytes E. pseudohyphae and budding yeast

Copyright 2007 by the American Academy of Pediatrics

page 102

2007 PREP SA on CD-ROM Critique: 49 Preferred Response: D

Transient neonatal pustular melanosis (TNPM) is a disorder of unknown cause that begins in utero. At birth, affected infants may exhibit pustules or small hyperpigmented macules (Item C49A) surrounded by a rim of scale, the remnant of the pustule roof. Pustules resolve within several days, but the hyperpigmented macules may persist for 1 to 3 months, as described for the infant in the vignette. TNPM is more common among African-American infants and may be observed on any skin surface, including the palms and soles. A Wright-stained preparation of the pustular contents reveals a predominance of neutrophils. No treatment is necessary for this self-limited condition. A number of neonatal skin conditions are characterized by pustules or vesicles. They may be distinguished from transient neonatal pustular melanosis clinically and by microscopic examination of lesional contents. The vesicles of erythema toxicum (Item C49B) contain eosinophils (Item C49C), the pustules caused by staphylococcal folliculitis (Item C49D) contain gram-positive cocci, the vesicles of herpes simplex virus infection (Item C49E) contain multinucleated giant cells (Item C49F), and the pustules of congenital candidiasis (Item C49G) contain pseudohyphae and budding yeast (Item C49H). References: Eichenfield L, Larralde M. Neonatal skin and skin disorders: congenital erosive and vesicular dermatosis, vesiculopustular and bullous diseases of the newborn. In: Schachner LA, Hansen RC, eds. Pediatric Dermatology. 3rd ed. St. Louis, Mo: Mosby; 2003:232-239 Weston WL, Lane AT, Morelli JG. Skin diseases in newborns: transient skin disease. In: Color Textbook of Pediatric Dermatology. 3rd ed. St. Louis, Mo: Mosby; 2002:299-307

Copyright 2007 by the American Academy of Pediatrics

page 103

2007 PREP SA on CD-ROM Question: 50

A 17-year-old boy who has a 5-year history of Crohn disease comes in with a flare of his illness, characterized by fever, diarrhea, and a 15-lb weight loss. He admits he has "forgotten to take his medicines lately." Physical examination demonstrates a very thin patient who has a perianal fistula. Abdominal computed tomography scan demonstrates thickening of the ileum (Item Q50A) and ascending colon. You are trying to decide whether to administer enteral nutrition (through a nasogastric tube) or begin parenteral nutrition. Of the following findings associated with Crohn disease, the BEST indication for instituting parenteral nutrition is

A. abdominal radiograph demonstrating air fluid levels B. active ileitis demonstrated on colonoscopy C. hypophosphatemia D. institution of 6-mercaptopurine therapy E. perianal abscess and fistula

Copyright 2007 by the American Academy of Pediatrics

page 104

2007 PREP SA on CD-ROM Critique: 50 Preferred Response: A

Crohn disease can cause many complications that require intensive nutritional therapy, including intestinal inflammation, fistulizing disease, and small bowel strictures. In most patients, such problems can be managed with enteral nutrition. Typically, a polymeric or enteral formula is delivered either by nasogastric tube, gastrostomy, or orally. The formula not only provides nutrition, but it may have a beneficial anti-inflammatory effect on the disease itself. Patients who have hypophosphatemia, ileitis, or perianal fistula and those receiving 6-mercaptopurine therapy all could benefit from enteral feedings. Small bowel obstruction, as indicated by air fluid levels on radiography, is a contraindication to enteral feedings. The principle for nourishing a chronically ill child is "if the gut works, use it." If tolerated, enteral nutrition is preferable to parenteral nutrition for many reasons. Enteral nutrition maintains the integrity of the gastrointestinal mucosa, stimulates the recovery of digestive enzymes such as lactase, and stimulates hepatobiliary and pancreatic secretions. Parenteral nutrition, although lifesaving in some circumstances, is associated with an increased risk of bacterial sepsis (from line infections), electrolyte abnormalities, hepatitis, and cholestasis. Currently available commercial formulas allow tube feeding in a variety of circumstances. Hydrolysates and amino acid-based formulas are available for children who have protein allergy, and low-fat formulas can be used for children who have fat malabsorption. Transpyloric tube placement allows enteral feeding while minimizing the risk of aspiration for children at risk for pulmonary aspiration due to reflux. The primary use of parenteral nutrition is in children who have intestinal disorders such as short bowel syndrome, Crohn disease, anatomic obstruction, ileus, pseudo-obstruction, or enteric fistulas. References: Courtney E, Grunko A, McCarthy T. Enteral nutrition. In: Hendricks KM, Duggan C, eds. Manual of Pediatric Nutrition. 4th ed. Hamilton, Ontario, Canada: BC Decker; 2005:252-316 Hyams JS. Inflammatory bowel disease. Pediatr Rev. 2005;26:314-320 Available at: http://pedsinreview.aappublications.org/cgi/content/full/26/9/314

Copyright 2007 by the American Academy of Pediatrics

page 105

2007 PREP SA on CD-ROM Question: 51

You are asked to see a term infant in the newborn nursery at 8 hours of age and consider her transfer to the neonatal intensive care unit. The problems and findings reported to you include: poor feeding, emesis, temperature instability (core temperature of 96F [35.5C]), hypoglycemia (whole blood glucose concentration of 25 mg/dL [1.4 mmol/L]), and polycythemia (hematocrit of 70% [0.70]). Of the following, the physical finding that is MOST likely to accompany these problems is

A. a tuft of hair over the sacral region B. birthweight of 1,800 g C. caf au lait macule on the left leg D. iris coloboma of the right eye E. isolated cleft of the hard palate

Copyright 2007 by the American Academy of Pediatrics

page 106

2007 PREP SA on CD-ROM Critique: 51 Preferred Response: B

Infants who are small for gestational age (<2,500 g) at birth may be affected by maternal disease states (eg, advanced maternal diabetes mellitus, chronic hypertension, cardiovascular disease); pregnancy-related problems, including poor nutritional status with poor weight gain or pregnancyinduced hypertension (PIH); or fetal conditions such as congenital infection, chromosome abnormality, or birth defects. The infants aberrant growth may be discerned prenatally (intrauterine growth restriction [IUGR]) or only noted at the time of birth. These infants have both increased morbidity and mortality compared with appropriately grown term infants (Item C51A). The fetus that experiences IUGR often has a poor tolerance for labor. Consequently, perinatal asphyxia is more common, and successful transition and adaptation to extrauterine life is fraught with complications. Such complications include temperature instability (hypothermia) related to a large head-to-body ratio, decreased subcutaneous fat, and small energy reserves, all of which require a thermoneutral environment and an avoidance of fasting. Hypoglycemia may be present and would be treated best with an intravenous dextrose infusion. Feeding problems that may preclude enteral feeding for a time or require the use of a nasogastric or orogastric feeding tube are not uncommon in the small-for-gestational age infant. Polycythemia may be present and reflects fetal adaptation to chronic hypoxia by increasing the erythrocyte mass to enhance oxygen-carrying capacity. Polycythemia also may be associated with impaired feeding, hypoglycemia, and jitteriness. When the hematocrit is greater than 70% (0.70) and there are clinical signs of altered neurologic function, a partial exchange transfusion with normal saline should be performed to reduce the hematocrit to 55% (0.55). The findings reported for the infant in the vignette most likely are associated with the infant being small for gestational age (<2,500 g birthweight). A tuft of hair over the sacral region may indicate occult spina bifida, but this finding would not be associated with growth impairment. Similarly, caf au lait macules (often seen in neurofibromatosis), an iris coloboma (Item C51B) (seen in the CHARGE syndrome), and isolated clefting of the hard palate are not commonly associated with growth restriction. References: Hay WW Jr, Catz CS, Grave GD, Yaffe SJ. Workshop summary: fetal growth: its regulation and disorders. Pediatrics. 1997;99:585-591. Available at: http://pediatrics.aappublications.org/cgi/content/full/99/4/585 Kliegman RM. Intrauterine growth restriction. In: Martin RJ, Fanaroff AA, Walsh MC, eds. Fanaroff and Martins Neonatal-Perinatal Medicine: Diseases of the Fetus and Infant. 8th ed. Philadelphia, Pa: Mosby-Elsevier; 2006:271-306 Linderkamp O. Blood viscosity of the neonate. NeoReviews. 2004;5:e406-e416. Available at: http://neoreviews.aappublications.org/cgi/content/full/5/10/e406 Thureen PJ, Anderson MS, Hay WW Jr. The small-for-gestational age infant NeoReviews. 2001;2:e139-e149. Available at: http://neoreviews.aappublications.org/cgi/content/full/2/6/e139

Copyright 2007 by the American Academy of Pediatrics

page 107

2007 PREP SA on CD-ROM Question: 52

A 12-month-old boy comes to the emergency department with a 3-day history of intractable vomiting and watery diarrhea. His mother reports decreased urine output for the past 24 hours. His heart rate is 180 beats/min, and his blood pressure is 85/40 mm Hg. He is lethargic but responds to stimulation. His mucous membranes are very dry, his skin turgor is decreased, and his capillary refill is 3 seconds. The remainder of his physical examination findings are unremarkable. Of the following, the laboratory data that are MOST consistent with this patient's clinical presentation are

A. Serum Sodium: High; Serum Osmolality: High; Urine Sodium: Low; Urine Osmolality: Low B. Serum Sodium: Low; Serum Osmolality: Low; Urine Sodium: Low; Urine Osmolality: High C. Serum Sodium: Low; Serum Osmolality: Low; Urine Sodium: High; Urine Osmolality: High D. Serum Sodium: Low; Serum Osmolality: Normal; Urine Sodium: High; Urine Osmolality: High E. Serum Sodium: Normal; Serum Osmolality: Normal; Urine Sodium: Low; Urine Osmolality: Low

Copyright 2007 by the American Academy of Pediatrics

page 108

2007 PREP SA on CD-ROM Critique: 52 Preferred Response: B

Diarrhea due to acute gastroenteritis usually causes a proportional water and sodium loss, resulting in isonatremic dehydration. Hyponatremic dehydration also may be seen in acute gastroenteritis, especially if the only fluids tolerated are low-sodium fluids, such as water or juice. The clinical signs of both hyponatremic and isonatremic dehydration are due to decreased extracellular fluid, primarily from the intravascular compartment. The signs include decreased skin turgor, delayed capillary refill, decreased tear production and urine output, sunken eyes and anterior fontanelle, and tachycardia. Hypotension is a late finding, indicating decompensated shock. Because the child described in the vignette displays these clinical signs, isonatremia or hyponatremia is likely. With hypernatremic dehydration, intravascular volume is relatively preserved, despite an overall body water loss, so the degree of dehydration may be underestimated. Hyponatremia resulting from gastroenteritis is associated with a low serum osmolality. In contrast, factitious hyponatremia may occur with high osmolar states such as hyperglycemia or hyperlipidemia. In a healthy child who has gastroenteritis, renal sodium-preserving and concentrating mechanisms remain intact, resulting in low urinary sodium concentrations and high urine osmolality, respectively. The findings of hyponatremia with normal-to-high urine sodium and urine osmolality values should alert the clinician to the possibility of the syndrome of inappropriate antidiuretic hormone secretion, which is not likely in the child in the vignette. References: Constantinescu AR. Parenteral fluid therapy for infants and children. In: Rakel RE, Bope ET, eds. Conns Current Therapy. 57th ed. Philadelphia, Pa: Elsevier Saunders; 2005:716-718 Moritz ML, Ayus JC. Disorders of water metabolism in children: hyponatremia and hypernatremia. Pediatr Rev. 2002;23:371-380 Available at: http://pedsinreview.aappublications.org/cgi/content/full/23/11/371 Roberts KB. Fluid and electrolytes: parenteral fluid therapy. Pediatr Rev. 2001;22:380-387 Available at: http://pedsinreview.aappublications.org/cgi/content/full/22/11/380

Copyright 2007 by the American Academy of Pediatrics

page 109

2007 PREP SA on CD-ROM Question: 53

A 2-year-old boy presents with a history of vomiting and diarrhea for several days. Physical examination reveals lethargy, poor oral intake, tachycardia, dry mucous membranes, and poor skin turgor. He refuses to take oral fluids, so you decide to begin intravenous fluid administration. Of the following, the BEST next step in fluid management is to

A. administer a bolus of hypotonic fluids at 20 mL/kg B. administer a bolus of isotonic fluid at 20 mL/kg C. administer a bolus of 3% saline at 10 mL/kg D. administer 5% dextrose and 0.25 normal saline with 20 mEq potassium chloride at
maintenance rate

E. await laboratory results before starting fluid therapy

Copyright 2007 by the American Academy of Pediatrics

page 110

2007 PREP SA on CD-ROM Critique: 53 Preferred Response: B

The child described in the vignette appears to be moderately to severely dehydrated, and his refusal to drink is somewhat ominous and implies depleted intravascular volume. Initial therapy of a severely dehydrated child includes intravenous administration of isotonic fluids (generally normal saline) at 20 mL/kg, with prompt re-evaluation after 30 minutes to 1 hour. Initial therapy for mild-to-moderate dehydration may be accomplished at least as effectively, if not more so, with oral rehydration, but it is labor-intensive, requiring one-to-one parent or nursing presence with the patient. After the initial intravenous fluid bolus, additional bolus fluid administration may be required. Careful follow-up and clinical assessment is needed after each bolus to determine if clinical improvement in hydration status has occurred and to prevent volume overload. Fluid therapy may need to be altered if there are signs of renal, cardiac, or respiratory impairment. Urine output, physical examination results, and improved alertness and willingness to drink guide further fluid therapy. If the child is more awake and willing to drink after an intravenous bolus, subsequent therapy may include replacement of a calculated fluid deficit of 5% to 10% via oral rehydration (50 to 100 mL/kg over 4 hours) or by continued administration of intravenous fluid. In the past, oral rehydration involved isotonic fluids, and intravenous rehydration and maintenance fluids often have been hypotonic, but some suggest that more hypotonic oral fluids are better tolerated and that isotonic saline be continued as maintenance intravenous fluid in some cases. Children who require intravenous rehydration should have serum electrolyte concentrations determined; a contraction alkalosis may be followed by a reperfusion acidosis that transiently worsens despite clinical improvement. Mildly dehydrated children who are managed with oral rehydration generally do not require measurement of serum electrolytes. It is never appropriate to delay fluid resuscitation while awaiting laboratory results because such results do not change initial management of volume depletion. However, interpretation of electrolyte values can guide further management once volume resuscitation is complete It is never appropriate to use hypertonic saline (3% saline) as initial fluid resuscitation in the dehydrated patient. Its use is limited to those who have severe hyponatremia unresponsive to isotonic saline boluses or who need concomitant fluid restriction. Ideally, hypertonic saline should be administered in an intensive care unit. Because dehydrated patients may have acute renal failure, they are at risk for lifethreatening hyperkalemia if potassium is included in their intravenous fluids. Therefore, it is important to assess renal function (urine output) before adding potassium to fluids in the severely dehydrated individual who may have suffered a renal insult. References: Centers for Disease Control and Prevention. Managing acute gastroenteritis among children: oral rehydration, maintenance, and nutritional therapy. MMWR Recomm Rep. 2003;52(RR-16):116. Available at: http://www.cdc.gov/mmwr/preview/mmwrhtml/rr5216a1.htm Duke T, Mathur A, Kukuruzovic R, McGuigan M. Hypotonic vs isotonic saline solutions for intravenous fluid management of acute infections. The Cochrane Database of Systematic Reviews. 2003;3:CD004169. Available at: http://www.mrw.interscience.wiley.com/cochrane/clsysrev/articles/CD004169/frame.html Finberg L. Dehydration in infancy and childhood. Pediatr Rev. 2002;23:277-282. Available at: http://pedsinreview.aappublications.org/cgi/content/full/23/8/277 Moritz ML, Ayus JC. Disorders of water metabolism: hyponatremia and hypernatremia. Pediatr Rev. 2002;23:371-380. Available at: http://pedsinreview.aappublications.org/cgi/content/full/23/11/371

Copyright 2007 by the American Academy of Pediatrics

page 111

2007 PREP SA on CD-ROM

Roberts KB. Fluids and electrolytes: parenteral fluid therapy. Pediatr Rev. 2001;22:380-387. Available at: http://pedsinreview.aappublications.org/cgi/content/full/22/11/380

Copyright 2007 by the American Academy of Pediatrics

page 112

2007 PREP SA on CD-ROM Question: 54

A 4-year-old girl is brought to the emergency department by her babysitter because the child has suddenly become clumsy, and her speech has become slurred over the last hour. On physical examination, the girl is afebrile and dysarthric. She has prominent vertical and horizontal nystagmus, along with truncal and appendicular ataxia. Deep tendon reflexes are normal, as are results of the remainder of the physical examination. Of the following, the MOST likely diagnosis is

A. brain tumor B. cerebellar hemorrhage C. Guillain-Barr syndrome D. meningoencephalitis E. toxic ingestion

Copyright 2007 by the American Academy of Pediatrics

page 113

2007 PREP SA on CD-ROM Critique: 54 Preferred Response: E

Ataxia is a broad term implying simply dyscoordination. Although acute ataxia often is associated with cerebellar pathology, this disturbance also can arise from defects in proprioception, vestibular function, vision, or even weakness. When evaluating the child who has acute ataxia, the clinician should consider categories of disease, including toxic ingestion (eg, alcohol, thallium used as a pesticide, and anticonvulsants such as phenytoin), neoplasm (eg, brainstem glioma, medulloblastoma), infection (labyrinthitis, postinfectious encephalomyelitis [also known as acute cerebellar ataxia], Guillain-Barr syndrome), vasculopathy (cerebellar hemorrhage), trauma, metabolic conditions (eg, Hartnup disease and abetalipoproteinemia), and conversion disorder. Specific clues from the history and physical examination may point to a diagnosis. For example, fever suggests meningoencephalitis, and acute cerebellar ataxia follows a viral infection, such as varicella or influenza. Deep tendon reflexes are depressed or absent in Guillain-Barr syndrome. Horizontal nystagmus is common with peripheral processes such as labyrinthitis or even cerebellar hemisphere lesions or sedative-hypnotic ingestion. Vertical nystagmus can be seen with central processes such as hemorrhage or Chiari malformation, but also is suggestive of phencyclidine, phenytoin, and lithium poisoning. The development of clumsiness, dysarthria, and nystagmus over only hours described for the child in the vignette suggests a very rapid process. The vertical plus horizontal nystagmus is typical of phenytoin ingestion. The lack of fever makes an infectious process unlikely. Some degree of obtundation would be expected with cerebellar hemorrhage. The normal reflexes are inconsistent with Guillain-Barr syndrome. Ataxia with a brain tumor develops over days or weeks. References: Dinolfo E. Evaluation of ataxia. Pediatr Rev. 2001;22:177-178. Available at: http://pedsinreview.aappublications.org/cgi/content/full/22/5/177 Johnston MV. Movement disorders: ataxias. In: Behrman RE, Kliegman RM, Jenson HB, eds. Nelson Textbook of Pediatrics. 17th ed. Philadelphia, Pa: WB Saunders Co; 2004:2019-2023 MacDonald GP. Ataxia of childhood. In: Berg BO, ed. Child Neurology: A Clinical Manual. 2nd ed. Philadelphia, Pa: JB Lippincott Co; 1994:287-305

Copyright 2007 by the American Academy of Pediatrics

page 114

2007 PREP SA on CD-ROM Question: 55

You are examining a 4-year-old child who has just undergone tonsillectomy and adenoidectomy for obstructive sleep apnea syndrome. He has just been extubated. While talking with his parents, you notice that his breathing is becoming more labored. On chest auscultation, you hear equal but poor air entry, no wheezes, and some crackles bilaterally. There are no murmurs, but a loud and palpable second heart sound is evident. You increase the oxygen level and reintubate the patient. After intubation, you obtain a chest radiograph. Of the following, the MOST likely finding on the chest radiograph is

A. bilateral perihilar infiltrates B. hemothorax C. normal pulmonary findings D. pulmonary edema E. tension pneumothorax

Copyright 2007 by the American Academy of Pediatrics

page 115

2007 PREP SA on CD-ROM Critique: 55 Preferred Response: D

Although most children who have obstructive sleep apnea syndrome (OSAS) do well with surgical treatment, respiratory compromise is a well-recognized adverse outcome of anesthesia and surgery in affected patients. Therefore, all children who have a history of OSAS should receive a comprehensive preanesthesia evaluation. Complications of tonsillectomy and adenoidectomy in children include hemorrhage, dehydration, and velopharyngeal insufficiency. Although each of these is reported to occur with a frequency of 3% or less, they are the most common complications reported. Respiratory complications can include laryngospasm, apnea, pulmonary hypertension, pulmonary hypertensive crisis, and pneumonia. Children who have OSAS are especially likely to have respiratory complications after surgery. A subgroup of patients who have OSAS and are undergoing surgery should receive particularly intensive perioperative management as well as inpatient monitoring following the procedure. These include patients younger than 3 years of age; those who have medical comorbidities, neuromuscular disease, bleeding diatheses, or syndromes such as trisomy 21; and those who have severe OSAS demonstrated on polysomnography. This latter group of patients may be at particular risk for development of the rare complication of postobstructive pulmonary edema. This entity should be suspected in any patient who has undergone surgical therapy for OSAS and demonstrates respiratory distress in the postoperative period, such as the boy described in the vignette. The diagnosis is confirmed with a chest radiograph that demonstrates pulmonary edema. Although bilateral infiltrates and hemo- or pneumothorax may occur following certain procedures, the clinical findings of the child in the vignette, along with the history of OSAS, are most consistent with the development of acute pulmonary edema. References: Bandla P, Brooks LJ, Trimarchi T, Helfaer M. Obstructive sleep apnea syndrome in children. Anesthesiol Clin North Am. 2005;23:535-549. Abstract available at: http://www.ncbi.nlm.nih.gov/entrez/query.fcgi?orig_db=PubMed&db=PubMed&cmd=Search&ter m=%22Anesthesiology+clinics+of+North+America%22[Jour]+AND+2005[pdat]+AND+Bandla+P[ author] Chan J, Edman JC, Koltai PJ. Obstructive sleep apnea in children. Am Fam Physician. 2004;69:1147-1154. Available at: http://www.aafp.org/afp/20040301/1147.html

Copyright 2007 by the American Academy of Pediatrics

page 116

2007 PREP SA on CD-ROM Question: 56

Physical examination of a newborn female reveals meningomyelocele; dysmorphic features, including a narrow bifrontal diameter, epicanthal folds, a broad and low nasal bridge, and midface hypoplasia; and a short systolic murmur. Results of echocardiography document coarctation of the aortic arch. Of the following, the MOST likely prenatal exposure to explain these findings is

A. alcohol B. lithium C. retinoic acid D. thiazide diuretic E. valproic acid

Copyright 2007 by the American Academy of Pediatrics

page 117

2007 PREP SA on CD-ROM Critique: 56 Preferred Response: E

The use of anticonvulsants during pregnancy is associated with an increased risk for fetal anomalies. In particular, three syndromes have been delineated in association with the maternal use of valproic acid, trimethadione, and hydantoin, respectively. Mothers should be informed of the possible consequences to the fetus of anticonvulsant therapy during pregnancy. However, the potential harmful consequences of uncontrolled seizures for both the mother and the fetus also must be considered before making any specific recommendations for discontinuation of anticonvulsant therapy during pregnancy. Fetal valproate syndrome (Item C56A) is a pattern of malformations characterized by craniofacial anomalies that include a narrow bifrontal diameter, high forehead, epicanthal folds, broad and low nasal bridge, midfacial hypoplasia, long philtrum, and small mouth. Exposed infants, such as the one described in the vignette, also are at increased risk for cardiovascular anomalies, cleft lip, and meningomyelocele. The risk for meningomyelocele has been reported to be as high as 5%, and approximately 30% of exposed infants have some features of the syndrome. Fetal trimethadione syndrome results from exposure to this anticonvulsant during the first trimester and is characterized by mental deficiency, prenatal-onset growth deficiency, and craniofacial abnormalities, including a short upturned nose, low nasal bridge, prominent forehead, synophrys (fusion of the eyebrows), and malformed ears. Cardiovascular defects, ambiguous genitalia, and cleft lip with/without cleft palate also have been reported. Most infants exposed to trimethadione display one or more of these features, and the prognosis is poor in some because of the combination of serious heart defects and mental retardation. The fetal hydantoin syndrome is characterized by digit and nail hypoplasia (Item C56B), an unusual facies, and mental deficiency. Growth deficiency has been observed both prenatally and postnatally. Approximately 10% of prenatal exposures to hydantoin result in the entire syndrome, with another 33% producing some of the effects. Prenatal exposure to other anticonvulsants, including carbamazepine and phenobarbital, also has been reported to result in facial features similar to those observed in hydantoin syndrome. Prenatal exposure to alcohol may result in the fetal alcohol syndrome, which is characterized by prenatal onset of growth deficiency, microcephaly, a typical facies (Item C56C), and cardiac septal defects. However, an increased incidence of meningomyelocele is not characteristic. Although the use of diuretics in pregnancy is not encouraged, the administration of thiazide diuretics during the first trimester is associated only minimally with an increased incidence of birth defects, and in later trimesters, there is no strong association. Prenatal exposure to lithium poses an increased risk for congenital heart defects, including Ebstein anomaly, but an increase in myelomeningocele and craniofacial abnormalities is not reported. Isotretinoin exposure is characterized by central nervous system defects (eg, hydrocephalus, microcephaly, structural errors of neuronal migration), but not meningomyelocele. Other associated abnormalities include facial asymmetry, microtia or anotia, conotruncal cardiac abnormalities, and mental deficiency. The risk for retinoic acid embryopathy is 35% in women who continue to ingest the agent past the 15th day following conception. References: Artama M, Auvinen A, Raudaskoski T, Isojarvi I, Isojarvi J. Antiepileptic drug use of women with epilepsy and congenital malformations in offspring. Neurology. 2005;64:1874-1878. Abstract available at: http://www.ncbi.nlm.nih.gov/entrez/query.fcgi?orig_db=PubMed&db=PubMed&cmd=Search&ter m=%22Neurology%22[Jour]+AND+1874[page]+AND+2005[pdat] Holmes LB, Harvey EA, Coull BA, et al. The teratogenicity of anticonvulsant drugs. N Engl J Med. 2001;344:1132-1138. Abstract available at: http://www.ncbi.nlm.nih.gov/entrez/query.fcgi?cmd=Retrieve&db=pubmed&dopt=Abstract&list_ui ds=11297704&query_hl=1&itool=pubmed_docsum

Copyright 2007 by the American Academy of Pediatrics

page 118

2007 PREP SA on CD-ROM Question: 57

A 16-year-old girl presents with the complaint of right upper quadrant pain, right shoulder pain, and nausea. She is afebrile. She has been taking combined oral contraceptive pills for 6 months. Results of ultrasonography of the gallbladder performed after an emergency department visit 2 days ago are normal. Of the following, the most appropriate NEXT step in the evaluation of this patient's pain is

A. computed tomography scan of the liver B. hepatobiliary scintography C. hydrogen breath test D. pelvic examination E. ultrasonography of the abdomen

Copyright 2007 by the American Academy of Pediatrics

page 119

2007 PREP SA on CD-ROM Critique: 57 Preferred Response: D

Fitz-Hugh-Curtis syndrome, a perihepatitis, is a complication of pelvic inflammatory disease (PID) associated with chlamydial or, less commonly, gonococcal infections. The perihepatitis involves the liver capsule and surrounding peritoneum and presents as right upper quadrant pain, as described for the girl in the vignette. The diagnosis of Fitz-Hugh-Curtis syndrome initially may be overlooked because of the absence of associated lower abdominal symptoms. Right upper quadrant pain coupled with a high index of suspicion, evidence of PID, and lack of evidence for hepatitis, gallbladder disease, or pancreatitis form the basis for diagnosing FitzHugh-Curtis syndrome. For the patient in the vignette, the most appropriate next step in evaluation of the pain is a pelvic examination for signs of salpingitis or PID (eg, cervical motion tenderness, adnexal tenderness, vaginal discharge). Although rarely necessary, in challenging cases, laparoscopic visualization of the liver can provide a definitive diagnosis of Fitz-Hugh-Curtis syndrome. Grayish granular exudates on the anterior surfaces of the liver and adjacent peritoneum are characteristic of an acute inflammatory process, and violin stringlike adhesions are characteristic of a chronic response associated with Fitz-Hugh-Curtis syndrome. Treatment is with the same regimens recommended for PID, and a significant improvement of the right upper quadrant pain is expected within 24 to 48 hours. A hydrogen breath test is particularly useful for the evaluation of lactose intolerance in adolescents who have recurrent abdominal pain. Computed tomography scan of the liver is useful for detecting discrete lesions such as tumors, abscesses, and cysts, which are not likely causes of the adolescents pain. Hepatobiliary scintography (study of radionuclide uptake in the liver and excretion in the bile) is particularly helpful for differentiating intrahepatic cholestasis from extrahepatic obstruction in infants. After a pelvic examination has established the absence of signs or symptoms of PID, ultrasonography of the abdomen often is indicated to investigate other causes of right upper quadrant pain (subphrenic abscess, renal stones, and appendicitis). References: Banikarim C, Chacko MR. Pelvic inflammatory disease in adolescents. Adolesc Med Clin. 2004;15:273-285 Bortot AT, Risser WL, Cromwell PF. Coping with pelvic inflammatory disease in the adolescent. Contemp Pediatr. 2004;21:33-48. Available at: http://www.contemporarypediatrics.com/contpeds/article/articleDetail.jsp?id=108024 Darville T. Neisseria gonorrhoeae (gonococcus). In: Behrman RE, Kleigman RM, Jenson HB, eds. Nelson Textbook of Pediatrics. 17th ed. Philadelphia, Pa: Saunders; 2004:899-904 Pletcher JR, Slap GB. Pelvic inflammatory disease. In: Neinstein LS, ed. Adolescent Health Care Practice Guide. 4th ed. Philadelphia, Pa: Lippincott, Williams & Wilkins; 2002:1161-1170 Shrier LA. Bacterial sexually transmitted infections: gonorrhea, chlamydia, pelvic inflammatory disease, and syphilis. In: Emans SJH, Laufer MR, Goldstein DP, eds. Pediatric and Adolescent Gynecology. 5th ed. Philadelphia, Pa: Lippincott, Williams & Wilkins; 2005:565-614

Copyright 2007 by the American Academy of Pediatrics

page 120

2007 PREP SA on CD-ROM Question: 58

A normal-appearing 9-month-old boy is discovered on routine herniorrhaphy to have bilateral Fallopian tubes and a rudimentary uterus. Biopsy of the gonads performed during this procedure revealed normal testicular tissue. On examination today, his phallus is normal in size and appearance, and his descended testes are both 2 mL in volume. Of the following, the MOST likely sex chromosome complement for this child is

A. XX B. XX/XY C. XXY D. XO/XY E. XY

Copyright 2007 by the American Academy of Pediatrics

page 121

2007 PREP SA on CD-ROM Critique: 58 Preferred Response: E

The boy described in the vignette, who has normal descended testes and a normal penis and is discovered to have fallopian tubes and a rudimentary uterus, has the persistent mllerian duct syndrome. A peptide hormone (mllerian inhibiting substance [MIS] or anti-mllerian hormone [AMH]) produced by the testis induces regression of the mllerian ducts during fetal development. These ducts become the fallopian tubes and uterus as well as the upper part of the vagina if there is no regression. Genetic defects that lead to failure of production of MIS or lack of receptors for this hormone are responsible for failure of regression of mllerian tissues in otherwise normal males. This is an autosomal recessive disorder and is associated with an XY chromosomal pattern. The SRY gene is the male-determining gene on the Y chromosome. Children who have an XX chromosome pattern rarely present with sex reversal (male phenotype) because of a translocation of the SRY gene to an X chromosome or an autosome. Affected children have gonads that generally are composed of testicular tissue and should not have mllerian remnants. Children who have an XX/XY chromosomal pattern usually have external genitalia that reflect true intersex, and gonads contain a variable mix of testicular or ovarian tissue that may differ in each gonad. Unilateral mllerian remnants may be present if mostly ovarian tissue is found on that side. Individuals who have the XXY complement have Klinefelter syndrome. They have normal male external and internal sexual development at birth, although testes and phallus may be somewhat small. As they grow, they express components of Klinefelter syndrome of variable severity, ranging from late-onset infertility to severe behavioral and neurodevelopmental problems. Children who have an XO/XY chromosomal pattern usually have normal male differentiation but may have somewhat small phalluses. They may have infertility and short stature. Some affected children have mixed gonadal dysgenesis that results in ambiguous genitalia and some manifestations of Turner syndrome. The presence of Y chromosomal material in a dysgenetic gonad increases the risk for malignancy within the gonad, although the risk is lower than previously believed. References: Houk CP, Levitsky LL. Evaluation of the infant with ambiguous genitalia. UptoDate. 2006;14.1. Available at: http://www.utdol.com/utd/content/topic.do?topicKey=pediendo/11554&type=P&selectedTitle=14~ 24 Houk CP, Levitsky LL. Management of the infant with ambiguous genitalia. UptoDate. 2006;14.1. Available at: http://www.utdol.com/utd/content/topic.do?topicKey=pediendo/12719&type=P&selectedTitle=15~ 24 Josso N, Belville C, di Clemente N, Picard JY. AMH and AMH receptor defects in persistent Mllerian duct syndrome. Hum Reprod Update. 2005;11:351-356. Abstract available at: http://www.ncbi.nlm.nih.gov/entrez/query.fcgi?orig_db=PubMed&db=PubMed&cmd=Search&ter m=%22Human+reproduction+update%22[Jour]+AND+351[page]+AND+2005[pdat]

Copyright 2007 by the American Academy of Pediatrics

page 122

2007 PREP SA on CD-ROM Question: 59

An 8-year-old boy has difficulty with reading; he reads slowly and makes many mistakes. He has a history of a febrile seizure at 1 year of age and a fall at age 2 years that resulted in a brief loss of consciousness. His mother read an article about interventions for improving vision function in children who have reading difficulties and would like your opinion. Findings on his physical examination are normal, except for vision of 20/40 bilaterally. Of the following, your BEST response is that you would like to

A. order brain magnetic resonance imaging B. order electroencephalography C. refer him for optometric evaluation D. refer him for a functional vision assessment E. request a psychoeducational evaluation

Copyright 2007 by the American Academy of Pediatrics

page 123

2007 PREP SA on CD-ROM Critique: 59 Preferred Response: E

The child described in the vignette has symptoms that are concerning for a reading learning disability. The most helpful intervention is further evaluation of his academic difficulty by requesting a psychoeducational evaluation. Reading is a complicated endeavor that involves the integration of multiple neurologic pathways. Although vision is necessary for reading, complex visual processing facilitates reading and comprehension of written language. Children who have common ophthalmologic disorders may have academic difficulty only when there is an inability to see adequately what is written on the page. Mild or correctable vision deficits rarely cause reading or academic problems. Of note, there is no evidence to support the value of eye exercises or the use of special tinted lenses as therapy for reading difficulties. Brain magnetic resonance imaging and electroencephalography are unlikely to provide useful information in a child who has normal findings on physical examination and a single febrile seizure. He should receive routine eye evaluations, but given the near-normal vision documented on your office screening, he is unlikely to have a vision disorder that is causing his reading difficulty. For children who have vision loss, a functional vision assessment, which evaluates how the child uses vision to perform routine tasks, can be useful. References: Committee on Children With Learning Disabilities American Academy of Pediatrics (AAP), and American Academy of Ophthalmology (AAO), and American Association for Pediatric Ophthalmology and Strabismus (AAPOS). Learning disabilities, dyslexia, and vision: a subject review. Pediatrics. 1998;102:1217-1219. Available at: http://pediatrics.aappublications.org/cgi/content/full/102/5/1217 Galaburda AM, Duchaine BC. Developmental disorders of vision. Neurol Clin. 2003;2:687-707 Olitsky SE, Nelson LB. Reading disorders in children. Pediatr Clin North Am. 2003;50:213-224

Copyright 2007 by the American Academy of Pediatrics

page 124

2007 PREP SA on CD-ROM Question: 60

A 4-year-old boy presents to your clinic for a second opinion. He has a 3-week history of diarrhea, abdominal pain, and tenesmus. The parents state that he seems to be getting worse, and nobody has been able to help them despite "a bunch of tests on his poop." His stool output has increased from four to five per day to eight to ten per day during the past week, and he now has a temperature of 102F (38.9C). They are starting to see what appears to be blood in the toilet after he goes to the bathroom. According to the parents, the boy was in good health until 1 week after they returned from a fishing trip on the Amazon river. Physical examination reveals a moderately ill-appearing boy who has diffuse abdominal pain. During your examination, he passes a very foul-smelling stool that appears to be a mixture of blood and pus, which you send to the laboratory for analysis. Of the following, the MOST appropriate next test is

A. abdominal ultrasonography B. barium enema C. colonic biopsy D. gallium scan E. liver function test

Copyright 2007 by the American Academy of Pediatrics

page 125

2007 PREP SA on CD-ROM Critique: 60 Preferred Response: A

The boy described in the vignette has a gastrointestinal infection caused by Entamoeba histolytica. The clinical presentation of amebiasis may range from asymptomatic cyst shedding to intestinal amebiasis to extraintestinal disease. Intestinal symptoms may occur within 2 weeks of infection and present with colicky abdominal pain and diarrhea. Patients who have diarrhea complain of tenesmus, and red and white blood cells often are present in the stool. Amebic colitis is especially common in children younger than 5 years of age and can produce toxic megacolon, fulminant colitis, ulceration of the colon, and rarely, bowel perforation. Occasionally, a chronic form of amebic colitis may occur that can mimic inflammatory bowel disease, and amebomas may develop. This annular lesion may be mistaken for changes associated with inflammatory bowel disease, a pyogenic abscess, or a colonic carcinoma. Such lesions usually respond to antiamebic therapy. Disseminated amebiasis also may occur, and a very serious form causes liver abscess. Because the patient in the vignette is very ill and presents with diffuse abdominal pain, abdominal ultrasonography is indicated to evaluate for a liver abscess (Item C60A). Amebiasis can be diagnosed by identifying trophozoites or cysts in the stool. Detection of serum antibody titers also may aid in diagnosing amebic colitis or extraintestinal disease. Colonic biopsy may demonstrate amebic organisms but is not the most appropriate test at this stage of illness. Liver function test results often are normal in the presence of liver abscesses because abscess formation does not acutely destroy liver cells. A gallium scan may help find a liver abscess, but ultrasonography is an easier and more cost-effective diagnostic tool. A barium enema is not indicated in a patient who has colitis due to the risk of developing toxic megacolon or perforation. Patients who have amebic disease require therapy to treat both the intraluminal and tissueinvading organisms. For asymptomatic patients who excrete cysts (intraluminal disease), paromomycin, iodoquinol, or diloxanide furoate may be used; patients who have all other forms should receive metronidazole followed by iodoquinol or paromomycin. References: American Academy of Pediatrics. Amebiasis. In: Pickering LK, ed. Red Book: 2006 Report of the Committee on Infectious Diseases. 27th ed. Elk Grove Village, Ill: American Academy of Pediatrics; 2006:204-206 Yost J. In brief: amebiasis. Pediatr Rev 2002;23:293-294. Available at: http://pedsinreview.aappublications.org/cgi/content/full/23/8/293

Copyright 2007 by the American Academy of Pediatrics

page 126

2007 PREP SA on CD-ROM Question: 61

A 12-year-old girl presents to the emergency department with nausea, vomiting, and abdominal pain of 1 month's duration. Physical examination reveals a large, smooth mass encompassing almost the entire lower abdomen. Computed tomography scan confirms a mass, and biopsy documents Burkitt lymphoma. She immediately begins receiving chemotherapy, and 12 hours later she develops the classic electrolyte and urinary findings consistent with tumor lysis syndrome (TLS). Of the following, the laboratory findings MOST consistent with TLS are

A. Serum Potassium: Elevated; Serum Phosphorous: Elevated; Serum lactate dehydrogenase:


Normal; Serum sodium: Elevated

B. Serum Potassium: Elevated; Serum Phosphorous: Normal; Serum lactate dehydrogenase:


Elevated; Serum sodium: Normal

C. Serum Potassium: Normal; Serum Phosphorous: Elevated; Serum lactate dehydrogenase:


Elevated; Serum sodium: Elevated

D. Serum Potassium: Normal; Serum Phosphorous: Normal; Serum lactate dehydrogenase:


Elevated; Serum sodium: Normal

E. Serum Potassium: Elevated; Serum Phosphorous: Elevated; Serum lactate dehydrogenase:


Elevated; Serum sodium: Normal

Copyright 2007 by the American Academy of Pediatrics

page 127

2007 PREP SA on CD-ROM Critique: 61 Preferred Response: E

Tumor lysis syndrome (TLS) is caused by any oncologic condition in which the cancer cells are being destroyed rapidly. Various intracellular substances are released into the bloodstream (eg, potassium, lactate dehydrogenase, phosphorous), resulting in significantly elevated levels of these electrolytes in the serum. Because only intracellular ions are affected in TLS, serum sodium concentrations should be normal. The kidney is primarily responsible for eliminating the excess electrolytes from the body but may become overwhelmed and unable to remove dangerous quantities of various substances with sufficient speed. In addition, some substances, such as uric acid and phosphorous, may be deposited in the kidney and cause tubular obstruction and damage. If this occurs, the body is deprived of the primary organ required to maintain electrolyte homeostasis. The girl described in the vignette has a Burkitt lymphoma, which is a generally very large tumor that contains an abundance of cells. Once chemotherapy is initiated, the tumor cells typically are very sensitive to the therapy and destroyed rapidly, resulting in hyperkalemia, hyperphosphatemia, and elevated serum lactate dehydrogenase. The kidney excretes massive quantities of various substances until its capacity is overwhelmed. Other oncologic conditions, such as acute lymphoblastic leukemia, especially when it is associated with exceedingly high white blood cell counts (>50x103/mcL [50x109/L]) also may cause TLS. Therapy for TLS includes aggressive hydration (at least twice maintenance fluid), alkalinization of the urine (to offset the deposition of uric acid in the kidney), and diuretic therapy (to promote further urine flow). Therapy to reduce the serum concentrations of uric acid with either allopurinol or rasburicase generally is provided early in the course of chemotherapy to patients at risk for TLS. Finally, if significant electrolyte abnormalities persist or acute renal failure develops, continuous renal replacement therapy is indicated. References: Cairo MS, Bishop M. Tumour lysis syndrome: new therapeutic strategies and classification. Br J Haematol. 2004;127:3-11. Abstract available at: http://www.ncbi.nlm.nih.gov/entrez/query.fcgi?cmd=Retrieve&db=pubmed&dopt=Abstract&list_ui ds=15384972&query_hl=31&itool=pubmed_docsum Lameire NH, Flombaum CD, Moreau D, Ronco C. Acute renal failure in cancer patients. Ann Med. 2005;37:13-25. Abstract available at: http://www.ncbi.nlm.nih.gov/entrez/query.fcgi?orig_db=PubMed&db=PubMed&cmd=Search&ter m=%22Annals+of+medicine%22[Jour]+AND+13[page]+AND+2005[pdat] Olgar S, Yetgin S, Cetin M, Aras T, Akhan O. Electrolyte abnormalities at diagnosis of acute lymphocytic leukemia may be a clue for renal damage in long-term period. J Pediatr Hematol Oncol. 2005;27:202-206. Abstract available at: http://www.ncbi.nlm.nih.gov/entrez/query.fcgi?orig_db=PubMed&db=PubMed&cmd=Search&ter m=%22Journal+of+pediatric+hematology/oncology+:+official+journal+of+the+American+Society+ of+Pediatric+Hematology/Oncology%22[Jour]+AND+202[page]+AND+2005[pdat]

Copyright 2007 by the American Academy of Pediatrics

page 128

2007 PREP SA on CD-ROM Question: 62

The mother of a child who is infected with human immunodeficiency virus (HIV) would like to enroll her child in a local child care center. Of the following, the circumstance that is MOST likely to exclude the child who has HIV infection from attending a child care center is

A. a child who exhibits aggressive behavior such as biting and scratching B. a child who has a history of occasional nose bleeds C. a child who is not yet toilet trained D. no circumstance of exclusion E. the parent(s) or guardian who does not want to disclose the HIV status of the child

Copyright 2007 by the American Academy of Pediatrics

page 129

2007 PREP SA on CD-ROM Critique: 62 Preferred Response: A

Other than blood exposure, human immunodeficiency virus (HIV) has not been transmitted through the types of daily contact that usually occur in child care centers or schools, including contact with saliva or tears. Therefore, children who have HIV infection generally should not be excluded from child care or school to protect personnel or other children. Some specific circumstances, however, dictate against HIV-infected children attending child care centers or schools. These include a child who has uncovered exudative skin lesions or uncontrolled aggressive behavior (eg, biting or scratching) or the desire to minimize exposure of the HIVinfected child to contagious illnesses. A history of mild eczema or occasional nose bleeds is not a reason to exclude an infected child from child care or school. Exposure to stool or urine is not a route of HIV transmission and, therefore, lack of toilet training is not a reason for exclusion. Disclosure of HIV infection to the child care center or school is not required. In general, to minimize possible transmission of any bloodborne disease, all child care centers and schools should follow the recommended standard precautions to avoid contact with blood or other body fluids from any child or staff member. Such procedures include: not sharing personal items that might be in contact with blood (eg, toothbrush) and promptly disinfecting spills of blood or body fluids (eg, vomitus, urine, or feces) with a commercial cleaner or diluted bleach (1:10). These protocols should be applied universally because unidentified HIV infections may be present. References: American Academy of Pediatrics. Human immunodeficiency virus infection. In: Pickering LK, ed. Red Book: 2006 Report of the Committee on Infectious Diseases. 27th ed. Elk Grove Village, Ill: American Academy of Pediatrics; 2006:378-401 American Academy of Pediatrics Committee on Pediatric AIDS. Education of children with human immunodeficiency virus infection. Pediatrics. 2000;105:1358-1360. Available at: http://pediatrics.aappublications.org/cgi/content/full/105/6/1358

Copyright 2007 by the American Academy of Pediatrics

page 130

2007 PREP SA on CD-ROM Question: 63

As part of your clinic responsibilities, you supervise allergy shots for patients who have allergic rhinitis. The nurse calls you about a 12-year-old girl who received her allergy shot 30 minutes ago and now is experiencing warmth and erythema over the injection site. On physical examination, the patient appears healthy and in no distress. Vital signs include a temperature of 97.5F (36.4C), heart rate of 90 beats/min, and respiratory rate of 18 breaths/min. At the injection site, there is a 4 x 4 cm raised, erythematous, warm area (Item Q63A). There is no edema of the tongue or uvula, and findings on her pulmonary, cardiovascular, and skin examinations are otherwise normal. Of the following, the NEXT most appropriate action is to

A. administer an oral antibiotic B. administer intramuscular epinephrine C. allow the patient to leave the clinic D. discontinue allergy shots E. observe the patient for an additional 30 minutes

Copyright 2007 by the American Academy of Pediatrics

page 131

2007 PREP SA on CD-ROM Critique: 63 Preferred Response: C

There are three general levels of management for allergic rhinitis: avoidance, pharmacologic management, and allergen immunotherapy (allergy shots). Immunotherapy can be considered in children ages 5 years and older who have significant allergic disease that is refractory to avoidance and pharmacologic therapy or who experience adverse effects from medications. During immunotherapy, a number of safety recommendations should be followed, including administration of the immunotherapy in a clinic, a mandatory wait time of 20 to 30 minutes after an allergy shot, prompt recognition of systemic reactions, and avoidance of allergy shots in children receiving beta-blockers or who have unstable asthma. The two primary complications from allergy shots, not including pain from the injection, are local reactions and anaphylaxis. Local reactions (Item C63A), as described for the girl in the vignette, are common and occur at the injection site. These reactions are self-limited, spontaneously improve within 4 to 72 hours, and do not require any specific therapy. Patients experiencing local reactions in the absence of other systemic symptoms should be allowed to leave the clinic. Because local reactions represent localized immunoglobulin E-mediated reactions and not an infection, administration of an oral antibiotic or epinephrine is not recommended. Some experts recommend observing patients who have asthma or who have experienced previous anaphylaxis for 30 to 60 minutes after an allergy injection. In the absence of systemic symptoms or previous reactions for the girl in the vignette, observation past the recommended 20 to 30 minutes is not indicated. Discontinuation of allergy shots may be warranted for patients who experience repeated episodes of anaphylaxis, but generally is not required for local reactions. Systemic anaphylaxis is the primary concern during allergen immunotherapy and occurs in approximately 1 in 200 injections. Most reactions occur during the first year of buildup, during peak pollen seasons, and begin within 30 minutes after an allergy shot. Common symptoms include generalized pruritus, hives, angioedema, sneezing fits, ocular erythema, difficulty breathing, and lightheadedness. In general, prompt administration of intramuscular epinephrine and supportive care are recommended. References: Atkins D, Leung DYM. Principles of treatment of allergic disease. In: Behrman RE, Kliegman RM, Jenson HB, eds. Nelson Textbook of Pediatrics. 17th ed. Philadelphia, Pa: WB Saunders Co 2004:752-758 Joint Task Force on Practice Parameters. Allergen immunotherapy: a practice parameter. American Academy of Allergy, Asthma and Immunology, American College of Allergy, Asthma and Immunology. Ann Allergy Asthma Immunol. 2003;90:1-40

Copyright 2007 by the American Academy of Pediatrics

page 132

2007 PREP SA on CD-ROM Question: 64

A 3-year-old boy who was involved in a motor vehicle crash sustained a significant head injury and had a Glasgow Coma Scale score of 8 on arrival in the emergency department. An endotracheal tube was placed shortly after his arrival, and the respiratory therapist is providing ventilation at 12 breaths/min with 100% oxygen via a bag-valve mask until a ventilator can be brought to the emergency department. Ten minutes later, the child becomes more restless and agitated. His heart rate has increased from an initial 110 beats/min to about 150 beats/min, and he appears flushed. The monitor indicates that his oxygen saturation is 96%. Of the following, a TRUE statement about this child's ventilatory status is that

A. a patient who has acute carbon dioxide retention due to hypoventilation still can have nearnormal blood oxygen saturation

B. an arterial blood gas determination likely would reveal a pH of approximately 7.45 C. his agitation is due to relative hypoxemia D. his agitation is not related to his ventilatory status; it is due to tracheal irritation from the
endotracheal tube

E. the oxygen saturation of 96% indicates that he is being ventilated adequately

Copyright 2007 by the American Academy of Pediatrics

page 133

2007 PREP SA on CD-ROM Critique: 64 Preferred Response: A

The child described in the vignette exhibits signs of acute hypercapnia, or carbon dioxide retention, likely due to the relatively low number of ventilations that are being provided per minute. The observed near-normal oxygen saturation is a reflection of the high inspired oxygen concentration. Even minimal oxygen therapy can maintain normal oxygenation in patients who are experiencing severe hypoventilation. For this reason, an oxygen saturation monitor is not an accurate means to assess the adequacy of ventilation. Hypoventilation leading to hypercapnia results in a pH well below 7.4 due to carbon dioxide retention. The oxygen saturation of 96% reported for the child in the vignette is not consistent with hypoxemia. Restlessness due to tracheal irritation is a possibility, especially if a patient has not received adequate sedation prior to beginning mechanical ventilation. However, the presence of flushing, one of the clinical manifestations of hypercapnia, is the best clue to the correct diagnosis. The additional manifestations of tachycardia and agitation could be observed with either condition. References: American Heart Association. Airway, ventilation, and management of respiratory distress and failure. In: PALS Provider Manual. Dallas, Tex: American Heart Association; 2002:81-126 Pope J, McBride J. Consultation with the specialist: respiratory failure in children. Pediatr Rev. 2004;25:160-167. Available at: http://pedsinreview.aappublications.org/cgi/content/full/25/5/160

Copyright 2007 by the American Academy of Pediatrics

page 134

2007 PREP SA on CD-ROM Question: 65

An adolescent male presents with a hypopigmented eruption (Item Q65A) composed of multiple oval macules arrayed across the upper trunk in a yoke distribution. Of the following, the PREFERRED management course consists of

A. administration of oral griseofulvin B. application of psoralen followed by ultraviolet radiation C. avoidance of the offending allergen D. topical application of moderately potent corticosteroid E. topical application of selenium sulfide

Copyright 2007 by the American Academy of Pediatrics

page 135

2007 PREP SA on CD-ROM Critique: 65 Preferred Response: E

Tinea versicolor typically manifests as multiple round or oval macules with superficial scale arrayed in a yoke distribution that includes the neck, chest, upper back, and shoulders, as described for the boy in the vignette. Commonly, individual macules coalesce to form patches. Lesions may be hypo- (Item C65A) or hyperpigmented (Item C65B), hence, the name versicolor. The infection occurs primarily in adolescents and young adults, perhaps because the organism responsible, Malassezia furfur (Pityrosporum ovale) uses sebum as a nutrient. The infection is most prevalent in hot, humid climates. A potassium hydroxide preparation performed on scale obtained from the lesions reveals the classic spaghetti and meatballs appearance (ie, short hyphae and spores) (Item C65C). Initial therapy of tinea versicolor consists of topical application of selenium sulfide 1% (present in some antiseborrheic shampoos) or 2.5% (available by prescription). Although many treatment regimens exist, a commonly employed one advises the application of selenium sulfide to all affected areas for 10 minutes, after which it is rinsed off. An alternative is topical ketoconazole, although this drug typically is more expensive. For patients who have resistant infection or who cannot use topical therapy, treatment with oral fluconazole or ketoconazole may be beneficial. Regardless of the initial treatment employed, infection often returns. For this reason, following initial treatment, topical selenium sulfide often is used prophylactically (ie, as a single 8- to 12-hour application monthly for 3 to 4 months). Patients should be counseled that return of normal skin pigmentation often requires several months. Topical administration of psoralen compounds followed by gradually increasing exposure to ultraviolet radiation may be indicated in the management of vitiligo (characterized by depigmented [not hypopigmented] macules and patches), but it does not have a role in the treatment of tinea versicolor. Postinflammatory hypopigmentation may follow episodes of atopic or contact dermatitis. However, the physical findings exhibited by the adolescent described in the vignette are not consistent with these diagnoses and, therefore, treatment with a topical corticosteroid and epicutaneous patch testing, respectively, are not warranted. Griseofulvin is used to treat certain dermatophyte infections, particularly tinea capitis, but it is not effective in tinea versicolor. References: Darmstadt GL, Sidbury R. Cutaneous fungal infections. In: Behrman RE, Kliegman RM, Jenson HB, eds. Nelson Textbook of Pediatrics. 17th ed. Philadelphia, Pa: WB Saunders Co; 2004:22302234 Falabella R. Pigmentary abnormalities: nongenetic disorders of hypopigmentation. In: Schachner LA, Hansen RC, eds. Pediatric Dermatology. 3rd ed. St. Louis, Mo: Mosby; 2003:504-513 Paller AS, Mancini AJ. Photosensitivity and photoreactions. In: Hurwitz Clinical Pediatric Dermatology. 3rd ed. Philadelphia, Pa: Elsevier Inc; 2006:503-5524 Paller AS, Mancini AJ. Eczematous eruptions in childhood. In: Hurwitz Clinical Pediatric Dermatology. 3rd ed. Philadelphia, Pa: Elsevier Inc; 2006:49-84 Paller AS, Mancini AJ. Disorders of pigmentation. In: Hurwitz Clinical Pediatric Dermatology. 3rd ed. Philadelphia, Pa: Elsevier Inc; 2006:265-306 Weston WL, Lane AT, Morelli JG. Fungal and year infections of the skin: yeast skin infections. In: Color Textbook of Pediatric Dermatology. 3rd ed. St. Louis, Mo: Mosby; 2002:71-76

Copyright 2007 by the American Academy of Pediatrics

page 136

2007 PREP SA on CD-ROM Question: 66

A 3-year-old boy who recently was adopted is brought to you for the first time. The adoptive mother explains that the biologic mother had a history of intravenous drug use, and the child is hepatitis C-positive. On physical examination, the happy, active child has no evidence of an enlarged liver or spleen. You discuss with the mother the possibility of referral to a specialist for antiviral therapy. Of the following, a TRUE statement about treatment of hepatitis C is that

A. antiviral therapy usually is administered for 12 weeks B. coinfection with human immunodeficiency virus is a contraindication to treatment C. patients whose alanine aminotransferase values remain persistently normal do not need to be
referred

D. the preferred treatment is oral ribavirin monotherapy E. viral genotype determines the likelihood of response to therapy

Copyright 2007 by the American Academy of Pediatrics

page 137

2007 PREP SA on CD-ROM Critique: 66 Preferred Response: E

Hepatitis C is estimated to infect 170 million individuals worldwide and up to 200,000 children in the United States. Infection primarily occurs after exposure to infected blood products and infected needles, sexual contact with an infected partner, or vertical transmission from mother to child. The incidence of transmission from infected blood has declined dramatically since screening was instituted in 1990. In contrast, the incidence of transmission from mother to child has stayed stable at approximately 5%, and there is no known intervention to reduce such risk. Approximately 80% of infected adults and 50% to 80% of infected children become chronic carriers, and up to 20% of infected individuals develop cirrhosis over time. Thus, early diagnosis and treatment is essential. Routine serologic testing for hepatitis C is recommended for injection drug users, recipients of blood products or organ transplants before July 1992, patients on longterm hemodialysis, people who have persistently abnormal alanine aminotransferase concentrations, or children born to high-risk parents. A full summary of recommendations for hepatitis C screening is included in the Red Book. Screening of high-risk individuals is performed by hepatitis C serology, which detects immunoglobulin G against the viral NS5 and NS3 antigens. Antibodies usually become positive within 8 weeks after an exposed person is infected. Antibody testing has a sensitivity of 97% and a positive predictive value of more than 95% in patients at risk, although the predictive value is only 25% in the general population. If a person has a positive antibody result, further confirmatory testing with recombinant immunoblot antibodies and hepatitis C quantitative viral RNA is performed. In appropriate individuals, liver biopsy is useful in predicting the extent of inflammation and hepatic cirrhosis. Treatment of individuals infected with hepatitis C currently involves the administration of either interferon alone or interferon with ribavirin for 6 to 12 months. Response typically is measured by reductions in viral load, improvement in transaminases, and reduction of inflammation on liver histology. Genotype determines response rate: patients who have genotype 1 usually have a poorer response rate (<40%), while other genotypes have a better response rate (50% to 70%). Although genotype status does predict response rate, therapy still can be used in patients who have the hepatitis C virus genotype 1. The response rate in adults is superior among those receiving both interferon and ribavirin, although ribavirin is associated with anemia. Ribavirin is contraindicated in pregnancy because it may cause birth defects. Ribavirin is not effective as monotherapy. Patients who have normal transaminase values may warrant treatment, depending on viral load and liver histology. Human immunodeficiency virus (HIV) infection is not a contraindication to therapy, although the response rate is poorer in patients who have HIV and hepatitis C coinfection. References: Gonzalez-Peralta RP, Jolley CD. Hepatitis C virus. In: Walker WA, Goulet O, Kleinman RE, Sherman PM, Shneider BL, Sanderson IR, eds. Pediatric Gastrointestinal Disease. 4th ed. Hamilton, Ontario, Canada: BC Decker; 2005:1156-1169 Jonas MM. Children with hepatitis C. Hepatology. 2002;36(suppl 1):S173-S178. Available at: http://www3.interscience.wiley.com/cgi-bin/abstract/112219084/ABSTRACT American Academy of Pediatrics. Hepatitis C. In: Pickering LK, ed. Red Book: 2006 Report of the Committee on Infectious Diseases. 27th ed. Elk Grove Village, Ill: American Academy of Pediatrics; 2006:358-359

Copyright 2007 by the American Academy of Pediatrics

page 138

2007 PREP SA on CD-ROM Question: 67

An infant is delivered by cesarean section at 34 weeks' gestation because of preterm labor. There is no history of ruptured membranes, maternal fever, or abnormalities in fetal heart rate monitoring. The infant requires assisted ventilation with a bag-mask device and 100% oxygen in the delivery room. His Apgar scores are 5 and 7 at 1 and 5 minutes, respectively. An umbilical cord arterial pH is 7.23 and base deficit is 3 mmol/L. He is admitted to the newborn nursery, but transferred to the neonatal intensive care unit (NICU) within 1 hour for respiratory distress manifested by tachypnea and grunting. His arterial blood gas results (obtained on room air) upon admission to the NICU are: pH, 7.20; Pco2, 70 mm Hg; Po2, 50 mm Hg, and base deficit 10 mmol/L. Of the following, the MOST common cause for this infant's respiratory distress is

A. aspiration of amniotic fluid B. hemothorax C. pneumopericardium D. pneumoperitoneum E. pneumothorax

Copyright 2007 by the American Academy of Pediatrics

page 139

2007 PREP SA on CD-ROM Critique: 67 Preferred Response: E

Grunting respirations, as described for the infant in the vignette, are indicative of an infant breathing against a partially closed glottis, allowing for a prolonged expiratory phase and an elevation of end-expiratory volume (described by some as self-induced positive end-expiratory pressure). This mechanism may be helpful to the newborn who has an effective loss of functional residual capacity (diffuse atelectasis or impaired pulmonary compliance), acidemia (metabolic or associated with sepsis) with associated hyperpnea as an attempt to accomplish compensatory respiratory alkalosis, and air-leak syndromes that result in grossly reduced functional residual capacity due to collapsed lung volume. Because the newborn in the vignette has received positive-pressure ventilation, the most likely cause of his grunting is a pneumothorax. Pneumopericardium and pneumoperitoneum are rare in newborns. Hemothorax is seen with trauma, coagulopathy, or chest surgery. Aspiration of amniotic fluid is unlikely in this scenario. The blood gas values reported for the infant are indicative of a mixed, but principally respiratory acidemia. The normal Pco2 is 35 to 45 mm Hg, and correcting the hypercarbia will restore the pH to greater than 7.35. The base deficit of 10 mmol/L is mildly increased and likely reflects the mild hypoxemia (Po2 <60 mm Hg) associated with the respiratory distress, collapsed lung, and possible hemodynamic embarrassment. References: Durand DJ, Phillips B, Boloker J. Blood gases: technical aspects and interpretation. In: Goldsmith JP, Karotkin EH, eds. Assisted Ventilation of the Neonate. 4th ed. Philadelphia, Pa: Saunders; 2003:279-292 Flidel-Rimon O, Shinwell ES. Respiratory distress in the term and near-term infant. NeoReviews. 2005;6:e289-e297. Available at: http://neoreviews.aappublications.org/cgi/content/full/6/6/e289 Rosti L, Coppa I, Perricone G, Mastretti A. Index of suspicion in the nursery. NeoReviews. 2003;4:e315-e316. Available at: http://neoreviews.aappublications.org/cgi/content/full/4/11/e315

Copyright 2007 by the American Academy of Pediatrics

page 140

2007 PREP SA on CD-ROM Question: 68

A 4 year-old-boy presents to your clinic with anal itching of 2 weeks' duration. His mother denies itching in other family members. Tape applied to his perianal skin shows oval structures (Item Q68A). Of the following, the most appropriate management of this patient is

A. albendazole administered three times daily for 7 days B. ivermectin administered in a single dose and repeated in 2 weeks C. ketoconazole administered daily for 7 days D. mebendazole administered in a single dose and repeated in 2 weeks E. praziquantel administered three times in 1 day

Copyright 2007 by the American Academy of Pediatrics

page 141

2007 PREP SA on CD-ROM Critique: 68 Preferred Response: D

The child described in the vignette has an anal pinworm infection, which is caused by the roundworm Enterobius vermicularis. Transmission is via the fecal-oral route, and the primary symptom is anal pruritus. The diagnosis may be made by documentation of adult worms (Item C68A) in the perianal area. If worms are not grossly visible, transparent adhesive tape may be applied to the perianal skin and then applied to a glass slide and examined under low power through a microscope. Ideally, three specimens should be obtained on consecutive mornings immediately upon awakening and before washing or stooling. Stool studies are not recommended because the eggs (Item C68B) are found in very small quantities in the stool. The treatment of pinworm infections is a single dose of mebendazole, pyrantel pamoate, or albendazole. A repeat dose should be administered 2 weeks after the first dose in case of reinfection. Ivermectin and ketoconazole are not used in the management of these infections. Praziquantel is used in the treatment of some tapeworm infections but is not indicated in the treatment of pinworm infections. In addition to medical treatment, counseling the parent on steps to prevent reinfection and hygiene measures is important. Washing of the infected childs pajamas, bedding, and underclothes is recommended, and measures should be taken to avoid scratching the affected area. Bathing in the morning helps to remove eggs from the area. The entire family should be treated if multiple infections or repeated infections occur. Child care workers should be educated on good hand washing techniques, but affected children may attend child care. References: American Academy of Pediatrics. Pinworm infection. In: Pickering LK, ed. Red Book: 2006 Report of the Committee on Infectious Diseases. 27th ed. Elk Grove Village, Ill: American Academy of Pediatrics; 2006:520-522 Robinson J. Infectious diseases in schools and child care facilities. Pediatr Rev. 2001;22:39-46. Available at: http://pedsinreview.aappublications.org/cgi/content/full/22/2/39

Copyright 2007 by the American Academy of Pediatrics

page 142

2007 PREP SA on CD-ROM Question: 69

A 12-year-old child who recently emigrated from southeast Asia has beta-thalassemia for which she has required frequent transfusions. She presents today with polyuria and polydipsia. On your initial evaluation, you detect a grade II/VI systolic murmur with a gallop rhythm, palpate the liver 4 cm below the costal margin, and determine that the girl is well below the 5th percentile for height. Of the following, the MOST likely diagnosis is

A. acute anemia B. congenital heart defect C. iron overload D. lead poisoning E. sickle crisis

Copyright 2007 by the American Academy of Pediatrics

page 143

2007 PREP SA on CD-ROM Critique: 69 Preferred Response: C

The child described in the vignette has signs and symptoms of iron overload. Children who are dependent on transfusions for optimal hemodynamic status, maintenance of normal coagulation, or hematologic stability due to thalassemia or sickle cell disease are at risk for iron overload or transfusion-related hemosiderosis. Hemosiderosis usually occurs in patients who have received more than 100 units of transfused red cells. In beta thalassemia, transfusions must be administered approximately every month, and each 200 mL of packed red blood cells contains 200 mg of iron that cannot be excreted physiologically. Unbound iron may damage cells via lipid peroxidation. Acute manifestations of hemosiderosis include cardiac failure due to cardiomyopathy, arrhythmia, and liver failure. Although iron may be deposited in the ventricular myocardium, it is deposited earlier in the conduction tissues of the heart. More insidious symptoms include endocrine failure (diabetes, hypothyroidism, and pubertal delay) as well as short stature out of proportion to that expected from the underlying disease. Chelation may be used to reduce the iron load by subcutaneous injection of desferoxime, and oral chelators are being investigated. The child described in the vignette is unlikely to have acute anemia and should be well compensated for her chronic anemia. A congenital heart defect cannot be excluded, but most likely would have been detected prior to age 12 years and would not produce polyuria and polydipsia. Sickle beta thalassemia hemoglobinopathy would have manifested with recurrent infection and pain crises by age 12 years, and her peripheral blood smear would reveal sickle forms. Even in this situation, however, the primary concern still would be transfusion-related hemosiderosis. Chronic lead poisoning would be expected to yield cognitive and behavioral deficits and not present with signs of cardiac or liver involvement. References: Martin PL, Pearson HA. Hemoglobinopathies and thelassemias. In: McMillan JA, DeAngelis CD, Feigin RD, Warshaw JB, eds. Oskis Pediatrics: Principles and Practice. 3rd ed. Philadelphia, Pa: Lippincott Williams & Wilkins; 1999:1450-1452 Orkin SH, Nathan DG. The thalassemias. In: Nathan DG, Ginsburg D, Orkin SH, Look AT, eds. Nathan and Oskis Hematology of Infancy and Childhood. 6th ed. Philadelphia, Pa: Saunders; 2003:842-920

Copyright 2007 by the American Academy of Pediatrics

page 144

2007 PREP SA on CD-ROM Question: 70

A 2-year-old girl is rushed by ambulance to the emergency department for sudden-onset ataxia. Her parents have yet to arrive. On physical examination, the girl is afebrile, yet diaphoretic, with some nystagmus on far lateral gaze. Her ataxia has resolved. The remainder of physical examination findings are normal. Of the following, the MOST likely diagnosis for this child is

A. basilar migraine B. benign paroxysmal vertigo C. cerebellar hemorrhage D. phenytoin intoxication E. seizure

Copyright 2007 by the American Academy of Pediatrics

page 145

2007 PREP SA on CD-ROM Critique: 70 Preferred Response: B

The sudden and self-limited ataxia with nystagmus, but without vomiting or loss of consciousness, followed by some residual horizontal nystagmus described for the girl in the vignette is typical of benign paroxysmal vertigo. Benign paroxysmal vertigo is characterized by abrupt, brief episodes of vertigo with ataxia in children ages 1 to 3 years. The child may appear frightened, have pallor, and may indicate feeling dizzy. Rapid eye movements or nystagmus may be observed if the eyes are open. Benign paroxysmal vertigo is believed to be a migraine variant. Vertigo is defined as an illusion of movement, most often a sensation of rotation. The simplest approach to evaluating this complaint is to consider its temporal pattern and whether hearing is affected. Acute vertigo without hearing loss may stem from benign paroxysmal vertigo in toddlers, vestibular neuronitis in adolescents, labyrinthine concussion, migraine, seizures (rarely), and benign paroxysmal positional vertigo (BPPV). Most of these disorders last minutes to sometimes hours, although BPPV lasts only 5 to 20 seconds and occurs with changes in head position. BPPV can be elicited by the Hallpike-Dix maneuver. The child is moved abruptly from a sitting to lying posture, with the head hanging 45 degrees below the horizontal and rotated 45 degrees to one side, evoking vertigo and nystagmus. Acute vertigo lasting hours to days accompanied by hearing loss may be due to labyrinthitis, otitis media, perilymphatic fistula, temporal bone fracture, and Mnire disease. Mnire disease is a recurrent disorder that involves tinnitus and progressive hearing loss. Continuous vertigo over days to weeks may indicate a cholesteatoma; posterior fossa tumor; vestibular schwannoma (associated with neurofibromatosis II); demyelinating disease; or drug toxicity from aminoglycosides, isoniazid, furosemide, or phenytoin. The 2-year-old child in the vignette has not displayed headache or cranial neuropathies, making basilar migraine unlikely. The spontaneous and rapid resolution of her symptoms excludes cerebellar hemorrhage and phenytoin intoxication. No loss of consciousness has been observed, making seizure unlikely. References: MacGregor DL. Vertigo. Pediatr Rev. 2002;23:10-16. Available at: http://pedsinreview.aappublications.org/cgi/content/full/23/1/10

Copyright 2007 by the American Academy of Pediatrics

page 146

2007 PREP SA on CD-ROM Question: 71

You are evaluating a 15-year-old boy in your office. He tells you that he experiences chest pain at times while at home and at school. He describes it as sharp, located in the left chest, exacerbated by deep breathing, and resolving spontaneously. He does not feel palpitations or lightheadedness and has not had syncope. His 54-year-old father and grandfather both have had hypertension and myocardial infarctions. At this time, he has no pain, and results of physical examination are normal. Of the following, the MOST appropriate approach for this patient at this time is

A. chest radiography B. echocardiography C. electrocardiography D. reassurance with clinical follow-up E. referral to pediatric cardiologist

Copyright 2007 by the American Academy of Pediatrics

page 147

2007 PREP SA on CD-ROM Critique: 71 Preferred Response: D

Chest pain in children and adolescents is a common problem in the office, clinic, urgent care practice, and emergency department. Societal attention has focused on the importance of early recognition of chest pain in adults as an important presenting sign of cardiovascular disease and potential emergency. Some of this attention accounts for the high level of concern that many parents and children have when chest pain presents in the pediatric population. In fact, the likelihood that chest pain in an otherwise well child represents cardiovascular disease is extremely low. Overall, the incidence of chest pain attributable to a cardiac cause is less than 1% in children. Several organ systems have the potential to cause chest pain in children. The history and physical examination are essential to determine which system likely is associated and, as importantly, which is not. Information about pain quality, intensity, location, and temporal relationships is essential. Pain that radiates to the back, neck, shoulders, or left arm should raise concern about a cardiac cause. Pain that is constant or frequent, dull, pressurelike, or associated with exercise is more likely to be associated with cardiac causes than is pain that is brief, infrequent, and sharp, as described for the boy in the vignette. Pain that worsens with inspiration is generally reassuring, suggesting a musculoskeletal or pulmonary rather than cardiac cause. It is not uncommon to identify a history of a recent cardiac problem in an older member of the patients family, a reminder of the importance of and the potential for psychological impact on somatic findings. In addition to the physical examination that includes palpation of the pulses in the right arm and the femoral region and auscultation of the chest and heart, the chest wall and the costochondral margin should be palpated. If the physical examination results are normal and the history is consistent with a noncardiac source of the pain, as described for the boy in the vignette, reassurance with clinical follow-up usually is the most appropriate management. Because the pain is pleuritic, transient, and spontaneously resolving in this well adolescent, no referral or laboratory testing is indicated. References: Cava JR, Sayger PL. Chest pain in children and adolescents. Pediatr Clin North Am. 2004;51:1553-1568. Abstract available at: http://www.ncbi.nlm.nih.gov/entrez/query.fcgi?orig_db=PubMed&db=PubMed&cmd=Search&ter m=%22Pediatric+clinics+of+North+America%22[Jour]+AND+1553[page]+AND+2004[pdat] Fahey J. Chest pain. In: Rudolph CD, Rudolph AM, Hostetter MK, Lister G, Siegel NJ, eds. Rudolphs Pediatrics. 21st ed. New York, NY: McGraw Hill; 2003:1894-1897

Copyright 2007 by the American Academy of Pediatrics

page 148

2007 PREP SA on CD-ROM Question: 72

A 10-year-old boy (individual III,1 in the pedigree) (Item Q72A) presents for evaluation of ligamentous laxity and multiple joint dislocations. The family history is notable for a father who has scoliosis and ligamentous laxity, a paternal aunt who has had retinal detachments and mitral valve prolapse, and a paternal grandmother who had joint dislocations and now has osteoarthritis. Based on the family history, the MOST likely pattern of inheritance of this connective tissue disorder is

A. autosomal dominant B. autosomal recessive C. mitochondrial D. X-linked dominant E. X-linked recessive

Copyright 2007 by the American Academy of Pediatrics

page 149

2007 PREP SA on CD-ROM Critique: 72 Preferred Response: A

To determine the pattern of inheritance for a given condition, it is helpful to draw a pedigree. In the pedigree of the child described in the vignette (Item C72A), there is vertical transmission of a connective tissue disorder through three generations, which rules out autosomal recessive inheritance if there is no consanguinity. X-linked inheritance is ruled out by the male-to-male transmission (from the father to the son). Male-to-male transmission also excludes mitochondrial inheritance because mitochondria are transmitted almost exclusively through our mothers. Therefore, the mode of transmission for the disorder described in the vignette is autosomal dominant. Hallmarks of autosomal dominant inheritance include father-to-son transmission, variable expressivity, and the potential for reduced penetrance. Many dominant conditions have a high rate of spontaneous mutation (eg, neurofibromatosis type 1 [50%] and achondroplasia [80%]). The recurrence risk is 50% for all individuals who have autosomal dominant conditions. References: Hoyme HE. Patterns of inheritance. In: Behrman RE, Kliegman RM, Jenson HB, eds. Nelson Textbook of Pediatrics. 17th ed. Philadelphia Pa: Saunders; 2004:376-382 Nussbaum RL, McInnes RR, Willard HF. Patterns of single-gene inheritance. In: Thompson & Thompson Genetics in Medicine. 6th ed, revised reprint. Philadelphia, Pa: Saunders; 2004:51-78 Pinsky L. Overview of genetic assessment. UpToDate. 2006; 14.1. Available at: http://www.utdol.com/utd/content/topic.do?topicKey=genr_med/27636&type=A&selectedTitle=1~ 85

Copyright 2007 by the American Academy of Pediatrics

page 150

2007 PREP SA on CD-ROM Question: 73

A 5-year-old girl presents with a foul-smelling vaginal discharge of 2 weeks' duration. She has a previous history of one urinary tract infection at age 3 years. Of the following, the MOST appropriate next step is

A. culture of the discharge for respiratory pathogens B. genital examination using the knee-chest position C. perianal adhesive tape test D. sitz baths and application of an estrogen cream E. urine for culture and sensitivity

Copyright 2007 by the American Academy of Pediatrics

page 151

2007 PREP SA on CD-ROM Critique: 73 Preferred Response: B

A vaginal discharge in a prepubertal girl frequently is accompanied by vulvitis. The evaluation of vaginal discharge or vulvovaginitis in a prepubertal girl is guided by clues from the history and findings on physical examination. The clinician must inquire about a history of sexual molestation or activity, including instrumentation, in all girls who have a vaginal discharge with or without genital pain, regardless of their age or pubertal status. Although less commonly encountered, clinicians evaluating males must remember that instrumentation of the penis may cause genital pain or urethral discharge. Documentation of a recent history of oral broad-spectrum antibiotic use is important because it increases the risk of candidal vulvovaginitis. The character of the discharge may suggest a specific cause: a green discharge with Neisseria gonorrhoeae and group A betastreptococcal infection; a malodorous discharge with a retained foreign body; and a bloody discharge with trauma, foreign body, and Shigella or group A beta-hemolytic streptococcal infections. Other important historical information includes recent infections, hygiene habits, masturbation, use of skin products, previous occurrences of vulvovaginitis, and prior treatments. Often no specific cause of vulvovaginitis is found for prepubertal girls, and the condition is attributed to poor hygiene. Causes of a specific vulvovaginitis in prepubertal girls include infections (respiratory and enteric pathogens, Candida, sexually transmitted diseases, and pinworms), foreign body, vulvar skin diseases (eg, lichen sclerosus, seborrhea, psoriasis, atopic dermatitis, contact dermatitis), and trauma (eg, excessive cleaning or masturbation). Less common causes include systemic illnesses such as varicella and genitourinary malformations such as an ectopic ureter that may cause perineal wetness. The physical examination of prepubertal girls who have vulvovaginitis should include an inspection of the external genitalia, including the perineum and anus. A speculum examination usually is not necessary; samples of vaginal fluid can be obtained with a saline-moistened swab through the hymenal opening, avoiding contact with the sensitive hymenal edges, or a vaginal wash for a wet preparation, potassium hydroxide preparation, and cultures. The knee-chest position in a cooperative child often allows adequate visualization of the vaginal canal for the discovery of retained foreign bodies. If a foreign body is found, removal followed by sitz baths to clear residual symptoms is adequate treatment. Culture of the discharge for respiratory pathogens is not necessary. A perianal adhesive tape test is performed if a pinworm infestation is suspected as cause of vulvovaginitis associated with pruritus or excoriations. Sitz baths and application of an estrogen cream may be prescribed for persistent nonspecific vulvovaginitis, but would not be appropriate for a patient who has a foul-smelling discharge that is suggestive of a foreign body without inspection of the vaginal canal. Vulvovaginitis that is not associated with other signs or symptoms to implicate the urinary system is an unlikely presentation of a urinary tract infection, so obtaining a urine specimen for culture and sensitivity is not appropriate. References: Emans SJ. Vulvovaginal problems in the prepubertal child. In: Emans SJH, Laufer MR, Goldstein DP, eds. Pediatric and Adolescent Gynecology. 5th ed. Philadelphia, Pa: Lippincott, Williams & Wilkins; 2005:83-119 Sanfilippo JS. Vulvovaginitis. In: Behrman RE, Kleigman RM, Jenson HB, eds. Nelson Textbook of Pediatrics. 17th ed. Philadelphia, Pa: Saunders; 2004:1828-1832

Copyright 2007 by the American Academy of Pediatrics

page 152

2007 PREP SA on CD-ROM Question: 74

You are asked to evaluate a 1-day-old infant who has mild clitoromegaly (Item Q74A) and palpable masses in the labial folds bilaterally. Chromosome studies reveal that the infant has an XY karyotype. Of the following, additional history is MOST likely to reveal that the infant's

A. father was exposed to pesticides B. maternal aunts are infertile C. mother took androgens during the pregnancy D. mother took progestogens during the pregnancy E. paternal aunts and uncles are infertile

Copyright 2007 by the American Academy of Pediatrics

page 153

2007 PREP SA on CD-ROM Critique: 74 Preferred Response: B

The baby described in the vignette, who has mild masculinization and palpable gonadal masses with an XY karyotype, is an undervirilized XY infant (male pseudohermaphroditism). Because gonads are present below the inguinal ligament, there is testicular tissue. Even with careful evaluation, an underlying cause cannot presently be determined in up to 40% of undervirilized XY infants. However, one of the most common findings is androgen insensitivity (AIS), which may be complete (CAIS) or partial (PAIS). The finding usually represents a defect in the androgen receptor and is inherited in an X-linked pattern. Therefore, some maternal aunts are likely to have the same disorder, have an XY chromosomal pattern, and be infertile. Because this disorder is not inherited through the father, a history of infertility in paternal relatives would not be expected. Failure of masculinization of an XY fetus cannot be attributed to maternal androgen or progesterone treatment or to paternal exposure to pesticides, even if they have antiandrogenic effects. Physicians faced with the diagnostic management and care of XY infants should consult with experienced endocrinologists, geneticists, and psychological support personnel. Additional diagnostic studies should rule out other genetic disorders, such as 5 alpha-reductase deficiency and 17-ketosteroid reductase defects. The difficult decision of determining the sex of rearing of the affected child must involve the parents. A trial of exogenous testosterone can help to define the likely adult response to androgen and assist in this decision. References: Ahmed SF, Cheng A, Dovey L, et al. Phenotypic features, androgen receptor binding, and mutational analysis in 278 clinical cases reported as androgen insensitivity syndrome. J Clin Endocrinol Metab. 2000;85:658-665. Available at: http://jcem.endojournals.org/cgi/content/full/85/2/658 Androgen Insensitivity Syndrome Support Group Web site. Available at: http://www.medhelp.org/www/ais/ Boehmer AL, Brinkmann O, Bruggenwirth H, et al. Genotype versus phenotype in families with androgen insensitivity syndrome. J Clin Endocrinol Metab. 2001;86:4151-4160. Available at: http://jcem.endojournals.org/cgi/content/full/86/9/4151 Gottlieb B, Beitel LK, Trifiro MA. Androgen insensitivity syndrome. Gene Reviews. 2004. Available at: http://www.geneclinics.org/profiles/androgen/details.html Houk CP, Lee PA. Intersexed states: diagnosis and management. Endocrinol Metab Clin North Am. 2005;34:791-810 Houk CP, Levitsky LL. Evaluation of the infant with ambiguous genitalia. UptoDate. 2006;14.1. Available at: http://www.utdol.com/utd/content/topic.do?topicKey=pediendo/11554&type=P&selectedTitle=14~ 24 Wilson BE. Androgen insensitivity syndrome. eMedicine Specialties: Pediatrics: Endocrinology. 2003. Available at: http://www.emedicine.com/ped/topic2222.htm

Copyright 2007 by the American Academy of Pediatrics

page 154

2007 PREP SA on CD-ROM Question: 75

You diagnose attention-deficit/hyperactivity disorder in a 10-year-old girl. She is growing well, and physical examination findings are normal. Of the following, the MOST appropriate test to confirm the diagnosis is

A. brain computed tomography B. brain magnetic resonance imaging C. electroencephalography D. no test at this time E. positron emission testing

Copyright 2007 by the American Academy of Pediatrics

page 155

2007 PREP SA on CD-ROM Critique: 75 Preferred Response: D

Attention-deficit/hyperactivity disorder (ADHD) is a common, yet heterogeneous disorder that has a complex etiology; genetic, environmental, and biologic factors all play roles. For example, ADHD is associated with both genetic syndromes such as fragile X and intrauterine toxic exposures such as fetal alcohol syndrome. It also has been shown in family and twin studies that ADHD is more common in close family members of those diagnosed with the disorder. Research continues into the neurobiologic basis of ADHD. Evidence is increasing that alterations in the frontal lobe and frontal subcortical connections play significant roles in the disorder. Affected children have difficulties with executive functioning, such as organization, impulse control, and inattention, that are common in other disorders involving impaired frontal lobe function. Neurodiagnostic studies such as magnetic resonance imaging, electroencephalography, computed tomography scan, and positron emission testing may be used in research settings, but they are not used routinely in the evaluation of children who have ADHD. Such studies may be appropriate if there are specific concerns regarding risk for seizure, brain malformation, or brain injury. References: American Academy of Pediatrics Committee on Quality Improvement, Subcommittee on Attention-Deficit/Hyperactivity Disorder. Clinical practice guideline: diagnosis and evaluation of the child with attention-deficit/hyperactivity disorder. Pediatrics. 2000;105:1158-1170. Available at: http://pediatrics.aappublications.org/cgi/content/full/105/5/1158 American Academy of Pediatrics Subcommittee on Attention-Deficit/Hyperactivity Disorder and Committee on Quality Improvement. Clinical practice guideline: treatment of the school-aged child with attention-deficit/hyperactivity disorder. Pediatrics. 2001;108:1033-1044. Available at: http://pediatrics.aappublications.org/cgi/content/full/108/4/1033 Reiff MI, Tippins S, LeTourneau AA. ADHD: A Complete and Authoritative Guide. Elk Grove Village, Ill: American Academy of Pediatrics; 2004 Sims MD. Attention-deficit/hyperactivity disorder. In: Behrman RE, Kliegman RM, Jenson HB, eds. Nelson Textbook of Pediatrics. 17th ed. Philadelphia, Pa: WB Saunders Co; 2004:107-110

Copyright 2007 by the American Academy of Pediatrics

page 156

2007 PREP SA on CD-ROM Question: 76

You have been treating a 2-year-old girl for pneumococcal meningitis for the past 5 days. Of the following, the MOST likely complication of her disease is

A. brain abscess B. cerebral infarct C. cranial nerve palsy D. hearing impairment E. sagittal sinus thrombosis

Copyright 2007 by the American Academy of Pediatrics

page 157

2007 PREP SA on CD-ROM Critique: 76 Preferred Response: D

Sensorineural hearing loss is the most common long-term sequela of bacterial meningitis. It has been described in up to 30% of patients who have pneumococcal disease and 10% of those who have meningococcal meningitis. The hearing loss is due to either a labyrinthitis or direct inflammation of the auditory nerve. Patients who have significant hearing loss often demonstrate significant truncal ataxia during their recovery because of middle ear inflammation. Other longterm sequelae include developmental or learning delays, behavioral problems, seizures, hydrocephalus, and cerebral palsy. To decrease the risk of long-term neurologic sequelae, the American Academy of Pediatrics recommends the administration of adjunctive therapy with dexamethasone for the treatment of bacterial meningitis due to Haemophilus influenzae type B and consideration of treatment for pneumococcal meningitis in infants and children older than 6 weeks of age. If dexamethasone is used, it should be given regardless of the clinical severity of the illness and administered as soon as possible in the course of treatment; best results occur when the first dose is administered prior to any antimicrobial therapy. Other acute complications of bacterial meningitis include the syndrome of inappropriate antidiuretic hormone secretion, seizures, and increased intracranial pressure. Fever may be present for 5 to 7 days during the illness or may disappear if dexamethasone therapy is used, only to return after the corticosteroids are stopped. Less common but more serious acute complications include infarction of the brain or spinal cord, cranial nerve palsy, cerebral herniation, thrombosis of the sagittal sinus, subdural effusions, and brain abscess formation. References: Prober CG. Central nervous system infections. In: Behrman RE, Kliegman RM, Jenson HB, eds. Nelson Textbook of Pediatrics. 17th ed. Philadelphia, Pa: WB Saunders Co; 2004:2038-2047 Koomen I, Grobbee DE, Roord JJ, Donders R, Jennekens-Schinkel A, Van Furth AM. Hearing loss at school age in survivors of bacterial meningitis: assessment, incidence, and prediction. Pediatrics. 2003;112:1049-1053. Available at: http://pediatrics.aappublications.org/cgi/content/full/112/5/1049 Feigin RD, Watson JT, Gerber SI. Use of corticosteroids in bacterial meningitis. Pediatr Infect Dis J. 2004;23:355-357

Copyright 2007 by the American Academy of Pediatrics

page 158

2007 PREP SA on CD-ROM Question: 77

You are evaluating a 7-year-old boy for hematuria and proteinuria. As part of the evaluation, you measure serum electrolytes. The serum creatinine is 1.1 mg/dL (97.2 mcmol/L). Of the following, the MOST accurate serum creatinine measurements for children of normal physical development are (All measurements mg/dL [mcmol/L])

A. 3 months old: 0.3 (26.5); 2 years old: 0.4 (35.4); 7 years old: 1.0 (88.4); 17 years old: 1.0
(88.4)

B. 3 months old: 0.6 (53.0); 2 years old: 0.8 (70.7); 7 years old: 1.0 (88.4); 17 years old: 1.2
(106.1)

C. 3 months old: 0.3 (26.5); 2 years old: 0.4 (35.4); 7 years old: 0.6 (53.1); 17 years old: 0.9
(79.6)

D. 3 months old: 0.6 (53.0); 2 years old: 0.4 (35.4); 7 years old: 0.7 (61.9); 17 years old: 0.7
(61.9)

E. 3 months old: 0.7 (61.9); 2 years old: 0.8 (70.7); 7 years old: 0.7 (61.9); 17 years old: 0.7
(61.9)

Copyright 2007 by the American Academy of Pediatrics

page 159

2007 PREP SA on CD-ROM Critique: 77 Preferred Response: C

The kidneys have a full complement of glomeruli and tubules at birth, but the nephrons grow during childhood, with most of the development occurring during the first 2 years. Thus, renal function may require up to 2 years to mature completely. Development of glomerular function is generally complete by age 2 years. Tubular function matures rapidly during the first year after birth, resulting in improved ability to reabsorb sodium, potassium, and water. In turn, the adaptability of the kidney to perturbations of normal body homeostasis (eg, dehydration) is more pronounced after the first postnatal year than during infancy. The level of the mothers serum creatinine generally determines the serum creatinine of the newborn in the first 7 to 10 postnatal days. The serum creatinine is relatively low during infancy and early childhood due to lower muscle mass (Item C77A). Additionally, blood flow to the kidneys is lower in early infancy due to increased peripheral resistance, thus reducing effective renal plasma flow. Taken together, these factors contribute to a lower serum creatinine and a relatively lower glomerular filtration rate (GFR) during early childhood. GFR is determined routinely from the serum creatinine, but there are several other methods to determine the GFR, including inulin clearance and measurement of serum cystatin A. References: Atiyeh BA, Dabbagh SS, Gruskin AB. Evaluation of renal function during childhood. Pediatr Rev. 1996;17:175-180 Bkenkamp A, Domanetzki M, Zinck R, Schumann G, Byrd D, Brodehl J. Cystatin Ca new marker of glomerular filtration rate in children independent of age and height. Pediatrics. 1998;101:875-881. Available at: http://pediatrics.aappublications.org/cgi/content/full/101/5/875 Schwartz GJ, Haycock GB, Edelmann CM, Spitzer A. A simple estimate of glomerular filtration rate in children derived from body length and plasma creatinine. Pediatrics. 1976;58:259-263. Abstract available at: http://pediatrics.aappublications.org/cgi/content/abstract/58/2/259

Copyright 2007 by the American Academy of Pediatrics

page 160

2007 PREP SA on CD-ROM Question: 78

A 6-month-old infant has been receiving high-dose amoxicillin therapy for bilateral otitis media. After 48 hours of therapy, she continues to be febrile, with a temperature of 102F (38.9C), and is irritable. Physical examination reveals erythematous, dull, and bulging tympanic membranes, with no movement on insufflation. Of the following, the MOST appropriate antibiotic to change this patient to is

A. azithromycin B. cefdinir C. clindamycin D. doxycycline E. trimethoprim-sulfamethoxazole

Copyright 2007 by the American Academy of Pediatrics

page 161

2007 PREP SA on CD-ROM Critique: 78 Preferred Response: B

The infant described in the vignette has otitis media that is not responding to high-dose amoxicillin therapy, suggesting that the causative pathogen is beta-lactamase-producing nontypeable Haemophilus sp or Moraxella catarrhalis. Accordingly, the child requires a betalactamase-resistant antibiotic, such as cefdinir, for treatment of her infection. Beta-lactamases are bacterial enzymes that hydrolyze the beta-lactam ring of beta-lactam antibiotics, inhibiting the ability of such antibiotics to interfere with cell wall polymerization of the penicillin-binding proteins, thereby decreasing the organisms susceptibility to the drugs. Betalactamase production is one of the most frequent mechanisms of antibiotic resistance used by gram-negative organisms such as Haemophilus sp, Moraxella catarrhalis, strains of Klebsiella pneumoniae, Escherichia coli, Salmonella sp, Pseudomonas aeruginosa, Enterobacter sp, Citrobacter sp, and Neisseria gonorrhoeae. Some of these organisms can produce extendedspectrum beta-lactamases (ESBLs) that can confer resistance to all penicillins, all cephalosporins, and aztreonam, making treatment of infections due to these organisms much more difficult. Testing for beta-lactamase production or ESBL-producing organisms is conducted in most clinical laboratories. The second- and third-generation oral cephalosporins are not affected by beta-lactamases produced by the most common otitis media pathogens. Clindamycin is effective only against gram-positive organisms and, therefore, is not effective against nontypeable Haemophilus sp or M catarrhalis. Doxycycline, although effective against these pathogens, should not be used in children younger than 8 years of age if other efficacious drugs are available. Azithromycin has activity against both nontypeable Haemophilus sp and M catarrhalis, but its activity is greatly decreased against beta-lactamase-producing isolates. The prevalence of resistance among bacteria causing otitis media and potential adverse effects make trimethoprim-sulfamethoxazole a less preferred agent. References: Christenson JC, Korgenski EK. Laboratory diagnosis of infection due to bacteria, fungi, parasites and rickettsiae. In: Long SS, Pickering LK, Prober CG, eds. Principles and Practice of Pediatric Infectious Diseases. 2nd ed. New York, NY: Churchill Livingstone, 2003:1380-1391 Doern GV, Jones RN. Antimicrobial susceptibility testing of Haemophilus influenzae, Branhamella catarrhalis, and Neisseria gonorrhoeae. Antimicrob Agents Chemother. 1988;32:1747-1753. Available at: http://www.pubmedcentral.gov/articlerender.fcgi?tool=pubmed&pubmedid=3149883 Samaha-Kfoury JN, Araj GF. Recent developments in beta lactamases and extended spectrum beta lactamases. BMJ. 2003;327:1209-1213. Available at: http://bmj.bmjjournals.com/cgi/content/full/327/7425/1209

Copyright 2007 by the American Academy of Pediatrics

page 162

2007 PREP SA on CD-ROM Question: 79

A medical student who is rotating in your clinic has just evaluated a 12-month-old girl who presented with a history of recurrent bacterial and viral infections. As part of your discussion with the medical student, you review the different aspects of the immune system and the evaluation of the infant's host defense. Of the following, the test that is the BEST measure of cell-mediated immunity is

A. Candida skin test B. complement 50 assay C. dihydrorhodamine flow cytometry D. isohemmaglutinins E. serum immunoglobulins (Ig) A, M, and G

Copyright 2007 by the American Academy of Pediatrics

page 163

2007 PREP SA on CD-ROM Critique: 79 Preferred Response: A

Primary care physicians evaluating children who have recurrent bacterial or viral infections need to maintain a high index of suspicion for underlying defects in the childs immune system. Host defense beyond the skin and gastrointestinal tract includes humoral, cellular, phagocytic, and complement. Investigation into one or more of these areas may be warranted, depending on the childs age, type and location of infections, and family medical history. A simple qualitative assessment of cell-mediated immunity that can be employed to assess the girl in the vignette is the Candida delayed-type hypersensitivity (DTH) intradermal skin test. Induration of 10 mm or greater at 48 hours virtually excludes all primary T-cell defects. Other antigens used for DTH testing include tetanus, mumps, and trichophyton. If DTH testing results are negative, other measurements of the cellular immune system, including lymphocyte counts, enumeration of T-cell subpopulations by flow cytometry, and lymphocyte stimulation testing, may be considered. Total complement assay (eg, complement 50) measures the intactness of the classic complement pathway. Low levels can result from a delay in specimen processing, early (C1 through C4) complement deficiencies, or late (C5 through C9) complement deficiencies. The dihydrorhodamine fluorescence (DHR 123) test is a flow cytometry-based oxidation assay that can measure neutrophil respiratory burst, a phagocytic function. In many centers, the DHR 123 test is replacing the nitroblue tetrazolium dye test for evaluation of chronic granulomatous disease. Isohemagglutinins and serum immunoglobulins are qualitative and quantitative measurements of the humoral immune system, respectively. Isohemagglutinins are antibodies to type A and B red blood cell polysaccharide antigens and normally are present in children ages 1 year and older who have blood type A, B, or O. Serum immunoglobulins (Igs) include IgG, IgA, and IgM. References: Ballow M, ONeil KM. Approach to the patient with recurrent infections. In: Adkinson NF Jr, Yunginger JW, Busse WW, Bochner BS, Holgate ST, Simons FER, eds. Middletons Allergy Principles and Practice. 6th ed. Philadelphia, Pa: Mosby; 2003:1043-1072 Buckley RH. Evaluation of suspected immunodeficiency. In: Behrman RE, Kliegman RM, Jenson HB, eds. Nelson Textbook of Pediatrics. 17th ed. Philadelphia, Pa: WB Saunders Co 2004:681682

Copyright 2007 by the American Academy of Pediatrics

page 164

2007 PREP SA on CD-ROM Question: 80

A frail 6-year-old child who has cystic fibrosis is transported by ambulance to the emergency department. She has had hemoptysis for the past 4 hours, yielding approximately 10 mL of bright red blood. She has had increased cough over the past 3 days. Physical examination findings include a respiratory rate of 38 breaths/min, heart rate of 90 beats/min, oxygen saturation of 92% on room air, blood pressure of 100/70 mm Hg, and temperature of 98.6F (37C). She is awake and alert but seems breathless when she tries to speak. On auscultation, you note diffuse crackles throughout her lung fields. Of the following, the BEST next step in the management of this patient is to

A. administer methylprednisolone B. begin therapy with ceftriaxone C. insert an endotracheal tube and begin positive pressure ventilation D. obtain blood for determination of prothrombin and partial thromboplastin time E. transfuse with O-negative blood

Copyright 2007 by the American Academy of Pediatrics

page 165

2007 PREP SA on CD-ROM Critique: 80 Preferred Response: D

Acute pulmonary hemorrhage, or hemoptysis, is an uncommon problem in pediatrics. Acute lower respiratory tract infection is the leading cause today, accounting for 40% or more of cases. Other causes include cystic fibrosis and congenital heart disease, both of which can manifest as recurrent bleeding. In children younger than 4 years of age, foreign body aspiration should be considered. Other, less common causes include bronchiectasis, clotting disorders, and trauma. Unlike in adults, neoplasm is an uncommon cause of hemoptysis in children. Massive hemoptysis is defined as acute bleeding of more than 240 mL in 24 hours. The first step in the evaluation of a child who has hemoptysis is to determine the source of the bleeding. Bleeding from the upper airway (epistaxis) and bleeding from the upper gastrointestinal tract (hematemesis) sometimes can be mistaken for hemoptysis. Blood produced via hemoptysis usually is bright red and frothy, in contrast to the dark or coffee ground material produced in hematemesis. The pH of the material also can be helpful: an alkaline pH is found in hemoptysis, and an acidic pH is found in hematemesis. Epistaxis generally can be established after careful examination of the oropharynx and nasopharynx. Initial laboratory tests should include a complete blood count to assess for blood loss. Coagulation studies, such as prothrombin and partial thromboplastin time, to rule out a coagulation defect also are helpful, particularly because patients who have cystic fibrosis, such as the girl in the vignette, may have derangements in their ability to clot due to vitamin K deficiency or liver disease. Other initial laboratory tests to consider include serum chemistry determinations to assess for renal pathology, urinalysis, and a sputum sample for Gram stain. A chest radiograph also should be ordered to aid in localizing the bleeding, although films appear normal in up to one third of patients. The source of the bleeding for the child in the vignette likely is either pulmonary infection or erosion of a blood vessel due to bronchiectasis. In neither case would methylprednisolone be of benefit. Initial therapy with antibiotics is appropriate only after collection of blood and sputum samples if pneumonia is suspected. However, ceftriaxone is not effective therapy against the expected pathogens. Most hemoptysis in children resolves spontaneously without the need for invasive measures. The girl in the vignette has not had massive hemoptysis and has adequate oxygen saturation on room air, so endotracheal intubation is not indicated. Transfusion with Onegative blood is indicated only for hypotension due to acute massive blood loss. Patients whose hemoptysis does not resolve spontaneously or who experience marked blood loss may require bronchoscopy to determine the source of the bleeding. Additional management options include bronchial washings with 1:10,000 epinephrine, endoscopic tamponade with a balloon catheter, or partial lung resection. References: Flume PA, Yankaskas JR, Ebeling M, Hulsey T, Clark LL. Massive hemoptysis in cystic fibrosis. Chest. 2005;128:729-738. Abstract available at: http://www.ncbi.nlm.nih.gov/entrez/query.fcgi?orig_db=PubMed&db=PubMed&cmd=Search&ter m=Chest[Jour]+AND+729[page]+AND+2005[pdat] Pianosi P, Al-sadoon H. Hemoptysis in children. Pediatr Rev. 1996;17:344-348 Sidman JD, Wheeler WB, Cabalka AK, Soumekh B, Brown CA, Wright GB. Management of acute pulmonary hemorrhage in children. Laryngoscope. 2001; 111:33-35. Abstract available at: http://www.ncbi.nlm.nih.gov/entrez/query.fcgi?orig_db=PubMed&db=PubMed&cmd=Search&ter m=Laryngoscope[Jour]+AND+33[page]+AND+2001[pdat] Sloniewsky D, Green TP. Pulmonary hemorrhage, embolism and infarction. In: Behrman RE, Kliegman RM, Jenson HB, eds. Nelson Textbook of Pediatrics. 17th ed. Philadelphia, Pa: Saunders; 2004:1457-1458

Copyright 2007 by the American Academy of Pediatrics

page 166

2007 PREP SA on CD-ROM Question: 81

A 1-month-old infant was born with an extensive erythematous patch (Item Q81A) on the right side of his face that has grown at a rate matching his somatic growth. Of the following, the child is at GREATEST risk for

A. congestive heart failure B. consumptive coagulopathy C. glaucoma D. ocular axis occlusion E. tethered spinal cord

Copyright 2007 by the American Academy of Pediatrics

page 167

2007 PREP SA on CD-ROM Critique: 81 Preferred Response: C

Port-wine stains (PWSs) are permanent capillary vascular malformations that are present from birth, as reported for the infant in the vignette. Unlike hemangiomas, PWSs do not proliferate, but grow proportionally with the childs somatic growth. Although a PWS may occur anywhere on the body, facial lesions should raise concern about possible Sturge-Weber syndrome (SWS). SWS is the association of a facial PWS in the V1 distribution with central nervous system leptomeningeal angiomatosis (that may cause seizures) and/or glaucoma. One study found that only 8% of individuals who have a facial PWS have SWS; all of those affected had a PWS in the distribution of V1 (upper lid) and/or V2 (lower lid). The risk of SWS was increased if the facial PWS was bilateral or if unilateral, involved the distribution of all three branches of the trigeminal nerve. Congestive heart failure is not associated with SWS; it is an occasional complication in children who have multiple cutaneous hemangiomas and liver involvement or in those who have arteriovenous malformations. Consumptive coagulopathy (along with thrombocytopenia and anemia) is a feature of Kasabach-Merritt syndrome (Item C81A), in which patients have an atypical-appearing hemangioma (actually a hemangioepithelioma or tufted angioma). Ocular axis occlusion that prevents stimulation of the visual cortex is a concern with enlarging hemangiomas located near the eye (Item C81B). A tethered spinal cord results when a thickened, ropelike filum terminale remains firmly attached, reducing cord mobility. Tension on the cord and compromised blood supply produce neurologic findings, such as abnormal gait and secondary enuresis. A midline lumbosacral PWS might herald the existence of this or another occult spinal lesion. References: Frieden I, Enjolras O, Esterly N. Vascular birthmarks, other abnormalities of blood vessels and lymphatics. In: Schachner LA, Hansen RC, eds. Pediatric Dermatology. 3rd ed. St. Louis, Mo: Mosby, 2003:833-862 Halsam RHA. Tethered cord. In: Behrman RE, Kliegman RM, Jenson HB, eds. Nelson Textbook of Pediatrics. 17th ed. Philadelphia, Pa: WB Saunders Co; 2004:2050 Weston WL, Lane AT, Morelli JG. Vascular lesions: vascular tumors and malformations. In: Color Textbook of Pediatric Dermatology. 3rd ed. St. Louis, Mo: Mosby; 2002:187-199

Copyright 2007 by the American Academy of Pediatrics

page 168

2007 PREP SA on CD-ROM Question: 82

A 14-year-old overweight girl developed an episode of abdominal pain and jaundice 1 week ago. At that time, total bilirubin measured 8 mg/dL (136.8 mcmol/L) and direct bilirubin measured 5 mg/dL (85.5 mcmol/L). She was admitted to the hospital, where ultrasonography demonstrated a dilated common bile duct and gallbladder stones. She underwent endoscopic retrograde cholangiopancreatography (ERCP), and a gallstone was extracted from her common bile duct. She presents to your office today for follow-up and is asymptomatic. Physical examination demonstrates a normal-size liver and spleen without tenderness. Repeat laboratory studies reveal a total bilirubin of 2.5 mg/dL (42.8 mcmol/L) and direct bilirubin of 1.5 mg/dL (25.7 mcmol/L). Transaminase measurements are normal. Of the following, the MOST appropriate next step is to

A. evaluate for autoimmune hepatitis B. initiate treatment with cholic acid C. obtain hepatitis B serologic studies D. repeat the ERCP E. repeat the bilirubin measurement in 2 weeks

Copyright 2007 by the American Academy of Pediatrics

page 169

2007 PREP SA on CD-ROM Critique: 82 Preferred Response: E

Hepatobiliary obstruction is characterized by elevation of three laboratory parameters: conjugated (direct) bilirubin, alkaline phosphatase, and gamma-glutamyl transpeptidase. None of these markers is completely specific for hepatobiliary obstruction. For example, direct bilirubin can be elevated by intrahepatic cholestasis (as seen in Alagille syndrome and chlorpromazine toxicity), alkaline phosphatase can be raised by bone disease, and gamma-glutamyl transpeptidase can be increased by anticonvulsants and alcohol consumption. Therefore, if biliary obstruction is suspected, hepatobiliary ultrasonography should be performed to evaluate for bile duct dilation, gallstones, and hepatobiliary sludge. Gallstones composed primarily of calcium bilirubinate are referred to as pigment stones; those composed primarily of cholesterol are termed cholesterol stones. "Biliary sludge" is an amorphous material composed of cholesterol, mucin, and bilirubinate that is thicker than regular bile but not as hard as a stone. Sludge in the common bile duct can contribute to both obstructive jaundice and pancreatitis. The patient described in the vignette had a recent gallstone removed endoscopically. Generally, such patients can be followed clinically, and repeat laboratory studies are not necessary if the patient is asymptomatic. This girls bilirubin concentration is mildly elevated, which is not uncommon after a recent episode of obstructive jaundice; repeat measurement in 2 weeks can determine whether the hyperbilirubinemia has resolved. Repeat endoscopic retrograde cholangiopancreatography is not necessary at this time, although it should be considered if there is evidence of another retained stone. Transaminase values are normal, making both autoimmune and viral hepatitis unlikely. For a patient who has jaundice due to persistent biliary sludge, oral administration of the synthetic bile acid ursodeoxycholic acid may help facilitate biliary flow and promote sludge dissolution. Cholic acid generally is not used in the chemical dissolution of biliary sludge and gallstones. References: Dourakis SP, Mayroyannis C, Alexopoulou A, Hadziyannis SJ. Prolonged cholestatic jaundice after endoscopic retrograde cholangiography. Hepatogastroenterology. 1997;44:677-680. Abstract available at: http://www.ncbi.nlm.nih.gov/entrez/query.fcgi?cmd=Retrieve&db=pubmed&dopt=Abstract&list_ui ds=9222670&query_hl=21&itool=pubmed_docsum Lee SP, Nicholls JF, Park HZ. Biliary sludge as a cause of acute pancreatitis. N Engl J Med. 1992;326:589-593. Abstract available at: http://www.ncbi.nlm.nih.gov/entrez/query.fcgi?cmd=Retrieve&db=pubmed&dopt=Abstract&list_ui ds=1734248&query_hl=24&itool=pubmed_docsum Pratt DS, Kaplan MM. Evaluation of abnormal liver-enzyme results in asymptomatic patients. N Engl J Med. 2000;342:1266-1271. Available at: http://content.nejm.org/cgi/content/extract/342/17/1266

Copyright 2007 by the American Academy of Pediatrics

page 170

2007 PREP SA on CD-ROM Question: 83

A 30 weeks' gestation infant weighing 1,500 g experiences respiratory distress that requires assisted ventilation in the first 4 hours after birth. Some tachypnea with mild subcostal and intercostal retractions is evident on physical examination. Current ventilator parameters are: positive end-expiratory pressure of 4 cm H2O, peak inspiratory pressure of 22 cm H2O, synchronized ventilator rate of 40 breaths/min, and fraction of inspired oxygen of 0.8 to maintain a Pao2 of 60 mm Hg. A chest radiograph reveals low lung volumes, diffuse microatelectasis, and dense ground-glass opacities. You order surfactant for intratracheal administration. Of the following, the MOST expected event in the 4 hours following surfactant administration is

A. decreased oxygen requirement B. decreased pulmonary compliance C. hypotension D. increased inspiratory pressure E. pulmonary hemorrhage

Copyright 2007 by the American Academy of Pediatrics

page 171

2007 PREP SA on CD-ROM Critique: 83 Preferred Response: A

Exogenous surfactant therapy for neonatal respiratory distress syndrome (RDS) is perhaps the greatest example of bench-to-bedside research that has benefited newborns. Since the late 1980s, when clinical trials demonstrated the efficacy and safety of intratracheally administered surfactant, it has become the standard therapy, in conjunction with assisted ventilation and judicious oxygen therapy, in treating millions of newborns who have RDS. Upon diagnosing RDS in a preterm infant by clinical, radiographic, and blood gas parameters, early administration of surfactant results in obvious improvement in each of these broad categories. Clinically, treated infants demonstrate less work of breathing, improved oxygenation and ventilation, a reduction in assisted ventilation (apparent by diminished Fio2), decreased mean airway pressure (most readily accomplished by a reduction in inspiratory pressure), and improved pulmonary compliance (reflected in improved changes in lung volume per unit of inspiratory pressure administered). Radiographically, the degree of lung aeration improves as microatelectasis is overcome, and lung volumes typically improve. Blood gas measurements indicate improved ventilation and oxygenation and resolution of acidemia. Hypotension is a complication of RDS in the critically ill newborn and may reflect impaired circulatory volume, venous return to the heart (preload), and cardiac output. With significant hypoxemia and acidemia, myocardial function may be impaired. Additionally, the ductus arteriosus may remain patent as a result of poor neonatal oxygenation. These conditions are not directly corrected by surfactant administration, but they are ameliorated as lung disease improves. Pulmonary hemorrhage is an infrequent adverse effect of surfactant therapy that is seen most commonly in the extremely low-birthweight infant (<1,000 g) and is associated with a patent ductus arteriosus. References: American Academy of Pediatrics Committee on Fetus and Newborn. Surfactant replacement therapy for respiratory distress syndrome. Pediatrics. 1999;103: 684-685. Available at: http://pediatrics.aappublications.org/cgi/content/full/103/3/684 Bancalari E, Claure N, Gonzalez A. Patent ductus arteriosus and respiratory outcome in premature infants. Biol Neonate. 2005;88:192-201. Abstract available at: http://www.ncbi.nlm.nih.gov/entrez/query.fcgi?cmd=Retrieve&db=pubmed&dopt=Abstract&list_ui ds=16210841&query_hl=30&itool=pubmed_docsum Halliday HL. History of surfactant from 1980. Biol Neonate. 2005;87:317-322. Available at: http://content.karger.com/produktedb/produkte.asp?typ=fulltext&file=BON2005087004317 Philip AGS, Avery ME. Historical perspectives: surfactant deficiency to surfactant use. NeoReviews. 2002;3:e239-e242. Available at: http://neoreviews.aappublications.org/cgi/content/full/3/12/e239 Rodriguez RJ, Martin RJ, Fanaroff AA. Respiratory distress syndrome and its management. In: Martin RJ, Fanaroff AA, Walsh MC, eds. Fanaroff and Martins Neonatal-Perinatal Medicine: Diseases of the Fetus and Infant. 8th ed. Philadelphia, Pa: Mosby-Elsevier; 2006:1097-1107

Copyright 2007 by the American Academy of Pediatrics

page 172

2007 PREP SA on CD-ROM Question: 84

You are examining a 2-day-old term newborn prior to discharge. On physical examination, you note a 5 x 5 cm mass in the right abdomen. The remainder of her physical examination findings are normal. Of the following, the MOST likely diagnosis is

A. hydrometrocolpos B. lymphoma C. multicystic dysplastic kidney D. neuroblastoma E. Wilms tumor

Copyright 2007 by the American Academy of Pediatrics

page 173

2007 PREP SA on CD-ROM Critique: 84 Preferred Response: C

The differential diagnosis for a child who has an abdominal mass differs according to age. In the neonate, the most common causes of abdominal or flank mass are hydronephrosis and multicystic dysplastic kidney. Hydronephrosis may be unilateral, as in ureteropelvic or ureterovesicular junction obstruction, or bilateral, as in posterior urethral valves. Multicystic dysplastic kidney presents with a unilateral flank mass, as reported for the newborn in the vignette. The diagnosis can be supported with renal ultrasonography, which shows multiple cysts and little renal parenchyma. Hydrometrocolpos may result from imperforate hymen. Affected infants may present with a midline lower abdominal mass that represents accumulation of mucus and other fluid in the vagina. Other, less common causes of abdominal mass in the neonate include solid renal tumors, adrenal hemorrhage, intestinal duplication cyst, and renal vein thrombosis. Lymphoma is very rare in infants younger than 1 year of age. Neuroblastoma may present with an abdominal mass in the neonatal period, but it is far more common later in infancy and toddlerhood. Wilms tumor presents with a unilateral flank mass, but it is rarely diagnosed before 6 months of age. References: Chandler JC, Gauderer MW. The neonate with an abdominal mass. Pediatr Clin North Am. 2004;51:979-997. Abstract available at: http://www.ncbi.nlm.nih.gov/entrez/query.fcgi?orig_db=PubMed&db=PubMed&cmd=Search&ter m=%22Pediatric+clinics+of+North+America%22[Jour]+AND+2004[pdat]+AND+Chandler[author] Jaffe N, Huff V. Neoplasms of the kidney. In: Behrman RE, Kliegman RM, Jenson HB, eds: Nelson Textbook of Pediatrics. 17th ed. Philadelphia, Pa: WB Saunders Co; 2004:1711-1714

Copyright 2007 by the American Academy of Pediatrics

page 174

2007 PREP SA on CD-ROM Question: 85

The parents of a child who was diagnosed at birth with Beckwith-Wiedemann syndrome bring in the baby for his 2-month evaluation. They ask about future health problems and his prognosis now that his omphalocele has been repaired. Of the following, the child is MOST at risk for

A. acute lymphocytic leukemia B. astrocytoma C. Hodgkin disease D. rhabdomyosarcoma E. Wilms tumor

Copyright 2007 by the American Academy of Pediatrics

page 175

2007 PREP SA on CD-ROM Critique: 85 Preferred Response: E

Children who have hemihypertrophy and other somatic overgrowth syndromes are at higher risk of developing Wilms tumor and hepatoblastomas than the general population. BeckwithWiedemann syndrome, which involves a chromosome 11p15.5 gene deletion, is characterized by somatic overgrowth and macroglossia (Item C85A). Affected children have a 3% to 5% risk of developing Wilms tumor compared with an incidence of about 8.5 per 1 million children in the general population. Wilms tumor occurs in hereditary and nonhereditary patterns. It presents as an asymptomatic abdominal mass in two thirds of cases. Surveillance for the development of this tumor should include measurement of serum alpha-fetoprotein, urinalysis for microscopic hematuria, and abdominal ultrasonography every 3 months for at least the first 5 years after birth. The most classic association with Wilms tumor is the WAGR syndrome (Wilms, aniridia, genitourinary anomalies, and mental retardation), which is related to chromosome 11p13 deletion. Although acute lymphocytic leukemia, astrocytoma, Hodgkin disease, and rhabdomyosarcoma are relatively common neoplasms in children, none is chromosomally related to the Beckwith-Wiedemann syndrome. References: Chintagumpala MM, Steuber CP. Wilms tumor. In: McMillan JA, DeAngelis CD, Feigin RD, Warshaw JB, eds. Oskis Pediatrics: Principles and Practice. 3rd ed. Philadelphia, Pa: Lippincott Williams & Wilkins; 1999:1515-1516

Copyright 2007 by the American Academy of Pediatrics

page 176

2007 PREP SA on CD-ROM Question: 86

A 4-year-old girl presents to the emergency department with a 3-day history of left-sided weakness. On physical examination, she has a temperature of 99F (37.2C), pulse of 50 beats/min, respiratory rate of 24 breaths/min, and blood pressure of 118/78 mm Hg. She cannot abduct her right eye, but has normal funduscopic examination results. Strength on the left is 4/5 in the upper and lower extremity and on the right is normal. Of the following, the MOST appropriate next step in the evaluation of this child is

A. cerebral angiography B. computed tomography scan of the brain C. electroencephalography D. positron emission tomography scan of the head E. single-photon emission computed tomography scan of the brain

Copyright 2007 by the American Academy of Pediatrics

page 177

2007 PREP SA on CD-ROM Critique: 86 Preferred Response: B

Increased intracranial pressure (ICP) is an emergency that must be recognized quickly by pediatricians. Elevated ICP often is insidious and nonspecific initially. Among affected school-age children, declining academic performance, fatigue, behavioral changes, and vague intermittent headaches are common. Over time, morning headaches, especially pain at the occipital or frontal region, along with vomiting and lethargy ensue. Horizontal diplopia becomes apparent once the abducens nerve becomes compressed by the tentorium. Papilledema may develop if the pressure is longstanding, but the optic fundus can appear normal early in the course of increased ICP. Hypertension with bradycardia and variable respiratory pattern (Cushing triad) may indicate critically elevated ICP that is nearing cerebral herniation. Irritability, anorexia, failure to thrive, and even developmental regression can be early signs of increased ICP in infants. Increased ICP may lead to macrocephaly, splitting of the cranial sutures, or a bulging anterior fontanelle. The setting sun sign, a downward deviation of the eyes, may be seen with attendant hydrocephalus. Once there is concern about elevated ICP, immediate neuroimaging is required with computed tomography (CT) scan of the head or magnetic resonance imaging (MRI) of the brain. CT scan without contrast can be performed in just minutes, without conscious sedation. Although MRI provides better definition of the underlying pathology, such neuroimaging takes about 30 or more minutes and may require sedation. Either modality can determine whether there is increased ICP from a mass lesion. The hypertension, bradycardia, and right abducens paresis reported for the child in the vignette are worrisome for increased ICP, and she should receive a CT scan of the brain for evaluation. Cerebral angiography only delineates the intracranial vasculature. Positron emission tomography and single-photon CT scan determine whether a lesion is metabolically active (eg, tumor or infection). Single-photon CT scan may be useful in identifying the structural origin of epilepsy immediately following a seizure. Electroencephalography is used to evaluate seizures and level of consciousness, but has limited, if any, utility in the delineation of mass lesions of the brain. References: DiCarlo JV, Frankel LR. Stabilization of the critically ill child: neurologic stabilization. In: Behrman RE, Kliegman RM, Jenson HB, eds. Nelson Textbook of Pediatrics. 17th ed. Philadelphia, Pa: WB Saunders Co; 2004:308-311 Larsen GY, Goldstein B. Consultation with the specialist: increased intracranial pressure. Pediatr Rev. 1999;20:234-239. Available at: http://pedsinreview.aappublications.org/cgi/content/full/20/7/234

Copyright 2007 by the American Academy of Pediatrics

page 178

2007 PREP SA on CD-ROM Question: 87

A 12-year-old girl presents to your office for the first time with a swollen, painful, erythematous right knee joint. She tells you that her left knee felt and looked similar yesterday, but now feels normal. She also is easily fatigued and has had fever. On physical examination, she has a temperature of 101.7F (38.7C), a heart rate of 125 beats/min, a respiratory rate of 24 breaths/min, and a blood pressure of 120/78 mm Hg. Her lungs are clear. On auscultation, you note a 3/6 holosystolic murmur (Item Q87A) at the cardiac apex with radiation to the axilla. Of the following, the BEST plan for management of this patient's joint swelling includes

A. antibiotic therapy with doxycycline B. anti-inflammatory therapy with aspirin C. aspiration of the right knee joint D. heat, elevation, and splinting of the right knee E. immunotherapy with azathioprine

Copyright 2007 by the American Academy of Pediatrics

page 179

2007 PREP SA on CD-ROM Critique: 87 Preferred Response: B

Rheumatic fever follows infections caused by certain strains of group A streptococci. This inflammatory disease affects the heart, central nervous system, joints, subcutaneous tissue, and skin. Rheumatic fever is a significant cause of cardiovascular morbidity in developing countries and continues to be seen in the United States, despite decades of declining incidence. The disease can be viewed as a nonsuppurative complication of group A streptococcal infections of the upper respiratory tract that typically occurs in children between the ages of 4 and 15 years. Patients who have had rheumatic fever in the past have an increased incidence of recurrent disease. The pathognomonic cardiac lesion in rheumatic fever is the Aschoff body, which is a perivascular infiltrate of large cells that have polymorphous nuclei and basophilic cytoplasm and are arranged in a rosette pattern. In 1992, the American Heart Association presented guidelines for the diagnosis of rheumatic fever that included an update of the original Jones criteria. The clinical manifestations of rheumatic fever can be divided into major criteria and minor criteria. The major manifestations of the disease are: Carditis Polyarthritis Subcutaneous nodules Erythema marginatum Chorea Minor manifestations are: Arthralgia (not considered a criterion if polyarthritis is present) Fever Increased acute-phase reactants (erythrocyte sedimentation rate, C-reactive protein) Prolonged P-R interval The diagnosis requires two of the major manifestations or one of the major manifestations plus two of the minor manifestations. The carditis of rheumatic fever may involve any of the three layers of the heart: the endocardium, myocardium, or pericardium. If all three layers are involved, pancarditis exists. Patients who have carditis may not have symptoms, but they likely have signs on cardiac examination. When signs or symptoms are present, they most often are the result of endocarditis, which is caused by hyaline degeneration of the affected valve. The mitral valve is the most commonly affected valve, followed by the aortic valve. It is rare for the tricuspid or pulmonary valve to be affected. Therefore, the clinician should auscultate for the apical, holosystolic murmur that radiates to the left axilla and represents mitral regurgitation (Item C87A). If the mitral regurgitation is severe, there may be a low-frequency mid- to late diastolic murmur of relative mitral stenosis, which is referred to as the Carey Coombs murmur. The clinician also should listen for the early diastolic decrescendo murmur of aortic regurgitation (Item C87B), which may be exaggerated when the patient is in the sitting position, leaning forward. Signs and symptoms of congestive heart failure occur in about 5% of patients who have rheumatic carditis. The patient described in the vignette has fever and two major Jones criteria: migratory arthritis and carditis represented by mitral regurgitation. Therefore, she meets the diagnostic criteria for rheumatic fever. The acute management of rheumatic fever encompasses three primary strategies: treat the infection causing the disease, control and alleviate the symptoms, and provide supportive care as indicated. Because rheumatic fever is caused by group A betahemolytic streptococci, antibiotic therapy should be directed at eradication of this pathogen.

Copyright 2007 by the American Academy of Pediatrics

page 180

2007 PREP SA on CD-ROM

Typically, a 10-day course of oral penicillin V is adequate. Alternatively, intramuscular or intravenous penicillin preparations may be used. For those who have a penicillin allergy, an alternate antibiotic such as erythromycin may be used. Doxycycline, a treatment for Lyme arthritis, is not the best choice for management of arthritis resulting from acute rheumatic fever. Anti-inflammatory agents are important for symptomatic relief and to combat the inflammatory process. Salicylates are particularly effective for the migratory arthritis, with improvement often occurring in the first 12 to 24 hours of therapy. In addition, these medications frequently relieve the accompanying fever. Aspirin typically is administered in high doses (80 to 100 mg/kg per day to a serum level of 25 mg/dL) for several weeks before a gradual tapering is begun. For patients who cannot tolerate aspirin, the use of other nonsteroidal anti-inflammatory drugs may be considered. Steroids generally are reserved for patients who demonstrate moderate-to-severe carditis with congestive heart failure. Supportive care has historically included the use of bed rest, especially during the acute phase of the illness. Joint aspiration, immunotherapy, and elevation and immobilization of the right knee do not address the underlying problem for the patient in the vignette. References: Ayoub EM. Rheumatic fever. In: Moller JH, Hoffman JIE, eds. Pediatric Cardiovascular Medicine. Philadelphia, Pa: Churchill Livingstone; 2000:740-751 Dajani A, Taubert K, Ferrieri P, Peter G, Shulman S. Treatment of acute streptococcal pharyngitis and prevention of rheumatic fever: a statement for health professionals. Committee on Rheumatic Fever, Endocarditis, and Kawasaki Disease of the Council on Cardiovascular Disease in the Young, American Heart Association. Pediatrics. 1995;96:758-764. Abstract available at: http://pediatrics.aappublications.org/cgi/content/abstract/96/4/758 Ferrieri P. Proceedings of the Jones Criteria workshop. Circulation. 2002;106:2521-2523. Available at: http://circ.ahajournals.org/cgi/content/full/106/19/2521 Hoffman J. Rhuematic fever. In: Rudolph CD, Rudolph AM, Hostetter MK, Lister G, Siegel NJ, eds. Rudolphs Pediatrics. 21st ed. New York, NY: McGraw Hill; 2003:1901-1904 Special Writing Group of the Committee on Rheumatic Fever, Endocarditis, and Kawasaki Disease of the Council of Cardiovascular Disease in the Young of the American Heart Association. Guidelines for the diagnosis of rheumatic fever. Jones criteria, 1992 update. JAMA. 1992;268;2069-2073. Abstract available at: http://www.ncbi.nlm.nih.gov/entrez/query.fcgi?orig_db=PubMed&db=PubMed&cmd=Search&ter m=%22JAMA+:+the+journal+of+the+American+Medical+Association%22[Jour]+AND+2069[page] +AND+1992[pdat]

Copyright 2007 by the American Academy of Pediatrics

page 181

2007 PREP SA on CD-ROM Question: 88

You are discussing the common indications for chromosome analysis with a group of third-year medical students. Of the following, the MOST appropriate statement to include in your discussion is that

A. a blood karyotype should be obtained in any newborn who has multiple congenital anomalies
and growth retardation

B. a blood karyotype should be obtained only for a girl who has short stature if a buccal smear is
negative

C. chromosome analysis is not necessary to confirm the diagnosis of Down syndrome if the
major clinical features are present

D. chromosome analysis must be obtained to assess the reproductive risk for a woman who has
a sibling who has trisomy 21

E. routine chromosome analysis is adequate for the diagnosis of microdeletion syndromes, such
as DiGeorge syndrome

Copyright 2007 by the American Academy of Pediatrics

page 182

2007 PREP SA on CD-ROM Critique: 88 Preferred Response: A

Blood chromosome analysis can detect both abnormalities in chromosome number and gross structural defects of individual chromosomes. The numerical abnormalities can involve extra sets of all the chromosomes (eg, triploidy), the presence of a single extra chromosome (eg, trisomy), or the absence of a chromosome (eg, monosomy). Structural changes include deletions, duplications, translocations, and inversions. In general, chromosomal defects are associated with a combination of dysmorphic features; multiple congenital anomalies; growth deficiency; and variable degrees of developmental delay, learning problems, and mental retardation. Therefore, it is appropriate to obtain a chromosome analysis in any newborn who has multiple congenital anomalies and growth retardation. Turner syndrome (TS) results from complete absence or structural abnormality of an X chromosome. Growth retardation is one feature of the syndrome. Although features of TS often include a webbed neck, congenital heart disease, and ovarian dysgenesis, affected girls may have short stature alone. Because a buccal smear is not a reliable screening test for TS, all patients in whom TS is suspected should have a blood karyotype. Chromosome analysis should be obtained in every infant suspected of having Down syndrome, which can result from trisomy 21 or structural rearrangements involving chromosome 21. In particular, it is important to determine if there is a translocation involving chromosome 21 because one of the parents also may carry the structural rearrangement in a balanced form. In this case, the risk to future pregnancies is increased markedly. The sibling of an individual who has trisomy 21 is not at any additional risk for having a child who has a chromosome abnormality. Microdeletion syndromes such as 22q11 syndrome usually cannot be detected on routine chromosome analysis. Therefore, it is recommended that fluorescent in situ hybridization studies be carried out using a molecular probe for the precise area of interest to determine if a microdeletion is present. References: Hoyme HE. Patterns of inheritance. In: Behrman RE, Kliegman RM, Jenson HB, eds. Nelson Textbook of Pediatrics. 17th ed. Philadelphia Pa: Saunders; 2004:376-382 Hsu LYF. Cytogenetic abnormalities in the embryo, fetus, and infant. UpToDate. 2006;14.1. Available at: http://www.utdol.com/utd/content/topic.do?topicKey=antenatl/5719&type=P&selectedTitle=66~81 Pinsky L. Overview of genetic assessment. UpToDate. 2006; 14.1. Available at: http://www.utdol.com/utd/content/topic.do?topicKey=genr_med/27636&type=A&selectedTitle=1~ 85 Roizen NJ, Stark AR. Epidemiology and genetics of Down syndrome. UpToDate. 2006:14.1. Available at: http://www.utdol.com/utd/content/topic.do?topicKey=dis_chld/2412&type=P&selectedTitle=27~54

Copyright 2007 by the American Academy of Pediatrics

page 183

2007 PREP SA on CD-ROM Question: 89

During a sports physical, you note that a 14-year-old boy has thoracic kyphosis (Item Q89A). He cannot pull his shoulders back to correct the curvature in the standing position. The kyphosis persists in the prone position. A standing lateral radiograph reveals a kyphotic curve of 50 degrees between T3 and T12, with anterior wedging of vertebrae at T7 and T8 and T9. The patient is at Risser stage 4. Of the following, the MOST appropriate management plan for this patient is to

A. follow up the patient in 6 months B. obtain pulmonary function tests C. obtain renal ultrasonography D. obtain tissue typing for human leukocyte antigen B27 E. recommend rigorous hyperextension exercises

Copyright 2007 by the American Academy of Pediatrics

page 184

2007 PREP SA on CD-ROM Critique: 89 Preferred Response: A

Kyphosis refers to the convex alignment in the sagittal plane of the thoracic spine. The normal range of convexity is 20 to 40 degrees. Those who have an increased convex alignment have a clinical sign of round back that may be flexible (postural) or structural. A structural, nonflexible, round back is called idiopathic kyphosis or Scheuermann disease. A round back commonly is noticed during observations of adolescent posture. During the physical examination, the round back is viewed best from the side with the patient in the forward bending position. Affected patients who have a flexible kyphosis can correct the posture actively with hyperextension in both the standing and prone positions. Although the posture often is a concern of parents, such flexible kyphosis has no associated adverse health consequences. Scheuermann disease, as exhibited by the patient described in the vignette, is a fixed kyphosis (Item C89A) that develops around the time of puberty, most commonly in the thoracic region, and affects 7% to 8% of the population. Affected patients cannot correct the round back appearance with active hyperextension. Other findings on physical examination include an accentuated dorsal kyphosis with a sharp contour (or steep apex), a compensatory lumbar lordosis with a protuberant abdomen, and less frequently, a compensatory lordosis of the cervical spine. Neurologic examination findings generally are normal. A definitive diagnosis of Scheuermann disease of the thoracic spine is based on an increased kyphotic angle and the pathognomonic finding on the standing lateral radiograph of anterior wedging (Item C89B) of at least 5 degrees in three or more adjacent vertebrae. The standard radiographic evaluation should include both standing anteroposterior and lateral radiographs of the entire spine. Patients who have Scheuermann kyphosis also may have spondylolisthesis and scoliosis, which are treated as separate conditions. Treatment of Scheuermann disease depends on several factors, including the age of the patient, degree of skeletal maturity, degree of deformity, progression of the deformity, pain in the apical region of the kyphosis, and cosmetic and psychosocial factors. Neurologic complications and cardiopulmonary complaints are rare. Symptoms, including pain, usually are associated with more severe, often short-segment deformities that are greater than 90 degrees. Fortunately, Scheuermann disease is primarily a cosmetic problem for most patients. Treatment decisions are individualized in consultation with an orthopedic surgeon; there are no absolute guidelines. The patient described in the vignette is asymptomatic and skeletally mature (Risser stage 4 [indicating near-complete fusion of the iliac crest apophysis]) (Item C89C) and has a curve that is modest and cosmetically acceptable. Therefore, he can be managed with education and guidance and followed without treatment. The use of the Milwaukee brace early in the growth spurt may be recommended in adolescents who have kyphotic deformities of at least 45 degrees and often greater than 60 degrees. The bracing is continued until skeletal maturity is attained. However, bracing therapy remains controversial; long-term benefits are debated by experts. Detractors believe that the gains from bracing are slowly lost with subsequent longterm progression. A physical therapy program probably will not alter the natural progression of the disease, but a program that incorporates thoracic extension exercises and emphasizes general conditioning may aid in reducing symptoms. At present, surgical intervention usually is reserved for patients who have a mature skeleton and a severe, painful deformity. More options may become available as advances are made in microsurgical techniques, instruments, and materials for the treatment of spinal conditions. Pulmonary function tests are not indicated for a patient who has an asymptomatic thoracic kyphosis of 50 degrees. Restrictive lung disease is uncommon even in patients who have severe curves of 90 degrees or more. Scheuermann disease is not associated with renal abnormalities and, therefore, ultrasonography is not indicated. Human leukocyte antigen B27 is associated with spondyloarthritis and Reiter syndrome, not Scheuermann disease. References: Lowe TG. Scheuermanns disease. Orthop Clin North Am. 1999;30:475-487. Abstract available at:

Copyright 2007 by the American Academy of Pediatrics

page 185

2007 PREP SA on CD-ROM

http://www.ncbi.nlm.nih.gov/entrez/query.fcgi?orig_db=PubMed&db=PubMed&cmd=Search&ter m=%22The+Orthopedic+clinics+of+North+America%22[Jour]+AND+475[page]+AND+1999[pdat] Murray PM, Weinstein SL, Spratt KF. The natural history and long-term follow-up of Scheuermann kyphosis. J Bone Joint Surg Am. 1993;75:236-248 Thompson GH. The spine. In: Behrman RE, Kliegman RM, Jenson HB, eds. Nelson Textbook of Pediatrics. 17th ed. Philadelphia, Pa: Saunders; 2004:2280-2288

Copyright 2007 by the American Academy of Pediatrics

page 186

2007 PREP SA on CD-ROM Question: 90

A 16-year-old girl who was treated successfully for leukemia that presented as a mediastinal mass at 2 years of age comes to your office complaining of a "lump" in her neck (Item Q90A) of 3 weeks' duration. On physical examination, you note a firm, hard, fixed mass that seems to be part of the left lobe of the thyroid and scattered shotty lymphadenopathy on the left. Of the following, the MOST informative laboratory study for this patient is

A. fine-needle aspiration thyroid biopsy B. serum calcitonin concentration C. technetium thyroid scan D. thyroid ultrasonography E. 131-iodine thyroid scan

Copyright 2007 by the American Academy of Pediatrics

page 187

2007 PREP SA on CD-ROM Critique: 90 Preferred Response: A

All children who have discrete thyroid masses should be referred to an endocrinologist. The previous chemotherapy and irradiation to the chest probably received by the girl in the vignette for treatment of leukemia places her at risk for a thyroid malignancy. Thyroid cancer accounts for about 10% of the second malignancies among cancer survivors and is especially common among survivors of Hodgkin lymphoma. A thyroid mass associated with lymphadenopathy that is firm, hard, and fixed is much more likely to be malignant than a soft, mobile mass without associated adenopathy. A fine-needle aspiration biopsy can provide pathologic confirmation of malignancy in most cases, leading to definitive surgery and other necessary therapy. The most common thyroid malignancies in children are papillary and follicular carcinomas. Serum calcitonin concentration should be measured if there is suspicion of medullary carcinoma of the thyroid, but this malignancy is not found frequently after irradiation. A thyroid scan (Item C90A), whether technetium or 131-iodine, probably can help to identify the nodule, but the differentiation between hot and cold nodules, once so important in diagnosis, no longer is made. Thyroid malignancies may be found in lesions that take up iodine or technetium. Thyroid ultrasonography may be very useful for choosing the area to biopsy and assessing the extent of the thyroid lesion. Some suggest that biopsy performed with ultrasonography is more likely to be informative, but in this case, the pathologic reading of the aspiration biopsy of the large mass should be definitive. References: Halac I, Zimmerman D. Thyroid nodules and cancers in children. Endocrinol Metab Clin North Am. 2005;34:725-744. Abstract available at: http://www.ncbi.nlm.nih.gov/entrez/query.fcgi?orig_db=PubMed&db=PubMed&cmd=Search&ter m=%22Endocrinology+and+metabolism+clinics+of+North+America%22[Jour]+AND+725[page]+ AND+2005[pdat] Amrikachi M, Ponder TB, Wheeler TM, Smith D, Ramzy I. Thyroid fine-needle aspiration biopsy in children and adolescents: experience with 218 aspirates. Diagn Cytopathol. 2005;32:189-192. Abstract available at: http://www.ncbi.nlm.nih.gov/entrez/query.fcgi?orig_db=PubMed&db=PubMed&cmd=Search&ter m=%22Diagnostic+cytopathology%22[Jour]+AND+189[page]+AND+2005[pdat] Corrias A, Einaudi S, Chiorboli E, et al. Accuracy of fine needle aspiration biopsy of thyroid nodules in detecting malignancy in childhood: comparison with conventional clinical, laboratory, and imaging approaches. J Clin Endocrinol Metab. 2001;86:4644-4648. Available at: http://jcem.endojournals.org/cgi/content/full/86/10/4644

Copyright 2007 by the American Academy of Pediatrics

page 188

2007 PREP SA on CD-ROM Question: 91

You and your colleagues are discussing implementation of routine developmental screening in your office. In your research, you have found that

A. early identification is effective in improving educational outcome B. most developmental screening tests have a sensitivity of approximately 90% C. screening for behavioral and developmental concerns requires separate questionnaires D. subsequent screening is not necessary after children pass two screening tests E. the use of developmental screening tools requires extensive staff training

Copyright 2007 by the American Academy of Pediatrics

page 189

2007 PREP SA on CD-ROM Critique: 91 Preferred Response: A

Developmental or behavioral disabilities are seen in approximately 15% to 18% of children in the United States. The American Academy of Pediatrics recommends that pediatricians use validated screening tools at each health supervision visit, but time constraints and other pressing issues can make it difficult to comply with this recommendation. However, early intervention has been shown to be effective in improving long-term educational and vocational outcomes for children who have developmental or behavioral disabilities as well as preventing teen pregnancy and criminality. The hundreds of assessment measures available in the United States can add to practitioners confusion when choosing a screening test. Different tools are used in varying age groups and have different screening focuses (behavioral versus developmental versus psychosocial). However, children who have disabilities and are not identified on an initial screening are likely to be identified at subsequent screenings, and children who are overidentified likely have unique needs even though they might not qualify for special services. Although separate questionnaires are available for behavioral and developmental screening, tools also are available to screen for both areas in the same questionnaire. Screening should be performed at all health supervision visits, even if the child has passed several screening tests. Developmental screening is simple and can be completed by parent questionnaire or administered by staff who have minimal training (Item C91A). References: American Academy of Pediatrics Committee on Children with Disabilities. Developmental surveillance and screening of infants and young children. Pediatrics. 2001;108:192-196. Available at: http://pediatrics.aappublications.org/cgi/content/full/108/1/192 Glascoe FP. Early detection of developmental and behavioral problems. Pediatr Rev. 2000;21:272-280. Available at: http://pedsinreview.aappublications.org/cgi/content/full/21/8/272

Copyright 2007 by the American Academy of Pediatrics

page 190

2007 PREP SA on CD-ROM Question: 92

An 18-year-old pregnant girl presents to your clinic with complaints of discomfort in her knees and hands. She denies any swelling or erythema of these areas or recent trauma. She takes no regular medications, and she has been healthy until 6 days ago, when she developed these complaints. She did receive a rubella vaccination about 1 month ago because on her first visit to an obstetrician she was found to have a negative serum titer to rubella. Her physical examination findings are normal. Of the following, the MOST likely diagnosis is

A. adverse effect of the rubella vaccine B. anicteric hepatitis B infection C. poststreptococcal arthritis D. reactive arthritis due to Salmonella sp E. recent infection with parvovirus

Copyright 2007 by the American Academy of Pediatrics

page 191

2007 PREP SA on CD-ROM Critique: 92 Preferred Response: A

The patient described in the vignette has joint complaints due to the rubella vaccine. Rubellaassociated joint complaints can follow natural infection or immunization. Complications of immunization or natural disease usually occur in women and are uncommon in preadolescent children and males. Arthralgias of the knees and hands usually begin within 7 days of natural illness and 10 to 28 days after immunization. Postinfectious arthritis/arthralgia usually follows a viral infection and is short-lived compared with reactive arthritis/arthralgia, which follows a gastrointestinal or genitourinary infection, has a variable course, and may progress to a chronic spondyloarthropathy. Postinfectious arthropathy following viral infections appears to involve the deposition of immune complexes containing viral antigens into the joints. Reactive arthropathy may represent an autoimmune response involving T lymphocytes that cross-react to antigens present in the joints. Postinfectious or reactive joint disease can be diagnosed once the symptoms have resolved. Acute joint complaints involving only one joint should raise the suspicion of another diagnosis (eg, pyogenic infection of a joint). It is important to remember that some other autoimmune diseases can present with joint complaints (eg, inflammatory bowel disease, juvenile rheumatoid arthritis). No specific treatment is necessary for postinfectious or reactive joint complaints except for relief of pain or discomfort with nonsteroidal anti-inflammatory medications. Anicteric hepatitis B infection, poststreptococcal arthritis, reactive arthritis due to Salmonella sp, and a recent infection with parvovirus all can produce joint complaints, but the history of rubella immunization described for the girl in the vignette makes these diagnoses less likely. References: Keat A. Reactive arthritis or post-infective arthritis? Best Pract Res Clin Rheumatol. 2002;16:507-522. Abstract available at: http://www.ncbi.nlm.nih.gov/entrez/query.fcgi?db=pubmed&cmd=Retrieve&dopt=Abstract&list_ui ds=12406424&query_hl=8&itool=pubmed_DocSum Miller ML, Cassidy JT. Postinfectious arthritis and related conditions. In: Behrman RE, Kliegman RM, Jenson HB, eds. Nelson Textbook of Pediatrics. 17th ed. Philadelphia, Pa: WB Saunders Co; 2004:808-809

Copyright 2007 by the American Academy of Pediatrics

page 192

2007 PREP SA on CD-ROM Question: 93

You have been the primary practitioner for a 15-year-old girl since she was 3 years old. She never has been hospitalized, and her history, vital signs, and physical examination findings on this health supervision visit are normal. At each of the past two yearly visits, urinalysis revealed no abnormalities except 2+ protein. Her urinalysis today again reveals 2+ protein with no other abnormalities. Her complete blood count, measurements of serum electrolytes and serum complements, and antinuclear antibody test results are all normal. Two successive 24-hour urine collections reveal 321 and 387 mg protein. You now refer the girl to a pediatric nephrologist. Of the following, the MOST important next step for the pediatric nephrologist is to

A. obtain a first morning urine specimen B. obtain renal ultrasonography C. perform a renal biopsy D. repeat the 24-hour urine collection E. repeat the urinalysis

Copyright 2007 by the American Academy of Pediatrics

page 193

2007 PREP SA on CD-ROM Critique: 93 Preferred Response: A

Detection of proteinuria on routine urine dipstick evaluation is an uncommon but perplexing dilemma for pediatricians because proteinuria may suggest underlying renal disease or may represent a benign condition. It is incumbent on the pediatrician to determine the appropriate steps to be undertaken in a child who has a urine dipstick positive for protein. Among the many benign conditions causing a positive dipstick result for urinary protein are one of the following: a very concentrated urine (specific gravity =1.020), alkaline urine (pH =7.5), or the presence of mucoproteins. In addition, acute illness may result in a small degree of proteinuria. The most important feature of these conditions is that the urinary dipstick result almost never exceeds a reading of 1+. In the absence of these conditions, it is vital to determine if the proteinuria is transient (orthostatic) or fixed. Orthostatic proteinuria (OP) occurs during the daytime but disappears when the person is supine (eg, asleep) for at least 2 hours. Although the exact cause of OP remains controversial, it is a benign condition that requires no follow-up or treatment. Children who have OP cannot have a history of renal disease, hematuria, or edema. In contrast, fixed proteinuria is present at all times of the day, regardless of body position. The teenager in the vignette is asymptomatic and has had several urinalyses (UAs) revealing 2+ protein but no other abnormalities. A negative first morning UA, with a urine proteinto-creatinine ratio less than 0.2, would establish the diagnosis of OP. Prior to referral to the pediatric nephrologist, obtaining a repeat UA within 2 weeks and subsequently obtaining a first morning UA likely would have established the diagnosis of OP, obviating the need for several other unnecessary tests (eg, electrolyte panel, complete blood count, serum complements). Renal ultrasonography may be helpful in detecting a potential urinary tract malformation in some patients who may have proteinuria, but it is not the most appropriate first step. If the patient has a positive (1+ or more) first morning UA result, renal biopsy may be indicated. A 24hour urine collection (instead of a random protein-to-creatinine ratio) rarely is necessary and does not aid in the diagnosis of OP. Proteinuria has been established firmly for this child, and waiting for results from another daytime UA would only delay the diagnosis. References: Chandar J, Gomez-Marin O, del Pozo R, et al. Role of routine urinalysis in asymptomatic pediatric patients. Clin Pediatr (Phila). 2005;44:43-48. Abstract available at: http://www.ncbi.nlm.nih.gov/entrez/query.fcgi?orig_db=PubMed&db=PubMed&cmd=Search&ter m=%22Clinical+pediatrics%22[Jour]+AND+43[page]+AND+2005[pdat] Hogg RJ, Portman RJ, Milliner D, Lemley KV, Eddy A, Ingelfinger J. Evaluation and management of proteinuria and nephrotic syndrome in children: recommendations from a pediatric nephrology panel established at the National Kidney Foundation Conference on Proteinuria, Albuminuria, Risk, Assessment, Detection, and Elimination (PARADE). Pediatrics. 2000;105:1242-1249. Available at: http://pediatrics.aappublications.org/cgi/content/full/105/6/1242 Wingo C, Clapp WL. Proteinuria: potential causes and approach to evaluation. Am J Med Sci. 2000;320:188-194. Abstract available at: http://www.ncbi.nlm.nih.gov/entrez/query.fcgi?orig_db=PubMed&db=PubMed&cmd=Search&ter m=%22The+American+journal+of+the+medical+sciences%22[Jour]+AND+188[page]+AND+200 0[pdat]

Copyright 2007 by the American Academy of Pediatrics

page 194

2007 PREP SA on CD-ROM Question: 94

A 10-year-old boy presents with a 2-day history of a temperature to 102.5F (39.2C) and a maculopapular rash that began on his trunk and now has spread to his extremities and back. Several of the lesions appear to be vesicles (Item Q94A), and he is starting to develop new lesions on his face. Of the following, the MOST rapid method to confirm the cause of this patient's infection is

A. complement fixation B. direct fluorescent antibody C. enzyme immunoassay D. polymerase chain reaction E. viral culture

Copyright 2007 by the American Academy of Pediatrics

page 195

2007 PREP SA on CD-ROM Critique: 94 Preferred Response: B

The availability of rapid and reliable viral diagnostic tests facilitates decision making in the prevention and treatment of viral infections and the institution of effective infection control measures. The available rapid testing methods detect viral antigen by immunologic methods, such as fluorescent antibody (FA) testing or enzyme immunoassay (EIA), and viral nucleic acid by labeled complementary molecular probes, either by direct hybridization or after some form of amplification such as polymerase chain reaction. Antigen detection tests are performed directly on specimens from the patient. Kits to perform EIA or the FA test are available for the detection of rotavirus and enteric adenovirus in stool specimens; respiratory syncytial virus, influenza A and B viruses, parainfluenza viruses, and adenoviruses in respiratory specimens; hepatitis B surface antigen and human immunodeficiency virus (HIV) p24 antigen in serum; herpes simplex virus (HSV) and varicellazoster virus in vesicle swab specimens; and cytomegalovirus (CMV) in bronchoalveolar lavage and blood specimens. Results of these tests usually are available within hours and are preferred when sensitive and specific test kits and reagents are commercially available. The disadvantages of antigen detection tests include lack of available test kits for some clinically important viruses, inferior sensitivity compared with culture for most viruses that can be cultured, and inferior specificity for all viruses. Molecular probes for nucleic acid are most useful for detecting and typing viruses for which reliable culture methods are not available, such as human papillomavirus (HPV). Molecular probes are available in commercial kits for the detection of HIV in plasma or white blood cells; HSV and enteroviruses in cerebrospinal fluid; CMV, hepatitis B virus, and hepatitis C virus in serum; HPV in cervical cells; and parvovirus in serum. However, for some viruses, the concentration of viral genomes in direct patient specimens may be too low to allow detection with adequate sensitivity. Molecular amplification technologies have been developed for many viruses to increase the sensitivity for detection of viral genomes. The use of nucleic acid detection techniques is appropriate when the virus cannot be detected by rapid isolation or when antigen detection methods are not available or are insensitive. The patient described in the vignette has varicella-zoster infection, and the most rapid method available to identify this virus is direct fluorescent antibody testing on a sample of vesicular fluid. Complement fixation and EIA require acute and convalescent serum specimens, making them less rapid approaches. Further, complement fixation has poor sensitivity. Polymerase chain reaction is very sensitive, but its availability is limited, and highly trained personnel and specific equipment are required to perform the test. Viral culture is sensitive and specific, but it is expensive and has limited availability. Results from both polymerase chain reaction testing and viral cultures take much longer to obtain than direct FA. References: American Academy of Pediatrics. Varicella-zoster infections. In: Pickering LK, ed. Red Book: 2006 Report of the Committee on Infectious Diseases. 27th ed. Elk Grove Village, Ill: American Academy of Pediatrics; 2006:711-725 Mazzulli T. Laboratory diagnosis of infection due to viruses, Chlamydia, and Mycoplasma. In: Long SS, Pickering LK, Prober CG, eds. Principles and Practice of Pediatric Infectious Diseases. 2nd ed. New York, NY: Churchill Livingstone; 2003:1392-1408 Naber SP. Molecular pathologydiagnosis of infectious disease. N Engl J Med. 1994;331:12121215

Copyright 2007 by the American Academy of Pediatrics

page 196

2007 PREP SA on CD-ROM Question: 95

A 12-month-old boy presents for evaluation of severe eczema. His parents state that their son's eczema started soon after birth and has not responded to topical corticosteroids. The parents also are concerned that the infant has had frequent otitis media, sinus infections, and one episode of Streptococcus pneumoniae pneumonia requiring hospitalization last month. During your review of the laboratory studies performed, you notice that the infant has thrombocytopenia (20x103/mcL [20x109/L]) and small platelets. Of the following, the MOST likely diagnosis for this infant is

A. Bruton (X-linked) agammaglobulinemia B. chronic granulomatous disease C. DiGeorge syndrome D. Wiskott-Aldrich syndrome E. X-linked severe combined immunodeficiency

Copyright 2007 by the American Academy of Pediatrics

page 197

2007 PREP SA on CD-ROM Critique: 95 Preferred Response: D

The clinical constellation of recurrent infections, severe eczema, and small platelets described for the patient in the vignette is pathognomonic for Wiskott-Aldrich syndrome (WAS). WAS is a primary immune deficiency that has been mapped to the Xp11.22-11.23 chromosome. Patients can present initially with prolonged bleeding after circumcision or bloody diarrhea during infancy. Recurrent infections and eczema develop prior to age 1 year. Treatment options include intravenous or subcutaneous immunoglobulin, prophylactic antibiotics, and ideally a human leukocyte antigen-identical bone marrow transplant. Bruton (X-linked) agammaglobulinemia is another X-linked primary immune deficiency resulting from defects in the B-cell tyrosine kinase protein. Patients have a profound defect in Bcell production, severe hypogammaglobulinemia, small-to-absent tonsils, and recurrent sinopulmonary infections. Symptoms typically occur after 6 months, coinciding with waning of passive maternal antibody transfer. Treatment consists of lifelong immunoglobulin infusions, evaluation and aggressive treatment of infections, and monitoring for development of lymphoreticular malignancies. Chronic granulomatous disease (CGD) is a disorder of the phagocytic system resulting from an inability to kill catalase-positive organisms (eg, Staphylococcus aureus, Aspergillus sp, Serratia marcescens, Burkholderia cepacia, Candida albicans, and Salmonella sp). Patients present with recurrent lymphadenitis, hepatic abscesses, recurrent skin infections, and osteomyelitis. Bone marrow transplant is the only known cure. Aggressive supportive care, interferon-gamma injections (50 mcg/m2, three times per week), and prophylactic antibiotics are the mainstay therapy in lieu of transplant. DiGeorge syndrome is a genetic disorder linked to chromosome 22q11.2 and the dysmorphogenesis of the third and fourth pharyngeal pouches that usually presents in infancy. Clinical manifestations vary, but patients who have complete DiGeorge syndrome have recurrent viral, fungal, and bacterial infections similar to children who have severe combined immunodeficiency (SCID). Treatment consists of either bone marrow or thymic tissue transplant. X-linked SCID is the most common form of SCID and results from a defect in the common gamma chain that encodes several interleukin receptors. Affected infants present within the first few postnatal months with frequent episodes of diarrhea, sinopulmonary infections, and skin infections. Prompt recognition and referral to an immunologist is vital because replacement immunoglobulin fails to halt the progressive worsening of this immunodeficiency. Unless bone marrow transplantation is performed, death usually occurs before the childs second birthday. References: Buckley RH. Primary defects of cellular immunity. In: Behrman RE, Kliegman RM, Jenson HB, eds. Nelson Textbook of Pediatrics. 17th ed. Philadelphia, Pa: WB Saunders Co; 2004:693-694 Thampakkul S, Ballow M. Replacement intravenous immune serum globulin therapy in patients with antibody immune deficiency. Immunol Allergy Clin North Am. 2001;21:165-184

Copyright 2007 by the American Academy of Pediatrics

page 198

2007 PREP SA on CD-ROM Question: 96

You inserted an endotracheal tube into a 2-year-old child who had severe head trauma after a motor vehicle crash. Initially, she had good chest rise, with oxygen saturations of 99% on 100% inspired oxygen delivered via bag-valve mask, but 10 minutes later, she suddenly deteriorates. Her oxygen saturation now is 60%, and her heart rate is 50 beats/min and falling rapidly. You note poor breath sounds on both sides of her chest and poor chest rise. Of the following, the BEST initial management of this patient is to

A. consult a surgeon for urgent placement of a chest tube B. insert an 18-gauge angiocatheter into the upper right chest to evacuate a possible
pneumothorax

C. order an immediate chest radiograph to determine if she has a pneumothorax D. perform deep suction through the endotracheal tube using an 8-French flexible catheter E. remove the endotracheal tube and resume ventilation with bag-valve mask

Copyright 2007 by the American Academy of Pediatrics

page 199

2007 PREP SA on CD-ROM Critique: 96 Preferred Response: E

The child in the vignette has had a sudden deterioration in pulmonary status shortly after endotracheal intubation. Rapid identification and correction of the problem is essential for survival. In this situation, the authors of the Pediatric Advanced Life Support course suggest the use of a mnemonic to help determine the cause. DOPE refers to Displacement of the endotracheal tube either into the right mainstem bronchus or completely out of the trachea, Obstruction of the tube, Pneumothorax, and Equipment failure such as interruption of the oxygen supply. This child is experiencing hypoxia and bradycardia and likely will develop full cardiac arrest within seconds unless rapid action is taken. Removal of the endotracheal tube and ventilation via bag-valve mask addresses all of the elements in the DOPE pneumonic except pneumothorax. If her oxygen saturation does not improve with this intervention, a rapid assessment for pneumothorax should be performed. If an intubated patient has a more gradual deterioration in pulmonary status, rapid assessment for all the elements of the DOPE mnemonic may be performed rather than immediately removing the endotracheal tube. Direct laryngoscopy can be performed to view the endotracheal tube passing through the vocal cords. A suction catheter can be passed down the tube to check for obstruction. Inspection of the equipment and circuit can identify obvious problems. If breath sounds are diminished on only one side of the chest, especially in the presence of falling oxygen saturation, hyperresonance to percussion, and decreasing blood pressure, a suspected tension pneumothorax should be decompressed immediately using an angiocatheter attached to a syringe. The catheter should be inserted through the second interspace in the midclavicular line. A rush of air with immediate improvement in the patients status helps to confirm the diagnosis. At that point, a chest tube is inserted for long-term management of the pneumothorax. A chest radiograph may be helpful in a patient who is experiencing marginal deterioration in pulmonary status, especially if quick assessment using the DOPE mnemonic does not identify the source of the problem. Radiographic confirmation of a suspected tension pneumothorax prior to decompression is neither required nor suggested in the patient who has had acute severe deterioration. References: American Heart Association. Airway, ventilation, and management of respiratory distress and failure. In: PALS Provider Manual. Dallas, Tex: American Heart Association; 2002:81-126 Kelly RE Jr, Isaacman DJ. Thoracic emergencies. In: Fleisher GR, Ludwig S, Henretig FM, eds. Textbook of Pediatric Emergency Medicine. 5th ed. Philadelphia, Pa: Lippincott Williams & Wilkins; 2006:1631-1654 Silbergleit R, Lee DC, Blank-Reid C, McNamara RM. Sudden severe barotrauma from selfinflating bag-valve devices. J Trauma. 1996;40:320-322. Abstract available at: http://www.ncbi.nlm.nih.gov/entrez/query.fcgi?orig_db=PubMed&db=PubMed&cmd=Search&ter m=%22The+Journal+of+trauma%22[Jour]+AND+1996[pdat]+AND+Silbergleit+R[author]

Copyright 2007 by the American Academy of Pediatrics

page 200

2007 PREP SA on CD-ROM Question: 97

A 10-year-old girl presents with progressive fatigue without pain. Physical examination reveals edema and a pink-to-violet discoloration of the upper eyelids and malar areas (Item Q97A). Scaly, red papules (Item Q97B) are observed over the knuckles. Of the following, the MOST appropriate laboratory investigation to support the diagnosis is

A. antinuclear antibody assay B. creatine phosphokinase concentration C. epicutaneous patch testing D. erythrocyte sedimentation rate E. skin biopsy

Copyright 2007 by the American Academy of Pediatrics

page 201

2007 PREP SA on CD-ROM Critique: 97 Preferred Response: B

The girl described in the vignette has findings consistent with the diagnosis of juvenile dermatomyositis (JDM). The most common of the pediatric inflammatory myopathies, JDM is a systemic vasculopathy that has characteristic cutaneous findings and focal areas of myositis resulting in progressive proximal muscle weakness that is generally responsive to the prompt institution of immunosuppressive therapy. Disease presentation is variable. Approximately 30% of affected children experience the rapid onset of rash, weakness, and pain. Another 50% have insidious development of muscle pain and progressive weakness. The remaining 20% have a subacute onset in which a rash appears long before muscular or constitutional symptoms. A small proportion of children have typical cutaneous findings but no evidence of muscle disease (amyopathic dermatomyositis). Classic clinical findings include pink-to-violet color of the eyelids or cheeks (heliotrope) (Item C97A) and scaly, erythematous papules symmetrically arrayed over extensor surfaces of the elbows, knees, or metacarpophalangeal or interphalangeal joints (Gottron papules) (Item C97B). JDM usually is diagnosed clinically by the pathognomonic rash and proximal muscle findings. Elevated serum concentrations of muscle-derived enzymes (eg, creatine phosphokinase, aldolase, aspartate aminotransferase, and lactic acid dehydrogenase) reflect the leaky muscle membranes. Elevations of antinuclear antibody assay (in 60% of cases) and the erythrocyte sedimentation rate may be present but are not specific for JDM. Because the appearance and location of the child's skin lesions are not consistent with a diagnosis of contact dermatitis, epicutaneous patch testing to identify an offending allergen is not indicated. Skin biopsy in JDM is not diagnostic. References: Barnett NK, Wright D, Kawasaki T, Treadwell PA. Collagen vascular, connective tissue diseases, and selected systemic diseases with skin manifestations: toxic oil syndrome and eosinophilia myalgia syndrome, juvenile dermatomyositis. In: Schachner LA, Hansen RC, eds. Pediatric Dermatology. 3rd ed. St. Louis, Mo: Mosby; 2003:955-960 Pachman LM. Juvenile dermatomyositis. In: Behrman RE, Kliegman RM, Jenson HB, eds. Nelson Textbook of Pediatrics. 17th ed. Philadelphia, Pa: WB Saunders Co; 2004:813-816 Weston WL, Lane AT, Morelli JG. Papulosquamous disorders: dermatomyositis In: Color Textbook of Pediatric Dermatology. 3rd ed. St. Louis, Mo: Mosby; 2002:136-138

Copyright 2007 by the American Academy of Pediatrics

page 202

2007 PREP SA on CD-ROM Question: 98

A 2-year-old who has a history of repaired biliary atresia presents to your office with fatigue and intermittent dark stools. On physical examination, he is afebrile and pale but active. His heart rate is 110 beats/min, liver and spleen are both enlarged, and abdomen is distended, with prominent abdominal veins. The hematocrit is 22% (0.22). Of the following, the MOST appropriate next step is to

A. arrange for outpatient endoscopy B. arrange for hospital-based care C. begin oral iron supplementation D. obtain abdominal ultrasonography E. refer the boy for therapeutic paracentesis

Copyright 2007 by the American Academy of Pediatrics

page 203

2007 PREP SA on CD-ROM Critique: 98 Preferred Response: B

The patient described in the vignette has a history of biliary atresia palliated with a Kasai procedure and now has melena. On physical examination, there is evidence of portal hypertension, pallor, and tachycardia, and the hematocrit demonstrates severe anemia. The most likely diagnosis is an upper gastrointestinal hemorrhage due to esophageal variceal bleeding, portal gastropathy, or an ulcer. The patient should be hospitalized, have his hemodynamic status stabilized through transfusion, and undergo endoscopy to determine the cause of the bleeding. The presence of melena in a child typically suggests gastrointestinal bleeding from a lesion cephalad to the ligament of Treitz. The black color of the stool probably is caused by the compound hematin, which is generated when heme is oxidized by intestinal bacteria. The differential diagnosis of melena in the infant and child varies with age. The most common lesions causing upper gastrointestinal bleeding are duodenal ulcer, gastric ulcer, gastritis, arteriovenous malformations, erosive esophagitis, or esophageal varices. Although large ulcers and esophageal varices typically present with hematemesis, low-grade bleeding may present with melena and no hematemesis. The patient's clinical status and underlying disease history determine the need for acute care. Patients who have hemodynamic instability, anemia, or serious comorbidities need urgent intervention. Initial care should include intravenous catheter placement and laboratory assessment, including a complete blood count, coagulation studies, and blood typing and crossmatching. Endoscopy can help characterize the precise lesion and may allow for therapeutic intervention. For example, a patient who has bleeding varices can have rubber bands placed around them to stop the bleeding (band ligation), and a patient who has a bleeding vessel within an ulcer can be treated by endoscopic hemostasis. Because of his tenuous status and anemia, elective endoscopy or oral outpatient therapy with iron are not appropriate as first-line treatments for the boy described in the vignette. Abdominal ultrasonography can demonstrate portal hypertension and may show varices, but it cannot identify the site of bleeding or treat the bleeding. Paracentesis is useful if spontaneous bacterial peritonitis is suspected, but it will not identify the bleeding lesion, which is in the lumen of the gastrointestinal tract. References: Gilger MA. Upper gastrointestinal bleeding. In: Walker WA, Goulet O, Kleinman RE, Sherman PM, Shneider BL, Sanderson IR, eds. Pediatric Gastrointestinal Disease. 4th ed. Hamilton, Ontario, Canada: BC Decker; 2005:258-265 Squires RH Jr. Gastrointestinal bleeding. Pediatr Rev. 1999;20:95-101. Available at: http://pedsinreview.aappublications.org/cgi/content/full/20/3/95

Copyright 2007 by the American Academy of Pediatrics

page 204

2007 PREP SA on CD-ROM Question: 99

A term infant is delivered vaginally following a pregnancy complicated by diabetes mellitus. His oral and nasal airways are suctioned and found to be patent and free of meconium. He has cyanosis and respiratory distress immediately following birth that requires intubation and assisted ventilation with 100% oxygen. Because no improvement is apparent in the next 5 minutes, he is admitted to the neonatal intensive care unit. His birthweight is 4,500 g. A chest radiograph reveals findings consistent with decreased pulmonary blood flow. Of the following, the MOST likely cause of respiratory distress in this infant is

A. anemia B. choanal atresia C. hyperglycemia D. hypermagnesemia E. persistent pulmonary hypertension

Copyright 2007 by the American Academy of Pediatrics

page 205

2007 PREP SA on CD-ROM Critique: 99 Preferred Response: E

An infant of a diabetic mother (IDM), such as the infant described in the vignette, is predisposed to some increased risk of respiratory distress syndrome associated with an immature pulmonary surfactant profile. However, findings on this infants radiograph are not consistent with this diagnosis, and the combination of cyanosis and respiratory distress that does not improve with intubation and assisted ventilation raises concerns for cyanotic heart disease and decreased pulmonary blood flow. Transposition of the great arteries is the most common cyanotic heart lesion in the IDM. The possibility of persistent pulmonary hypertension of the newborn (PPHN) must be evaluated. The findings on chest radiograph vary with the cause of PPHN. Diffuse ground-glass densities and low lung volumes are observed with congenital pneumonia. Patchy infiltrates and hyperinflation are seen in meconium aspiration syndrome. In idiopathic cases of PPHN, the lung fields are clear, with evidence for decreased pulmonary blood flow. Echocardiography is vital to discerning this infants diagnosis. A hyperoxia test (assisted ventilation with 100% oxygen and blood gas analysis from preductal and postductal sites) to determine the presence of a fixed right-to-left extrapulmonary shunt is also an important evaluative tool. The IDM who has macrosomia is at risk for hypoglycemia, which also may present with cyanosis. Choanal atresia is characterized by cyanosis that resolves when an infant cries and an inability to pass a suction catheter through the nares. Anemia is associated with pallor, and in the IDM, polycythemia and plethora are more likely. Additionally, hypocalcemia and hypomagnesemia are common in the IDM. References: Bernstein D. Epidemiology and genetic basis of congenital heart disease. In: Behrman RE, Kliegman RM, Jenson HB, eds. Nelson Textbook of Pediatrics. 17th ed. Philadelphia, Pa: Saunders 2004;1499-1501 Bernstein D. Evaluation of the infant or child with congenital heart disease. In: Behrman RE, Kliegman RM, Jenson HB, eds. Nelson Textbook of Pediatrics. 17th ed. Philadelphia, Pa: Saunders 2004;1502-1503 Cowett R. The infant of the diabetic mother. Neoreviews. 2002;3:e173-e189. Available at: http://neoreviews.aappublications.org/cgi/content/full/3/9/e173 Flidel-Rimon O, Shinwell ES. Respiratory distress in the term and near-term infant. NeoReviews. 2005;6: e289-e297. Available at: http://neoreviews.aappublications.org/cgi/content/full/6/6/e289 Nold JL, Georgieff MK. Infants of diabetic mothers. Pediatr Clin North Am. 2004;51:619-637. Abstract available at: http://www.ncbi.nlm.nih.gov/entrez/query.fcgi?cmd=Retrieve&db=pubmed&dopt=Abstract&list_ui ds=15157588&query_hl=32&itool=pubmed_docsum Stoll BJ, Kliegman RM. Respiratory tract disorders. In: Behrman RE, Kliegman RM, Jenson HB, eds. Nelson Textbook of Pediatrics. 17th ed. Philadelphia, Pa: Saunders; 2004:573-588 Stoll BJ, Kliegman RM. The endocrine system. In: Behrman RE, Kliegman RM, Jenson HB, eds. Nelson Textbook of Pediatrics. 17th ed. Philadelphia, Pa: Saunders; 2004:613-615

Copyright 2007 by the American Academy of Pediatrics

page 206

2007 PREP SA on CD-ROM Question: 100

A 4-week-old boy has been vomiting all of his feedings for the last 2 days. His mother reports decreased urine output but no fever. On physical examination, he is alert, appears hungry, and is moderately dehydrated. Abdominal examination reveals a small mass in the epigastrium. When he is offered a bottle, he takes it vigorously and then vomits forcefully. You order laboratory tests. Of the following, the MOST likely abnormality to expect is

A. hyperchloremia B. hyperkalemia C. hypernatremia D. metabolic acidosis E. metabolic alkalosis

Copyright 2007 by the American Academy of Pediatrics

page 207

2007 PREP SA on CD-ROM Critique: 100 Preferred Response: E

The infant described in the vignette has forceful, nonbilious vomiting consistent with a gastric outlet obstruction. Pyloric stenosis is the most common cause of gastric obstruction in neonates. It results from hypertrophy of the pylorus and is most common in males older than 2 to 3 weeks of age. Affected infants generally appear well, with the exception of dehydration, and they often are very hungry after vomiting. Abdominal examination may reveal a gastric peristaltic wave after eating and a small olive-shaped mass in the epigastrium, which may be appreciated more easily with the infant in a prone position. Normal findings on abdominal examination are not unusual and do not rule out the diagnosis. The forceful vomiting of all stomach contents results in massive fluid loss and dehydration over time. Hypochloremic metabolic alkalosis develops from the progressive loss of hydrogen ion and chloride. The serum potassium concentration may be low or normal, and hyperbilirubinemia may be seen. Hypernatremia, hyperkalemia, hyperchloremia, and metabolic acidosis are not typical electrolyte abnormalities in pyloric stenosis. The diagnostic examination of choice is pyloric ultrasonography, which reveals thickening and elongation of the pylorus. Care should be taken to rehydrate the infant and correct any electrolyte abnormalities before undertaking surgical correction. The prognosis after surgical correction is excellent. References: Wyllie R. Pyloric stenosis and congenital anomalies of the stomach. In: Behrman RE, Kliegman RM, Jenson HB, eds. Nelson Textbook of Pediatrics. 17th ed. Philadelphia, Pa: WB Saunders Co; 2004:1229-1231 Dinkevich E, Ozuah PO. In brief: pyloric stenosis. Pediatr Rev. 2001;21:249-250. Available at: http://pedsinreview.aappublications.org/cgi/content/full/21/7/249

Copyright 2007 by the American Academy of Pediatrics

page 208

2007 PREP SA on CD-ROM Question: 101

A 2-year-old girl presents for evaluation of fussiness, low-grade fever, and what her parents describe as "growing pains." On physical examination, you palpate a nontender mass deep in the right periumbilical area and note mild purple discoloration of the eyelids. Of the following, the MOST likely diagnosis is

A. hepatoblastoma B. Hirschsprung disease C. intussusception D. neuroblastoma E. Wilms tumor

Copyright 2007 by the American Academy of Pediatrics

page 209

2007 PREP SA on CD-ROM Critique: 101 Preferred Response: D

Neuroblastoma is the most common extracranial solid tumor of childhood. It occurs most frequently in children younger than the age of 4 years and has a slight male predilection. It may arise anywhere in the sympathetic nervous system, but most commonly presents in the abdomen. Although it may present as an abdominal mass, as for the child in the vignette, it is diagnosed most commonly as widely disseminated disease. The discoloration of the periorbital tissues reported for the child in the vignette results from mass effects of involved lymphoid tissue in the periorbital area. Paraneoplastic manifestations of neuroblastoma, such as opsoclonus or dancing eyes, may result from a postulated immune mechanism and may persist despite surgical or chemotherapeutic cure of the underlying malignancy. Surgical staging was used for prognostic purposes until the late 1980s, but recent advances in tumor molecular biology have changed the method by which neuroblastoma is treated and prognosed. Tumors with amplification of the N-myc protooncogene or with multidrug resistance protein expression have a poorer prognosis. Wilms tumor often presents as a painless abdominal mass with few systemic symptoms and frequently is associated with somatic overgrowth symptoms or other genetic syndromes. Intussusception usually has an abrupt onset but may involve a palpable intussusceptum and typically is characterized by bloody stools and vomiting. Hirschsprung disease may present with a palpable toxic megacolon and fever, but there is often a history of constipation (diarrhea may be common in neuroblastoma due to catecholamine excess). Hepatoblastoma is rare but should be considered in children who have chronic hepatitis B infection or a somatic overgrowth syndrome. References: Strother DR, Dreyer ZE. Neuroblastoma. In: McMillan JA, DeAngelis CD, Feigin RD, Warshaw JB, eds. Oskis Pediatrics: Principles and Practice. 3rd ed. Philadelphia, Pa: Lippincott, Williams & Wilkins; 1999:1517-1519

Copyright 2007 by the American Academy of Pediatrics

page 210

2007 PREP SA on CD-ROM Question: 102

A 7-year-old girl presents with a 4-week history of an erythematous rash that involves the eyelids and malar areas and now has occurred at the elbows and knees. Her mother reports that the girl has had a poor appetite, low-grade fevers, muscle aches, and weakness during the past week. Physical examination confirms the rash (Item Q102A). She has difficulty rising from a chair. Other findings on her physical examination are normal. Serum creatine kinase is 677 U/L. Of the following, the test MOST likely to lead to this child's diagnosis is

A. erythrocyte sedimentation rate B. molecular testing for fascioscapulohumeral dystrophy C. muscle biopsy D. polymerase chain reaction testing for dystrophin E. serum antinuclear antibody

Copyright 2007 by the American Academy of Pediatrics

page 211

2007 PREP SA on CD-ROM Critique: 102 Preferred Response: C

Weakness can be a broad complaint in a child, but identifying the neurologic origin of the weakness, its temporal pattern, and associated findings can point to appropriate neurodiagnostic tests and help establish the diagnosis. In general, muscular weakness tends to be greater proximally than distally, affecting most the hips and shoulders. Creatine kinase is often elevated in myopathy. Neuropathy also produces symmetric weakness, but the deficit is greater distally than proximally and associated with hyporeflexia and sometimes, sensory loss. Nerve conduction velocities in neuropathy demonstrate diminished velocity or amplitude. Neuromuscular junction problems tend to worsen throughout the day, and in children are more notable rostrally than caudally. Myelopathy produces weakness inferior to a spinal lesion, often with a sensory deficit inferior to a specific dermatome. Weakness associated with brainstem processes often is asymmetric and characterized by cranial neuropathies. Cerebral processes with weakness produce hemiparesis and unilateral hyperreflexia and may cause visual field loss or speech problems. Acute causes of weakness include stroke (cerebrum or brainstem); acute intermittent porphyria, diphtheria, poliomyelitis, West Nile virus infection, or tick paralysis (neuropathy); organophosphate poisoning or botulism (neuromuscular junction); and periodic paralysis and acute infectious myositis (muscle). Subacute processes include tumor or abscess (cerebrum or spine), transverse myelitis (spine), myasthenia gravis (neuromuscular junction), Guillain-Barr syndrome (nerve), and polymyositis and dermatomyositis (muscle). Muscular dystrophies tend to present chronically. The 7-year-old girl described in the vignette displays weakness that is proximal more than distal, as well as elevated creatine kinase concentrations, suggesting a myopathy. The 4-week duration plus the erythematous malar rash raise suspicion about dermatomyositis. Muscle biopsy can demonstrate the fiber type variability, central nuclei that are vesicular, infiltration with inflammatory cells, and necrosis that characterize the condition. The erythrocyte sedimentation rate is a nonspecific test; it is elevated in myopathy, infections, and multiple other disorders. This child does not have the chronic course of the three most common dystrophies (Duchenne, Becker, and fascioscapulohumeral). Therefore, polymerase chain reaction testing for dystrophin or genetic testing for fascioscapulohumeral dystrophy is not warranted. Serum antinuclear antibody may be positive in patients who have dermatomyositis, but it is a nonspecific finding. It is most useful in the evaluation of patients who may have systemic lupus erythematosus, but lupus will not produce the myopathic weakness reported for this girl. References: Jacobson RD. Approach to the child with weakness or clumsiness. Pediatr Clin North Am. 1998;45:145-168. Abstract available at: http://www.ncbi.nlm.nih.gov/entrez/query.fcgi?cmd=Retrieve&db=pubmed&dopt=Abstract&list_ui ds=9491091&query_hl=14&itool=pubmed_docsum Roland EH. Muscular dystrophy. Pediatr Rev. 2000;21:233-238. Available at: http://pedsinreview.aappublications.org/cgi/content/full/21/7/233 Sarnat HB. Neuromuscular disorders. In: Behrman RE, Kliegman RM, Jenson HB, eds. Nelson Textbook of Pediatrics. 17th ed. Philadelphia, Pa: WB Saunders Co; 2004:2053-2082 Turner TL, Boom JA. Index of suspicion. Case 1. Pediatr Rev. 2000;21:389-392. Available at: http://pedsinreview.aappublications.org/cgi/content/full/21/11/389

Copyright 2007 by the American Academy of Pediatrics

page 212

2007 PREP SA on CD-ROM Question: 103

During a health supervision visit for a 16-year-old boy, you learn that he has experienced chest pain twice with intense exercise during practice for his high school varsity soccer team. Each time the pain felt like pressure, radiated to his left shoulder, and was associated with lightheadedness. He did not seek medical attention after either episode. His father, who is age 49 years, has hypertension and uses lipid-lowering medication. Results of the boy's physical examination are normal, including his blood pressure and cardiovascular examination. Of the following, the BEST management plan is

A. cardiology referral, with only light exercise pending evaluation B. cardiology referral, with restriction of all exercise pending evaluation C. echocardiography, with clearance for exercise if results are normal D. electrocardiography, with clearance for exercise if results are normal E. trial of bronchodilator therapy for exercise-induced asthma

Copyright 2007 by the American Academy of Pediatrics

page 213

2007 PREP SA on CD-ROM Critique: 103 Preferred Response: B

Chest pain in children and adolescents is a common problem. The causes of chest pain in the pediatric population are varied and can be considered by organ systems: musculoskeletal, respiratory, gastrointestinal, psychological, and cardiac. Among the musculoskeletal causes are chest wall strain, trauma, costochondritis, and the precordial catch syndrome. Respiratory causes include asthma, pneumonia, pneumothorax, pneumomediastinum, and chronic cough. Chest pain may be due to gastritis, esophagitis, or indigestion. Chest pain also may result from or be exacerbated by psychogenic processes, including anxiety, fear, and attention-seeking behaviors. Perhaps the most common causes of chest pain in pediatrics are those referred to as idiopathic. This diagnosis often is given to the patient who presents with a 1- to 2-week history of intermittent, brief, sharp, or stabbing pain that is not associated with exercise or exertion. The cardiac causes of chest pain are important to recognize because they can be associated with significant morbidity and mortality. Chest pain can result from pericardial pain, angina and myocardial ischemia, arrhythmias, and aortic dissection. Pericardial pain results from inflammation and often is associated with pericarditis. The pain typically is substernal, positional, and can be severe. Affected patients frequently prefer to sit and may refuse to lie down. Pain that results from angina and myocardial ischemia is rare in children but can occur in those who have anomalies of the coronary arteries. The pain often is referred to as a pressure sensation with burning, and there may be radiation to the neck, shoulder, or arm. As for the boy described in the vignette, the pain typically occurs during or following exercise or activity, and it may improve with rest. The clinician should consider myocardial ischemia strongly in the patient who has had Kawasaki disease or cardiac surgery. It also is important to consider use of illicit drugs, such as cocaine and other adrenergic stimulators, as potential causes of coronary vasospasm. Some of the tachyarrhythmias, such as supraventricular tachycardia, may present with chest pain, although this usually is described as a discomfort associated with palpitation and other symptoms. Although it may be associated with exercise, this is not the typical presentation. Pain from aortic dissection is usually acute and sharp and may present in the anterior chest or the back, depending on the area of the aorta that is affected. A history of Marfan syndrome or Ehlers-Danlos syndrome should be pursued in affected patients and their families. The pain experienced by the patient in the vignette has several qualities that suggest angina and possible myocardial ischemia. The best management plan is to seek cardiology referral, with a complete restriction of all activity until his evaluation is complete. Often, light activity and sports practice are as strenuous as games and competition. Imaging studies and electrocardiography, although important to the evaluation, should not be used as a surrogate for a complete history, physical examination, and specialized diagnostic testing from those trained in the evaluation of cardiac disease in children and adolescents. Similarly, although exerciseinduced bronchospasm is common, the concerning history of pressure-like pain referred to the left arm in this patient mandates a complete cardiac evaluation prior to initiation of other therapies. References: Cava JR, Sayger PL. Chest pain in children and adolescents. Pediatr Clin North Am. 2004;51:1553-1568. Abstract available at: http://www.ncbi.nlm.nih.gov/entrez/query.fcgi?orig_db=PubMed&db=PubMed&cmd=Search&ter m=%22Pediatric+clinics+of+North+America%22[Jour]+AND+1553[page]+AND+2004[pdat] Fahey J. Chest pain. In: Rudolph CD, Rudolph AM, Hostetter MK, Lister G, Siegel NJ, eds. Rudolphs Pediatrics. 21st ed. New York, NY: McGraw Hill; 2003:1894-1897

Copyright 2007 by the American Academy of Pediatrics

page 214

2007 PREP SA on CD-ROM Question: 104

The pregnant mother of a 3-year-old girl in your practice is concerned because her obstetrician just told her that her serum alpha-fetoprotein value is markedly increased. Of the following, the MOST appropriate advice to give to this woman is that she should consider

A. having an ultrasonographic examination to date her pregnancy and to search for fetal
anomalies

B. having another blood sample drawn to repeat the test C. having chorionic villus sampling as soon as possible to determine the chromosome
complement of the fetus

D. obtaining further testing only if she is older than age 35 E. being concerned only if there is a history of open neural tube defects in her family

Copyright 2007 by the American Academy of Pediatrics

page 215

2007 PREP SA on CD-ROM Critique: 104 Preferred Response: A

Alpha-fetoprotein (AFP) is produced by the fetal liver and crosses the placenta to enter the maternal circulation. Any defect that causes a breech in fetal skin can result in increased levels of AFP in the maternal circulation due to leaking of the protein. These fetal defects include open neural tube defects, anencephaly, gastroschisis, and omphalocele. Maternal serum AFP (MSAFP) concentrations also can be increased in twin pregnancies, those in which there is fetal demise or impending fetal demise, and in pregnancies in which the fetus has congenital nephrosis. Because the measurement of MSAFP is simple and inexpensive, the American College of Obstetricians and Gynecologists, the American Society of Human Genetics, and the American Academy of Pediatrics recommend offering MSAFP screening to all pregnant women at 16 to 18 weeks of gestation. However, such screening should be undertaken only if there is adequate counseling, access to high-quality laboratory services, and appropriate facilities for follow-up testing (ie, qualified diagnostic centers that offer conventional and high-resolution ultrasonography and amniocentesis). More recently, other biochemical markers have been added to this screening test to permit the identification of pregnancies at increased risk for chromosomal abnormalities, most notably trisomy 21. Depending on the cutoff used by the laboratory to define an elevated MSAFP concentration, which is usually 2 to 2.5 times the median value for gestational age, MSAFP screening detects most fetuses that have open neural tube defects. However, because it is a screening test, MSAFP results are abnormal in approximately 1% to 5% of pregnant women. Because the incidence of open neural tube defects is generally 1 in 1,000 or less, most findings of elevated MSAFP are due to other reasons (eg, incorrect dating of the pregnancy, other congenital anomalies, intrauterine growth restriction, multiple gestations, fetal demise). Incorrect dating is the most common reason for a false-positive MSAFP result, and ultrasonography should be used to date the pregnancy. If the dating of the pregnancy is changed by ultrasonography, the MSAFP value should be recalculated to reassess the risk. If the dating is correct, the patient should be offered high-resolution fetal ultrasonography to search for anomalies. If ultrasonography does not provide an explanation for the abnormal result (as occurs in about 50% of cases), amniocentesis should be offered to measure amniotic fluid AFP. Some laboratories request a second sample after an initial elevated MSAFP finding; others proceed immediately to follow-up ultrasonography. In general, second samples should be obtained only if the initial MSAFP concentration is minimally elevated and there is sufficient time for processing a second specimen. Clinical trials in pregnant women who have had a prior pregnancy affected by a neural tube defect have demonstrated that folic acid supplements substantially reduce the risk of recurrent neural tube defects. In one such trial, administration of 4 mg of folic acid daily, beginning at least 1 month before conception through the first trimester, reduced the recurrence risk of neural tube defects from 3.5% to 1.0%. Trials in women who have not had a prior affected pregnancy also have shown a beneficial effect. It is now recommended that all women of childbearing age take folic acid supplements to reduce the risk for neural tube defects. All women still should undergo maternal serum screening because folic acid supplements do not prevent all neural tube defects. Because elevated MSAFP concentrations are associated primarily with open neural tube defects, which usually do not result from chromosome abnormalities, chorionic villus sampling (performed at 10 to 12 weeks gestation), which is used to determine fetal karyotype, is not indicated for the woman described in the vignette. In addition, AFP cannot be measured in chorionic villi. Open neural tube defects are among the most common birth defects, occurring in approximately 1 in 1,000 pregnancies. In most cases, there is no family history, although those who do have a positive family history may be at increased risk over the general population. There is no association between the occurrence of neural tube defects and maternal age. References: American Academy of Pediatrics Committee on Genetics. Maternal serum alpha-fetoprotein screening. Pediatrics. 1991;88:1282-1283

Copyright 2007 by the American Academy of Pediatrics

page 216

2007 PREP SA on CD-ROM

Canick JA, Messerlian GM, Farina A. General principles of second trimester maternal serum screening for Down syndrome. UpToDate. 2006;14.1. Available at: http://www.utdol.com/utd/content/topic.do?topicKey=antenatl/12590&type=A&selectedTitle=3~54 Cunniff C, Committee on Genetics. Prenatal screening and diagnosis for pediatricians. Pediatrics. 2004;114:889-894. Available at: http://pediatrics.aappublications.org/cgi/content/full/114/3/889

Copyright 2007 by the American Academy of Pediatrics

page 217

2007 PREP SA on CD-ROM Question: 105

A 12-year-old basketball player has complained of right knee pain immediately after games for 2 weeks. Although he had complained of knee pain intermittently earlier in the season, there is no history of swelling or injury of the knee. Physical examination findings of the knee are remarkable for full range of motion no effusion, and swelling over the proximal right tibia (Item Q105A) that is tender to palpation. Of the following, the MOST likely diagnosis is

A. jumper's knee B. Osgood-Schlatter disease C. patellofemoral syndrome D. prepatellar bursitis E. Sinding-Larsen Johansson disease

Copyright 2007 by the American Academy of Pediatrics

page 218

2007 PREP SA on CD-ROM Critique: 105 Preferred Response: B

The chronic knee pain and swelling and tenderness to palpation over the area of the tibial tuberosity described for the young adolescent in the vignette are consistent with OsgoodSchlatter disease, an overuse injury that is a common cause of chronic anterior knee pain in young athletes. During adolescence, the tibial tuberosity develops as a specialized growth center called an apophysis. The tuberosity is the site of attachment of the quadriceps muscle via the patellar tendon. Repetitive traction stress applied to the apophysis results in inflammation or small fractures, with resultant pain and swelling (Item C105A). Pain is aggravated by activities that involve quadriceps contraction (eg, running and jumping) or by kneeling on the tuberosity. The disease may be bilateral, although unilateral presentations or predominance of pain are more common. The onset of Osgood-Schlatter disease often coincides with the period of rapid adolescent growth and poses a chronic, intermittent problem for several months or occasionally years before abating. After symptoms resolve, permanent prominence of the tuberosity may remain. The diagnosis generally is made clinically, although a radiograph may be obtained when symptoms or history are strikingly atypical or unilateral to rule out a bony neoplasm or complication (eg, avulsion of the tibial tubercle or a loose ossicle in the patellar tendon). Prepatellar bursitis, a cause of anterior knee pain, presents with swelling overlying the patella that is visible and palpable. The clinician usually can demonstrate full extension of the knee. The bursitis commonly is the result of direct trauma and friction or repetitive flexion and extension overuse that presents as an abrupt and rapid swelling. The mainstay of therapy is rest and the use of anti-inflammatory medication. Protective pads are recommended for use in sports that are associated with knee friction or trauma (eg, volleyball, wrestling). Patients who have very large swelling or recurrent episodes may need to be referred to an orthopedist for evaluation that may include aspiration, analysis, and culture of the fluid. The differential diagnosis includes septic bursitis that is commonly caused by Staphylococcus aureus. The term jumpers knee is given to knee extensor tendonitis (patellar and quadriceps tendonitis), which affects older and more skeletally mature adolescents. The associated pain is localized to the superior or inferior pole of the patella. Pain at the inferior pole may be caused by a partial tear of deep layers of the patellar tendon. Sinding-Larsen-Johansson disease also is associated with tenderness at the inferior pole of the patella rather than over the anterior tibial tuberosity, as in Osgood-Schlatter disease. Similar to Osgood-Schlatter disease, it is believed to be an apophysitis caused by repetitive stress injuries at the junction of the patella and the patellar tendon during early adolescent growth. Patellofemoral syndrome is another common cause of chronic anterior knee pain. Physical examination findings consistent with this diagnosis are tenderness to palpation of the underside of the patella medially and laterally, crepitus with flexion and extension, and pain with patellar compression. References: Fein FD, Ching D, Kim E. Septic bursitis: experience in a community practice. Orthopedics. 1991;14:981-984. Abstract available at: http://www.ncbi.nlm.nih.gov/entrez/query.fcgi?orig_db=PubMed&db=PubMed&cmd=Search&ter m=%22Orthopedics%22[Jour]+AND+981[page]+AND+1991[pdat] Hogan KA, Gross RH. Overuse injuries in pediatric athletes. Orthop Clin North Am. 2003;34:405415. Abstract available at: http://www.ncbi.nlm.nih.gov/entrez/query.fcgi?orig_db=PubMed&db=PubMed&cmd=Search&ter m=%22The+Orthopedic+clinics+of+North+America%22[Jour]+AND+405[page]+AND+2003[pdat] Pasque CB, McGinnis DW. Knee. In: Sullivan JA, Anderson SJ, eds. Care of the Young Athlete. Elk Grove Village, Ill: American Academy of Orthopaedic Surgeons and American Academy of Pediatrics; 2000:377-404

Copyright 2007 by the American Academy of Pediatrics

page 219

2007 PREP SA on CD-ROM Question: 106

You see a 7-day-old baby boy the day that his newborn screening test is reported to show a low thyroxine concentration of 7.8 mcg/dL (100.4 nmol/L) and a thyroid-stimulating hormone concentration of 25 mcU/mL. The baby had a birthweight of 3,000 g and now weighs 3,100 g and is 48.3 cm long. He looks healthy, and the thyroid is not palpable. The mother reports that she is breastfeeding, and the baby seems to be feeding well. He is her first child. The mother tells you that she takes thyroid hormone for an underactive thyroid and has needed to take calcium and vitamin D since she was a small child "to keep her calcium up." You look at her more closely and realize that she is plump, quite short (perhaps 4 ft 10 in), has a round face, and has short stubby fingers. She says that her mother also had short stature and similar problems. She has one brother who has similar problems and a sister who is 5 ft 6 in and does not have a problem with her calcium. Of the following, this baby and his mother MOST likely have

A. McCune Albright syndrome B. multiple endocrine autoimmune syndrome C. pseudohypoparathyroidism D. Noonan syndrome E. vitamin D resistance

Copyright 2007 by the American Academy of Pediatrics

page 220

2007 PREP SA on CD-ROM Critique: 106 Preferred Response: C

Pseudohypoparathyroidism is an autosomal dominant disorder usually associated with mutations in the stimulatory G protein alpha - subunit. As a result, the body does not respond to parathyroid hormone and other hormone effects mediated through this G protein mechanism. Individuals who have this disorder tend to be short and stocky, with short fourth metacarpals and metatarsals (brachydactyly) (Item C106A) and relatively slow mentation or mild mental retardation. Because the disorder is imprinted, children born to women who have this disorder have the full clinical expression with hypocalcemia and hyperphosphatemia, but children born to men who have the disorder have the phenotype without the signs and symptoms of hormone resistance (sometimes termed pseudopseudohypoparathyroidism). Genomic imprinting is the mechanism for sex-linked transmission of some specific genetic information. It is the result of zygotic alterations in the methylation pattern of genes that leads to inactivation of some genes derived from the maternal or paternal lines. Examples of other imprinted disorders include Beckwith-Wiedemann syndrome, Prader-Willi syndrome, and Angelman syndrome. Although hypocalcemia and subcutaneous ossifications are the most common manifestations of pseudohypoparathyroidism, resistance to thyroid-stimulating hormone (TSH), gonadotropins, and vasopressin also is common. This leads to elevated TSH concentrations in a newborn as a marker of hypothyroidism. McCune Albright syndrome is a disorder associated with polyostotic fibrous dysplasia, irregular caf au lait skin macules, and sexual precocity. Other endocrine and somatic abnormalities, including hyperthyroidism, Cushing syndrome, and gigantism (growth hormone excess), have been reported. Multiple endocrine autoimmune disorders do not present with the same physical appearance as reported for the mother in the vignette, although hypothyroidism and hypoparathyroidism may be prominent endocrine deficiency disorders as a result of autoimmune destruction of these glands. Noonan syndrome usually is inherited as an autosomal dominant disorder and is associated with short stature, delayed puberty, classic facial appearance (Item C106B), and mild mental retardation in many cases. The disorder is not associated with hypocalcemia or hypothyroidism. Vitamin D-resistant rickets is not associated with hypothyroidism and the physical findings reported for this family. References: Abraham MR, Khardori R. Pseudohypoparathyroidism. eMedicine Specialities: Medicine, Ob/Gyn, Psychiatry, and Surgery: Endocrinology. 2005. Available at: http://www.emedicine.com/med/topic1940.htm Bastepe M, Juppner H. Pseudohypoparathyroidism and mechanisms of resistance toward multiple hormones: molecular evidence to clinical presentation [editorial]. J Clin Endocrinol Metab. 2003;88:4055-4058. Available at: http://jcem.endojournals.org/cgi/content/full/88/9/4055 Weinstein LS, Yu S, Warner DR, Liu J. Endocrine manifestations of stimulatory G protein alphasubunit mutations and the role of genomic imprinting. Endocr Rev. 2001;22:675-705. Available at: http://edrv.endojournals.org/cgi/content/full/22/5/675

Copyright 2007 by the American Academy of Pediatrics

page 221

2007 PREP SA on CD-ROM Question: 107

A 10-year-old child recently had cognitive testing that showed a full scale intelligence quotient of 105. On achievement testing, he performed at the 3rd percentile in reading and at the 50th percentile in math. Of the following, these findings MOST likely represent

A. attention-deficit/hyperactivity disorder B. mental retardation C. poor testing conditions for the achievement test D. specific learning disability E. vision impairment

Copyright 2007 by the American Academy of Pediatrics

page 222

2007 PREP SA on CD-ROM Critique: 107 Preferred Response: D

Learning disabilities are characterized by an unexpected difficulty in one academic area in children who otherwise have the intelligence, motivation, and educational opportunity to learn. The child described in the vignette has normal cognition, with a full scale intelligence quotient of 105 and a normal math performance, but subnormal reading performance on achievement testing, a pattern that is concerning for a specific learning disability. For the boy in the vignette, the disability appears to be in reading. Children who have learning disabilities may appear inattentive in academic situations in which they do not understand the material, and attention-deficit/hyperactivity disorder (ADHD) may be comorbid with learning disability. However, ADHD is unlikely to be the primary diagnosis for the boy in the vignette. Children who have mental retardation have intelligence quotients of less than 70 plus associated limitations in adaptive functioning. Testing conditions for the boy in the vignette are unlikely to have been poor, given his performance on cognitive testing and the math portion of the achievement testing. Only severe vision impairment would affect the ability to read and would have been obvious through history or physical examination. References: Lyon GR, Shaywitz SE, Shaywitz BA. Specific reading disability (dyslexia). In: Behrman RE, Kliegman RM, Jenson HB, eds. Nelson Textbook of Pediatrics. 17th ed. Philadelphia, Pa: WB Saunders Co; 2004:110-112 Silver L. Developmental learning disorders. In: Lewis M, ed. Child and Adolescent Psychiatry: A Comprehensive Textbook. 3rd ed. Philadelphia, Pa: Lippincott Williams & Wilkins; 2002:621-629

Copyright 2007 by the American Academy of Pediatrics

page 223

2007 PREP SA on CD-ROM Question: 108

You are seeing a 14-year-old boy in your office who complains of fever, facial pain, and cough at night. His mother states that he has had problems with sinusitis for the past few months and has required repeated antimicrobial therapy during this period. Physical examination reveals a temperature of 101F (38.3C), yellow-green nasal discharge, and tenderness to palpation of the maxillary and frontal sinus regions. Of the following, the MOST likely pathogen associated with chronic sinusitis is

A. Moraxella catarrhalis B. nontypeable Haemophilus influenzae C. Staphylococcus aureus D. Streptococcus pneumoniae E. Streptococcus pyogenes

Copyright 2007 by the American Academy of Pediatrics

page 224

2007 PREP SA on CD-ROM Critique: 108 Preferred Response: C

The signs and symptoms of chronic sinusitis are similar to those of an acute infection except for persistent (>90 days) nasal discharge, congestion, and cough (day or night), as reported for the boy in the vignette. Other symptoms include headache, sore throat, halitosis, intermittent fever, malaise, and poor appetite. The pathogens associated with chronic sinusitis are similar to those of acute disease (eg, Streptococcus pneumoniae, nontypeable Haemophilus influenzae, Moraxella catarrhalis), but Staphylococcus aureus, other streptococci, fungi, and anaerobes play a more prominent role. Complications of chronic sinusitis include orbital cellulitis and central nervous system extension of infection into the cranium (eg, brain abscess, subdural abscess). Due to the lack of data, treatment of patients who have chronic sinusitis is largely empiric. Antibiotic treatment is problematic because of the large amount of beta-lactamase-producing bacteria in the sinuses, the prevalence of methicillin-resistant S aureus, and the unproven efficacy of therapy. Most experts suggest starting a regimen that would treat acute sinusitis for patients who have chronic sinusitis and are being considered for antibiotic therapy. High-dose amoxicillin (75 to 90 mg/kg per day) or amoxicillin-clavulanate is appropriate. Alternative treatments include cephalosporins (eg, cefprozil, cefuroxime axetil, cefpodoxime), clindamycin, and trimethoprim-sulfamethoxazole. The use of decongestants, antihistamines, mucolytics, and intranasal corticosteroids is controversial; most experts suggest that they have no role in the treatment of chronic sinusitis in children. References: Pappas DE, Hendley JO. Sinusitis. In: Behrman RE, Kliegman RM, Jenson HB, eds. Nelson Textbook of Pediatrics. 17th ed. Philadelphia, Pa: WB Saunders Co; 2004:1391-1393 Steele RW. Chronic sinusitis in children. Clin Pediatr (Phila). 2005;44:465-471. Abstract available at: http://www.ncbi.nlm.nih.gov/entrez/query.fcgi?orig_db=PubMed&db=PubMed&cmd=Search&ter m=%22Clinical+pediatrics%22[Jour]+AND+465[page]+AND+2005[pdat]

Copyright 2007 by the American Academy of Pediatrics

page 225

2007 PREP SA on CD-ROM Question: 109

An 11-year-old Caucasian boy who has no significant past medical history presents to the emergency department with a 3-day history of brown urine. He reports no dysuria, urgency, frequency, or abdominal or flank pain. His vital signs reveal: temperature, 99F (37.2C); blood pressure, 141/84 mm Hg; heart rate, 92 beats/min; and respiratory rate, 24 breaths/min. Significant findings on physical examination include moderate periorbital and leg edema. His urinalysis reveals moderate blood and 4+ protein. The serum complement 3 (C3) and C4 concentrations are both low. Of the following, the MOST likely cause of his hematuria is

A. immunoglobulin A nephropathy B. membranoproliferative glomerulonephritis C. postinfectious acute glomerulonephritis D. urinary tract infection E. urolithiasis

Copyright 2007 by the American Academy of Pediatrics

page 226

2007 PREP SA on CD-ROM Critique: 109 Preferred Response: B

Gross hematuria is defined as discolored urine. The precise color may aid in establishing an origin of the blood. It is important to recognize that not all patients suspected of having gross hematuria have red blood cells (RBCs) in the urine. Indeed, the urinalysis in some patients who have presumed gross hematuria may lack RBCs, suggesting the presence of myoglobin, hemoglobin, or porphyrins, all of which may discolor the urine. Additionally, some foods (eg, beets), drugs, or additives (red dye) may discolor the urine. A history should be obtained to ascertain a history of renal disease or urinary tract malformation; the duration of gross hematuria; associated symptoms such abdominal, flank, or suprapubic pain (suggesting an infection or renal malformation); fever (suggesting an infection); or the passage of sand, gravel, or stones. The next step in determining the cause of gross hematuria is to obtain a urinalysis to assess for blood, protein, and RBCs. The association of blood and protein in the urine with an elevated blood pressure reported for the boy in the vignette strongly indicates new-onset glomerular disease. The low serum complement values are consistent with acute glomerulonephritis (AGN). Of the options listed, only membranoproliferative GN (MPGN) and postinfectious AGN (PIAGN) result in reduced serum complements, and of these two conditions, the C3 and C4 are reduced only in patients who have MPGN. Although PIAGN is much more common than MPGN, only C3 is low in PIAGN. Patients who have immunoglobulin A nephropathy typically develop recurrent episodes of gross hematuria, but the serum complement values are normal. Patients who have infections may develop gross hematuria, but this is rare and usually is limited to viral cystitis. Finally, gross hematuria is common in patients who have nephrolithiasis, but they usually experience pain and do not develop significant proteinuria or hypocomplementemia. One of the most common causes of gross hematuria in children is hypercalciuria, with or without renal stones. Hypercalciuria usually is idiopathic, but it may be due to causes of hypercalcemia (eg, hyperparathyroidism, malignancy, Addison disease, sarcoidosis, vitamin D therapy) or result from increased urinary excretion of calcium (eg, furosemide therapy, Bartter syndrome, Dent disease [inherited hypercalciuria], distal renal tubular acidosis). Gross hematuria may occur in patients who have sickle cell disease or trait due to intrarenal sickling and sludging. References: Bergstein J, Leiser J, Andreoli S. The clinical significance of asymptomatic gross and microscopic hematuria in children. Arch Pediatr Adolesc Med. 2005;159:353-355. Abstract available at: http://www.ncbi.nlm.nih.gov/entrez/query.fcgi?orig_db=PubMed&db=PubMed&cmd=Search&ter m=%22Archives+of+pediatrics+%26+adolescent+medicine%22[Jour]+AND+353[page]+AND+20 05[pdat] Feld LG, Waz WR, Perez LM, Joseph DB. Hematuria. An integrated medical and surgical approach. Pediatr Clin North Am. 1997;44:1191-1210. Abstract available at: http://www.ncbi.nlm.nih.gov/entrez/query.fcgi?cmd=Retrieve&db=pubmed&dopt=Abstract&list_ui ds=9326958&query_hl=39&itool=pubmed_docsum Patel HP, Bissler JJ. Hematuria in children. Pediatr Clin North Am. 2001;48:1519-1537. Abstract available at: http://www.ncbi.nlm.nih.gov/entrez/query.fcgi?orig_db=PubMed&db=PubMed&cmd=Search&ter m=%22Pediatric+clinics+of+North+America%22[Jour]+AND+1519[page]+AND+2001[pdat]

Copyright 2007 by the American Academy of Pediatrics

page 227

2007 PREP SA on CD-ROM Question: 110

You are evaluating a 2-year-old boy in your office for recurrent cellulitis of his right thigh. The patient has had three episodes in the last 4 months. According to his mother, all the episodes start with a "red bump" that progressively enlarges and, in most cases, drains spontaneously. He has received two courses of cephalexin in the past, but there was no clinical improvement until the abscess spontaneously drained. On one occasion, an incision and drainage procedure had to be performed. Except for pain with walking, the patient has been afebrile and experienced no other systemic symptoms. On physical examination, you note a 6x6 cm area of induration and erythema on the lateral right thigh that is warm, firm, and tender to palpation. There is no active drainage from the site. Of the following, the MOST appropriate antibiotic for treatment of this patient is

A. amoxicillin B. amoxicillin/clavulanic acid C. clindamycin D. cefdinir E. cephalexin

Copyright 2007 by the American Academy of Pediatrics

page 228

2007 PREP SA on CD-ROM Critique: 110 Preferred Response: C

The most common etiologic agents associated with skin and soft-tissue infections are Staphylococcus aureus and group A streptococci. The patient described in the vignette has had recurrent episodes of cellulitis and abscess that did not respond to cephalexin, a beta-lactam antibiotic. He most likely has an infection that is caused by a strain of community-acquired methicillin-resistant S aureus. Over the last 5 years, this organism has emerged as the predominant pathogen causing skin and soft-tissue infections nationwide. The pathogen is not susceptible to any of the beta-lactam antibiotics, including amoxicillin, amoxicillin/clavulanic acid, cefdinir, and cephalexin. Clindamycin, trimethoprim/sulfamethoxazole, vancomycin, the tetracyclines, linezolid, and levofloxacin are the drugs of choice for the treatment of infections caused by these organisms. However, because of growing resistance to these drugs, susceptibility testing should be performed, especially when treating staphylococcal infections. Staphylococci that are susceptible to clindamycin but resistant to erythromycin may exhibit inducible clindamycin resistance (resistance develops while the patient is receiving clindamycin therapy), which should be confirmed using a double-disk diffusion test. Regardless of the antibiotic selected, appropriate management of an abscess includes incision and drainage. Clindamycin is a semisynthetic antibiotic that is a member of the lincosamide family of antibiotics whose mechanism of action is binding to the bacterial 50S ribosomal subunit, thereby interfering with protein synthesis. It is considered a bacteriostatic agent but has bactericidal activity against some strains of staphylococci, streptococci, and anaerobes, including Bacteroides fragilis, depending on drug concentration and bacterial growth conditions. It also has demonstrated the ability to inhibit production of staphylococcal exotoxin associated with toxic shock syndromes and to facilitate opsonization and phagocytosis of bacteria, even at low drug concentrations. The drug is well absorbed following oral administration, reaching peak serum concentrations within 1 hour, with intravenous administration resulting in peak concentrations within 20 to 45 minutes. Clindamycin reaches therapeutic concentrations in most body fluids and tissues, including bones and joints; exceptions include cerebrospinal fluid and bile. The drug is metabolized primarily in the liver, with most of the parent drug and its metabolites eliminated in the bile. Clindamycin is active against most gram-positive cocci, most anaerobes, and certain protozoa. Of the aerobic gram-positive cocci, it is active against streptococci (groups A, B, C and G; S bovis; microaerophilic streptococci; and most strains of pneumococci and viridans streptococci), staphylococci (methicillin-sensitive S aureus, certain strains of methicillin-resistant S aureus, and S epidermidis), and Corynebacterium diphtheriae. However, the drug has no activity against enterococci. Among the anaerobes, it is active against most gram-positive cocci (Peptostreptococcus sp), gram-positive nonspore-forming bacilli (Actinomyces sp, Propionibacterium sp), clostridia (excluding C difficile and a significant percentage of some nonperfringens clostridial species), and gram-negative bacilli (Bacteroides, Prevotella, Fusobacterium sp). It also has activity against Chlamydia trachomatis and certain protozoal pathogens (when used in combination with other agents), such as Plasmodium sp, Pneumocystis carinii, Toxoplasma gondii, and Babesia sp. References: Bradley J. Newer antistaphylococcal agents. Curr Opin Pediatr. 2005;17:71-77. Abstract available at: http://www.ncbi.nlm.nih.gov/entrez/query.fcgi?orig_db=PubMed&db=PubMed&cmd=Search&ter m=%22Current+opinion+in+pediatrics%22[Jour]+AND+71[page]+AND+2005[pdat] Falagas ME, Gorbach SL. Clindamycin and metronidazole. Med Clin North Am. 1995;79:845867. Available at: http://www.ncbi.nlm.nih.gov/entrez/query.fcgi?orig_db=PubMed&db=PubMed&cmd=Search&ter m=%22The+Medical+clinics+of+North+America%22[Jour]+AND+845[page]+AND+1995[pdat] Kasten MJ. Clindamycin, metronidazole, and chloramphenicol. Mayo Clin Proc. 1999;74:825-

Copyright 2007 by the American Academy of Pediatrics

page 229

2007 PREP SA on CD-ROM

833. Available at: http://www.ncbi.nlm.nih.gov/entrez/query.fcgi?db=pubmed&cmd=Retrieve&dopt=Abstract&list_ui ds=10473362&query_hl=11&itool=pubmed_docsum Marcinak JF, Frank AL. Treatment of community-acquired methicillin-resistant Staphylococcus aureus in children. Curr Opin Infect Dis. 2003;16:265-269. Abstract available at: http://www.ncbi.nlm.nih.gov/entrez/query.fcgi?orig_db=PubMed&db=PubMed&cmd=Search&ter m=%22Current+opinion+in+infectious+diseases%22[Jour]+AND+265[page]+AND+2003[pdat] Weingarten-Arams J, Adam HM. In brief: clindamycin. Pediatr Rev. 2002;23:149-150. Available at: http://pedsinreview.aappublications.org/cgi/content/full/23/4/149

Copyright 2007 by the American Academy of Pediatrics

page 230

2007 PREP SA on CD-ROM Question: 111

You are seeing a 2-month-old male infant who has trisomy 21 for a health supervision visit. The boy's mother expresses concern that the infant has been having "noisy breathing" during the past 2 to 3 weeks. The infant has been exclusively formula-fed and has had no choking or difficulty feeding. According to the mother, the noise, which occurs on inspiration, is louder when the infant is supine and when crying. She has not noticed any rhinorrhea, cough, or other upper respiratory viral illness symptoms. The infant was born via an uneventful vaginal delivery that did not require forceps. Apgar scores were 8 and 9 at 1 and 5 minutes, respectively. On physical examination, the infant, whose physical appearance is consistent with trisomy 21, is resting comfortably. His vital signs are appropriate for age, but you hear an audible noise during inspiration. Of the following, the MOST likely explanation for the infant's respiratory symptoms is

A. laryngomalacia B. subglottic tracheal web C. tracheomalacia D. vascular ring E. vocal cord paralysis

Copyright 2007 by the American Academy of Pediatrics

page 231

2007 PREP SA on CD-ROM Critique: 111 Preferred Response: A

When evaluating an infant who presents with noisy breathing, it is important to obtain a thorough birth history and current medical history, observe breathing patterns in different positions, and auscultate the airway. Secondary evaluation tools include radiography, spirometry, and direct airway visualization. In general, breathing noises can be classified as inspiratory, expiratory, or biphasic. Disorders at the level of and superior to the vocal cords result in a harsh, inspiratory noise called stridor (Item C111A). Lesions inferior to the vocal cords typically result in wheezing, a high-pitched expiratory noise. Laryngomalacia is the most common congenital laryngeal abnormality resulting in stridor. Symptoms may begin shortly after birth, although they typically occur between 1 and 2 months after birth. As with the child described in the vignette, infants are happy, thriving, and not having difficulty during feedings, but stridor usually worsens during supine positioning, increased crying or agitation, or a viral illness. Direct visualization of an omega-shaped epiglottis that prolapses (Item C111B) during inspiration is a hallmark of the condition. Severe cases may require surgical correction, although laryngomalacia usually improves spontaneously by 2 years of age. A laryngeal web results from failure of the embryonic airway to recanalize. Most laryngeal webs occur at the level of the vocal cords and present at birth with stridor. The delayed onset of stridor in the infant in the vignette makes a laryngeal web unlikely. Tracheomalacia (Item C111C) results from defective cartilaginous rings causing flaccidity of the tracheal wall. Both tracheomalacia and laryngomalacia present at similar ages and are exacerbated by crying, viral respiratory infections, and supine positioning. However, infants who have tracheomalacia present with wheezing instead of stridor. Extrinsic compression of the trachea by vascular anomalies such as a vascular ring (Item C111D) can result in recurrent wheezing that is worsened with crying, feeding, or neck flexion. The right-sided aortic arch with left ligamentum arteriosum and the double aortic arch account for most cases. Unilateral or bilateral vocal cord paralysis (Item C111E) can be associated with inspiratory stridor, but often results in a weak cry, coughing, and choking. When vocal cord paralysis is suspected, a thorough search for underlying causes (eg, congenital central lesions, atrial enlargement, birth trauma, traumatic forceps delivery, prior surgical procedures) should be undertaken. References: Bertrand P, Navarro H, Caussade S, Holmgren N, Sanchez I. Airway anomalies in children with Down syndrome: endoscopic findings. Pediatr Pulmonol. 2003;36:137-141. Abstract available at: http://www.ncbi.nlm.nih.gov/entrez/query.fcgi?orig_db=PubMed&db=PubMed&cmd=Search&ter m=Pediatr+Pulmonol[Jour]+AND+137[page]+AND+2003[pdat] Boat TF. Chronic or recurrent respiratory symptoms. In: Behrman RE, Kliegman RM, Jenson HB, eds. Nelson Textbook of Pediatrics. 17th ed. Philadelphia, Pa: WB Saunders Co; 2004:14011404 Paston F, Bye M. In brief: tracheomalacia. Pediatr Rev. 1996;17:328 Vicencio AG, Parikh S. Laryngomalacia and tracheomalacia: common dynamic airway lesions. Pediatr Rev. 2006; 27:e33-35. Available at http://pedsinreview.aappublications.org/cgi/content/full/27/4/e33

Copyright 2007 by the American Academy of Pediatrics

page 232

2007 PREP SA on CD-ROM Question: 112

An 11-year-old child is brought to the emergency department with massive trauma after a motor vehicle crash. His Glasgow Coma Scale score is 3. Rapid assessment reveals marked bruising of the face and blood from the mouth, nose, and ears. In addition, the child has large areas of bruising and abrasions across the chest, abdomen, and pelvis as well as bright red blood from the urethral meatus and a deformity of the left femur. Among the initial major interventions are insertion of an endotracheal tube and establishment of two large-bore intravenous lines. Of the following, the MOST appropriate next step is to

A. administer broad-spectrum antibiotics to treat his contaminated wounds B. administer tetanus toxoid C. obtain upright abdominal films to assess for ruptured viscus D. order contrast-enhanced magnetic resonance imaging of his head E. place a Foley catheter into his bladder to monitor urine output

Copyright 2007 by the American Academy of Pediatrics

page 233

2007 PREP SA on CD-ROM Critique: 112 Preferred Response: B

Tetanus toxoid should be administered to all victims of massive trauma who have contaminated wounds and have not received this vaccine within the past 5 years or whose immunization status is not known. For the patient described in the vignette, the widespread areas of abrasion put him at particular risk for tetanus, much like a burn victim. A patient who has blood at the urethral meatus, as described in the vignette, should be assumed to have a urethral injury. Blind placement of a Foley catheter is discouraged because such action may convert a partial urethral tear into a complete transection. Retrograde urethrography should be performed to evaluate for this injury. Upright abdominal films have no role in the evaluation of a patient who has had massive trauma. The cervical, thoracic, and lumbosacral spine must remain immobilized during the initial management of such patients. Optimal radiologic evaluation of the abdomen in major trauma victims is accomplished by computed tomography. Ideally, this should be performed using both intravenous and oral contrast. Computed tomography of the head, not magnetic resonance imaging, is an important part of this patients evaluation. Contrast adds no information to the scan, so it should not be administered. Prophylactic antibiotics are not warranted for the patient in the vignette. Even in the case of severe burns, prophylactic antibiotics do not significantly decrease the risk of secondary infections, and they may select for resistant organisms. References: American Academy of Pediatrics. Tetanus. In: Pickering LK, ed. Red Book: 2003 Report of the Committee on Infectious Diseases. 26th ed. Elk Grove Village, Ill: American Academy of Pediatrics; 2003:611-616 Garcia CT. Genitourinary trauma. In: Fleisher GR, Ludwig S, Henretig FM, eds. Textbook of Pediatric Emergency Medicine. Philadelphia, Pa: Lippincott Williams & Wilkins; 2006:1463-1474 American College of Surgeons. Abdominal trauma. In: Advanced Trauma Life Support Program for Doctors. 6th ed. Chicago, Ill: American College of Surgeons; 1997:157-175

Copyright 2007 by the American Academy of Pediatrics

page 234

2007 PREP SA on CD-ROM Question: 113

A 6-week-old infant presents with excessive tearing of the right eye, as evidenced by overflow of tears onto the eyelid and cheek, and frequent appearance of mucoid material. The child exhibits no systemic symptoms. Of the following, the MOST appropriate primary management is

A. administration of systemic antibiotics B. fluorescein staining of the cornea C. instillation of ophthalmic corticosteroid drops D. nasolacrimal massage and cleansing of the lids E. referral to ophthalmology for nasolacrimal duct probing

Copyright 2007 by the American Academy of Pediatrics

page 235

2007 PREP SA on CD-ROM Critique: 113 Preferred Response: D

Tears enter either the upper or lower punctum at the medial aspect of the eye and drain into the nasolacrimal duct (NLD). Among neonates, obstruction of the NLD frequently occurs at the site where the duct passes through the maxilla into the nose. Some 6% of newborns exhibit obstruction (Item C113A) when the embryonic structures fail to fully canalize. NLD is observed with increased frequency in certain populations, including preterm infants and children who have Down syndrome. Signs of obstruction include pooling or overflowing of tears, which may not be observed until normal tear production develops. Often, there is accumulation of mucoid material at the medial canthus. Stasis of tears in the obstructed NLD results in conditions favorable for bacterial infection. Progression of infection can lead to cellulitis of the overlying soft tissue. Preferred management of the infant described in the vignette is to recommend massage overlying the NLD two to three times daily and cleansing of the lids as necessary. When infection complicates the obstruction of the drainage system, instillation of a topical ophthalmic antibiotic is justified, but systemic antibiotics are not indicated. NLD obstruction typically resolves by 9 to 12 months of age. Persistence of symptoms beyond this time justifies referral to an ophthalmologist for possible probing of the NLD. For the patient in the vignette, the cornea is not involved, and fluorescein staining would not be beneficial. Instillation of a topical ophthalmic corticosteroid is not indicated in the management of this disorder and is associated with significant risks. References: Olitsky SE, Nelson LB. Disorders of the lacrimal system. In: Behrman RE, Kliegman RM, Jenson HB, eds. Nelson Textbook of Pediatrics. 17th ed. Philadelphia, Pa: WB Saunders Co; 2004:2099 Prasad S. Congenital nasolacrimal duct obstruction. Pediatr Rev. 1994:15:88.

Copyright 2007 by the American Academy of Pediatrics

page 236

2007 PREP SA on CD-ROM Question: 114

A 16-year-old girl presents to your office with a recent diagnosis of peptic ulcer. The gastroenterologist has prescribed sucralfate. Of the following, the condition that is a relative contraindication to sucralfate treatment is

A. congenital heart disease B. congenital hepatic fibrosis C. diabetes mellitus D. end-stage renal disease E. steroid-dependent asthma

Copyright 2007 by the American Academy of Pediatrics

page 237

2007 PREP SA on CD-ROM Critique: 114 Preferred Response: D

In 2005, the primary therapy of a gastric or duodenal ulcer involves treatment with a proton pump inhibitor such as omeprazole, lansoprazole, or pantoprazole. If Helicobacter pylori also is present, the pathogen should be eradicated through combination therapy with a proton pump inhibitor (eg, omeprazole, lansoprazole, pantoprazole) and two antibiotics (eg, tetracycline + clarithromycin, amoxicillin + metronidazole, amoxicillin + clarithromycin). At times, other medications, including histamine-2 receptor blockers, antacids, and sucralfate, are used as adjunctive therapy. Histamine-2 blockers reduce acid secretion, antacids neutralize gastric acid, and sucralfate coats the inflamed area and forms a protective barrier. Sucralfate is an aluminum hydroxide complex of sucrose that is effective both for treatment and prevention of peptic ulcers; it is generally less effective as monotherapy compared with either histamine blockers or proton pump inhibitors. It also has been used in a wide variety of other gastrointestinal conditions, including radiation-induced mucositis and proctitis, nonulcer dyspepsia, and bile reflux gastritis. Although the precise mechanism of action of sucralfate is unclear, it is believed to coat the ulcer region, leaving it less susceptible to acid-induced injury. Sucralfate generally is well tolerated, but it may cause constipation and decrease the bioavailability of certain medications (eg, fluoroquinolones, ketoconazole, warfarin). In addition, because it contains aluminum and has been reported to increase plasma aluminum concentrations in patients who have renal failure, it is not recommended for use in such patients. Congenital heart disease, congenital hepatic fibrosis, diabetes mellitus, and steroid-dependent asthma are not contraindications to sucralfate treatment. References: Jacobson K, Chiba N, Chen Y, et al. Gastric acid secretory response in Helicobacter pyloripositive patients with duodenal ulcer disease. Can J Gastroenterol. 2001;15:29-39. Abstract available at: http://www.ncbi.nlm.nih.gov/entrez/query.fcgi?cmd=Retrieve&db=pubmed&dopt=Abstract&list_ui ds=11173324&query_hl=26&itool=pubmed_DocSum Sherrard DJ. Aluminummuch ado about nothing [comment]. N Engl J Med. 1991;324:558-559

Copyright 2007 by the American Academy of Pediatrics

page 238

2007 PREP SA on CD-ROM Question: 115

You are making rounds in the neonatal intensive care unit and discussing the complications of necrotizing enterocolitis (NEC) with the residents. Of the following, the MOST likely long-term complication following NEC is

A. encopresis B. failure to thrive C. gastroesophageal reflux disease D. intestinal stricture E. pneumatosis intestinalis

Copyright 2007 by the American Academy of Pediatrics

page 239

2007 PREP SA on CD-ROM Critique: 115 Preferred Response: D

The diagnosis of neonatal necrotizing enterocolitis (NEC) is based on clinical manifestations (poor feeding, vomiting, gastrointestinal tract bleeding, septic shock); laboratory data and physical findings (thrombocytopenia, abdominal distention); and radiologic evidence of an ileus (paucity of bowel gas), intramural air (intestinal pneumatosis) (Item C115A), portal venous air, or free air in the abdomen indicative of bowel perforation (pneumoperitoneum) (Item C115B). Early radiographic findings may be nonspecific and include dilated loops of intestine and bowel wall thickening consistent with edema. One third of cases lack radiographic findings, and in these, NEC is diagnosed only at surgery or autopsy. Very low-birthweight (<1,500 g) infants are affected most frequently by NEC, with an incidence of approximately 10% in this population in modern neonatal intensive care units (NICUs). The overall mortality for NEC ranges from 10% to 30%. It is a multifactorial disorder, the precise causes of which have not been completely elucidated. Risk factors include prematurity (most frequent), congenital heart disease, early and rapid feeding, hyperosmolar formulas, maternal cocaine use, respiratory distress syndrome, and perinatal asphyxia. The clinical staging of NEC is given in Item C115C. Initial management of NEC includes making the infant nil per os, passing a nasogastric tube and applying low intermittent suction, providing intravenous fluids and nutrition, and administering appropriate antibiotic therapy. Additional monitoring of abdominal radiographic findings and laboratory parameters (complete blood count, platelet count, coagulation profile, and serum electrolyte concentrations) is essential. Surgical intervention may be necessary in up to 50% of infants and includes placing a peritoneal drain, performing an exploratory laparotomy, or bowel resection. Complications from NEC can be separated into acute or short-term problems encountered in the NICU or longer-term problems encountered after weeks or months. Among the short-term concerns are surgical site, anastomotic, or stoma problems; dehiscence or fistula development; infection; and recurrent NEC. Longer-term complications are either related to nutrition and growth (associated with degree of short bowel syndrome, ability to feed enterally, or dependance on parenteral nutrition) and late-occurring surgical complications such as strictures. Strictures (Item C115D) form at the site of either old medically managed NEC or at a surgical anastomotic site. More than 10% of infants who have had NEC develop intestinal strictures. Pneumatosis intestinalis is an acute finding with active NEC. Gastroesophageal reflux disease is no more common in infants who have NEC than in other preterm infants. The incidence of encopresis is not increased in children who have a history of NEC. Failure to thrive may occur in infants who have a history of NEC and short bowel syndrome. Although short bowel syndrome may occur in up to 25% of infants who have NEC, management with balanced enteral and parenteral nutrition largely avoids overt failure to thrive. Recent evidence attests to the negative impact that prematurity combined with NEC and associated lengthy hospitalization has on overall neurodevelopment in extremely low-birthweight infants (<1,000 g). Up to 50% of those who survive NEC experience some neurodevelopmental delay. References: Blakely ML, Lally KP, McDonald S, et al. Postoperative outcomes of extremely low birth-weight infants with necrotizing enterocolitis or isolated intestinal perforation: a prospective cohort study by the NICHD Neonatal Research Network. Ann Surg. 2005;241:984-989. Available at: http://www.annalsofsurgery.com/pt/re/annos/abstract.00000658-20050600000016.htm;jsessionid=Ej4fiXcDG59KsIFpGCZWXdkTOzFTsC7BXVazr8Wl2NPl0lsrxBzx!1861031787!-949856144!9001!-1 Caplan MS, Jilling T. The pathophysiology of necrotizing enterocolitis. NeoReviews. 2001;2:e103e109. Available at: http://neoreviews.aappublications.org/cgi/content/full/2/5/e103 Dimmitt RA, Moss RL. Clinical management of necrotizing enterocolitis. NeoReviews.

Copyright 2007 by the American Academy of Pediatrics

page 240

2007 PREP SA on CD-ROM

2001;2:e110-e117. Available at: http://neoreviews.aappublications.org/cgi/content/full/2/5/e110 Hintz SR, Kendrick DE, Stoll BJ, et al for the NICHD Neonatal Research Network. Neurodevelopmental and growth outcomes of extremely low birth weight infants after necrotizing enterocolitis. Pediatrics. 2005;115:696-703. Available at: http://pediatrics.aappublications.org/cgi/content/full/115/3/696 Springer SC, Annibale DJ. Necrotizing enterocolitis. eMedicine Specialties: Pediatrics: Neonatology. 2002. Available at: http://www.emedicine.com/ped/topic2601.htm Stoll BJ, Kliegman RM. Digestive system disorders: neonatal necrotizing enterocolitis. In: Behrman RE, Kliegman RM, Jenson HB, eds. Nelson Textbook of Pediatrics. 17th ed. Philadelphia, Pa: Saunders; 2004:590-592

Copyright 2007 by the American Academy of Pediatrics

page 241

2007 PREP SA on CD-ROM Question: 116

A 2-year-old boy is brought to your clinic because he has a nighttime cough. According to his mother, several times over the past few months he has awakened with a barking, nonproductive cough that improves by the next morning. She denies fever and rhinorrhea with the episodes. Evaluation of his lungs yields normal results. Of the following, the clinical feature that is MOST suggestive of spasmodic croup rather than recurrent laryngotracheobronchitis in this boy is

A. age of the patient B. barking nature of the cough C. lack of rhinorrhea and fever D. nonproductive nature of the cough E. normal findings on physical examination

Copyright 2007 by the American Academy of Pediatrics

page 242

2007 PREP SA on CD-ROM Critique: 116 Preferred Response: C

Acute laryngotracheobronchitis, or croup, is the most common cause of acute upper airway obstruction in children. It usually is caused by a virus (typically parainfluenza), and it may affect infants, toddlers, and school-age children. Common features include low-grade fever, rhinorrhea, and mild cough, followed 1 to 3 days later by a harsh, nonproductive, barking cough (Item C116A) and inspiratory stridor. Laryngoscopy rarely is indicated, but if performed, reveals erythema and edema of the tracheal epithelium. Symptoms usually abate within 1 week. Recurrent infections may occur, especially in children who attend child care and are repeatedly exposed to viral illnesses. Spasmodic croup is similar to acute laryngotracheobronchitis in that the cough is often barking and nonproductive. It also may affect young children. Allergic and psychological factors are believed to be primarily responsible for spasmodic croup. Therefore, the classic infectious signs of rhinorrhea and fever are not seen. Symptoms may last for only a few hours and generally not as long as with infectious croup. If performed, laryngoscopy reveals boggy, pale edema of the epithelium, with little to no inflammatory changes. References: Cressman WR, Myer CM 3rd. Diagnosis and management of croup and epiglottitis. Pediatr Clin North Am. 1994;41:265-276. Abstract available at: http://www.ncbi.nlm.nih.gov/entrez/query.fcgi?cmd=Retrieve&db=pubmed&dopt=Abstract&list_ui ds=8139876&query_hl=3&itool=pubmed_docsum Malhotra A, Krilov LR. Viral croup. Pediatr Rev. 2001;22:5-12. Available at: http://pedsinreview.aappublications.org/cgi/content/full/22/1/5 Roosevelt GE. Acute inflammatory upper airway obstruction. In: Behrman RE, Kliegman RM, Jenson HB, eds: Nelson Textbook of Pediatrics. 17th ed. Philadelphia, Pa: WB Saunders Co; 2004:1405-1409

Copyright 2007 by the American Academy of Pediatrics

page 243

2007 PREP SA on CD-ROM Question: 117

A 4-year-old boy presents for a health supervision visit, and in the course of the visit, his mother discloses that ever since the birth of his 2-month-old sister, the boy has resumed bedwetting, which had ceased to be a regular occurrence. He is dry during the day and has no stool incontinence. Past medical history reveals that the child has never been hospitalized, had a negative urine culture at age 9 months associated with a fever, and was toilet trained completely at age 3, with only episodic bedwetting about once every month. Physical examination reveals a happy, playful child whose growth and developmental parameters are normal for age and who has a circumcised penis and normal findings on scrotal examination. Urinalysis of a specimen obtained via clean catch urination shows normal results. Of the following, the next BEST step in the management of this boy is to

A. begin imipramine therapy at bedtime B. obtain urine for culture and sensitivity C. perform renal ultrasonography D. reassure the mother that this is most likely a temporary regression E. refer the boy for voiding cystourethrography

Copyright 2007 by the American Academy of Pediatrics

page 244

2007 PREP SA on CD-ROM Critique: 117 Preferred Response: D

Bedwetting (nocturnal enuresis) is primary if the child has never been dry at night. It is secondary when a child has previously sustained dryness at night and subsequently resumes bedwetting. Only 2% to 3% of children who have nocturnal enuresis have a contributing physical problem; most have either diminished bladder capacity (also evidenced by daytime wetting or dribbling) or diminished nocturnal arousal (also evidenced by failure to awaken with wet clothes). This is a maturational problem, and parents should be reassured that the problem generally resolves over time. Essentially, a child who has enuresis is unable to awaken to the sensation of a full bladder or unable to awaken to urinate in the toilet. Secondary enuresis usually represents a relapse of physiologic primary enuresis. True regression due to psychological stressors often results in diurnal enuresis or other behavioral regressions. No specific evaluation with laboratory tests or imaging is required for either primary or secondary enuresis other than a clean voided specimen for urinalysis to exclude diabetes and infection. Enuresis may be treated with expectant management. Other treatment modalities include alarms and medications. Alarms may have a better long-term success rate than medications such as desmopressin, imipramine, or oxybutynin. These medications have good short-term success rates but also are associated with more adverse effects. References: Glazener CMA, Evans JHC, Peto R. Tricyclic and related drugs for nocturnal enuresis in children. The Cochrane Database of Systematic Reviews. 2003;3:CD002117. Available at: http://www.mrw.interscience.wiley.com/cochrane/clsysrev/articles/CD002117/frame.html Lawless MR, McElderry DH. Nocturnal enuresis: current concepts. Pediatr Rev. 2001;22:399407. Available at: http://pedsinreview.aappublications.org/cgi/content/full/22/12/399 Schmitt BD. Nocturnal enuresis. Pediatr Rev. 1997;18:183-191. Available at: http://pedsinreview.aappublications.org/cgi/content/full/18/6/183

Copyright 2007 by the American Academy of Pediatrics

page 245

2007 PREP SA on CD-ROM Question: 118

The parents of a 4-month-old bring the boy to you because of failure to thrive. The baby was born after a term gestation, and there were no complications at delivery. The mother reports that he has a weak suck, and his body seems limp. On physical examination, the child's weight is 4.1 kg, length is 52 cm, and occipitofrontal circumference is 40.5 cm. His penis appears small for age (Item Q118A), and the child exhibits diffuse hypotonia. Deep tendon reflexes are normal. Other findings on the physical examination are normal. Of the following, the test MOST likely to establish this child's diagnosis is

A. arylsulfatase A measurement B. electromyography with nerve conduction velocities C. fluorescent in situ hybridization testing D. magnetic resonance imaging of the brain E. thyroid-stimulating hormone measurement

Copyright 2007 by the American Academy of Pediatrics

page 246

2007 PREP SA on CD-ROM Critique: 118 Preferred Response: C

The limpness or floppiness reported for the infant in the vignette points to hypotonia, which often is disproportionate to weakness. The trunk and extremities of the infant who has hypotonia hang limply when the child is suspended horizontally. When the clinician suspends the child vertically by holding the infants axillae, the baby will slip through the examiners hands, rather than adducting his or her shoulders. Hypotonia is found in a wide variety of pathophysiologic processes arising throughout the motor system. Cerebral, or central, causes of hypotonia include chromosomal abnormalities (Down syndrome, Turner syndrome, Prader-Willi syndrome), metabolic abnormalities (hypothyroidism, leukodystrophies), sepsis, and perinatal trauma (hypoxia-ischemia or intracranial hemorrhage). The most worrisome cause of hypotonia localizing to the brainstem and spine is spinal muscular atrophy. Neuropathy is rarely a cause of infant hypotonia (congenital polyneuropathy, familial dysautonomia), but peripheral hypotonia commonly stems from the neuromuscular junction (botulism, myasthenia gravis, or myasthenic syndrome) or muscle (muscular dystrophy and congenital myopathies). Clues on physical examination often help lead to the diagnosis. Depressed or absent deep tendon reflexes are seen with spinal muscular atrophy, neuropathies, and even botulism. Tongue fasciculations are common with spinal muscular atrophy. Botulism produces rostral greater than caudal weakness. Low birthweight, almond-shaped palpebral fissures, and hypogonadism are typical of Prader-Willi syndrome. Laboratory investigations are targeted toward the disorders suggested by physical examination findings. The child in the vignette has physical findings typical for Prader-Willi syndrome; fluorescent in situ hybridization testing for the gene locus at the 15q11-13 chromosomal region is indicated. Arylsulfatase A testing assesses for metachromatic leukodystrophy, which develops later in the first postnatal year. Measuring thyroid-stimulating hormone is not likely to be useful because the boy lacks signs of hypothyroidism. Electromyography and nerve conduction studies are used to assess a congenital myopathy or spinal muscular atrophy. Magnetic resonance imaging of the brain would be useful if there were a history of perinatal trauma or concern about cerebral dysgenesis. References: Berg BO. The hypotonic infant. In: Child Neurology: A Clinical Manual. 2nd ed. Philadelphia, Pa: JB Lippincott Co; 1994:279-286 Stiefel L. In brief: hypotonia in infants. Pediatr Rev. 1996;17:104-105

Copyright 2007 by the American Academy of Pediatrics

page 247

2007 PREP SA on CD-ROM Question: 119

The mother of a 6-month-old girl calls you because the infant has not been feeding well all day and appears pale. You tell her to meet you at the emergency department. Upon arrival, you find an awake, alert, but irritable child who wants to take her bottle but fatigues quickly thereafter. On physical examination, she has no fever, a respiratory rate of 50 breaths/min, heart rate of 250 beats/min, and blood pressure of 80/50 mm Hg. Her oxygen saturation is 98% in room air. Her lungs are clear, with good aeration. Her rapid pulse is palpable in all extremities, and she has a capillary refill time of 2 seconds. You administer oxygen and place the child on a cardiac monitor, which reveals a rapid heart rate with a narrow QRS complex (Item Q119A). Of the following, the BEST management plan for this infant is

A. carotid massage B. electrical cardioversion C. intravenous administration of adenosine D. intravenous administration of verapamil E. oral administration of digoxin

Copyright 2007 by the American Academy of Pediatrics

page 248

2007 PREP SA on CD-ROM Critique: 119 Preferred Response: C

Any sustained or frequently recurring tachyarrhythmia should prompt medical attention to evaluate the mechanism and cause of the arrhythmia and initiate treatment. The medical treatment of a tachyarrhythmia can be approached in terms of acute and chronic therapy. Acute therapy depends on the patients clinical presentation. The evaluation should begin with assessment of the airway, breathing, and the circulation (ABCs). For patients who have altered consciousness, diminished perfusion, and shock, immediate and emergent therapy to address the ABCs is undertaken. Following this, emergent treatment of the arrhythmia may include the use of electric cardioversion or medications. A more deliberate approach to diagnosis and management can be undertaken for patients who are hemodynamically stable. For arrhythmias that use the atrioventricular node (AVN) as part of the re-entrant circuit, blocking conduction through the AVN can be therapeutic. Increasing vagal tone affects the AVN by slowing conduction through it. Although vagal maneuvers such as ocular pressure and carotid massage are NOT recommended in children, the resulting changes in vagal input on the heart are likely the mechanism by which these maneuvers may be effective. The fastest and safest therapy to block the AVN is the use of intravenous adenosine, which works by interrupting conduction through the AVN. Its half-life is extremely short (seconds), requiring intravenous push administration followed by a bolus of 5 to 10 mL of normal saline, so that the medication quickly reaches the central circulation. Adverse effects of adenosine are generally minor and short-lived and may include flushing and a pressure sensation or feeling of tightness in the chest. The patient described in the vignette has a narrow QRS complex tachycardia that is consistent with a supraventricular tachycardia or paroxysmal atrial tachycardia. Such arrhythmias invariably use the AVN to complete the circuit and, therefore, typically respond well to the use of medications that block conduction through the AVN, such as adenosine. Verapamil, a calcium channel blocker, also affects the AVN, but its use is contraindicated in children younger than 1 year of age because of reports that it can cause hypotension, bradycardia, and even sudden death in infants who have supraventricular tachycardia. Carotid massage is not performed in children for several reasons, including the risk of soft-tissue injury. Oral digoxin would not be used in this case because its onset of action is significantly longer than that of adenosine. References: Balaji S. Supraventricular tachycardia in children. Curr Treat Options Cardiovasc Med. 2000;2:521-528. Abstract available at: http://www.ncbi.nlm.nih.gov/entrez/query.fcgi?orig_db=PubMed&db=PubMed&cmd=Search&ter m=%22Current+treatment+options+in+cardiovascular+medicine%22[Jour]+AND+2000[pdat]+AN D+Balaji+S[author] Chun TU, Van Hare GF. Advances in the approach to treatment of supraventricular tachycardia in the pediatric population. Curr Cardiol Rep. 2004;6:322-326. Abstract available at: http://www.ncbi.nlm.nih.gov/entrez/query.fcgi?orig_db=PubMed&db=PubMed&cmd=Search&ter m=%22Current+cardiology+reports%22[Jour]+AND+2004[pdat]+AND+Chun[author] Dixon J, Foster K, Wyllie J, Wren C. Guidelines and adenosine dosing in supraventricular tachycardia. Arch Dis Child. 2005;90:1190-1191. Abstract available at: http://www.ncbi.nlm.nih.gov/entrez/query.fcgi?cmd=Retrieve&db=pubmed&dopt=Abstract&list_ui ds=16243875&query_hl=29&itool=pubmed_docsum

Copyright 2007 by the American Academy of Pediatrics

page 249

2007 PREP SA on CD-ROM Question: 120

A 42-year-old primigravida is in her 16th week of pregnancy. She is concerned about the possibility of her child having myelomeningocele. Of the following, the MOST useful diagnostic evaluation is

A. amniocentesis B. chorionic villus sampling C. cordocentesis D. fetal ultrasonography E. maternal alpha-fetoprotein screening

Copyright 2007 by the American Academy of Pediatrics

page 250

2007 PREP SA on CD-ROM Critique: 120 Preferred Response: D

Fetal ultrasonography performed between 12 and 24 weeks of pregnancy is most useful for early detection of structural anomalies such as a myelomeningocele. Its sensitivity and specificity as a screening test for fetal anomalies can be improved substantially if the screening is performed by trained, experienced ultrasonographers. It is noninvasive, safe, and accurate in experienced hands. Advantages of early fetal ultrasonography include improved diagnosis of multiple gestation and accurate dating of the pregnancy. This imaging technique may be technically difficult in certain conditions, such as maternal obesity, and the uncertain outcome of some malformations and the late appearance of others may complicate interpretation of findings. Although concentrations of alpha-fetoprotein (AFP) in amniotic fluid are increased in the fetus that has open defects of the spinal cord, this test has limited sensitivity and specificity for the diagnosis of a myelomeningocele. Early amniocentesis, performed between 12 and 14 weeks of pregnancy, is most useful for detecting fetal chromosomal abnormalities such as trisomy 21 and genetic defects such as cystic fibrosis, hemoglobinopathy, and certain inborn errors of metabolism. The desquamated fetal cells (amniocytes) from the amnion provide a source of mitotically active cells that may be used for cytogenetic evaluation or DNA analysis. Midtrimester amniocentesis, performed between 15 and 25 weeks of pregnancy, is similarly useful. The most common indication for amniocentesis performed in the third trimester of pregnancy is assessment of fetal lung maturity. This invasive procedure has potential risks of bleeding, leakage of fluid, cramping, infection, and fetal loss. Chorionic villus sampling, which entails a biopsy of the placenta between 10 and 12 weeks of pregnancy, is not useful for diagnosing a myelomeningocele. It is most helpful for detecting cytogenetic abnormalities in the fetus early in gestation, but there may be an increased risk for complications over amniocentesis (risk is operator-dependent). Cordocentesis, which entails aspiration of fetal blood by direct puncture of the umbilical cord under ultrasonographic guidance, generally is limited to pregnancies of more than 19 weeks gestation and is not useful for the diagnosis of a myelomeningocele. It is most suitable for assessing the fetus that is at risk for acidosis, cytogenetic abnormalities, or hemolytic anemia. Cordocentesis also can aid in diagnosing other conditions, including coagulation disorders, blood cell abnormalities, and congenital infections such as toxoplasmosis. This invasive procedure has potential risks of bleeding, thromboembolism, placental abruption, fetal-maternal hemorrhage, infection, and fetal loss. Although maternal serum AFP screening performed between 15 and 20 weeks of pregnancy is safe and useful for identification of myelomeningocele in the fetus, it does not delineate the extent and location of the lesion. Maternal serum AFP is increased in other fetal conditions, such as abdominal wall defects and urogenital malformations, and it is decreased in Down syndrome. References: ACOG Practice Bulletin No. 58. Ultrasonography in pregnancy. Obstet Gynecol. 2004;104:14491458 Cunniff C, Committee on Genetics. Prenatal screening and diagnosis for pediatricians. Pediatrics. 2004;114:889-894. Available at: http://pediatrics.aappublications.org/cgi/content/full/114/3/889 Ghidini A. Fetal blood sampling: indications and invasive fetal therapy. UpToDate. 2006;14.1. Available at: http://www.utdol.com/utd/content/topic.do?topicKey=antenatl/6605&type=A&selectedTitle=1~12 Ghidini A, McLaren R. Chorionic villus sampling: risks, complications, and techniques. UpToDate. 2006;134.1. Available at: http://www.utdol.com/utd/content/topic.do?topicKey=antenatl/4910&type=A&selectedTitle=1~16

Copyright 2007 by the American Academy of Pediatrics

page 251

2007 PREP SA on CD-ROM Question: 121

A 16-year-old boy presents with a very swollen, painful right knee (Item Q121A). He is a soccer player, but there is no history of recent injury. During the interview, you notice the boy has injected conjunctivae (Item Q121B). Of the following, further evaluation MOST likely will reveal

A. alopecia areata B. Gottron papules C. Kayser-Fleischer rings D. malar rash E. urethritis

Copyright 2007 by the American Academy of Pediatrics

page 252

2007 PREP SA on CD-ROM Critique: 121 Preferred Response: E

The teenage boy described in the vignette displays two physical findings, arthritis and conjunctivitis, that are typical and suggestive of Reiter syndrome (RS). The third clinical finding of the classic triad for RS is urethritis. A finding of leukocyte esterase on a urine dipstick from a male patient suggests the presence of urethritis and is confirmed by the presence of 10 or more polymorphonuclear leukocytes per high-power field in the centrifuged sediment of a first morning void. (Although RS most often is recognized in males, it also may occur in females who have urethritis or cervicitis.) Dermatologic findings of RS are common and include balanitis; painless ulcers on the tongue, palate, pharynx, and buccal mucosa; onycholysis; and vesicles and papules that mimic psoriasis (keratoderma blennorrhagicum). The arthritis associated with RS is oligo/monoarticular, often affecting the knees, ankles, feet, or wrists. RS is believed to result from an autoimmune response to an infectious disease, most commonly the sexually transmitted Chlamydia trachomatis infection. Individuals who have human leukocyte antigen B-27 are at increased risk of developing RS. Although the cause of alopecia areata (Item C121A) is unknown, there is an infrequent association with autoimmune diseases such as Hashimoto thyroiditis, Addison disease, vitiligo, and collagen vascular disease. However, these diseases are unlikely to present with monoarticular arthritis and conjunctivitis. Gottron papules (erythematous papules located over the metacarpal, phalangeal, and interphalangeal joints) (Item C121B) and violaceous discoloration of the upper eyelids (Item C121C) are associated with juvenile dermatomyositis (JDM). Other common clinical findings of JDM include arthralgia, fatigue, and proximal muscle weakness. Kayser-Fleischer rings are found in the corneas of patients who have Wilson disease, a rare genetic disorder of copper metabolism. Monoarticular arthritis and conjunctivitis are not part of the usual clinical picture for JDM or Wilson disease. The finding of a malar rash (Item C121D) is suggestive of systemic lupus erythematosus (SLE), an unlikely diagnosis for the patient described in the vignette. Although musculoskeletal findings of SLE include arthralgia, arthritis, tendonitis, and myositis, the clinical presentation of monoarticular arthritis is not typical, and conjunctivitis is not an associated finding. In addition, SLE presenting after puberty has a female predominance of 8 to 1. References: Amor B. Reiters syndrome: diagnosis and clinical features. Rheum Dis Clin North Am. 1998;24:677-695. Abstract available at: http://www.ncbi.nlm.nih.gov/entrez/query.fcgi?orig_db=PubMed&db=PubMed&cmd=Search&ter m=%22Rheumatic+diseases+clinics+of+North+America%22[Jour]+AND+677[page]+AND+1998[ pdat] Mazziotta JM, Ahmed N. Conjunctivitis and cervicitis. J Fam Prac. 2004;53:121-123. Available at: http://www.jfponline.com/Pages.asp?AID=1648 Shrier LA. Bacterial sexually transmitted infections: gonorrhea, chlamydia, pelvic inflammatory disease, and syphilis. In: Emans SJH, Laufer MR, Goldstein DP, eds. Pediatric and Adolescent Gynecology. 5th ed. Philadelphia, Pa: Lippincott, Williams & Wilkins; 2005:565-614 Simpson T, Oh K. Urethritis and cervicitis in adolescents. Adolesc Med Clin. 2004;15:253-271

Copyright 2007 by the American Academy of Pediatrics

page 253

2007 PREP SA on CD-ROM Question: 122

A mother brings her 14-year-old son to your office complaining that she can never get him to clean his neck and asks you to talk with him. You note that he is overweight, with a body mass index of 32 mg/m2, and he has a Sexual Maturity Rating 3 for pubic hair, with testes that are about 10 mL. He has velvety dark pigmentation (Item Q122A) on the back of his neck and under his arms. On laboratory testing, the boy is most likely to have

A. elevated cortisol concentration B. elevated fasting glucose value C. elevated hemoglobin A1c measurement D. low high-density lipoprotein cholesterol value E. low triglyceride concentrations

Copyright 2007 by the American Academy of Pediatrics

page 254

2007 PREP SA on CD-ROM Critique: 122 Preferred Response: D

The dark velvety skin pigmentation described for the boy in the vignette is acanthosis nigricans (Item C122A) and is a result of insulin stimulation of skin insulin-like growth factor-1 receptors. Hyperinsulinism in obese children is due to insulin resistance induced by adipose tissue and inactivity. Puberty also intensifies insulin resistance so that findings frequently are more obvious after the beginning of sexual maturation. Insulin resistance is associated with a constellation of physical findings, including obesity, increased abdominal girth, hypertension, and acanthosis nigricans. Laboratory findings include low levels of high-density lipoprotein cholesterol, high triglyceride concentrations, and glucose intolerance or frank diabetes. Although Cushing syndrome, which is characterized by elevated cortisol concentrations, could be associated with insulin resistance, acanthosis nigricans, and obesity, it is a relatively rare disorder compared with exogenous obesity and insulin resistance. Frank type 2 diabetes with fasting hyperglycemia or elevated hemoglobin A1c as a marker for hyperglycemia is much less common than insulin resistance with compensatory hyperinsulinism and normal glucose tolerance. References: Burke JP, Hale DE, Hazuda HP, Stern MP. A quantitative scale of acanthosis nigricans. Diabetes Care. 1999;22:1655-1659. Available at: http://care.diabetesjournals.org/cgi/reprint/22/10/1655 Hirschler V, Aranda C, Calcagno Mde L, Maccalini G, Jadzinsky M. Can waist circumference identify children with the metabolic syndrome? Arch Pediatr Adolesc Med. 2005;159:740-744. Abstract available at: http://www.ncbi.nlm.nih.gov/entrez/query.fcgi?orig_db=PubMed&db=PubMed&cmd=Search&ter m=%22Archives+of+pediatrics+%26+adolescent+medicine%22[Jour]+AND+740[page]+AND+20 05[pdat] Ten S, Maclaren N. Insulin resistance syndrome in children. J Clin Endocrinol Metab. 2004;89:2526-2539. Available at: http://jcem.endojournals.org/cgi/content/full/89/6/2526

Copyright 2007 by the American Academy of Pediatrics

page 255

2007 PREP SA on CD-ROM Question: 123

You diagnose attention-deficit/hyperactivity disorder in a 10-year-old boy and recommend treatment with methylphenidate. His mother asks about potential adverse effects of the treatment. Of the following, the MOST common adverse effect of treatment is

A. delayed sleep onset B. depression C. hallucinations D. tics E. weight gain

Copyright 2007 by the American Academy of Pediatrics

page 256

2007 PREP SA on CD-ROM Critique: 123 Preferred Response: A

Most children who receive medication for treatment of attention-deficit/hyperactivity disorder (ADHD) are prescribed stimulant medications, such as methylphenidate or dextroamphetamine. Stimulants generally are considered safe, with generally mild and short-lived adverse effects. The most common adverse effects are decreased appetite, irritability, and delayed sleep onset. Approximately 15% to 30% of children experience motor tics, most of which are transient and do not represent an absolute contraindication to continuing with the medication. Although stimulants do not cause Tourette syndrome, they may unmask the tic symptoms. The safety profile of stimulants is being reviewed, and in February 2006, the United States Food and Drug Administration placed a black box warning for cardiovascular risk, including sudden death and stroke, especially in children who have underlying heart disease. Other medications for ADHD include atomoxetine and tricyclic antidepressants. Atomoxetine is a norepinephrine reuptake inhibitor that had a similar adverse effect profile to stimulants in clinical trials, but has been associated with nausea, hepatic injury, and risk of suicidal ideation. Tricyclic antidepressants typically cause sedation and related anticholinergic adverse effects such as dry mouth and constipation. If the child in the vignette experiences delayed sleep onset, the condition can be managed by changing the timing of the medication dose or decreasing an afternoon dose. Depression and hallucinations are rare adverse effects of stimulant medication. Tics are less common than changes in sleep onset. A child who receives stimulant medication for treatment of ADHD is more likely to have weight loss or lack of weight gain related to decreased appetite than weight gain. References: American Academy of Pediatrics Committee on Quality Improvement, Subcommittee on Attention-Deficit/Hyperactivity Disorder. Clinical practice guideline: diagnosis and evaluation of the child with attention-deficit/hyperactivity disorder. Pediatrics. 2000;105:1158-1170. Available at: http://pediatrics.aappublications.org/cgi/content/full/105/5/1158 American Academy of Pediatrics Subcommittee on Attention-Deficit/Hyperactivity Disorder and Committee on Quality Improvement. Clinical practice guideline: treatment of the school-aged child with attention-deficit/hyperactivity disorder. Pediatrics. 2001;108:1033-1044. Available at: http://pediatrics.aappublications.org/cgi/content/full/108/4/1033 MTA Cooperative Group. National Institute of Mental Health multimodal treatment study of ADHD follow-up: 24-month outcomes of treatment strategies for attention-deficit/hyperactivity disorder. Pediatrics. 2004;113:754-761. Available at: http://pediatrics.aappublications.org/cgi/content/full/113/4/754 Reiff MI, Tippins S, LeTourneau AA. ADHD: A Complete and Authoritative Guide. Elk Grove Village, Ill: American Academy of Pediatrics; 2004 Sims MD. Attention-deficit/hyperactivity disorder. In: Behrman RE, Kliegman RM, Jenson HB, eds. Nelson Textbook of Pediatrics. 17th ed. Philadelphia, Pa: WB Saunders Co; 2004:107-110

Copyright 2007 by the American Academy of Pediatrics

page 257

2007 PREP SA on CD-ROM Question: 124

A 2-year-old boy presents to the emergency department and requires intubation due to apnea. Subsequent tests reveal a diagnosis of meningococcal meningitis. Because of the emergent nature of the intubation, you were not able to put on a mask prior to performing this task. Therefore, you will need to take prophylactic medications to prevent transmission of the organism to you. Of the following, the MOST appropriate antimicrobial agent for prophylaxis is

A. azithromycin B. cefdinir C. doxycycline D. penicillin E. rifampin

Copyright 2007 by the American Academy of Pediatrics

page 258

2007 PREP SA on CD-ROM Critique: 124 Preferred Response: E

Postexposure prophylaxis with rifampin, ceftriaxone, or ciprofloxacin should be given to high-risk contacts of a patient who has meningococcal meningitis. All persons who have had contact with the patients oral secretions or those who have examined the throat are considered high risk. In addition, those living in the same house as the patient or those having significant contact with the patient (eg, best friends, child care, school) also should be considered for prophylaxis. The drug of choice is rifampin (10 mg/kg [maximum, 600 mg] orally every 12 hours for 2 days). Ceftriaxone in a single dose of 125 mg for those younger than 15 years of age and 250 mg for those 15 years of age and older can be used, but it is expensive and should be reserved for patients who are unable to take rifampin or ciprofloxacin (eg, pregnant women). A single dose of ciprofloxacin (500 mg) can be administered to patients older than 18 years of age. Meningococcal vaccine can be used as an adjunct to chemoprophylaxis when outbreaks are caused by a serogroup contained within available vaccines (eg, A, C, Y, W-135). Azithromycin, cefdinir, doxycycline, and penicillin are not indicated for the prevention of meningococcal disease because they do not eradicate nasopharyngeal colonization. Other uses for rifampin include: 1) in combination with antistaphylococcal regimens to treat invasive or recurrent staphylococcal disease (eg, endocarditis, osteomyelitis); 2) in combination with other medications (eg, isoniazid, pyrazinamide) to treat disease due to Mycobacterium tuberculosis; and 3) to eradicate the nasal carriage of other bacteria (eg, Haemophilus influenzae type b, Staphylococcus aureus, Streptococcus pyogenes). References: Adam HM. Rifampin. Pediatr Rev. 2004;25:216-217. Available at: http://pedsinreview.aappublications.org/cgi/content/full/25/6/216 American Academy of Pediatrics. Meningococcal infections. In: Pickering LK, ed. Red Book: 2006 Report of the Committee on Infectious Diseases. 27th ed. Elk Grove Village, Ill: American Academy of Pediatrics; 2006:452-460 Kaye D. Current use for old antibacterial agents: polymyxins, rifampin, and aminoglycosides. Infect Dis Clin North Am. 2004;18:669-689

Copyright 2007 by the American Academy of Pediatrics

page 259

2007 PREP SA on CD-ROM Question: 125

You are asked to evaluate a 15 year-old boy who presented to the emergency department with gross hematuria that began the day before. He ran his first marathon yesterday. He reports no dysuria, urgency, frequency, or abdominal or flank pain. His vital signs and physical examination findings are normal. Urinalysis reveals: specific gravity, 1.010; pH, 6.0; large blood; no protein; 0 to 2 red blood cells; and 0 to 2 white blood cells. A serum creatinine is 2.7 mg/dL (238.7 mcmol/L). His complete blood count is normal. Of the following, the MOST likely cause of his hematuria and renal failure is

A. hemoglobinuria B. immunoglobulin A nephropathy C. myoglobinuria D. urinary tract obstruction E. urolithiasis

Copyright 2007 by the American Academy of Pediatrics

page 260

2007 PREP SA on CD-ROM Critique: 125 Preferred Response: C

Because routine screening of urine has become a common practice, many children who are otherwise asymptomatic are being discovered to have microscopic hematuria. For most of these children who have no other urinary findings and are asymptomatic, it is unlikely that an underlying cause will be ascertained. In contrast, determining a specific cause of gross hematuria in children is far more likely. History and physical examination are essential for assessing the child who has gross hematuria. Pertinent information includes the presence of prior renal disease and symptoms such as fever, pain, or edema. The physical examination should focus on probing for edema, bruising, joint swelling, or rashes. Regardless of the findings, a urinalysis is mandatory. Blood and significant (greater than 1+) protein are strongly suggestive of glomerular or interstitial disease. Microscopic analysis of a centrifuged urine specimen is necessary to determine if red blood cells (RBCs) are present. In the absence of RBCs, other causes of a dipstick positive for blood include hemoglobin, myoglobin, or porphyrins. The likelihood of myoglobinuria is strong for the boy described in the vignette because of the absence of RBCs in the urine and the antecedent strenuous exercise. He probably has developed acute renal failure due to rhabdomyolysis, myoglobin deposition in the kidney, and urinary tract obstruction. Hemoglobinuria is rare in the absence of hemolysis. Immunoglobulin A nephropathy may present with gross hematuria, but RBCs should be detected on microscopic analysis. Although patients who have urinary tract infections often have microscopic hematuria, gross hematuria is unusual. Finally, patients who have urolithiasis may develop gross hematuria, but concomitant colicky pain is usually present. Myoglobinuria is caused by any process that involves excessive destruction of muscle or rhabdomyolysis. This may occur after excessive, strenuous exercise in hot weather. Other causes of myoglobinuria include crush injury to muscles from trauma, mitochondrial disorders, other inherited muscle diseases (eg, carnitine palmitoyltransferase II deficiency, McArdle disease), malignant hyperthermia, or rarely after systemic viral illness or due to medication usage (eg, HMG-CoA reductase inhibitors). References: Allison RC, Bedsole DL. The other medical causes of rhabdomyolysis. Am J Med Sci. 2003;326:79-88. Abstract available at: http://www.ncbi.nlm.nih.gov/entrez/query.fcgi?db=pubmed&cmd=Retrieve&dopt=AbstractPlus&li st_uids=12920439 Malinoski DJ, Slater MS, Mullins RJ. Crush injury and rhabdomyolysis. Crit Care Clin. 2004;20:171-192. Abstract available at: http://www.ncbi.nlm.nih.gov/entrez/query.fcgi?db=pubmed&cmd=Retrieve&dopt=AbstractPlus&li st_uids=14979336 Melli G, Chaudhry V, Cornblath DR. Rhabdomyolysis: an evaluation of 475 hospitalized patients. Medicine. 2005;84:377-385. Abstract available at: http://www.ncbi.nlm.nih.gov/entrez/query.fcgi?cmd=Retrieve&db=pubmed&dopt=Abstract&list_ui ds=16267412&query_hl=41&itool=pubmed_docsum

Copyright 2007 by the American Academy of Pediatrics

page 261

2007 PREP SA on CD-ROM Question: 126

You are evaluating a 7-year-old boy who has sickle cell disease for a 5-day history of fever, rhinorrhea, headache, and worsening fatigue. Physical examination reveals a tired-appearing, pale boy who has a temperature of 101.5F (38.6C); heart rate of 150 beats/min; very pale conjunctivae and mucous membranes; and a faint diffuse, erythematous, lacy rash that is most prominent on his cheeks and trunk. Laboratory tests show a white blood cell count of 12x103/mcL (12x109/L), with 50% neutrophils, 45% lymphocytes, and 5% monocytes; hemoglobin of 4 g/dL (40 g/L); hematocrit of 16% (0.16); and reticulocyte count of less than 1% (0.01). Of the following, the MOST likely cause for this patient's present illness is infection with

A. coxsackievirus B. cytomegalovirus C. Epstein-Barr virus D. parainfluenza virus E. parvovirus B19

Copyright 2007 by the American Academy of Pediatrics

page 262

2007 PREP SA on CD-ROM Critique: 126 Preferred Response: E

Erythema infectiosum (fifth disease) is caused by human parvovirus B19, a nonenveloped, single-stranded DNA virus that infects only humans and replicates primarily in human erythrocyte precursors. The receptor for the virus is the erythrocyte P antigen, which is found in the red blood cell precursor cell line and in a variety of other tissues, including myocardium, endothelium, placenta, megakaryocyte, and fetal liver. The modes of transmission include contact with respiratory tract secretions, percutaneous exposure to blood or blood products, and vertical transmission from mother to fetus. Most infections are asymptomatic or mild. The most common findings in symptomatic disease are fever in 15% to 30% of patients and a distinctive rash. The facial rash is intensely erythematous, giving the patient a slapped cheek appearance (Item C126A), and often is accompanied by circumoral pallor. A symmetric, maculopapular, lacy rash (Item C126B) also may develop on the trunk, as described for the boy in the vignette, and subsequently spread to involve the buttocks and extremities. The rash can be exacerbated by environmental temperature, exercise, hot baths, stress, or sunlight and can persist for weeks to months. As reported for the boy in the vignette, a brief, mild, nonspecific prodromal illness consisting of fever, malaise, myalgias, and headache often precedes the development of the rash by 7 to 10 days. Arthralgia and arthritis, or both, of the small joints of the hands and occasionally the ankles, knees, and wrists can occur, especially in infected adult women. Among patients who have chronic hemolytic anemias (eg, sickle cell disease, glucose-6phosphate dehydrogenase deficiency, and autoimmune hemolytic anemia) and other conditions associated with low hemoglobin concentrations, including hemorrhage, severe anemia, and thalassemia, parvovirus B19 infection can cause a transient aplastic crisis lasting for up to 10 days. Parvovirus infects and destroys red blood cell precursors preferentially, causing reticulocytopenia and decreased red blood cell synthesis. Although infection with coxsackievirus, cytomegalovirus, Epstein-Barr virus, and parainfluenza virus may present with fever, rhinorrhea, headache, and worsening fatigue, they would not cause severe anemia or the rash described. Parvovirus B19 infection occurring during pregnancy can damage the placenta and leads to infection in approximately 30% of fetuses. This may cause fetal hydrops and death, with the risk of fetal death following infection being between 2% and 6%; the greatest risk for death is when infection occurs during the first half of pregnancy. Parvovirus B19 infection is not believed to be teratogenic, and children who have had infections in utero usually are healthy at follow-up. References: American Academy of Pediatrics. Parvovirus B19 (erythema infectiosum, fifth disease). In: Pickering LK, ed. Red Book: 2006 Report of the Committee on Infectious Diseases. 27th ed. Elk Grove Village, Ill: American Academy of Pediatrics; 2006:484-487 Brown KE. Human parvovirus B19. In: Long SS, Pickering LK, Prober CG, eds. Principles and Practice of Pediatric Infectious Diseases. 2nd ed. New York, NY: Churchill Livingstone; 2003:1101-1104 Chorba T, Coccia P, Holman RC, et al. The role of parvovirus B19 in aplastic crisis and erythema infectiosum (fifth disease). J Infect Dis. 1986;154:383-393. Abstract available at: http://www.ncbi.nlm.nih.gov/entrez/query.fcgi?orig_db=PubMed&db=PubMed&cmd=Search&ter m=%22The+Journal+of+infectious+diseases%22[Jour]+AND+383[page]+AND+1986[pdat] Miller E, Fairley CK, Cohen BJ, Seng C. Immediate and long term outcome of human parvovirus B19 infection in pregnancy. Br J Obstet Gynaecol. 1998;105:174-178. Abstract available at: http://www.ncbi.nlm.nih.gov/entrez/query.fcgi?db=pubmed&cmd=Retrieve&dopt=AbstractPlus&li st_uids=9501782 Young NS, Brown KE. Parvovirus B19. N Engl J Med. 2004;350:586-597

Copyright 2007 by the American Academy of Pediatrics

page 263

2007 PREP SA on CD-ROM Question: 127

A 3-year-old boy presents to the emergency department following the abrupt onset of coughing and wheezing. You order a chest radiograph for evaluation of a suspected foreign body aspiration. Of the following, the MOST appropriate statement regarding foreign body aspiration is that

A. most foreign body aspirations present within 24 hours B. nonfood items (eg, coins, pins, pencaps) are the most common items aspirated by infants and
toddlers

C. the classic triad of cough, wheeze, and decreased breath sounds is present in most cases D. the majority of aspirated foreign bodies are located in the larynx or trachea E. toy balloons are a common cause of foreign body aspirations

Copyright 2007 by the American Academy of Pediatrics

page 264

2007 PREP SA on CD-ROM Critique: 127 Preferred Response: A

The presentation of a foreign body aspiration depends on whether the event was witnessed, the age of the child, the type of object aspirated, the elapsed time since the event, and the location of the object. Most foreign body aspirations (50% to 75%) present and are diagnosed within 24 hours. Although an acute choking or coughing episode accompanied by wheezing is highly suspicious for a foreign body aspiration, some infants and children present with few or no symptoms. The classic triad of coughing, wheezing, and decreased breath sounds is present in less than 50% of cases. Food is the most common item aspirated by infants and toddlers (eg, sunflower seeds, nuts, beans, carrots, corn); nonfood items (eg, coins, paper clips, pins, pen caps) more commonly are aspirated by older children. Although a rare cause of foreign body aspiration, toy balloon aspiration can be fatal. Most aspirated foreign bodies lodge in the right lung rather than the larynx or trachea (Item C127A). Long-term complications of undiagnosed or retained foreign bodies include recurrent pneumonia, atelectasis, and bronchiectasis. References: Holinger LD. Foreign bodies of the airway. In: Behrman RE, Kliegman RM, Jenson HB, eds. Nelson Textbook of Pediatrics. 17th ed. Philadelphia, Pa: WB Saunders Co; 2004:1410-1411 Karakoc F, Karadag B, Akbenlioglu C, et al. Foreign body aspiration: what is the outcome? Pediatr Pulmonol. 2002;34:30-36. Abstract available at: http://www.ncbi.nlm.nih.gov/entrez/query.fcgi?cmd=Retrieve&db=pubmed&dopt=Abstract&list_ui ds=12112794&query_hl=111&itool=pubmed_docsum

Copyright 2007 by the American Academy of Pediatrics

page 265

2007 PREP SA on CD-ROM Question: 128

A 14-year-old girl presents to your office with complaints of a red and "irritated" eye for the past 12 hours. She wears contact lenses, but has not worn them since the previous evening. Her pain and inflammation have continued to worsen despite removing the contact lenses. On physical examination, you note diffuse moderate injection of the bulbar conjunctiva of the left eye. Her extraocular motions and pupillary reflexes are normal. Results of funduscopic examination are normal. There is no discharge. Fluorescein examination reveals diffuse uptake of stain but no evidence of corneal abrasion. Of the following, the MOST appropriate next step in the management of this child is to

A. apply a soft patch to the eye and refer to an ophthalmologist within 2 to 3 days B. arrange for urgent consultation with an ophthalmologist C. prescribe an ophthalmic antibiotic and ask her to return for re-examination in 2 to 3 days D. prescribe no antibiotics but ask her to return for re-examination in 2 to 3 days E. prescribe ophthalmic antihistamine drops

Copyright 2007 by the American Academy of Pediatrics

page 266

2007 PREP SA on CD-ROM Critique: 128 Preferred Response: B

A red eye in a patient who wears contact lenses may represent dangerous corneal infections due to gram-negative bacteria or fungi or to breakdown and ulceration of corneal epithelium. Such patients should have their contact lenses removed and be evaluated by an ophthalmologist within 12 hours. Empiric antibiotics are not recommended; antibiotic therapy should be directed by the ophthalmologist, depending on the problem identified. For those who do not wear contact lenses and who have either corneal abrasion or bacterial conjunctivitis, empiric therapy with a topical antibiotic is appropriate. Common causes for red eye in contact lens wearers include microbial keratitis, contact lensinduced peripheral ulcer (CLPU), and contact lens-induced acute red eye (CLARE). Microbial keratitis is an infection of the cornea characterized by excavation and necrosis of tissue. Inappropriate treatment or a significant delay in treatment can result in visual impairment. Removal of contact lenses and frequent instillation of sterile saline drops generally result in complete recovery from CLPU and CLARE. Fluorescein stain is taken up by damaged ocular epithelial cells. In a corneal abrasion, fluorescein uptake is focal at the site of injury. With microbial keratitis and CLARE, the uptake is both multifocal and diffuse. Prescribing ophthalmic antihistamine drops and asking the patient to return for reexamination in 2 to 3 days are unnecessary delays to further evaluation and treatment by an ophthalmologist. References: Hume EBH, Stapleton F, Willcox MDP. Evasion of cellular ocular defenses by contact lens isolates of Serratia marcescens. Eye Contact Lens. 2003;29:108-112. Abstract available at: http://www.ncbi.nlm.nih.gov/entrez/query.fcgi?orig_db=PubMed&db=PubMed&cmd=Search&ter m=Eye+Contact+Lens[Jour]+AND+108[page]+AND+2003[pdat] Levin AV. EyeRed. In: Fleisher GR, Ludwig S, Henretig FM, eds. Textbook of Pediatric Emergency Medicine. 5th ed. Philadelphia, Pa: Lippincott Williams & Wilkins; 2006:267-271 Kim J. The use of vital dyes in corneal disease. Curr Opin Ophthalmol. 2000;11:241-247. Abstract available at: http://www.ncbi.nlm.nih.gov/entrez/query.fcgi?cmd=Retrieve&db=pubmed&dopt=Abstract&list_ui ds=10977768&query_hl=11&itool=pubmed_docsum Sweeney DF, Jalbert I, Covey M, et al. Clinical characterization of corneal infiltrative events observed with soft contact lens wear. Cornea. 2003;22:435-442. Abstract available at: http://www.ncbi.nlm.nih.gov/entrez/query.fcgi?orig_db=PubMed&db=PubMed&cmd=Search&ter m=Cornea[Jour]+AND+435[page]+AND+2003[pdat]

Copyright 2007 by the American Academy of Pediatrics

page 267

2007 PREP SA on CD-ROM Question: 129

The mother of a 4-year-old otherwise healthy-appearing boy brings him in for evaluation because he had profound generalized body odor (bromhidrosis) for the past several days. Upon careful questioning, the mother recalls an episode of sneezing followed by a 3- to 4-day history of a purulent, blood-tinged nasal discharge. Of the following, the most likely cause of the body odor is

A. allergic rhinitis B. epidermolytic hyperkeratosis C. nasal foreign body D. pachyonychia congenita E. premature adrenarche

Copyright 2007 by the American Academy of Pediatrics

page 268

2007 PREP SA on CD-ROM Critique: 129 Preferred Response: C

Insertion of small objects such as beads, berries, seeds, and paper into the nares is a common practice of young children. Undetected nasal foreign bodies can lead to local irritation, mucosal edema, and possible infection, producing a bloody, mucous discharge. This sequence of events frequently results in a noticeably unpleasant odor that may be perceived as generalized body odor (bromhidrosis), as described for the boy in the vignette. Allergic rhinitis generally produces thin, watery secretions that occur seasonally or yearround. Bromhidrosis is not present. Epidermolytic hyperkeratosis (EHK), an autosomal dominant disorder, is characterized by extensive skin scaling at birth, erythroderma, and recurrent bullae formation. By school age, bromhidrosis may result, as thick, dirty-yellow scales (Item C129A) develop on palms, soles, elbows, knees and become malodorous due to bacterial colonization. Purulent nasal secretions are not characteristic of EHK. Pachyonychia congenita is a rare genodermatosis in which combinations of ectodermal defects produce thickened, tented nails (Item C129B); palmoplantar hyperkeratosis; and hyperhidrosis that leads to marked foot odor. Premature adrenarche is an early maturational event of adrenal androgen production in which sexual hair appears before the age of 9 years in boys who have no other evidence of pubertal development. Axillary odor is common. References: Garibaldi L. Incomplete (partial) precocious development. In: Behrman RE, Kliegman RM, Jenson HB, eds. Nelson Textbook of Pediatrics. 17th ed. Philadelphia, Pa: WB Saunders Co; 2004:1868 Haddad J Jr. Acquired disorders of the nose. In: Behrman RE, Kliegman RM, Jenson HB, eds. Nelson Textbook of Pediatrics. 17th ed. Philadelphia, Pa: WB Saunders Co; 2004:1387 Weston WL, Lane AT, Morelli JG. Genodermatoses: ichthyosis. In: Color Textbook of Pediatric Dermatology. 3rd ed. St. Louis, Mo: Mosby; 2002:270-274

Copyright 2007 by the American Academy of Pediatrics

page 269

2007 PREP SA on CD-ROM Question: 130

A 15-year-old girl who has juvenile rheumatoid arthritis has been treated with ibuprofen 30 mg/kg per day for 3 months. She has had epigastric abdominal pain for 1 month that has been unresponsive to empiric therapy with omeprazole 20 mg/day. You are considering adding misoprostol 100 mcg four times daily to her current treatment regimen. Of the following, a TRUE statement regarding misoprostol is that the drug

A. frequently causes abdominal pain B. frequently causes constipation C. is a cyclooxygenase-2 inhibitor D. is contraindicated in pregnancy E. is effective treatment for bleeding ulcers

Copyright 2007 by the American Academy of Pediatrics

page 270

2007 PREP SA on CD-ROM Critique: 130 Preferred Response: D

The gastric epithelium is exposed constantly to both gastric acid (secreted by the parietal cells) and the proteolytic enzyme pepsin (secreted by the chief cells). To prevent cell damage and gastric ulcer formation, the gastric epithelium is coated by a protective layer primarily composed of mucin and bicarbonate. Prostaglandin E stimulates mucus production and is important in maintaining the integrity of the protective mucin glycoprotein-bicarbonate layer. The protective layer can be damaged by Helicobacter pylori organisms and nonsteroidal anti-inflammatory drugs (NSAIDs) such as ibuprofen and aspirin, leading to peptic ulcer disease. Misoprostol is a synthetic prostaglandin used to prevent peptic ulcer disease in high-risk patients. It is employed most commonly in patients receiving long-term NSAID therapy, such as the girl described in the vignette, who is taking aspirin for treatment of arthritis. Although misoprostol is effective as a preventive agent, it is not useful in the acute treatment of bleeding peptic ulcers. Common adverse events include nausea and diarrhea; abdominal pain and constipation are not frequent adverse effects. In addition, because it is a synthetic prostaglandin, it may stimulate uterine contractions. Thus, misoprostol can cause spontaneous abortion and is contraindicated in pregnancy. Because ulcer prophylaxis with proton pump inhibitors is more effective and better tolerated than misoprostol, gastroenterologists rarely use misoprostol today. References: Lanza FL. A guideline for the treatment and prevention of NSAID-induced ulcers. Members of the Ad Hoc Committee on Practice Parameters of the American College of Gastroenterology. Am J Gastroenterol. 1998;93:2037-2046 Stupnicki T, Dietrich K, Gonzlez-Carro P, et al. Efficacy and tolerability of pantoprazole compared with misoprostol for the prevention of NSAID-related gastrointestinal lesions and symptoms in rheumatic patients. Digestion. 2003;68:198-208. Available at: http://content.karger.com/produktedb/produkte.asp?typ=fulltext&file=DIG2003068004198

Copyright 2007 by the American Academy of Pediatrics

page 271

2007 PREP SA on CD-ROM Question: 131

The mother of an infant born at 34 weeks' gestation asks you whether any of the medications she was given prior to delivery will have any effect on her daughter's growth and development. Of the following, the MOST correct statement concerning drugs commonly used in labor is that

A. beta-adrenergic tocolytic agent safety for the fetus is inversely related to the dose and duration
of maternal treatment

B. hypoglycemia may complicate the neonatal course of infants whose mothers were treated with
beta-adrenergic tocolytic agents

C. neonatal hypermagnesemia is an asymptomatic incidental electrolyte problem that rarely


follows maternal treatment with magnesium sulfate

D. opioids used for analgesia are safest when administered within 4 hours of delivery E. tocolysis with indomethacin is preferred over beta-adrenergic agents because of its lack of
adverse effects for the fetus and newborn

Copyright 2007 by the American Academy of Pediatrics

page 272

2007 PREP SA on CD-ROM Critique: 131 Preferred Response: B

In addition to antibiotics, numerous medications may be administered to a woman in labor (Item C131A). These may be prescribed for analgesia, management of pregnancy-induced hypertension, seizure prophylaxis, tocolysis, or for fetal indications (eg, to enhance fetal lung maturity). Beta-adrenergic tocolytics (eg, terbutaline) may be administered subcutaneously for shortterm tocolysis or orally for longer-term effects. Beta-adrenergic effects are related directly to the dose and duration of treatment. One of the adverse effects is fetal hyperinsulinemia and subsequent neonatal hypoglycemia. Indomethacin is only recommended for short-term tocolysis (<48 h) and only before 32 weeks completed gestation due to its hemodynamic risks to the fetus. Neonatal hypermagnesemia can follow maternal treatment with magnesium sulfate and is associated with a number of adverse effects. Opioid analgesic risks to the newborn are reduced if more than 4 hours have passed since the mother received her last dose, but, if necessary, can be reversed with naloxone. References: Benitz WE, Druzin ML. Drugs that affect neonatal resuscitation. NeoReviews. 2005;6:e189e195. Available at: http://neoreviews.aappublications.org/cgi/content/full/6/4/e189 Slattery MM, Morrison JJ. Preterm delivery. Lancet. 2002;360:1489-1497. Abstract available at: http://www.ncbi.nlm.nih.gov/entrez/query.fcgi?cmd=Retrieve&db=pubmed&dopt=Abstract&list_ui ds=12433531&query_hl=36&itool=pubmed_docsum

Copyright 2007 by the American Academy of Pediatrics

page 273

2007 PREP SA on CD-ROM Question: 132

A 3-month-boy who has been previously healthy is brought to the emergency department with a 3-day history of rhinorrhea, mild cough, and wheezing. He has been afebrile and has had some difficulty feeding. His pulse oximetry reading is 90% on room air, respiratory rate is 60 breaths/min, and heart rate is 130 beats/min. Chest examination reveals mild subcostal retractions, scattered wheezes, and coarse crackles bilaterally. The rest of the physical examination findings are normal. Of the following, the pathogen that is MOST likely responsible for his symptoms is

A. adenovirus B. Chlamydia trachomatis C. Haemophilus influenzae D. respiratory syncytial virus E. Streptococcus pneumoniae

Copyright 2007 by the American Academy of Pediatrics

page 274

2007 PREP SA on CD-ROM Critique: 132 Preferred Response: D

The acute onset of wheezing, shortness of breath, and rhinorrhea plus nonfocal wheezes and bilateral crackles described for the infant in the vignette are most consistent with acute viral bronchiolitis. Bronchiolitis is a lower respiratory tract illness in which the bronchioles are obstructed with mucus and edema. Clinical features include a mild upper respiratory tract infection with rhinorrhea, occasional low-grade fever, and congestion, followed within 1 to 3 days by wheezing, worsening cough, and dyspnea. Focal findings are not typical, but coarse crackles, wheezing, and transmitted upper airway sounds are prominent. Tachypnea may be marked and may interfere with feeding, and hypoxia is common. Radiographic findings include hyperinflation (Item C132A) and patchy atelectasis. More than 50% of cases of bronchiolitis are caused by respiratory syncytial virus; the remainder are caused by adenovirus, parainfluenza, influenza, and other pathogens. Infants born to mothers infected with Chlamydia trachomatis may present with tachypnea and a persistent, staccato cough at 1 to 3 weeks of age. These infants are typically afebrile, crackles are present on physical examination, and wheezing is rare. Radiographic findings include hyperinflation and interstitial infiltrates. Children who have bacterial pneumonia caused by Haemophilus influenzae or Streptococcus pneumoniae usually present with fever, dyspnea, and focal findings on chest examination, such as decreased breath sounds and rales. Chest radiography reveals a focal infiltrate with normal inflation. References: Goodman D. Inflammatory disorders of the small airway. In: Behrman RE, Kliegman RM, Jenson HB, eds: Nelson Textbook of Pediatrics. 17th ed. Philadelphia, Pa: WB Saunders Co; 2004:14141417 McCarthy CA, Hall CB. Respiratory syncytial virus: concerns and control. Pediatr Rev. 2003;24:301-309. Available at: http://pedsinreview.aappublications.org/cgi/content/full/24/9/301

Copyright 2007 by the American Academy of Pediatrics

page 275

2007 PREP SA on CD-ROM Question: 133

A 2-week-old boy whose birthweight was 5 kg and whom you saw in the nursery presents for evaluation. He is exclusively breastfed. The father is concerned that the child's penis is too small. The testes are descended bilaterally. The stretched penile length is 3.8 cm. Of the following, the MOST appropriate step in the evaluation of the child is to

A. obtain thyroid studies B. reassure the father that the penile length is normal C. obtain a genetics consultation D. obtain a karyotype E. obtain a pediatric urology consultation

Copyright 2007 by the American Academy of Pediatrics

page 276

2007 PREP SA on CD-ROM Critique: 133 Preferred Response: B

Assessment of penile length in a child who has a prominent pubic fat pad requires gentle stretching of the penis to its full length and measuring from the symphysis pubis. For an infant born at term, the mean penile length during the first postnatal 5 months is 3.9 cm. Accordingly, the father in the vignette can be reassured that his sons penile length is normal; no tests or consultations are necessary. If the penile length is less than 2.5 cm (less than 2 standard deviations from the mean), endocrine evaluation for hypothalamic hypopituitarism, androgen insensitivity, or chromosomal disorder is appropriate. For older boys who are overweight, the penis may disappear into the pubic fat pad and must be measured in the manner described previously. Tables for penile length should be consulted for age-appropriate norms. References: Jones KL. Smiths Recognizable Patterns of Human Malformations. 6th ed. Philadelphia, Pa: Elsevier Saunders; 2006:863 Weiner DE. Micropenis. Pediatr Rev. 1994;15:288

Copyright 2007 by the American Academy of Pediatrics

page 277

2007 PREP SA on CD-ROM Question: 134

A father brings in his 6-year-old boy who has chronic constipation because the child now has developed enuresis during both night and day. On physical examination, you discern a small, firm sacral dimple just to the right of the gluteal cleft. The remainder of the physical examination findings are normal. Of the following, the MOST appropriate next step in the evaluation of this child is

A. magnetic resonance imaging of the spine B. radiographs of the abdomen and pelvis C. ultrasonography of the kidneys, ureters, and bladder D. urine culture E. voiding cystourethrography

Copyright 2007 by the American Academy of Pediatrics

page 278

2007 PREP SA on CD-ROM Critique: 134 Preferred Response: A

Occult spinal dysraphism (spina bifida occulta) can be insidious and nonspecific upon initial presentation. The pathology can include syringomyelia, diastematomyelia, a spinal cord tethered to the inferior thecal sac, sacral agenesis, or dermoid sinus. Dermoid sinuses pass from the skin through the dura, acting as conduits for the entry of bacteria into the nervous system. Clinical manifestations of occult spinal dysraphism can be subtle. In some instances, a lipoma, patch of hair, hemangioma, or discoloration of skin lies over the lumbosacral spine. A sacral dimple superior to the gluteal cleft or lateral to the cleft with displacement of the gluteal fold, as described for the boy in the vignette, should raise suspicion for dysraphism. Children later may develop high-arched feet, discrepancy in strength between lower extremities, or a gait abnormality. Back pain or absent perineal sensation, urinary incontinence, lower urinary tract infection, or fecal soiling also may occur. Children younger than age 6 months, whose vertebral bones have not ossified fully, can be evaluated for spina bifida by ultrasonography. Older children require initial evaluation with magnetic resonance imaging of the spine. Radiographs might be helpful to evaluate accompanying constipation, but will not evaluate dysraphism adequately. Urinary tract abnormalities should be managed after the dysraphism is identified, and the management approach is similar to that in other children who have myelodysplasia. Ultrasonography of the kidneys, ureter, and bladder might be helpful, as can urine culture or voiding cystourethrography, in evaluation of the urinary tract. References: Drolet BA. Cutaneous signs of neural tube dysraphism. Pediatr Clin North Am. 2000;47:813-823. Abstract available at: http://www.ncbi.nlm.nih.gov/entrez/query.fcgi?cmd=Retrieve&db=pubmed&dopt=Abstract&list_ui ds=10943258&query_hl=16&itool=pubmed_docsum Johnston MV, Kinsman S. Congenital anomalies of the central nervous system: neural tube defects (dysraphism). In: Behrman RE, Kliegman RM, Jenson HB, eds. Nelson Textbook of Pediatrics. 17th ed. Philadelphia, Pa: WB Saunders Co; 2004:1983

Copyright 2007 by the American Academy of Pediatrics

page 279

2007 PREP SA on CD-ROM Question: 135

A 15-year-old previously healthy girl had an episode of syncope in gym class. She awoke after 1 minute and was taken to the emergency department where physical examination findings were normal. Results of electrocardiography are shown in Item Q135A. Of the following, the BEST assessment of the electrocardiographic result is

A. atrioventricular block B. Brugada syndrome C. prolonged QT syndrome D. sinus rhythm with premature contractions E. Wolff-Parkinson-White syndrome

Copyright 2007 by the American Academy of Pediatrics

page 280

2007 PREP SA on CD-ROM Critique: 135 Preferred Response: C

Syncope or presyncope during exercise is abnormal and should prompt a thorough cardiovascular evaluation because such symptoms could be caused by disorders that have been associated with sudden cardiac death in young athletes. Among these conditions are hypertrophic cardiomyopathy (HCM), anomalies of the coronary arteries, ruptured aortic aneurysm (as can be seen in Marfan syndrome), and arrhythmias (eg, long QT syndrome). It is critical to obtain a detailed history, including family history for sudden death, arrhythmia, and deafness, because several of these entities (HCM, Marfan syndrome, and long QT syndrome) can have autosomal dominant inheritance patterns. Familial syndromes such as the Jervell and Lange-Nielsen syndrome, an autosomal recessive condition of prolonged QT syndrome associated with deafness, may become apparent with a detailed family history. As part of the initial evaluation of any patient who has syncope during exercise, a complete (at least 12 leads) electrocardiogram (ECG) should be obtained. The timing of the voltage intervals is particularly important in terms of looking for evidence of heart block, QRS abnormalities, prolonged QT interval corrected for heart rate, and sinus node dysfunction. The ECG also can be used to evaluate for pre-excitation syndromes such as the Wolff-ParkinsonWhite syndrome and to diagnose such entities as the Brugada syndrome. Because the patient in the vignette has had syncope with exercise, she should be removed from all gym classes and participation in sports until her evaluation is complete. Her ECG tracing shows clear evidence of a prolonged QT interval, even when the interval is corrected for the heart rate (dividing the QT interval by the square root of the R-R interval) (Item C135A). Abnormal corrected QT intervals (QTc) are greater than 450 msec. There is no evidence of atrioventricular block (Item C135B) in the patient because the PR interval is normal and there is a direct relationship between every P wave and every QRS complex. Brugada syndrome (Item C135C), often a familial syndrome, is the combination of a right bundle branch block pattern and ST segment elevation on ECG, with the potential for sudden death. Neither of these ECG findings is seen on this tracing. Wolff-Parkinson-White syndrome requires ventricular preexcitation (early depolarization of the QRS complex) (Item C135D), which is not present on the ECG tracing reported for the patient. Although the ECG tracing demonstrates a sinus rhythm, there is no evidence of premature contractions. References: Berger S, Utech L, Fran Hazinski M. Sudden death in children and adolescents. Pediatr Clin North Am. 2004;51:1653-1677. Abstract available at: http://www.ncbi.nlm.nih.gov/entrez/query.fcgi?cmd=Retrieve&db=pubmed&dopt=Abstract&list_ui ds=15561179&query_hl=2&itool=pubmed_docsum Chiang CE. Congenital and acquired long QT syndrome. Current concepts and management. Cardiol Rev. 2004;12:222-234. Available at: http://www.cardiologyinreview.com/pt/re/cardiorev/abstract.00045415-20040700000007.htm;jsessionid=EGmnHdsk2bBzGB8lmmKJ2CksqGuCo0Oai31wLs7zmc3bNCcedP3e!469394890!-949856145!9001!-1 Maron BJ. Sudden cardiac death in the young athlete and the preparticipation cardiovascular evaluation. In: Moller JH, Hoffman JIE, eds. Pediatric Cardiovascular Medicine. Philadelphia Pa: Churchill Livingstone; 2000:891-900 Rhodes LA. Syncope. In Moller JH, Hoffman JIE, eds. Pediatric Cardiovascular Medicine. Philadelphia, Pa: Churchill Livingstone; 2000:885-890

Copyright 2007 by the American Academy of Pediatrics

page 281

2007 PREP SA on CD-ROM Question: 136

A newborn has microcephaly, dysmorphic features (including a prominent forehead, protuberant ears, and micrognathia), bilateral hip dislocation, clinodactyly, and a single transverse palmar crease bilaterally. Peripheral blood chromosome analysis reveals an unbalanced translocation that results in partial trisomy 9. Of the following, the MOST appropriate statement to include when discussing the diagnosis with the parents of this infant is that

A. chromosome analysis should be obtained on both parents to determine if they are balanced
translocation carriers

B. chromosome analysis of their other children is unnecessary C. mental development should be normal because their infant does not have a full trisomy D. their risk for a similarly affected child in future pregnancies is very small E. they are at increased risk for other trisomies (eg, Down syndrome) in future pregnancies

Copyright 2007 by the American Academy of Pediatrics

page 282

2007 PREP SA on CD-ROM Critique: 136 Preferred Response: A

The finding of an unbalanced translocation in an infant always should prompt examination of the parental blood karyotypes to determine if the translocation is familial. Translocations usually involve the exchange of genetic material between two chromosomes. Robertsonian translocations result from fusion of the centromere of two acrocentric chromosomes (chromosomes 13, 14, 15, 21, and 22); reciprocal translocations result from the breakage of two chromosomes away from the centromere, with exchange of the chromosomal material distal to the breakpoints. During meiosis, distribution of the translocated chromosomes can give rise to normal gametes, gametes that carry the balanced rearrangement, or gametes that have a duplication or deficiency of some chromosomal material. Unless the translocation has disrupted a gene, carriers of balanced translocations usually are phenotypically normal. However, carriers are at risk for having abnormal offspring if unbalanced gametes are formed. Thus, it is important to study the chromosomes of the parents of any infant who has a translocation to assess their risk for recurrence in future pregnancies. Some couples in whom one member carries a balanced translocation may have a history of multiple miscarriages, which is explained by the loss of embryos/fetuses that have unbalanced translocations incompatible with survival. Trisomy for the short arm of chromosome 9 is the most frequent partial trisomy and results in the phenotypic features noted in the infant described in the vignette. Intellectual impairment always is present, although to varying degrees. The phenotypically normal child of a parent who carries a balanced translocation would be expected to have either a normal karyotype or to be a balanced translocation carrier. Such children should be offered chromosomal analysis and genetic counseling by the time they are of reproductive age. Balanced translocation carriers are primarily at risk for having children who have an unbalanced chromosome complement, not trisomies. References: Jones KL. Trisomy 9 mosaic syndrome. In: Smith's Recognizable Patterns of Human Malformation. 6th ed. Philadelphia, Pa: Elsevier Saunders Co; 2006:26-27 Pinsky L. Overview of genetic assessment. UpToDate. 2006; 14.1. Available at: http://www.utdol.com/utd/content/topic.do?topicKey=genr_med/27636&type=A&selectedTitle=1~ 85

Copyright 2007 by the American Academy of Pediatrics

page 283

2007 PREP SA on CD-ROM Question: 137

A 17-year-old girl presents with amenorrhea of 6 months' duration. One year ago she joined the cross country team at school. At that time, her periods had been regular, about every 26 days, and remained so for the 3-month running season. At the end of the season, she continued to run 5 miles a day to be more competitive for the subsequent season. She noticed that her menstrual flow was lighter for a few months preceding the amenorrhea. You suspect exercise-induced amenorrhea and recommend a decrease in exercise. Of the following, the factor MOST likely to be associated with a low bone density and stress fractures in this patient is

A. cigarette smoking B. early onset of puberty C. high body mass index D. use of antidepressant medication E. use of oral contraceptives

Copyright 2007 by the American Academy of Pediatrics

page 284

2007 PREP SA on CD-ROM Critique: 137 Preferred Response: A

Polycystic ovary syndrome (PCOS) and exercise-induced amenorrhea are two common causes of secondary amenorrhea during the adolescent years. The patient described in the vignette is a runner and, like other athletes who have intensive training schedules, is at risk for exercise-induced amenorrhea. Such serious athletes also have higher rates of disordered eating. The female athlete triad includes disordered eating, amenorrhea, and decreased bone density. Included in the group of athletes who have secondary amenorrhea are patients who have anorexia nervosa. A history and physical examination can reveal possible causes of the amenorrhea and guide the choice of laboratory tests, which should include a pregnancy test. Low levels of estradiol (E2) have been implicated in the loss of bone mineralization and increased risk of stress fractures found in patients who have exercise-induced amenorrhea. (A negative result to a progesterone withdrawal test is expected with low E2 concentrations.) Other factors associated with low bone density in female athletes are low weight, low percentage of body fat, delayed puberty, longer duration of amenorrhea, low calcium and protein intake, and a family history of osteoporosis. Adolescents who have exercise-induced amenorrhea should be advised to reduce the intensity of their training and increase their caloric intake. Resumption of menses is necessary to prevent further bone demineralization. The benefits of hormonal replacement have not been established. Patients also are advised to take 1,500 mg/d of calcium and encouraged to abstain from smoking tobacco because it increases the risk for stress fractures. Although obesity frequently is associated with PCOS, it is not always present. The diagnosis should be suspected in any adolescent female who has amenorrhea and clinical features of hyperandrogenism (eg, hirsutism, acne). Delayed puberty, rather than early puberty, is associated with a low bone density. A low percentage of body fat, reflected in a low body mass index, also is associated with a low bone density. The use of antidepressants has not been associated with a low bone density or increased risk of stress fractures. There is some evidence that stress fractures occur less frequently in athletes who have used oral contraceptives. References: Emans SJ. Amenorrhea in the adolescent. In: Emans SJH, Laufer MR, Goldstein DP, eds. Pediatric and Adolescent Gynecology. 5th ed. Philadelphia, Pa: Lippincott, Williams & Wilkins; 2005:214-269 Hergenroeder AC, Chorley JN. Female athletes: menstrual problems and the risk for osteopenia. In: Behrman RE, Kliegman RM, Jenson HB, eds. Nelson Textbook of Pediatrics. 17th ed. Philadelphia, Pa: Saunders; 2004:2315-2316

Copyright 2007 by the American Academy of Pediatrics

page 285

2007 PREP SA on CD-ROM Question: 138

You are asked to see a 14-year-old girl who developed pubic hair at age 11 years and breast buds at age 12 years, but has not reached menarche. She is a gymnast who practices 2 hours a day. Breast tissue is Sexual Maturity Rating (SMR) 2 and pubic hair is SMR 4. She is 57 in tall and weighs 86 lb. The results of gonadotropin laboratory studies are a luteinizing hormone concentration of 18 mIU/mL (18 IU/L) (normal adult female, 2 to 70 mIU/mL [2 to 70 IU/L]) and a follicle-stimulating hormone concentration of 40 mIU/mL (40 IU/L) (normal adult female, 1 to 30 mIU/mL [1 to 30 IU/L]). Of the following, the MOST likely cause of the primary amenorrhea in this patient is

A. autoimmune ovarian failure B. excessive exercise C. imperforate hymen D. prolactinoma E. Turner syndrome

Copyright 2007 by the American Academy of Pediatrics

page 286

2007 PREP SA on CD-ROM Critique: 138 Preferred Response: E

Most girls who have Turner syndrome are identified at a relatively young age because of short stature or growth attenuation. Thelarche (breast development) does not occur in most affected girls because streak ovaries present at birth never develop follicles. However, a substantial minority of girls who have X chromosome abnormalities at least begin puberty before developing complete ovarian failure. Adrenarche (adrenal puberty) usually is not delayed in girls who have Turner syndrome, so they have normal development of pubic and axillary hair. Any girl who has had a failure or delay of pubertal maturation (failure of menarche within 3 years of development of initial age-appropriate thelarche) should have gonadotropins (luteinizing hormone [LH] and follicle-stimulating hormone [FSH]) measured. An LH elevation can be seen during the midcycle surge of this hormone, but elevations of FSH are pathognomonic for ovarian failure. Girls who have ovarian failure and elevated FSH values are said to have hypergonadotropic hypogonadism, and the two most common causes of this finding are Turner syndrome and autoimmune ovarian failure. Such findings in a girl who is short and has had pubertal arrest, such as the girl described in the vignette, strongly suggest Turner syndrome. Autoimmune ovarian failure usually is not associated with short stature. Chromosome studies are mandated in any girl who has ovarian failure. Excessive exercise can cause pubertal delay, but this is associated with low levels of LH and FSH because the excessive exercise leads to hypogonadotropic hypogonadism. Hyperprolactinemia, which can be caused by prolactinoma, also suppresses puberty, but ovaries are intact, and affected girls also have low concentrations of LH and FSH. Girls who have imperforate hymens complete puberty but do not have normal menses. Levels of LH and FSH are normal. References: Misra M, Park-Bennett S. Disorders of puberty. In: Burg FD, Ingelfinger JR, Polin RA, Gershon AA, eds. Gellis & Kagans Current Pediatric Therapy. Philadelphia, Pa: WB Saunders; 2002:706710 Saenger P. Clinical manifestations and diagnosis of Turner syndrome (gonadal dysgenesis). UptoDate. 2006;14;1. Available at: http://uptodateonline.com/utd/content/topic.do?file=r_endo_f/13147&type=A&selectedTitle=1~36 Sedlmeyer IL, Palmert MR. Delayed puberty: analysis of a large case series from an academic center. J Clin Endocrinol Metab. 2002;87:1613-1620. Available at: http://jcem.endojournals.org/cgi/content/full/87/4/1613 Sybert VP, McCauley E. Turner's syndrome. N Engl J Med. 2004;351:1227-1238

Copyright 2007 by the American Academy of Pediatrics

page 287

2007 PREP SA on CD-ROM Question: 139

You diagnose attention-deficit/hyperactivity disorder in an 8-year-old girl and initiate therapy with a daily morning dose of long-acting methylphenidate. Her mother asks about using stimulant medication after school, on weekends, and during the summer. Of the following, your BEST response is that

A. an after-school dose is not necessary with a long-acting form of methylphenidate B. dosing outside school hours allows parents to monitor medication effect C. immediate-release medication should be used during holidays D. summer dosing increases the risk of tolerance to medication E. weekend dosing should be decreased by 50%

Copyright 2007 by the American Academy of Pediatrics

page 288

2007 PREP SA on CD-ROM Critique: 139 Preferred Response: B

Children who have attention-deficit/hyperactivity disorder (ADHD) commonly have difficulty attending to and completing tasks both at home and school. Families may resist giving medication outside of school hours because they assume that the same degree of attention needed in a school setting is not necessary in the childs other settings. However, best results from stimulant medication therapy have been noted when children receive medication in all of their typical settings. Additionally, parents will be more aware of medication adverse effects and adequacy of dosing if they observe the child when he or she is taking the medication. An after-school dose of medication may be necessary for the child to perform successfully in after-school and home activities if the medication effects have worn off at the end of the school day. There is no need to change the type or the dosing of medication used during weekends and holidays. Continuous dosing throughout the year does not make a child more tolerant to stimulant medications, and drug holidays are not necessary. References: American Academy of Pediatrics Subcommittee on Attention-Deficit/Hyperactivity Disorder and Committee on Quality Improvement. Clinical practice guideline: treatment of the school-aged child with attention-deficit/hyperactivity disorder. Pediatrics. 2001;108:1033-1044. Available at: http://pediatrics.aappublications.org/cgi/content/full/108/4/1033 Reiff MI, Tippins S, LeTourneau AA. ADHD: A Complete and Authoritative Guide. Elk Grove Village, Ill: American Academy of Pediatrics; 2004 Wender EH. Managing stimulant medication for attention-deficit/hyperactivity disorder. Pediatr Rev. 2001;22:183-190. Available at: http://pedsinreview.aappublications.org/cgi/content/full/22/6/183 Wender EH. Managing stimulant medication for attention-deficit/hyperactivity disorder: an update. Pediatr Rev. 2002;23:234-236. Available at: http://pedsinreview.aappublications.org/cgi/content/full/23/7/234

Copyright 2007 by the American Academy of Pediatrics

page 289

2007 PREP SA on CD-ROM Question: 140

A 4-year-old boy presents to your clinic with a 4-day history of fever. Over the past 24 hours, he developed swelling over the left side of his face, and his left eye is starting to close. On physical examination, his temperature is 102F (38.9C), and his left cheek and lower eyelid are swollen (Item Q140A) but not red or warm. On oral examination, you note a severe decay of the second maxillary molar and elicit pain when you tap on this tooth. Of the following, the MOST appropriate therapy is

A. azithromycin B. cefdinir C. cephalexin D. penicillin E. trimethoprim-sulfamethoxazole

Copyright 2007 by the American Academy of Pediatrics

page 290

2007 PREP SA on CD-ROM Critique: 140 Preferred Response: D

Antimicrobial therapy is indicated for patients who have dental infections if there is associated cellulitis or facial swelling, as described for the patient in the vignette. Many dentists want to try to treat the infection with antimicrobial agents prior to repairing or removing the involved tooth. Penicillin remains the antibiotic of choice for dental infections. If the patient is allergic to penicillin, clindamycin or erythromycin is an acceptable alternative. Azithromycin, cefdinir, cephalexin, and trimethoprim-sulfamethoxazole are not indicated for dental infections. Dental caries caused by Streptococcus mutans and other streptococci destroy most of the tooth and invade the dental pulp. Pulpitis (inflammation of the pulp) can lead to extension of the infection beyond the tooth to the periapical tissues of the periodontal ligament, possibly resulting in an abscess. Depending on the location of the tooth involved, the patient may develop swelling on the face or below the jaw. Fever, facial swelling, and tooth pain that is exaggerated when the tooth is touched usually indicate an abscess. Abscesses can have a polymicrobial etiology, including anaerobes (eg, Actinobacillus actinomycetemcomitans), especially when periodontal disease also is present. Careful attention to dental hygiene, especially in high-risk patients (eg, those who have complex congenital heart disease) is important to prevent systemic illness. References: Tinanoff N. Dental caries. In: Behrman RE, Kliegman RM, Jenson HB, eds. Nelson Textbook of Pediatrics. 17th ed. Philadelphia, Pa: WB Saunders Co; 2004:1209-1212 Wayne DB, Trajtenberg CP, Hyman DJ. Tooth and periodontal disease: a review for the primarycare physician. South Med J. 2001;94:925-932. Abstract available at: http://www.ncbi.nlm.nih.gov/entrez/query.fcgi?cmd=Retrieve&db=pubmed&dopt=Abstract&list_ui ds=11592756&query_hl=7&itool=pubmed_docsum

Copyright 2007 by the American Academy of Pediatrics

page 291

2007 PREP SA on CD-ROM Question: 141

You are seeing a newborn boy for the first time. Several prenatal ultrasonographic examinations revealed bilateral hydronephrosis. The boy's weight and height are appropriate for gestation, and his physical examination findings are unremarkable. Postnatal ultrasonography reveals severe bilateral hydronephrosis. Of the following, the MOST likely cause of the hydronephrosis is

A. polycystic kidney disease B. posterior urethral valves C. ureteropelvic junction obstruction D. vesicoureteral reflux E. Wilms tumor

Copyright 2007 by the American Academy of Pediatrics

page 292

2007 PREP SA on CD-ROM Critique: 141 Preferred Response: B

Urinary tract obstruction (UTO) may indicate an abnormality in the kidney(s), ureter(s), bladder, or urethra. A logical and stepwise approach to determine the cause usually results in reaching an accurate diagnosis. UTO generally is found in utero or early in infancy. The advent of routine fetal ultrasonography has dramatically increased the detection of UTO prior to birth. In most other cases of UTO, a urinary tract infection in infancy stimulates evaluation of the system. The practitioners goal is to ascertain the site of obstruction. Renal ultrasonography is the primary imaging technique used to determine if UTO is present, and if so, the degree of obstruction. In general, hydronephrosis is described as mild, moderate, or severe. Most studies show that renal function is better preserved in mild compared with moderate-to-severe hydronephrosis. Another invaluable piece of information is whether the hydronephrosis is unilateral or bilateral. Obviously, bilateral hydronephrosis increases the likelihood of developing renal insufficiency, but it also may provide an indication of the origin of obstruction. Generally, the origin of obstruction is lower with bilateral versus unilateral hydronephrosis. It is essential to determine the cause of any obstruction; if left untreated, it may result in recurrent urinary tract infections and renal scarring and damage. The bilateral hydronephrosis reported for the boy in the vignette strongly suggests the presence of UTO, although the hydronephrosis may be nonobstructive. Voiding cystourethrography is necessary to determine if there is vesicoureteral reflux (VUR) (Item C141A), a ureteral abnormality such as a duplicated ureter, or posterior urethral valves (PUV) (Item C141B). PUV must be considered a likely cause of bilateral hydronephrosis in a male. The most common presentation of autosomal recessive or dominant polycystic kidney disease (Item C141C) is enlarged kidneys in the newborn period, with or without small cysts; these kidneys do not display hydronephrosis. Although ureteropelvic junction obstruction (UPJO) and VUR commonly cause hydronephrosis, the sex of the child and bilateral hydronephrosis makes PUV more common. However, it should be emphasized that children who have PUV may have concomitant VUR or UPJO. Wilms tumor (Item C141D), the most common abdominal tumor in infants, presents most commonly as a solid mass, occasionally with concomitant hydronephrosis. References: Aksu N, Yavascan O, Kangin M, et al. Postnatal management of infants with antenatally detected hydronephrosis. Pediatr Nephrol. 2005;20:1253-1259. Abstract available at: http://www.ncbi.nlm.nih.gov/entrez/query.fcgi?orig_db=PubMed&db=PubMed&cmd=Search&ter m=%22Pediatric+nephrology+(Berlin,+Germany)%22[Jour]+AND+1253[page]+AND+2005[pdat] Chandrasekharam VV, Shah MA. Outcome of patients with antenatally detected pelviureteric junction obstruction. Pediatr Nephrol. 2005;20:547 Onen A, Jayanthi VR, Koff SA. Long-term followup of prenatally detected severe bilateral newborn hydronephrosis initially managed nonoperatively. J Urol. 2002;168:1118-1120. Abstract available at: http://www.ncbi.nlm.nih.gov/entrez/query.fcgi?orig_db=PubMed&db=PubMed&cmd=Search&ter m=%22The+Journal+of+urology%22[Jour]+AND+1118[page]+AND+2002[pdat]

Copyright 2007 by the American Academy of Pediatrics

page 293

2007 PREP SA on CD-ROM Question: 142

Over the past week, you and your colleagues have seen a number of previously healthy children younger than 2 years of age who present with fever, coryza, cough, vomiting, and diarrhea. Several of the children have required hospitalization for respiratory distress and pneumonia. You suspect that influenza virus infection is the cause of these illnesses. Of the following, the MOST rapid test to confirm the diagnosis in these patients is

A. enzyme immunoassay antigen detection B. immunoglobulin M (IgM) titers on acute sera C. IgM titers on acute and convalescent sera D. polymerase chain reaction E. viral culture

Copyright 2007 by the American Academy of Pediatrics

page 294

2007 PREP SA on CD-ROM Critique: 142 Preferred Response: A

Influenza is a common respiratory disease that is caused by the influenza A and B viruses and is associated with annual epidemics. It is spread person-to-person by respiratory droplets via coughing or sneezing or through direct contact with infected nasopharyngeal secretions. The clinical manifestations of the disease differ, depending on the age of the person who is infected. In young infants, the presentation can range from fever and a crouplike cough to a picture resembling sepsis. In children, the most common presenting signs and symptoms are fever, malaise, cough, rhinitis, pharyngitis, vomiting, and diarrhea, although in some children, the disease manifests as a simple upper respiratory tract infection or a febrile illness with few other symptoms. In adolescents, the illness has a similar presentation to that seen in adults and is characterized by the sudden onset of high fever, malaise, myalgia, headache, chills, rigors, pharyngitis, and a nonproductive cough. In uncomplicated cases, symptoms usually resolve in 3 to 7 days, although cough and malaise may persist for 2 weeks or more. The treatment of influenza is supportive, but specific antiviral therapy should be considered in high-risk patients or those who have severe disease (Item C142A). Amantadine and rimantadine are in the M2 class of antiviral drugs and work by inhibiting viral replication. These agents are only licensed for the treatment of influenza A disease. Oseltamivir and zanamivir are neuraminidase inhibitors that are effective in the treatment of both influenza A and B infection. Amantadine and oseltamivir may be used in children 1 year of age and older; rimantadine is licensed only for use in children 13 years of age or older. Zanamivir is approved for use in children who are 7 years of age and older and is administered by inhalation. A 2-week course of chemoprophylaxis should be considered in patients in high-risk groups who have been exposed to influenza. Amantadine and rimantadine are licensed for prophylaxis in children 1 year of age and older, oseltamivir can be used for prophylaxis in children 13 years of age and older, and zanamivir is not currently licensed for prophylaxis. The accurate and early diagnosis of influenza in patients who have respiratory illnesses can reduce inappropriate use of antibiotics and provide the option of using antiviral therapy. A number of diagnostic tests are available, but the most rapid is enzyme immunoassay antigen detection. Paired acute and convalescent serology can aid in the diagnosis of a recent influenza infection, although results are not available for 2 or more weeks. Similarly, polymerase chain reaction and viral culture do not provide rapid results. A single acute serum specimen is not valuable in diagnosis. Annual immunization against influenza is the best approach to prevention. Two types of influenza vaccines may be used, both of which contain the same three influenza strains (two influenza A strains and one influenza B strain) that are believed to be circulating in the United States in a given year. Vaccination is strongly recommended for all healthy children 6 to 23 months of age, all children and adults who have chronic underlying diseases (eg, asthma, chronic cardiac and pulmonary conditions, immunocompromise), persons living in chronic care facilities or nursing homes, women who are pregnant (>14 weeks gestation), household members of high-risk patients, and health-care workers. The inactivated intramuscular influenza vaccine may be used in children 6 months of age and older. Children younger than 9 years of age should receive two doses 1 month apart for their initial vaccination, then a single dose each subsequent fall. The live, attenuated, intranasal vaccine is licensed only for use in healthy individuals ages 5 to 49 years. Children 5 to 9 years of age should receive two doses 6 weeks apart for their initial vaccination, then a single dose each year subsequently. Efficacy in the pediatric population is similar for both vaccines, ranging from 70% to 90%. References: Abramson JS, Katz SL, Offit PA, McMillan JA. Reducing the risk of pediatric influenza: prevention strategies help both the young and the old. Contemp Pediatr. 2005;22(suppl 1):1-8 Advanced Studies in Medicine. Prevention of influenza virus infection among children. ASIMCME 2002;2(8):286-323. Available at: http://www.jhasim.com/template.cfm?TEMPLATE=include_program.cfm&ID=38&ZoneID=8&pag

Copyright 2007 by the American Academy of Pediatrics

page 295

2007 PREP SA on CD-ROM

eName=Volume%202,%20(8) American Academy of Pediatrics. Influenza. In: Pickering LK, ed. Red Book: 2006 Report of the Committee on Infectious Diseases. 27th ed. Elk Grove Village, Ill: American Academy of Pediatrics; 2006:401-411 Harper SA, Fukuda K, Uyeki TM, Cox NJ, Bridges CB. Prevention and control of influenza. Recommendations of the Advisory Committee on Immunization Practices (ACIP). MMWR Recomm Rep. 2005;54:1-40. Available at: www.cdc.gov/mmwr/preview/mmwrhtml/rr54e713a1.htm

Copyright 2007 by the American Academy of Pediatrics

page 296

2007 PREP SA on CD-ROM Question: 143

A 4-year-old boy presents with a history of chronic upper and lower respiratory tract infections. His weight is 15 kg (25th percentile), height is 97 cm (10th percentile), temperature is 98.1F (36.8C), and pulse oximetry is 96% on room air. On physical examination, he coughs intermittently and has mild clubbing. On nasal examination, you note purulent rhinorrhea and nasal polyps (Item Q143A). Auscultation of the heart reveals a regular rate and rhythm, with the point of maximal impulse displaced to the right. Of the following, the MOST likely diagnosis is

A. cystic fibrosis B. human immunodeficiency virus infection C. primary ciliary dyskinesia (Kartagener syndrome) D. severe combined immunodeficiency E. X-linked (Bruton) agammaglobulinemia

Copyright 2007 by the American Academy of Pediatrics

page 297

2007 PREP SA on CD-ROM Critique: 143 Preferred Response: C

The chronic sinopulmonary infections, nasal polyps, and right-sided heart (dextrocardia) described for the boy in the vignette are virtually diagnostic for primary ciliary dyskinesia (PCD). Approximately 50% of patients who have PCD have situs inversus, a reversal of the visceral organs (Kartagener syndrome). The diagnosis can be confirmed by identification of abnormal cilia orientation or missing dynein arms by electron microscopy. Cystic fibrosis can present similarly to PCD, with chronic sinopulmonary infections, bronchiectasis, nasal polyps, and clubbing, but neither dextrocardia nor situs inversus is a typical finding. Primary immunodeficiencies such as X-linked agammaglobulinemia and severe combined immunodeficiency present with recurrent bacterial and viral infections. Nasal polyps may result from chronic sinusitis, although digital clubbing and dextrocardia are not observed. Clinical manifestations of human immunodeficiency virus infection vary widely. Initial symptoms can include lymphadenopathy, hepatosplenomegaly, failure to thrive, diarrhea, and thrush. Recurrent otitis media, sinusitis, and lymphocytic interstitial pneumonia are common, but bronchiectasis and nasal polyps are rare. References: Chilvers MA, Rutman A, OCallaghan C. Ciliary beat pattern is associated with specific ultrastructural defects in primary ciliary dyskinesia. J Allergy Clin Immunol. 2003;112:518-524 Haddad GG. Primary ciliary dyskinesia (immotile cilia syndrome). In: Behrman RE, Kliegman RM, Jenson HB, eds. Nelson Textbook of Pediatrics. 17th ed. Philadelphia, Pa: WB Saunders Co; 2004:1450-1451 Welch JE, Hogan MB, Wilson NW. Ten-year experience using a plastic, disposable curette for the diagnosis of primary ciliary dyskinesia. Ann Allergy Asthma Immunol. 2004;93:189-192

Copyright 2007 by the American Academy of Pediatrics

page 298

2007 PREP SA on CD-ROM Question: 144

A 12-year-old child is struck in the right eye by a baseball. He now complains of double vision when he looks to the left. On physical examination, you note marked swelling and bruising circumferentially around the boy's right eye. His pupillary reflexes are normal, he has no blood in the anterior chamber of the eye, and his optic disc margins are sharp. He has dysconjugate gaze to the left but not to the right. His vision is 20/20 out of the left eye and 20/30 out of the right eye. Other findings on his neurologic examination are normal. Of the following, the MOST likely diagnosis is

A. blow-out fracture B. corneal abrasion C. detached retina D. hyphema E. traumatic iritis

Copyright 2007 by the American Academy of Pediatrics

page 299

2007 PREP SA on CD-ROM Critique: 144 Preferred Response: A

The findings described for the boy in the vignette are indicative of an internal orbital or blow-out fracture, which is defined as a fracture of the orbital floor or orbital wall with an intact orbital rim. This is an uncommon injury before 8 years of age, possibly due to poorly pneumatized sinuses in very young children. Rupture of the orbital wall often is accompanied by extrusion of orbital soft tissue, including periosteum, fat, or muscle. Entrapment of the inferior rectus muscle and, less commonly, the medial rectus muscle is manifested by impaired eye movement. Impaired eye movement results in intermittent diplopia, depending on the direction of eye movement. Surgical reduction is required within 4 to 10 days to prevent necrosis and subsequent fibrosis of the entrapped tissues. A child who has a suspected blow-out fracture should be evaluated by orbital computed tomography (CT) scan; skull films are of limited value. CT scan is especially valuable because it allows for evaluation of both the bony orbit and the globe. Associated injuries include ruptured globe, retrobulbar hemorrhage, and traumatic optic neuropathy, the first two of which can be demonstrated on CT scan. A child in whom a blow-out fracture is confirmed should be examined immediately by an ophthalmologist to determine the need for surgical reduction, assess for associated injuries, and monitor for complications. Corneal abrasion, detached retina, hyphema, and traumatic iritis can occur in conjunction with a blow-out fracture. Corneal abrasion causes severe pain and tearing but does not cause dysconjugate gaze or diplopia. Hyphema (Item C144A) refers to blood in the anterior chamber, which may cause visual impairment but generally not diplopia. Traumatic iritis is characterized by pain and severe photophobia but not diplopia. Detached retina can cause a curtainlike deficit in the peripheral vision but does not cause dysconjugate gaze. A detached retina is an ophthalmologic emergency. References: American College of Surgeons. Ocular trauma. In: Advanced Trauma Life Support Program for Doctors. 6th ed. Chicago, Ill: American College of Surgeons; 1997:411-418 Cruz AA, Eichenberger GCD. Epidemiology and management of orbital fractures. Curr Opin Ophthalmol. 2004;15:416-421. Abstract available at: http://www.ncbi.nlm.nih.gov/entrez/query.fcgi?orig_db=PubMed&db=PubMed&cmd=Search&ter m=Curr+Opin+Ophthalmol[Jour]+AND+416[page]+AND+2004[pdat] Klenk G, Kovacs A. Blow-out fracture of the orbital floor in early childhood. J Craniofac Surg. 2003;14:666-671. Abstract available at: http://www.ncbi.nlm.nih.gov/entrez/query.fcgi?orig_db=PubMed&db=PubMed&cmd=Search&ter m=J+Craniofac+Surg[Jour]+AND+666[page]+AND+2003[pdat] Neuman MI, Eriksson E. Facial trauma. In: Fleisher GR, Ludwig S, Henretig FM, eds. Textbook of Pediatric Emergency Medicine. 5th ed. Philadelphia, Pa: Lippincott Williams & Wilkins; 2006:1475-1484

Copyright 2007 by the American Academy of Pediatrics

page 300

2007 PREP SA on CD-ROM Question: 145

A 14-month-old infant who is breastfed with bottle supplementation presents with early evidence of dental caries (Item Q145A). Of the following, the MOST appropriate practice to disrupt this process is to

A. discontinue bottle feeding B. discontinue bottle feeding except for juices C. encourage brushing teeth with fluoride-containing toothpaste D. switch from formula to whole milk for bottle feeding E. use bottled water to prepare formula

Copyright 2007 by the American Academy of Pediatrics

page 301

2007 PREP SA on CD-ROM Critique: 145 Preferred Response: A

Nursing infants with erupted dentition and toddlers who are allowed to fall asleep while taking a bottle containing high sugar content liquid such as juices and formula are at risk for the development of nursing bottle caries (Item C145A). Recently, prolonged nocturnal breastfeeding also has been recognized as a contributing factor. Characteristically, the sequence of caries development progresses from maxillary incisors to maxillary molars and subsequently mandibular dentition. Brushing before bedtime and elimination of feeding in bed should greatly decrease the appearance of nursing caries. Bottle feeding should be discontinued for the 14-month-old infant described in the vignette. Continuing juices by bottle would compound the problem. Preparing formula with bottled water would have no impact on the disease process. Changing from formula to whole milk is appropriate for the infant, but nocturnal bottle feeding of whole milk would continue to place the toddler at risk for caries. Brushing teeth is a good practice, but discontinuation of bottle feeding is most important. References: American Academy of Pediatrics. Policy statement: oral health risk assessment timing and establishment of the dental home. Pediatrics. 2003;111:1113-1116. Available at: http://pediatrics.aappublications.org/cgi/content/full/111/5/1113 Tinanoff N. Dental caries. In: Behrman RE, Kliegman RM, Jenson HB, eds. Nelson Textbook of Pediatrics. 17th ed. Philadelphia, Pa: WB Saunders Co; 2004:1209-1211 Zitelli BJ, Davis HW. Caries. In: Atlas of Pediatric Physical Diagnosis. 3rd ed. St. Louis, Mo: Mosby-Wolfe; 1997:613

Copyright 2007 by the American Academy of Pediatrics

page 302

2007 PREP SA on CD-ROM Question: 146

During the health supervision visit for a 1-month-old infant, you note mild jaundice, but no fever and clear lungs. Cardiac examination demonstrates a 2/6 systolic murmur at the left sternal border. Abdominal examination demonstrates a firm liver 3 cm below the costal margin and extending past the midline. The alanine aminotransferase is 140 U/L, aspartate aminotransferase is 130 U/L, total bilirubin is 5.5 mg/dL (94.1 mcmol/L), and direct bilirubin is 3.0 mg/dL (51.3 mcmol/L). Of the following, the finding that MOST strongly suggests the need for referral to a surgeon for intraoperative cholangiography is

A. abdominal ultrasonography demonstrating a prominent gallbladder B. brain computed tomography scan demonstrating cerebral calcifications C. echocardiography demonstrating pulmonary artery stenosis D. dimethyl iminodiacetic acid scan demonstrating no tracer in the bowel E. liver biopsy demonstrating periodic acid-Schiff stain-positive globules in the hepatocytes

Copyright 2007 by the American Academy of Pediatrics

page 303

2007 PREP SA on CD-ROM Critique: 146 Preferred Response: D

In a 1-month-old infant, the liver edge can be palpated below the costal margin. As the infant grows, more of the liver is covered by the ribcage, making it unusual to feel a liver edge in children older than 1 year of age. A preferred approach to assessing liver size in all children involves percussion of the liver span; the dull sound of the liver can be differentiated easily from the more hollow sound of the bowel and chest. Because chest hyperinflation can push the liver down, thus mimicking "hepatomegaly," determination of liver span can help differentiate true enlargement from liver displacement. Hepatomegaly should be suspected when the liver edge extends more than 1 cm below the costal margin, if the liver is hard to palpation, or if there is a prominent left lobe in the epigastrium. The causes of hepatomegaly in the infant and child are diverse and include primary liver disease, infection, malignancy, and storage disorders. For an infant younger than 2 months of age who has direct hyperbilirubinemia and hepatomegaly, such as the infant described in the vignette, the primary responsibility of the physician is to diagnose or rule out extrahepatic biliary atresia. This condition must be differentiated from other anatomic, infectious, and metabolic causes of neonatal cholestasis. Biliary atresia occurs in as many as 30% of infants who have conjugated hyperbilirubinemia. The initial evaluation of an infant who has suspected biliary atresia typically involves exclusion of infectious causes by appropriate cultures and serologies, hepatobiliary ultrasonography to exclude gallstones, and radionuclide scintigraphy. The scintigraphic examination involves injecting an infant with a radionuclide tracer such as dimethyl iminodiacetic acid. A liver that is affected by biliary atresia demonstrates tracer uptake into the liver but no excretion into the bowel (Item C146A). In contrast, a liver affected by cholestasis from other causes but that has a patent biliary system demonstrates delayed uptake in the liver but excretion into the bowel. As a caveat, however, infants who have severe cholestasis may have false-positive test results, in which tracer uptake is so slow that there is no significant excretion into the bowel. If this is the case, additional testing with liver biopsy and intraoperative cholangiography is necessary to exclude biliary atresia. Abdominal ultrasonography demonstrating a prominent gallbladder makes biliary atresia less likely because the gallbladder in biliary atresia is typically shrunken or not visible. Brain calcifications are seen in congenital toxoplasmosis and cytomegalovirus infection, two infectious causes of liver disease. Peripheral pulmonary artery stenosis is characteristic of Alagille syndrome, a genetic cause of intrahepatic bile duct paucity. A liver biopsy with periodic acidSchiff stain-positive granules suggests a storage disorder (glycogen storage disease, alpha-1antitrypsin deficiency) as a cause of the child's hepatomegaly. References: Campbell KM, Bezerra JA. Biliary atresia. In: Walker WA, Goulet O, Kleinman RE, Sherman PM, Shneider BL, Sanderson IR, eds. Pediatric Gastrointestinal Disease. 4th ed. Hamilton, Ontario, Canada: BC Decker; 2005:1122-1138 Wolf AD, Lavine JR. Hepatomegaly in neonates and children. Pediatr Rev. 2000;21:303-310. Available at: http://pedsinreview.aappublications.org/cgi/content/full/21/9/303

Copyright 2007 by the American Academy of Pediatrics

page 304

2007 PREP SA on CD-ROM Question: 147

You are evaluating a very low-birthweight (VLBW) preterm infant who experienced polyuria in the first 72 hours after birth. No diuretics have been prescribed, and there is no glycosuria, hematuria, or obvious anasarca on examination. You collect a urine sample to measure electrolytes and creatinine and simultaneously obtain a blood sample to measure serum electrolytes and creatinine. Of the following, the MOST correct statement regarding sodium handling in the VLBW infant is that

A. fractional excretion of sodium is lower than in term infants B. intravenous sodium supplementation is necessary from birth C. phototherapy increases sodium requirements D. sodium excretion increases with gestational age E. water losses generally exceed sodium losses

Copyright 2007 by the American Academy of Pediatrics

page 305

2007 PREP SA on CD-ROM Critique: 147 Preferred Response: E

The fractional excretion of sodium (FENa) is a measure of renal handling of solute. The equation for calculating the FENa is: FENa = [(Urine Na x Plasma Cr)/(Plasma Na x Urine Cr)] x 100 Example: [(90 x 0.9)/(145 x 120)] x 100 = 0.5 Sodium is reabsorbed in the proximal renal tubule. In the preterm kidney, a number of factors affect the renal handling of sodium. First, the renal blood flow increases throughout gestation in the fetus and in the first week of postnatal life. Second, the glomerular filtration rate increases throughout the first postnatal week. Third, the extracellular fluid compartment, where most of the total body content of sodium is located, is greatest in the most preterm infants. Depending on the degree of prematurity, the functional number of nephrons may be reduced significantly (nephrogenesis is not complete until after 36 weeks gestation), and renal efficiency in handling solute load is reduced accordingly because sodium transporter activity in the renal tubules is immature. Additionally, the premature kidney has poor concentrating ability, with maximal urinary concentration of 600 to 800 mOsm/L in the first 2 weeks of postnatal life. This results in a high risk of hypervolemia and hyponatremia (due to dilution) for the preterm infant who is given too much water. However, because of postnatal adjustments in fluid compartments (Item C147A), the addition of sodium to intravenous fluids for the preterm neonate is largely unnecessary in the first 24 to 72 hours after birth. The very low-birthweight (VLBW) newborn described in the vignette has polyuria, generally defined as a urine output of greater than 6 mL/kg per hour. Causes typically include iatrogenic overhydration, diabetes insipidus (genetic or sometimes related to intracranial pathology), hyperglycemia with a corresponding osmotic drag of water and the occurrence of glycosuria, or anatomic renal problems such as an obstruction that has been relieved. Water losses in the VLBW newborn generally exceed solute (sodium) losses, especially in cases of polyuria, and dehydration with elevated serum sodium concentrations may result. As noted previously, sodium excretion (FENa) decreases with increasing gestational age and postnatal age. Accordingly, the FENa is higher in a preterm newborn than in a term infant. Phototherapy increases fluid (water) requirements for the VLBW newborn, but not sodium requirements. Intravenous sodium supplementation is not necessary in most VLBW newborns in the first 24 to 72 hours of postnatal life. References: Chua AN, Sarwal MM. Acute renal failure management in the neonate. NeoReviews. 2005;6:e369-e376. Available at: http://neoreviews.aappublications.org/cgi/content/full/6/8/e369 Dell KM, Davis ID. Fluid and electrolyte management. In: Martin RJ, Fanaroff AA, Walch MC, eds. Fanaroff and Martins Neonatal-Perinatal Medicine: Diseases of the Fetus and Infant. 8th ed. Philadelphia, Pa: Mosby-Elsevier; 2006:695-702

Copyright 2007 by the American Academy of Pediatrics

page 306

2007 PREP SA on CD-ROM Question: 148

A young mother brings in her toddler immediately after seeing him drink an unknown amount of mineral spirits. He appears tired and has a persistent cough. His respiratory rate is 40 breaths/min, and his lungs are clear bilaterally. There are no other abnormal findings on physical examination. A chest radiograph shows no abnormality. Of the following, the MOST appropriate management option is

A. administration of activated charcoal B. administration of intravenous corticosteroids C. discharge home with follow-up the next day D. gastric lavage E. hospitalization for observation and supportive care

Copyright 2007 by the American Academy of Pediatrics

page 307

2007 PREP SA on CD-ROM Critique: 148 Preferred Response: E

Hydrocarbons, such as mineral spirits, kerosene, gasoline, turpentine, and pine oil, are among the most common ingestions in young children. The hydrocarbons that have the lowest viscosity, such as furniture polish, are associated with the highest risk for aspiration. Children who ingest hydrocarbons are at potentially very high risk for complications and should be evaluated carefully. Children who have ingested hydrocarbons may have a slight cough initially, which does not necessarily indicate an aspiration. However, those who exhibit persistent cough, tachypnea, or dyspnea within a few hours are at high risk of aspiration. Some children, including the boy described in the vignette, develop signs of central nervous system involvement, such as irritability or lethargy, which is largely due to hypoxia, not direct toxic effects to the brain. Fever may be present initially but is not an indicator of aspiration pneumonitis. Intestinal symptoms, such as vomiting and abdominal pain, may occur because of mucosal irritation. Children who have confirmed or suspected hydrocarbon ingestion should be evaluated with a chest radiograph, but early radiographs (within 1 hour of ingestion) may appear normal, even in the presence of aspiration. Findings indicative of aspiration include bilateral, patchy opacities (Item C148A), but these findings correlate poorly with clinical manifestations. Blood gas analysis may reveal hypoxemia and should be considered in children who have low pulse oximetry readings. Even children who do not have respiratory symptoms should be observed in the emergency department for 4 to 6 hours before discharge. Hospitalization is indicated for those who have respiratory symptoms, altered mental status, abnormal chest radiographs, or inadequate followup, such as the boy in the vignette. Management is largely supportive. Oxygen should be administered, and intubation with ventilatory support should be performed for those who have impending respiratory failure. Gastric decontamination with lavage is only indicated in cases when the ingested hydrocarbon solution contains other harmful elements, such as camphor, aromatics, heavy metals, or pesticides. Activated charcoal is not effective and should not be administered because it may increase the risk of emesis, thereby further increasing the risk of aspiration. Corticosteroids and antibiotics have not been shown to have a role in the management of hydrocarbon ingestions. Respiratory symptoms of hydrocarbon ingestion typically resolve within 7 days, although chest radiograph abnormalities may be present for weeks. Complications such as pneumatoceles, pneumothorax, empyema, bacterial pneumonia, and respiratory distress syndrome occur in a small number of patients. A few patients may develop chronic pulmonary function abnormalities dues to proliferative alveolar thickening. Another population in which hydrocarbon ingestion is important is adolescents. Adolescents may inhale hydrocarbons chronically and are at risk for long-term symptoms such as peripheral neuropathy and encephalopathy. They are also at risk for sudden cardiac death. References: Lee DC. Hydrocarbons. In: Marx JA, Hockberger RS, Walls RM, eds. Rosens Emergency Medicine: Concepts and Clinical Practice. 5th ed. St. Louis, Mo: Mosby; 2002:2160-2163 McGuigan ME. Poisoning potpourri. Pediatr Rev. 2001;22:295-302. Available at: http://pedsinreview.aappublications.org/cgi/content/full/22/9/295 Tucker JF. Aliphatic hydrocarbons. In: Ford MD, Delaney KA, Ling LJ, Erickson T, eds. Clinical Toxicology. Philadelphia, Pa: WB Saunders Co; 2001:789-793

Copyright 2007 by the American Academy of Pediatrics

page 308

2007 PREP SA on CD-ROM Question: 149

A 2-year-old boy who has been followed by your practice since birth presents for a health supervision visit. On careful physical examination, you have difficulty locating the right testis. In reviewing your records, you note that this is a new finding. Of the following, the MOST likely diagnosis is

A. agenesis of the testis B. indirect inguinal hernia C. pseudohermaphroditism D. retractile testis E. undescended testis

Copyright 2007 by the American Academy of Pediatrics

page 309

2007 PREP SA on CD-ROM Critique: 149 Preferred Response: D

Because the boy described in the vignette has had normal findings on previous testicular examinations, it is likely that the cremasteric reflex is causing retractile testis, preventing examination of the testis on this visit. A warm examination room, warm compresses to the inguinal area, or having the boy sit in the tailors position (sitting with knees flexed and ankles crossed on the examination table) or squatting may facilitate descent of the normal testis back into the scrotal sac. Retractile testis is not common in early infancy or after puberty. Agenesis of the testis is unlikely because the testes were palpable on previous examinations. Indirect inguinal hernia would be associated with an inguinal mass, but the testis would still be palpable on examination. True undescended testis is the single most common genitourinary problem in boys. Bilateral undescended testes may be associated with chromosomal or endocrine disorders, including pseudohermaphroditism as well as other syndromes and should be evaluated with careful attention to the presence of other malformations or metabolic disturbances. A normal-appearing penis implies intact testicular tissue, but chromosomal and endocrine studies should be performed. An endocrine consultation may be appropriate in an infant who has bilateral undescended testes, but it is not necessary for a child who has a unilateral undescended testis. Failure of the testis to descend along the appropriate track from the abdomen through the inguinal ring and into the scrotal sac may be associated with infertility and up to a 40% increase in the risk for testicular cancer, which would be undetected in the abdominal location. Structural changes in the undescended testis have been documented as early as 2 years of age and diminished sperm production as young as 6 years. Although administration of beta human chorionic gonadotropin has been used infrequently to effect testicular descent, it is not believed to be very effective. Adverse effects include premature closure of the epiphyses and premature onset of secondary sex characteristics (reversible). Laparoscopic orchiopexy of the presumably present unilateral undescended testis is the current treatment of choice and usually is performed at 1 year of age. References: Callaghan P. In brief: undescended testis. Pediatr Rev. 2000;21:395. Available at: http://pedsinreview.aappublications.org/cgi/content/full/21/11/395 Roth DR, Gonzales ET. Disorders of renal development and anomalies of the collecting system, bladder, penis and scrotum. In: McMillan JA, DeAngelis CD, Feigin RD, Warshaw JB, eds. Oskis Pediatrics: Principles and Practice. 3rd ed. Philadelphia, Pa: Lippincott, Williams & Wilkins; 1999:1555-1556

Copyright 2007 by the American Academy of Pediatrics

page 310

2007 PREP SA on CD-ROM Question: 150

A 15-year-old girl is rushed to the emergency department after slipping off a diving board and striking her head on cement. On physical examination, her mental status evaluation results are completely normal. She can abduct her upper extremities at the shoulder, but cannot flex or extend her arms and minimally moves her fingers. She cannot move her limp lower extremities. You order emergent magnetic resonance imaging of the brain and spine. Of the following, the MOST important therapy to implement before the patient is sent for imaging is

A. fosphenytoin 18 phenytoin equivalents/kg intravenously B. dexamethasone 1 mg/kg orally C. low-molecular weight heparin 1 mg/kg subcutaneously D. mannitol 1 g/kg intravenously E. methylprednisolone 30 mg/kg intravenously

Copyright 2007 by the American Academy of Pediatrics

page 311

2007 PREP SA on CD-ROM Critique: 150 Preferred Response: E

The possibility of occult or overt spinal trauma should be considered with any injury. For any notable body trauma, the cervical and lower spine should be immobilized until spinal trauma has been eliminated as a possibility. Common sites for fracture dislocation spinal cord injuries are C1 through C2, C5 through C6, and T12 through L1 spinal segments. Fracture dislocations of the vertebral column are the most common causes of spinal cord injury, although injury can be as severe as complete cord transection without any visible trauma on radiography. Sudden whiplash (flexion-extension) injuries may contuse (Item C150A) or lacerate the cervical cord. Typical symptoms and signs of spinal trauma include focal pain along the spinal axis or any weakness or sensory loss in the extremities or trunk. Spinal shock is characterized by loss of all voluntary movement and sensation inferior to the site of injury. Reflex function in the cord is lost temporarily, resulting in ileus and bladder distention. While the spine is immobilized, the child should undergo a thorough neurologic examination. If there is high index of suspicion for spinal trauma, as for the girl described in the vignette, prompt initiation of methylprednisolone 30 mg/kg administered intravenously over 1 hour, followed by 5.4 mg/kg per hour the next 23 hours, is a protocol that can lead to improved neurologic outcomes. The cervical spine typically is evaluated by radiographs (including lateral, anterior-posterior, and odontoid views), but if there is lingering concern, computed tomography scan or magnetic resonance imaging is warranted. Intravenous dexamethasone is useful for spinal cord compression with tumors, but its efficacy has not been investigated fully for spinal trauma. Anticonvulsants sometimes are administered prophylactically after severe head trauma, but this is controversial. Mannitol is useful only if the patient displays signs of increased intracranial pressure. Low-molecular weight heparin to prevent deep venous thrombosis might be useful if the patient is not expected to regain ambulatory status for several days. References: Bracken MB, Shepard MJ, Collins WF, et al. A randomized, controlled trial of methylprednisolone or naloxone in the treatment of acute spinal-cord injury. Results of the Second National Acute Spinal Cord Injury Study. N Engl J Med. 1990;322:1405-1411. Abstract available at: http://www.ncbi.nlm.nih.gov/entrez/query.fcgi?cmd=Retrieve&db=pubmed&dopt=Abstract&list_ui ds=2278545&query_hl=19&itool=pubmed_docsum Haslam RHA. Spinal cord disorders: spinal cord trauma. In: Behrman RE, Kliegman RM, Jenson HB, eds. Nelson Textbook of Pediatrics. 17th ed. Philadelphia, Pa: WB Saunders Co; 2004:2050 Temkin NR, Dikmen SS, Wilensky AJ, Keihm J, Chabal S, Winn HR. A randomized, double-blind study of phenytoin for the prevention of post-traumatic seizures. N Engl J Med. 1990;323:497502. Abstract available at: http://www.ncbi.nlm.nih.gov/entrez/query.fcgi?cmd=Retrieve&db=pubmed&dopt=Abstract&list_ui ds=2115976&query_hl=21&itool=pubmed_docsum

Copyright 2007 by the American Academy of Pediatrics

page 312

2007 PREP SA on CD-ROM Question: 151

You care for a 17-year-old boy who is overweight. He has gained 44 lb (20 kg) in the last year, especially in his abdominal area. On examination today, his blood pressure is 158/90 mm Hg using a large, appropriately sized cuff. His mother has a similar body habitus and is being treated for type 2 diabetes mellitus. Of the following, the MOST likely abnormalities to expect in this patient if his present condition continues into adulthood is

A. high triglyceride concentrations, low high-density lipoprotein cholesterol concentrations B. hypoglycemia from insulin sensitivity C. low concentrations of C-reactive protein in the blood D. low fibrinogen concentrations with bleeding diatheses E. low triglyceride concentrations, low low-density lipoprotein cholesterol concentrations

Copyright 2007 by the American Academy of Pediatrics

page 313

2007 PREP SA on CD-ROM Critique: 151 Preferred Response: A

The metabolic syndrome is a series of clinical and laboratory findings that include truncal obesity, elevated concentrations of serum triglycerides, decreased concentrations of highdensity lipoprotein cholesterol, and an elevation in blood pressure. Sex- and age-specific values exist for each of these values in the adult population. The metabolic syndrome increases the risk for type 2 diabetes and cardiovascular disease. In some recent studies of adult patients, the incidence of the metabolic syndrome was as high as 23%. The metabolic syndrome affects people from all ethnic and cultural backgrounds, but is most prevalent in the Hispanic population. The increasing incidence of the metabolic syndrome in the adult population is likely the result of the growing number of children who develop obesity. The growth in numbers of obese children has paralleled the increased incidence in type 2 diabetes and impaired glucose tolerance. The patient described in the vignette has abdominal obesity, hypertension, and a family history of type 2 diabetes, which places him at risk for the metabolic syndrome and its attendant risks for the development of cardiovascular disease as he ages. The most likely laboratory abnormalities that will be identified with the metabolic syndrome are elevated triglyceride concentrations and decreased concentrations of high-density lipoprotein cholesterol. Insulin resistance, which often is found in metabolic syndrome, may lead to hyperglycemia. C-reactive protein and fibrinogen values often are elevated in patients who have metabolic syndrome. References: Cruz M L, Weigensberg MJ, Huang TT, Ball G, Shaibi GQ, Goran MI. The metabolic syndrome in overweight Hispanic youth and the role of insulin sensitivity. J Clin Endocrinol Metab. 2004;89:108-113. Available at: http://jcem.endojournals.org/cgi/content/full/89/1/108 Ford ES, Giles WH, Dietz WH. Prevalence of the metabolic syndrome among US adults: findings from the third National Health and Nutrition Examination Survey. JAMA. 2002;287:356-359. Abstract available at: http://www.ncbi.nlm.nih.gov/entrez/query.fcgi?orig_db=PubMed&db=PubMed&cmd=Search&ter m=%22JAMA+:++the+journal+of+the+American+Medical+Association%22[Jour]+AND+2002[pdat ]+AND+Ford+E[author]

Copyright 2007 by the American Academy of Pediatrics

page 314

2007 PREP SA on CD-ROM Question: 152

A newborn female has loose neck skin (Item Q152A) and nonpitting edema of the lower extremities (Item Q152B). Of the following, the MOST appropriate evaluation for this infant is

A. blood chromosome analysis B. magnetic resonance imaging of the brain C. slitlamp ophthalmologic examination D. ultrasonography of the liver E. voiding cystourethrography

Copyright 2007 by the American Academy of Pediatrics

page 315

2007 PREP SA on CD-ROM Critique: 152 Preferred Response: A

The finding of loose neck skin (Item C152A), which is suggestive of the presence of a cystic hygroma in fetal life, and nonpitting edema of the lower extremities in a newborn female should raise the suspicion of Turner syndrome (TS). Congenital lymphedema (Item C152B), which occurs in up to 80% of affected females, typically disappears during infancy, leaving only a puffy appearance to the hands and feet, although in some patients it reappears when estrogen therapy is initiated. The redundant posterior neck skin can persist as the pterygium colli, or webbed neck. The clinical diagnosis of TS should be confirmed by peripheral blood chromosome analysis, which will reveal monosomy X, mosaic monosomy X, or the presence of an abnormal X chromosome that contains a deletion. A buccal smear lacks sensitivity and should not be used to make the diagnosis of TS. Affected girls also have short stature, ovarian dysgenesis, a broad chest with wide-spaced nipples (Item C152C), ear anomalies, cubitus valgus, and renal (eg, pelvic kidney, horseshoe kidney) and cardiac (eg, bicuspid aortic valve, coarctation of the aorta) defects. Intelligence typically is normal. Estrogen replacement therapy at the expected time of puberty is required in most cases, and treatment with growth hormone also should be offered. Magnetic resonance imaging of the brain, slitlamp ophthalmologic examination, and ultrasonography of the liver are not routinely indicated. Although renal defects may be present, the most common abnormality is horseshoe kidney, which can be detected by ultrasonography and usually has no clinical effects that would prompt the need for voiding cystourethrography. References: Frias JL, Davenport ML, Committee on Genetics and Section on Endocrinology. Health supervision for children with Turner syndrome. Pediatrics. 2003;111:692-702. Available at: http://pediatrics.aappublications.org/cgi/content/full/111/3/692 Sybert VP, McCauley E. Turner's syndrome. N Engl J Med. 2004;351:1227-1238

Copyright 2007 by the American Academy of Pediatrics

page 316

2007 PREP SA on CD-ROM Question: 153

A 16-year-old boy presents for his annual sports physical before wrestling season. He has been participating in a weight lifting program and proudly reports his increased muscle mass and strength. You are concerned that he may be using performance-enhancing substances (eg, creatine). Of the following, the use of creatine is MOST likely to be revealed by

A. abnormal liver enzyme concentrations B. an interview with the patient C. increase in hemoglobin concentration D. presence of gynecomastia E. serum drug screening

Copyright 2007 by the American Academy of Pediatrics

page 317

2007 PREP SA on CD-ROM Critique: 153 Preferred Response: B

Creatine and other substances such as anabolic steroids, androstenedione, growth hormone, and erythropoietin have been used by athletes because of their performance-enhancing reputations. Most adolescent users of performance-enhancing substances (PESs) seek to improve their athletic performance, although some use these agents to enhance their physical appearance by developing a more muscular body. All athletes, whether in organized sports or individual training programs (eg, weight lifting), should be asked about the use of PESs. The diagnosis of PES use is aided by a high index of suspicion and a history that includes the recognition of signs of use and adverse effects. Creatine enhances the ability of muscles to maintain power in short maximal exercise efforts. Creatine cannot work without strength training; those who lift weights achieve a greater benefit compared with those who do not lift. Adverse effects include muscle cramping and gastrointestinal symptoms of vomiting and diarrhea. The use of creatine is not recommended for adolescent athletes because its safety has not been established. The use of creatine and other PESs is most likely to be revealed by an interview with an adolescent. There is no specific assay to identify creatine use. Creatine has not been reported to cause abnormal liver enzyme concentrations, increased hemoglobin concentrations, or gynecomastia. The lifetime prevalence of illegal steroid use among high school students in the United States has been reported to be as high as 6%. Users of anabolic steroids anticipate gains in muscle mass and strength. Deepening of the voice and hirsutism in a female and gynecomastia and a high voice in a male should heighten the clinicians suspicion for the use of anabolic steroids. Virilization in the female also may result in hair loss. Hypogonadism or a decrease in testicular size associated with oligospermia/azoospermia and decreased sexual function/desire may occur in male users. Acne, often severe, can be observed in both male and female abusers. Other adverse effects associated with anabolic steroid abuse include hypertension, hyperaggressiveness, mood swings, psychosis, and mania. Depression can occur during withdrawal. Prepubertal abusers of anabolic steroids are at risk for premature closure of their epiphyseal growth plates. Nonspecific evidence of anabolic steroid use may be found in laboratory study results, including elevated liver enzyme concentrations and decreased highdensity and increased low-density lipoprotein cholesterol levels. Laboratory evidence of orally administered steroids can be present in the urine for days to weeks; injected steroids can be present for 6 months or longer. Androstenedione has received considerable attention in the media and is purported to increase muscle mass and strength. As a precursor to testosterone, use theoretically can induce premature puberty or the premature closure of growth plates and cause other adverse effects that have been attributed to anabolic steroid use. Assays (urine screens confirmed by gas chromatography) included in some mandatory drug screens of collegiate athletes are available to detect androstenedione by specialized laboratories. Growth hormone (GH) use theoretically increases protein anabolism and lipolysis; thus, it has a particular lure for bodybuilders. The anticipated signs of GH abuse are clinical findings consistent with acromegaly. These findings include coarsening of facial features, growth of facial bones (including the mandible, with increased separation of teeth), increase in head circumference, broadening of the nose, enlargement of the tongue, and thickness of fingers and toes. GH abuse may be associated with adverse behavior changes, hypertension, diabetes, cardiovascular disease, and peripheral neuropathy. There is no readily available, reliable test or metabolic marker to detect the use of exogenous GH. Erythropoietin increases red blood cell mass and improves oxygen delivery to skeletal muscle, thereby improving exercise endurance. Specialized laboratory testing can identify the use of natural and recombinant erythropoietin. The increased red blood cell mass also increases blood viscosity and may cause vascular occlusions. References: Coupey SM. Specific drugs. In: Schydlower M, ed. Substance Abuse: A Guide for Health Professionals. 2nd ed. Elk Grove Village, Ill: American Academy of Pediatrics; 2002:191-276

Copyright 2007 by the American Academy of Pediatrics

page 318

2007 PREP SA on CD-ROM

Greydanus DE, Patel DR. Sports doping in the adolescent athlete: the hope, hype, and hyperbole. Pediatr Clin North Am. 2002;49:829-855. Abstract available at: http://www.ncbi.nlm.nih.gov/entrez/query.fcgi?orig_db=PubMed&db=PubMed&cmd=Search&ter m=%22Pediatric+clinics+of+North+America%22[Jour]+AND+829[page]+AND+2002[pdat] Hergenroeder AC, Neinstein LS. Guidelines in sports medicine. In: Neinstein LS, ed. Adolescent Health Care A Practice Guide. 4th ed. Philadelphia, Pa: Lippincott Williams & Wilkins; 2002:380438 Koch JJ. Performance-enhancing substances and their use among adolescent athletes. Pediatr Rev. 2002;23:310-317. Available at: http://pedsinreview.aappublications.org/cgi/content/full/23/9/310

Copyright 2007 by the American Academy of Pediatrics

page 319

2007 PREP SA on CD-ROM Question: 154

A 14-year-old boy has a body mass index that is greater than the 95th percentile for age and an accelerating weight curve. His blood pressure is 135/85 mm Hg. His mother has type 2 diabetes that developed during her first pregnancy, and several paternal relatives also have type 2 diabetes. The family emigrated from the Caribbean when he was a toddler. He has dark velvety thickening (Item Q154A) of the skin on his neck and under his arms. Of the following, the findings that are MOST supportive of a diagnosis of metabolic syndrome in this young man are

A. acanthosis nigricans, hypertension, obesity B. acanthosis nigricans and maternal history of diabetes C. acanthosis nigricans, obesity D. hypertension, obesity E. obesity and maternal history of diabetes

Copyright 2007 by the American Academy of Pediatrics

page 320

2007 PREP SA on CD-ROM Critique: 154 Preferred Response: D

The constellation of findings that lead to the diagnosis of metabolic syndrome in adults varies, depending upon the criteria used, although hypertension and obesity are common to all definitions. The latest criteria from the International Diabetes Federation include ethnicity-specific waist circumference criteria or a body mass index greater than 30 kg/M2 plus two of the following: Fasting triglyceride concentration greater than 150 mg/dL (1.7 mmol/L) or treatment for this finding High-density lipoprotein cholesterol concentration less than 40 mg/dL (1.03 mmol/L) in men and 50 mg/dL (1.29 mmol/L) in women Hypertension manifested by systolic blood pressure of 130 mm Hg or greater and diastolic pressure of 85 mm Hg or greater or previously treated hypertension Fasting plasma glucose value of 100 mg/dL (5.5 mmol/L) or greater or previously diagnosed type 2 diabetes Definitions in childhood are not established, but usually consist of similar criteria adjusted to age-appropriate values greater than the 85th percentile for blood pressure, weight, and waist circumference. A recent review of the criteria for the diagnosis of metabolic syndrome concludes that metabolic syndrome may not be an entity in adults that is greater in severity than the sum of its components. Acanthosis nigricans is not included as a criterion for the diagnosis of metabolic syndrome, although it often is found. Maternal or family history of diabetes frequently is found and increases the risk of type 2 diabetes in the child, but it is not one of the criteria for the diagnosis of metabolic syndrome. References: Boney CM, Verma A, Tucker R, Vohr BR. Metabolic syndrome in childhood: association with birth weight, maternal obesity, and gestational diabetes mellitus. Pediatrics. 2005;115:e290-e296. Available at: http://pediatrics.aappublications.org/cgi/content/full/115/3/e290 Ford ES. Prevalence of the metabolic syndrome defined by the International Diabetes Federation among adults in the U.S. Diabetes Care. 2005;28:2745-2749. Available at: http://care.diabetesjournals.org/cgi/content/full/28/11/2745 Hirschler V, Aranda C, Calcagno Mde L, Maccalini G, Jadzinsky M. Can waist circumference identify children with the metabolic syndrome? Arch Pediatr Adolesc Med. 2005;159:740-744. Abstract available at: http://www.ncbi.nlm.nih.gov/entrez/query.fcgi?orig_db=PubMed&db=PubMed&cmd=Search&ter m=%22Archives+of+pediatrics+%26+adolescent+medicine%22[Jour]+AND+740[page]+AND+20 05[pdat] Janssen I, Katzmarzyk PT, Srinivasan SR, et al. Combined influence of body mass index and waist circumference on coronary artery disease risk factors among children and adolescents. Pediatrics. 2005;115:1623-1630. Available at: http://pediatrics.aappublications.org/cgi/content/full/115/6/1623 Kahn R, Buse J, Ferrannini E, Stern M; American Diabetes Association; European Association for the Study of Diabetes. The metabolic syndrome: time for a critical appraisal: joint statement from the American Diabetes Association and the European Association for the Study of Diabetes. Diabetes Care. 2005;28:2289-2304. Available at: http://care.diabetesjournals.org/cgi/content/full/28/9/2289

Copyright 2007 by the American Academy of Pediatrics

page 321

2007 PREP SA on CD-ROM

Ten S, Maclaren N. Insulin resistance syndrome in children. J Clin Endocrinol Metab. 2004;89:2526-2539. Available at: http://jcem.endojournals.org/cgi/content/full/89/6/2526

Copyright 2007 by the American Academy of Pediatrics

page 322

2007 PREP SA on CD-ROM Question: 155

You are evaluating a 10-year-old boy who has behavior problems. He hits his younger brother, talks back to his parents, and refuses to perform his assigned household chores. He does well in school, but is oppositional in the classroom. His parents ask what they can do to improve his behavior. Of the following, your BEST response is that

A. he needs cognitive testing B. he should be referred for individual counseling C. medication is the preferred first-line treatment D. school-based behavior programs are preferred to home-based programs E. the parents should be trained in behavior modification techniques

Copyright 2007 by the American Academy of Pediatrics

page 323

2007 PREP SA on CD-ROM Critique: 155 Preferred Response: E

Behavior modification is a general category of therapy that refers to principles and techniques based on learning theory and used to change behavior. Behavioral techniques are used to either strengthen or maintain desired behaviors or to decrease or eliminate undesired behaviors. For example, parental praise or token economies are examples of techniques used to reinforce behavior, and time-out is a technique used to decrease or eliminate a behavior. The child described in the vignette has oppositional behavior that may respond to behavioral management. He is doing well in school and, therefore, probably does not need cognitive testing. Individual counseling may be used as an adjunct, but he is more likely to respond to consistent behavior management. Behavioral approaches should be tried before using medication to control behavior. The behavioral plan should be instituted both at home and in school, with communication between teacher and parents to maintain consistency. References: Boris NW, Dalton R. Disruptive behavioral disorders. In: Behrman RE, Kliegman RM, Jenson HB, eds. Nelson Textbook of Pediatrics. 17th ed. Philadelphia, Pa: WB Saunders Co; 2004:88-90 Vitulano LA. Child and adolescent behavior therapy. In: Lewis M, ed. Child and Adolescent Psychiatry: A Comprehensive Textbook. 3rd ed. Philadelphia, Pa: Lippincott Williams & Wilkins; 2002:998-1014

Copyright 2007 by the American Academy of Pediatrics

page 324

2007 PREP SA on CD-ROM Question: 156

A 14-month-old girl is brought to the emergency department with a 12-hour history of fever and rash. Her mother became frightened when it was difficult to arouse the girl after her nap. Findings on physical examination include a temperature of 104F (40C), a heart rate of 164 beats/min, a respiratory rate of 42 breaths/min, and a blood pressure of 75/45 mm Hg. There are petechiae and purpura (Item Q156A) on the chest, arms, and legs. As you are establishing intravenous access and drawing blood for initial laboratory evaluations, you request an immediate dose of antibiotics. Of the following, the MOST appropriate therapy is

A. ceftriaxone B. penicillin C. vancomycin D. vancomycin plus ceftriaxone E. vancomycin plus gentamicin

Copyright 2007 by the American Academy of Pediatrics

page 325

2007 PREP SA on CD-ROM Critique: 156 Preferred Response: D

The findings reported for the patient in the vignette suggest that she has sepsis due to Neisseria meningitidis, but nearly all pathogenic bacteria can cause petechiae and purpura. When treating patients who have potentially life-threatening infections, the initial choice of antimicrobial agents should be broad enough to cover the endemic pathogenic bacteria in the local community. Once a bacterial pathogen is isolated and susceptibility data are known, the antimicrobial therapy can be simplified. The treatment of sepsis varies with age of the patient. Vancomycin and ceftriaxone represent the best choice for most patients older than 3 months who have potentially life-threatening infections. Vancomycin provides coverage for penicillin- and cephalosporin-resistant Streptococcus pneumoniae, Staphylococcus aureus, and Enterococcus, and ceftriaxone is effective against N meningitidis, most S pneumoniae, and other gram-negative organisms such as Haemophilus influenzae. Single agents (eg, ceftriaxone, penicillin, vancomycin) do not provide enough coverage in a critically ill patient. Vancomycin and gentamicin provide broad coverage, but gentamicin is not active against H influenzae and does not penetrate the central nervous system as effectively as ceftriaxone. For most neonates, ampicillin in addition to cefotaxime or gentamicin is an appropriate choice. Many experts suggest that if central nervous system disease has not or cannot be ruled out, cefotaxime is preferable because the cephalosporins penetrate the central nervous system better than the aminoglycosides. Vancomycin rarely is included in the treatment of neonates who have community-acquired sepsis. Certain situations may call for the addition of acyclovir. Patients between 1 and 3 months of age are more problematic. It is during this age that a crossover can occur from pathogens commonly seen in the neonatal period (eg, group B Streptococcus, Escherichia coli, Listeria monocytogenes, Enterococcus) to those usually encountered in children older than 3 months of age (eg, S pneumoniae, N meningitidis, S aureus). Clinicians should understand the common bacterial pathogens and their resistance patterns in their community when making antibiotic choices. If there is concern for methicillinresistant S aureus or penicillin- or cephalosporin-resistant S pneumoniae, vancomycin should be part of the empiric regimen. References: Stormorken A, Powell KR. Sepsis and shock. In: Behrman RE, Kliegman RM, Jenson HB, eds. Nelson Textbook of Pediatrics. 17th ed. Philadelphia, Pa: WB Saunders Co; 2004:846-850 American Academy of Pediatrics. Meningococcal infections. In: Pickering LK, ed. Red Book: 2006 Report of the Committee on Infectious Diseases. 27th ed. Elk Grove Village, Ill: American Academy of Pediatrics; 2006:452-460 Durbin WJ. Pneumococcal infections. Pediatr Rev. 2004;25:418-424. Available at: http://pedsinreview.aappublications.org/cgi/content/full/25/12/418

Copyright 2007 by the American Academy of Pediatrics

page 326

2007 PREP SA on CD-ROM Question: 157

You are evaluating a newborn boy who has lax abdominal musculature (Item Q157A) and bilateral undescended testes. Other findings on physical examination are normal. Of the following, the MOST likely urologic abnormality in this boy is

A. hydronephrosis B. renal cysts C. ureterocele D. ureteropelvic junction obstruction E. vesicoureteral reflux

Copyright 2007 by the American Academy of Pediatrics

page 327

2007 PREP SA on CD-ROM Critique: 157 Preferred Response: A

Prune belly (Eagle-Barrett) syndrome (PBS) is a relatively uncommon condition resulting from poorly developed abdominal musculature. Affected children may have a variety of urinary tract anomalies, including undescended testes, hydronephrosis, posterior urethral valves (PUV), and bladder dysfunction. The lax abdominal musculature (Item C157A) and bilateral undescended testes described for the boy in the vignette suggest the diagnosis of PBS. Abdominal ultrasonography and voiding cystourethrography are indicated in all children who have PBS to study the entire urinary tract. Hydronephrosis is a common anomaly associated with PBS. The hydronephrosis usually is obstructive and due to PUV or vesicoureteral reflux and ureteropelvic junction obstruction. Renal cysts are very uncommon, and ureteroceles are seen only occasionally in children who have PBS. Some affected children may have nonobstructive hydronephrosis. The best test to differentiate obstructive from nonobstructive hydronephrosis is renal scintigraphy. In obstructive hydronephrosis, the tracer cannot escape the kidney, but in most kidneys with nonobstructive hydronephrosis, some amount of the tracer exits the kidney. The renal outcome for children who have PBS is generally poor. Meticulous attention must be paid to the prevention of urinary tract infections (UTIs). Thus, it is imperative to institute antibiotic prophylaxis if obstruction is present, and in some cases of moderate-to-severe hydronephrosis with obstruction, early surgical repair is recommended. Urinary diversion via a vesicostomy or Mitrofanoff procedure (using the appendix to connect the bladder to the abdomen, with exit near the umbilicus) may be indicated to prevent urinary stasis and UTIs. Unfortunately, renal growth usually is compromised because of multiple abnormalities within the urinary tract, and most children who have PBS develop renal insufficiency. References: Denes FT, Arap MA, Giron AM, Silva FA, Arap S. Comprehensive surgical treatment of prune belly syndrome: 17 years' experience with 32 patients. Urology. 2004;64:789-793. Abstract available at: http://www.ncbi.nlm.nih.gov/entrez/query.fcgi?orig_db=PubMed&db=PubMed&cmd=Search&ter m=%22Urology%22[Jour]+AND+789[page]+AND+2004[pdat] Jennings RW. Prune belly syndrome. Semin Pediatr Surg. 2000;9:115-120. Abstract available at: http://www.ncbi.nlm.nih.gov/entrez/query.fcgi?orig_db=PubMed&db=PubMed&cmd=Search&ter m=%22Seminars+in+pediatric+surgery%22[Jour]+AND+115[page]+AND+2000[pdat] Strand WR. Initial management of complex pediatric disorders: prunebelly syndrome, posterior urethral valves. Urol Clin North Am. 2004;31:399-415. Abstract available at: http://www.ncbi.nlm.nih.gov/entrez/query.fcgi?orig_db=PubMed&db=PubMed&cmd=Search&ter m=%22The+Urologic+clinics+of+North+America%22[Jour]+AND+399[page]+AND+2004[pdat]

Copyright 2007 by the American Academy of Pediatrics

page 328

2007 PREP SA on CD-ROM Question: 158

A 15-year-old boy was camping with friends 1 week ago. Over the last 4 days, he has developed fever, worsening headache, increasing sleepiness, and combativeness. You suspect arboviral meningoencephalitis. Of the following, the MOST frequently used method to confirm the diagnosis is

A. detection of myelin basic protein in the cerebrospinal fluid (CSF) B. detection of viral antigen in brain biopsy tissue C. detection of viral nucleic acid in the CSF by polymerase chain reaction D. isolation of the virus by culture of the CSF E. measurement of acute and convalescent antibody titers in serum or CSF

Copyright 2007 by the American Academy of Pediatrics

page 329

2007 PREP SA on CD-ROM Critique: 158 Preferred Response: E

Arboviruses are an important cause of encephalitis in the United States during the late spring and summer months. These agents are transmitted by the bites of mosquitos, ticks, or sandflies. Examples of arboviral encephalitis encountered in North America include St. Louis encephalitis, La Crosse encephalitis, Western and Eastern equine encephalitis, California encephalitis, West Nile encephalitis, and Colorado tick fever. The diagnosis should be suspected in any child who presents with symptoms of fever, irritability or change in mental status, and headache. Cerebrospinal fluid (CSF) shows pleocytosis (usually <500 cells with a mononuclear predominance), normal-to-mildly elevated CSF protein concentration, and a normal CSF glucose concentration. Most arboviral infections resolve within 2 weeks, but there is a mortality rate of 10%. Eastern equine encephalitis has a much higher mortality rate of up to 50%. Confirming the diagnosis of arboviral encephalitis remains difficult because rapid diagnostic tests such as polymerase chain reaction are unavailable, and culture of the CSF is rarely helpful. Isolation of virus or immunohistochemical detection of viral antigen from brain tissue may provide positive results, but this requires a brain biopsy, which rarely is performed. Measurement of myelin basic protein, which can be elevated in patients who have multiple sclerosis and other demyelinating conditions, is not helpful in the diagnosis of arboviral infections. Serology continues to be the mainstay of diagnosis for arboviral infections, with several different serologic techniques available. The presence of virus-specific immunoglobulin M (IgM) detected by IgM-capture enzyme immunoassay in the CSF is diagnostic, and the finding of virus-specific IgM in the serum is presumptive evidence of infection. The demonstration of a fourfold elevation of virus-specific IgG in acute and convalescent (4 to 6 weeks) sera, as measured by immunofluorescence, hemagglutination inhibition, complement fixation, or neutralization, is the most common method of confirming an arboviral infection. An elevated virusspecific IgG titer in a single serum sample is also strong evidence of a recent infection. References: American Academy of Pediatrics. Arboviruses (including California encephalitis [primarily La Crosse], Eastern and Western equine encephalitis, Powassan encephalitis, St Louis encephalitis, Venezuelan equine encephalitis, West Nile encephalitis, Colorado tick fever, dengue, Japanese encephalitis, and yellow fever). In: Pickering LK, ed. Red Book: 2006 Report of the Committee on Infectious Diseases. 27th ed. Elk Grove Village, Ill: American Academy of Pediatrics; 2006:211-217 Hayes EB, OLeary DR. West Nile virus infection: a pediatric perspective. Pediatrics. 2004;113:1375-1381. Available at: http://pediatrics.aappublications.org/cgi/content/full/113/5/1375 Solomon T. Flavivirus encephalitis. N Engl J Med. 2004;351:370-378. Abstract available at: http://www.ncbi.nlm.nih.gov/entrez/query.fcgi?orig_db=PubMed&db=PubMed&cmd=Search&ter m=%22The+New+England+journal+of+medicine%22[Jour]+AND+370[page]+AND+2004[pdat]

Copyright 2007 by the American Academy of Pediatrics

page 330

2007 PREP SA on CD-ROM Question: 159

An 18-year-old girl who has asthma presents for an annual evaluation. She states that she has had asthma for the past 6 years. Her symptoms include cough, wheezing, and chest tightness, primarily triggered by upper respiratory tract infections, strenuous exercise, and pollen exposure. She never has been hospitalized for her asthma, but she had to visit the emergency department 2 years ago for an exacerbation. After reviewing her symptom frequency, medication use, and metered-dose inhaler technique, she performs a pulmonary function test (Item Q159A). The prebronchodilator FEV1 is 74% of expected and the FEV1/FVC is 79% of expected. Of the following, the BEST characterization of her asthma severity, based on results of her pulmonary function test, is

A. mild obstructive with reversibility B. mild restrictive without reversibility C. moderate obstructive with reversibility D. moderate restrictive without reversibility E. normal

Copyright 2007 by the American Academy of Pediatrics

page 331

2007 PREP SA on CD-ROM Critique: 159 Preferred Response: C

Spirometry, also referred to as a pulmonary function test (PFT), can measure inspiratory and expiratory respiratory effort. When interpreting spirometry, measurements of forced vital capacity (FVC), volume of air exhaled during the first second (FEV1), and the FEV1/FVC ratio help determine obstructive versus restrictive disease. If obstructive disease is suspected, the severity of obstructive lung disease can be classified further as mild, moderate, or severe (Item C159A). The adolescent in the vignette demonstrated a prebronchodilator FEV1 of 74% and FEV1/FVC of 79%, which is consistent with moderate obstructive severity. Reversibility (Item C159B) is defined as an increase in FEV1 of 12% and 200 mL after a short-acting bronchodilator. The adolescent in the vignette demonstrated a 35% increase (+700 mL) in FEV1 after a bronchodilator treatment, which is consistent with airway reversibility. Causes of obstructive lung disease include asthma and cystic fibrosis; restrictive lung disease may indicate interstitial fibrosis, scoliosis, or neuromuscular disease. References: Guill MF. Asthma update: clinical aspects and management. Pediatr Rev. 2004;25:335-344. Available at: http://pedsinreview.aappublications.org/cgi/content/full/25/10/335 Liu AH, Spahn JD, Leung DYM. Childhood asthma. In: Behrman RE, Kliegman RM, Jenson HB, eds. Nelson Textbook of Pediatrics. 17th ed. Philadelphia, Pa: WB Saunders Co 2004:760-774

Copyright 2007 by the American Academy of Pediatrics

page 332

2007 PREP SA on CD-ROM Question: 160

A 16-year-old boxer presents to your office 2 hours after a blow delivered to one ear during sparring. The ear is swollen and painful. On physical examination, you note a 2-cm diameter area of bruising and fluctuance along the upper aspect of the pinna anteriorly. Of the following, the MOST appropriate next step in the management of this patient is to

A. apply ice to the ear to limit further swelling B. apply a pressure dressing C. arrange for consultation with an otorhinolaryngologist within 1 to 3 days D. evacuate the hematoma with needle and syringe E. provide antibiotic prophylaxis to prevent infection

Copyright 2007 by the American Academy of Pediatrics

page 333

2007 PREP SA on CD-ROM Critique: 160 Preferred Response: D

A tangential blow to the ear can cause a shearing force that separates the skin from the underlying perichondrium or the perichondrium from the cartilage. The resulting hematoma (Item C160A) may cause long-term disfigurement if not managed properly. The patient described in the vignette has an auricular hematoma that should be evacuated by fine-needle aspiration before a clot forms. The classic sequela of an auricular hematoma is the cauliflower ear commonly found in boxers, kickboxers, and wrestlers, which is caused by infiltration of fibroblasts into the hematoma and eventual neocartilage formation. Such migration can be prevented by early fineneedle evacuation of the hematoma. Larger hematomas and partially organized hematomas may require an open approach. Another potential consequence of an auricular hematoma is necrosis of the cartilage due to separation of the perichondrium from the cartilage, which deprives the cartilage of its blood supply. The clot can be evacuated by an open or closed method. Postevacuation care sometimes includes splints or pressure dressings, but no randomized studies clearly demonstrate the need for these additional measures or the optimal management strategy. Refractory auricular hematomas may require more aggressive treatment, such as open debridement. Ice may be applied to the ear after evacuation of the hematoma, but it is not adequate therapy by itself. Similarly, antibiotic prophylaxis against skin flora, including Staphylococcus aureus and group A staphylococci, is recommended, but only after hematoma evacuation. Consultation with an otorhinolaryngologist is appropriate, but the clot will have stabilized within 1 to 3 days, making needle evacuation ineffective if delayed until then. References: Ghanem T, Rasamny JK, Park SS. Rethinking auricular trauma. Laryngoscope. 2005;115:12511255. Abstract available at: http://www.ncbi.nlm.nih.gov/entrez/query.fcgi?orig_db=PubMed&db=PubMed&cmd=Search&ter m=Laryngoscope[Jour]+AND+1251[page]+AND+2005[pdat] Jones SEM, Mahendran S. Interventions for acute auricular haematoma. The Cochrane Database of Systematic Reviews. 2004;2:CD004166. Available at: http://www.mrw.interscience.wiley.com/cochrane/clsysrev/articles/CD004166/frame.html Kazahaya K, Handler SD. Otolaryngologic trauma. In: Fleisher GR, Ludwig S, Henretig FM, eds. Textbook of Pediatric Emergency Medicine. 5th ed. Philadelphia, Pa: Lippincott Williams & Wilkins; 2006:1497-1505

Copyright 2007 by the American Academy of Pediatrics

page 334

2007 PREP SA on CD-ROM Question: 161

A 7-year-old child presents with a scalp lesion characterized by patchy areas of hair loss with palpable suboccipital nodes (Item Q161A). Of the following, the MOST likely diagnosis is

A. alopecia areata B. nevus sebaceus C. seborrheic dermatitis D. tinea amiantacea E. tinea capitis

Copyright 2007 by the American Academy of Pediatrics

page 335

2007 PREP SA on CD-ROM Critique: 161 Preferred Response: E

The scalp is a common site of infection by fungal dermatophytes. Organisms most frequently responsible in children include Trichophyton tonsurans and Microsporum canis. T tonsurans is responsible for 95% of infections in North America. Infection may produce one or several discrete patches of hair loss (Item C161A) within which are scale and the remnants of broken hairs within follicles (ie, black dot hairs). In some patients, infection may be more diffuse, resulting in widespread scaling (Item C161B) and less obvious alopecia (ie, the seborrheic form). Other manifestations include pustules or a tender boggy mass (ie, a kerion) (Item C161C). The presence of enlarged suboccipital or posterior cervical lymph nodes in association with hair loss, as described for the child in the vignette, should raise suspicion for tinea capitis. Alopecia areata (Item C161D) is characterized by the sudden appearance of sharply circumscribed areas of complete hair loss. The scaling, black dot hairs, and lymphadenopathy seen in tinea capitis are not observed. Nevus sebaceus (of Jadassohn) (Item C161E) is associated with alopecia but is present at birth as a slightly raised, oval or linear yellow-orange plaque. Scalp involvement in seborrheic dermatitis is characterized by the accumulation of greasy scale; hair loss is uncommon. Tinea (pityriasis) amiantacea (Item C161F) is a manifestation of psoriasis that appears as thick, white asbestos-like scale that adheres to the hair and scalp. The thickness of the scale and absence of alopecia distinguish this disorder from tinea capitis. References: Hogan PA. Papulosquamous disease: psoriasis vulgaris. In: Schachner LA, Hansen RC, eds. Pediatric Dermatology. 3rd ed. St. Louis, Mo: Mosby; 2003:643-652 Tay Y-K. Hair disorders: nonscarring alopecia. In: Schachner LA, Hansen RC, eds. Pediatric Dermatology. 3rd ed. St. Louis, Mo: Mosby; 2003:544-552 Weston WL, Lane AT, Morelli JG. Birthmarks. In: Color Textbook of Pediatric Dermatology. 3rd ed. St. Louis, Mo: Mosby; 2002:308-316 Weston WL, Lane AT, Morelli JG. Dermatitis: seborrheic dermatitis. In: Color Textbook of Pediatric Dermatology. 3rd ed. St. Louis, Mo: Mosby; 2002:39-40 Weston WL, Lane AT, Morelli JG. Fungal and yeast infections of the skin: hair infections (tinea capitis). In: Color Textbook of Pediatric Dermatology. 3rd ed. St. Louis, Mo: Mosby; 2002:63-66 Weston WL, Lane AT, Morelli JG. Hair disorders: common forms of hair loss. In: Color Textbook of Pediatric Dermatology. 3rd ed. St. Louis, Mo: Mosby; 2002:222-224

Copyright 2007 by the American Academy of Pediatrics

page 336

2007 PREP SA on CD-ROM Question: 162

A 15-year-old girl is admitted to your facility with severe anorexia nervosa and amenorrhea. She weighs 35 kg and is 160 cm tall. She has bradycardia and orthostatic hypotension. You plan to stabilize her medically and begin nasogastric tube feeding. Of the following, the electrolyte abnormality that is MOST likely to occur during the first week of her treatment is

A. hypercalcemia B. hyperphosphatemia C. hypocalcemia D. hyponatremia E. hypophosphatemia

Copyright 2007 by the American Academy of Pediatrics

page 337

2007 PREP SA on CD-ROM Critique: 162 Preferred Response: E

The girl described in the critique is severely malnourished and about to undergo intensive nutritional rehabilitation with enteral feeding. Although this therapy is lifesaving, it also may result in refeeding syndrome. In this syndrome, malnourished patients given oral, enteral, or intravenous nutrition develop fluid retention and electrolyte abnormalities. The most common electrolyte abnormality reported is hypophosphatemia, which can occur in approximately 25% of patients who have anorexia nervosa during refeeding. Other electrolyte and micronutrient abnormalities, including hyponatremia, hypocalcemia, hypokalemia, hyperglycemia, and thiamine deficiency, also may occur, but are less prevalent than hypophosphatemia. Hypercalcemia and hyperphosphatemia generally do not occur. With careful monitoring of electrolytes and slow refeeding, signs and symptoms of refeeding syndrome can be avoided. However, the clinical manifestations include edema, muscle weakness, and cardiac arrhythmias. The precise mechanism by which phosphorus levels are lowered during refeeding has not yet been characterized fully. Malnourished patients are depleted in total body phosphorus, despite normal serum concentrations. The carbohydrate challenge during refeeding induces the release of insulin, which causes fluid and electrolyte shifts. In addition, malnourished patients being refed synthesize the phosphate-rich compounds creatine phosphokinase, 2,3 diphosphoglycerate, and adenosine triphosphate. Thus, total body stores of phosphorus may be depleted even further during energy synthesis. To minimize the risk of refeeding syndrome, supplemental feedings should be introduced gradually to malnourished patients (beginning at 25% of recommended calories and advancing to full calories over 5 to 7 days). In addition, serum electrolytes, blood glucose, calcium, phosphorus, and magnesium should be measured at least daily for the first week of feeding. Many centers automatically prescribe a phosphorus supplement (500 mg twice a day) to patients who have anorexia nervosa and are being refed. References: Frates SE. Eating disorders. In: Hendricks KM, Duggan C, eds. Manual of Pediatric Nutrition. 4th ed. Hamilton, Ontario, Canada: BC Decker; 2005:494-504 Marinella MA. Refeeding syndrome and hypophosphatemia. J Intensive Care Med. 2005;20:155159. Abstract available at: http://www.ncbi.nlm.nih.gov/entrez/query.fcgi?cmd=Retrieve&db=pubmed&dopt=Abstract&list_ui ds=15888903&query_hl=2&itool=pubmed_docsum

Copyright 2007 by the American Academy of Pediatrics

page 338

2007 PREP SA on CD-ROM Question: 163

You are present at the birth of an infant in whom bilateral hydronephrosis was diagnosed in utero. A fetal shunt was placed in each flank between the renal pelvis and the amniotic cavity. Nonetheless, the infant has bilaterally palpable flank masses, and the shunts are not apparent at birth. The infant shows no dysmorphisms and has no respiratory distress. Renal ultrasonography reveals bilateral hydronephrosis. Of the following, the MOST correct statement regarding the fetus/neonate who has obstructive uropathy is that

A. urinary tract infection, hydronephrosis, and respiratory distress all can be treated and resolved
with a fetal shunt

B. urinary tract infection, hydronephrosis, and respiratory distress typically lead to fetal or
neonatal death

C. urinary tract infection is common, hydronephrosis often persists, and respiratory distress is
not uncommon

D. urinary tract infection is uncommon, hydronephrosis resolves spontaneously, and respiratory


distress is uncommon

E. urinary tract infection is uncommon, hydronephrosis resolves spontaneously, and respiratory


distress results from apnea

Copyright 2007 by the American Academy of Pediatrics

page 339

2007 PREP SA on CD-ROM Critique: 163 Preferred Response: C

The infant described in the vignette has a prenatal diagnosis of hydronephrosis and palpably enlarged kidneys on physical examination. Despite a history of shunting in utero, there are no apparent shunts present on examination. Because this is not uncommon when upper urinary tract obstruction is the problem, present techniques generally address direct drainage of renal calyces or cysts by needle under ultrasonographic guidance. Hydronephrosis in the fetus, which may be physiologic and transient, or pathologic, is observed in 1 in 500 to 700 fetuses. Pathologic obstructive uropathy most frequently results from ureteropelvic junction obstruction or, less commonly, ureterovesical obstruction. In males, the most common obstructive uropathy is posterior urethral valves (Item C163A), an obstruction of the bladder outlet. Obstruction of the emptying of the renal collecting system, ureters, or bladder may result in retrograde hydrostatic pressure and destruction of the renal parenchyma. Rupture of the fetal urinary collecting system under these conditions is believed to cause urinary ascites. When an obstruction is identified in a fetus, a vesicoamniotic shunt to relieve pressure and preserve renal function may be considered. The optimal timing of delivery may be determined in balancing the effects of oligohydramnios and pulmonary hyperplasia against progressive renal parenchymal loss. Unfortunately, efforts to mitigate the obstruction and consequential renal impairment by placing shunts have not met with great success. Fetal shunts may become dislodged or obstructed, the accumulation of urine continues, and their placement imposes risks of preterm labor and chorioamnionitis. Amniotic fluid volumes may be diminished in the presence of fetal urinary tract obstruction, which may, in turn, lead to impaired fetal lung development, especially if the amniotic fluid volume is reduced during the canalicular stage of lung development (16 to 24 weeks gestation). Postnatal death from pulmonary hypoplasia is well described in affected patients. Although certain fetuses who have lower urologic obstructions may benefit from vesicoamniotic shunting, patient selection is difficult, and no large randomized trials have demonstrated clear benefit. At present, placement is reserved for fetuses whose delivery is not imminent and who have a high risk for pulmonary hypoplasia due to the presence of oligohydramnios. Postnatally, infants who have obstructive uropathy are at increased risk for urinary tract infection, vesicoureteral reflux, hydronephrosis, and possibly respiratory distress, but these conditions do not typically lead to fetal or neonatal death. Respiratory distress may be related to impingement of renal masses on the diaphragm, urinary ascites, or an element of pulmonary hypoplasia. Although simple, transient, physiologic hydronephrosis may resolve spontaneously, obstructive uropathy does not. References: Chevalier RL. Perinatal obstructive nephropathy. Semin Perinatol. 2004;28:124-131. Abstract available at: http://www.ncbi.nlm.nih.gov/entrez/query.fcgi?cmd=Retrieve&db=pubmed&dopt=Abstract&list_ui ds=15200251&query_hl=38&itool=pubmed_docsum Crombleholme TM. Surgical treatment of the fetus. In: Martin RJ, Fanaroff AA, Walsh MC, eds. Fanaroff and Martins Neonatal-Perinatal Medicine: Diseases of the Fetus and Infant. 8th ed. Philadelphia, Pa: Mosby-Elsevier; 2006:231-254 Kennedy WA II. Assessment and management of fetal hydronephrosis. NeoReviews. 2002;3:e214-e219. Available at: http://neoreviews.aappublications.org/cgi/content/full/3/10/e214

Copyright 2007 by the American Academy of Pediatrics

page 340

2007 PREP SA on CD-ROM Question: 164

During the health supervision visit of a 2-year-old boy who is new to your practice, you note a productive cough that his mother says has been present for several months. His weight is below the 5th percentile. Fine crackles are present in his lungs bilaterally, and he has mild clubbing of the fingers. The remainder of his physical examination findings are unremarkable. A chest radiograph (Item Q164A) reveals poorly defined bronchovascular markings and a few cystic spaces bilaterally, prompting you to order computed tomography scan of the chest, which reveals diffuse bronchiectasis. Of the following, the MOST likely diagnosis is

A. allergic bronchopulmonary aspergillosis B. cystic fibrosis C. foreign body aspiration D. Mycobacterium tuberculosis infection E. primary ciliary dyskinesia

Copyright 2007 by the American Academy of Pediatrics

page 341

2007 PREP SA on CD-ROM Critique: 164 Preferred Response: B

Bronchiectasis is characterized by abnormal dilatation of the bronchi and bronchioles. It may be localized or diffuse, and the changes in the bronchial tree are irreversible. The most common cause of diffuse bronchiectasis in children is cystic fibrosis, and affected children also may exhibit chronic diarrhea and failure to thrive. Recurrent infections and inflammation cause chronic obstruction and hypertrophy of the wall of the bronchi. Less common causes of diffuse bronchiectasis include allergic bronchopulmonary aspergillosis, primary ciliary dyskinesia, immunodeficiency, and infections such as tuberculosis, pertussis, and measles. Foreign body aspiration may lead to local bronchiectasis, but diffuse bronchiectasis is unusual. Other causes of local bronchiectasis include lobar pneumonia, right middle lobe syndrome, and extrinsic compression by enlarged lymph nodes. Clinical symptoms of bronchiectasis are cough productive of copious sputum, hemoptysis, and anorexia. Localized rales or wheezing may be present on physical examination. Chest radiographs typically reveal nonspecific findings such as atelectasis, focal pneumonitis, and increased bronchovascular markings. The diagnosis may be made by high-resolution computed tomography (CT) scan of the chest, which reveals enlargement of the bronchial tree. CT scan is used to differentiate between the types of bronchiectasis, which are cylindrical, varicose, and saccular (the most severe type). If bronchiectasis is diagnosed, an evaluation for cystic fibrosis, immunodeficiencies, and infections is warranted. In general, treatment of bronchiectasis consists of chest physiotherapy, bronchodilators, and antibiotics. References: Barker AF. Bronchiectasis. N Engl J Med. 2002;346:1383-1393 Davis PB. Cystic fibrosis. Pediatr Rev. 2001;22:257-264. Available at: http://pedsinreview.aappublications.org/cgi/content/full/22/8/257 Lakser O. Bronchiectasis. In: Behrman RE, Kliegman RM, Jenson HB, eds. Nelson Textbook of Pediatrics. 17th ed. Philadelphia, Pa: WB Saunders Co; 2004:1436

Copyright 2007 by the American Academy of Pediatrics

page 342

2007 PREP SA on CD-ROM Question: 165

A child presents for her 2-week evaluation after being delivered by a midwife at home. The parents are concerned that they have never seen her turn her head, which makes it difficult for her to feed at the breast. They also note that her back does not appear normal. On physical examination, her hairline appears low posteriorly. You confirm that she does not turn her head, and when placed prone, does not turn her head to the side. Her right scapula appears to be higher than the left, and you note that the spine does not appear to be perfectly straight, suggesting congenital scoliosis. You obtain an anteroposterior radiograph of the cervical spine, which shows multilevel segmentation anomalies (Item Q165A) in the mid to lower cervical spine. Of the following, the condition that is MOST consistent with these findings is

A. cleidocranial dysostosis B. Klippel-Feil syndrome C. Noonan syndrome D. Sandifer syndrome E. Turner syndrome

Copyright 2007 by the American Academy of Pediatrics

page 343

2007 PREP SA on CD-ROM Critique: 165 Preferred Response: B

The Klippel-Feil syndrome involves the fusion of cervical vertebrae (Item C165A) and occurs in approximately 1 in 42,000 births, with a 65% female predominance. It is usually a sporadic event. Due to neck immobility, affected individuals are at risk of cervical spine injury. Associated defects may include deafness (conduction or sensorineural, occurring in up to 30% of patients), congenital heart defects (usually ventricular septal defect), rib defects, hemivertebrae, Sprengel anomaly (elevation of the scapula), scoliosis, and renal anomalies. Noonan and Turner syndrome may be associated with a webbed neck, but the spine is not involved. Sandifer syndrome refers to the tonic extension of the neck associated with gastroesophageal reflux and is functional rather than anatomic. Cleidocranial dysostosis presents with absence of the clavicles, yielding a droopy chest rather than the elevated scapula and shoulders that appear in Klippel-Feil syndrome. References: Klippel-Feil sequence. In: Jones KL. Smiths Recognizable Patterns of Human Malformations. 6th ed. Philadelphia, Pa: Elsevier Saunders; 2006:716-718 Weaver DD, Wheeler PG. Skeletal dysplasias. In: McMillan JA, DeAngelis CD, Feigin RD, Warshaw JB, eds. Oskis Pediatrics: Principles and Practice. Philadelphia, Pa: Lippincott Williams & Wilkins; 1999:2134-2154

Copyright 2007 by the American Academy of Pediatrics

page 344

2007 PREP SA on CD-ROM Question: 166

A mother brings her 12-month-old boy to you because he holds his head tilted to the right (Item Q166A). She tells you that he periodically draws up both his legs and cries. The child has developed normally, but does not yet walk or cruise. On physical examination, he has mildly increased deep tendon reflexes in the upper and lower extremities, but other findings are normal. Of the following, the study MOST likely to establish this boy's diagnosis is

A. audiometry B. electroencephalography C. magnetic resonance imaging of the brain and cervical spine D. pH probe of the distal esophagus E. radiographs of the cervical spine

Copyright 2007 by the American Academy of Pediatrics

page 345

2007 PREP SA on CD-ROM Critique: 166 Preferred Response: C

A head tilt commonly is produced by muscular torticollis, but the examiner always should consider a number of other disorders that can produce a twisted neck. Muscular torticollis results from bleeding within the sternocleidomastoid muscle shortly after birth. Swelling can be palpated within the muscle. The head is tipped toward the injured muscle, and the chin is rotated toward the other. In older children, muscular torticollis can result from minor trauma or inflammation of the cervical muscles. However, a high index of suspicion should be maintained for other processes producing head tilt. Neurologic processes causing head tilt are perhaps the most worrisome. Posterior fossa or cervical cord tumor (Item C166A) should be considered, along with syringomyelia, dystonia (from drug or dystonia musculorum deformans), or visual disturbance (trochlear nerve palsy more commonly than abducens or oculomotor nerve paresis). Congenital disorders such as hemivertebrae and Klippel-Feil syndrome can masquerade as muscular torticollis in the newborn. Other causes of head tilt include atlantoaxial rotary subluxation, lymphadenitis, Sandifer syndrome (gastroesophageal reflux), spasmus nutans, and paroxysmal torticollis. Paroxysmal torticollis is a migraine variant with onset in the first year after birth. Infants experience recurrent attacks of head tilt for minutes, sometimes with pallor, vomiting, or agitation. Episodes cease by age 2 to 3 years. No treatment is indicated, although affected children often develop migraines later in childhood. The 12-month-old boy described in the vignette is on the borderline of being delayed in cruising. Although his periodic withdrawal of arms and legs could be interpreted as due to reflux, such behavior also is seen with infantile spasms or cervical spine pain. The increased deep tendon reflexes point to an upper motor neuron process. Magnetic resonance imaging of the brain and cervical spine is warranted, particularly to exclude a tumor; radiographs would visualize only the bony anatomy. A pH probe would be useful only if gastroesophageal reflux were more likely. Audiometry can exclude hearing loss causing a head tilt, and electroencephalography can exclude seizures with head posturing. References: Robin NH. In brief: congenital muscular torticollis. Pediatr Rev. 1996;19:374-375 Thompson GH. The neck: torticollis. In: Behrman RE, Kliegman RM, Jenson HB, eds. Nelson Textbook of Pediatrics. 17th ed. Philadelphia, Pa: WB Saunders Co; 2004:2288-2289

Copyright 2007 by the American Academy of Pediatrics

page 346

2007 PREP SA on CD-ROM Question: 167

You are seeing a 6-week-old infant who was born with trisomy 21 and a large atrioventricular septal defect. Over the previous week, she has tired with feeding and has not gained weight. Her respiratory rate is 60 breaths/min and heart rate is 150 beats/min. Auscultation reveals mild retractions and a 2/6 systolic murmur with a gallop rhythm. The liver is palpable at 2 cm below the costal margin, and the perfusion is good. You decide to increase the caloric content of the formula to 24 kcal/oz, and you contact her pediatric cardiologist to discuss referral for surgical repair. Of the following, the BEST therapeutic option while awaiting surgical repair is

A. captopril B. furosemide C. hydralazine D. propranolol E. verapamil

Copyright 2007 by the American Academy of Pediatrics

page 347

2007 PREP SA on CD-ROM Critique: 167 Preferred Response: B

Congestive heart failure (CHF) is a clinical syndrome that reflects the inability of the heart to meet the metabolic requirements of the body, including those for growth. Congenital heart defects are the most common reason for pediatric heart failure, and the failure develops most frequently during early infancy. CHF results from excessive workload imposed on the cardiac muscle, usually caused by structural defects. The cardiac defects may impose an excessive volume load on the left ventricle (eg, large ventricular septal defect, atrioventricular septal defect), an excessive pressure load on the ventricle (eg, aortic stenosis), or a combination of volume and pressure load (eg, ventricular septal defect and coarctation). Less commonly, CHF may result from an intrinsic alteration in myocardial performance, which could be due to an inflammatory or an infectious process directly affecting the myocardium and depressing its contractile function. The child described in the vignette presents with symptoms that have been developing over weeks and are typical of progressive CHF in a patient who has a large-volume shunting lesion such as an atrioventricular septal defect. Poor feeding results from the infants inability to generate a prolonged suck while maintaining nasal breathing. This is complicated further by tachypnea caused by pulmonary congestion. Decreased caloric intake coupled with increased caloric expenditure due to tachypnea and tachycardia makes it difficult for the infant to gain weight. Indeed, weight loss is common. The murmur in this infant is consistent with mitral regurgitation, which often accompanies a complete atrioventricular septal defect (strong association with a cleft in the mitral valve). This defect leads to left-to-right shunting at the atrial and ventricular levels and is one of the most common forms of congenital heart disease in the child who has Down syndrome. A gallop rhythm, usually an S3, results from a volume-loaded, dilated, stiff left ventricle. Liver enlargement reflects increased filling pressures in the right heart. Treatment of CHF resulting from a left-to-right volume load in the infant is directed at diuresis to address the volume overload on the pulmonary circuit and left heart. Furosemide, a welltolerated and effective loop diuretic, can provide the necessary diuresis. With some unloading of the lungs and volume of the left heart, feeding may improve, and caloric intake may increase. Although medical treatment may provide some immediate relief, patients who have CHF affecting their growth should be considered for surgical repair of the defect. Even when diuresis does not yield a significant improvement in symptoms, it may be useful in improving lung function preoperatively. Afterload reduction with captopril can be effective in the management of CHF, but it does not provide the same level of symptomatic improvement as diuresis. Hydralazine, an antihypertensive and vasodilator, generally is not used for the management of CHF in the infant. It can lead to sinus tachycardia that can exacerbate left heart filling and compound CHF. The use of beta blockers in CHF resulting from myocardial dysfunction may be warranted, but they generally are not indicated for patients who have large-volume shunting lesions. Verapamil, a calcium channel blocker, is contraindicated in infants younger than 1 year of age. References: Balfour I. Management of chronic congestive heart failure in children. Curr Treat Options Cardiovasc Med. 2004;6:407-416. Abstract available at: http://www.ncbi.nlm.nih.gov/entrez/query.fcgi?orig_db=PubMed&db=PubMed&cmd=Search&ter m=%22Current+treatment+options+in+cardiovascular+medicine%22[Jour]+AND+2004[pdat]+AN D+Balfour+I[author] Dreyer WJ, Fisher DJ. Clinical recognition and management of chronic congestive cardiac failure. In: Garson A Jr, Bricker JT, Fisher DJ, Neish SR, eds. The Science and Practice of Pediatric Cardiology. 2nd ed. Baltimore, Md: Williams & Wilkins; 1998:2309-2325 Talner NS, McGovern JJ, Carboni MP. Congestive heart failure. In: Moller JH, Hoffman JIE, eds. Pediatric Cardiovascular Medicine. Philadelphia, Pa: Churchill Livingstone; 2000:817-829

Copyright 2007 by the American Academy of Pediatrics

page 348

2007 PREP SA on CD-ROM Question: 168

Numerous therapeutic agents are known to have teratogenic effects on the developing fetus. Of the following, the findings in the newborn that are MOST suggestive of prenatal exposure to an angiotensin-converting enzyme inhibitor are

A. deafness and cataracts B. microtia and conotruncal malformation C. nasal hypoplasia and stippled epiphyses D. neonatal anuria and patent ductus arteriosus E. smooth philtrum and lip

Copyright 2007 by the American Academy of Pediatrics

page 349

2007 PREP SA on CD-ROM Critique: 168 Preferred Response: D

Angiotensin-converting enzyme (ACE) inhibitors are used in the treatment of hypertension and the management of heart failure. Studies of this class of agents in pregnant animals have revealed an increased incidence of renal defects and fetal death. In humans, the use of ACE inhibitors throughout pregnancy or during the second and third trimesters has been associated with a number of complications, including fetal oligohydramnios and neonatal anuria due to persistent inhibition of the reninangiotensin system as well as fetal hypotension with subsequent poor perfusion of tissues. Patent ductus arteriosus also has been reported and has been hypothesized to result from the effect of these agents on kinase II, which increases production of prostaglandin. Hypoplasia of the skull, spontaneous abortion, and intrauterine demise also have been reported. Thus, ACE inhibitors should not be used during pregnancy. Microtia and conotruncal malformations are associated with prenatal exposure to retinoic acid. Nasal hypoplasia and stippled epiphyses are characteristic of the fetal warfarin syndrome, which results from first-trimester exposure to coumarin derivatives. Deafness and cataracts (Item C168A) are associated with rubella infections during the first trimester of pregnancy. Smooth philtrum and a thin upper lip are part of the facial characteristics of infants who have fetal alcohol syndrome (Item C168B). References: August P, Rose BD. Angiotensin converting enzyme inhibitors and receptor blockers in pregnancy. UpToDate. 2006;14.1. Available at: http://www.utdol.com/utd/content/topic.do?topicKey=hyperten/6229&type=A&selectedTitle=9~21 2 Pietrement C, Malot L, Santerne B, Roussel B, Motte J, Morville P. Neonatal acute renal failure secondary to maternal exposure to telmisartan, angiotensin II receptor antagonist. J Perinatol. 2003;23:254-255. Abstract available at: http://www.ncbi.nlm.nih.gov/entrez/query.fcgi?orig_db=PubMed&db=PubMed&cmd=Search&ter m=%22Journal+of+perinatology+:+official+journal+of+the+California+Perinatal+Association%22[J our]+AND+2003[pdat]+AND+Pietrement+C[author]

Copyright 2007 by the American Academy of Pediatrics

page 350

2007 PREP SA on CD-ROM Question: 169

The director of a community after-school program for adolescent boys is organizing an orientation session for new mentors. He wants some information about current substance abuse trends. Of the following, you are MOST likely to report that among adolescents,

A. athletes who use performance-enhancing substances are unlikely to use other illicit drugs B. daily cigarette smoking by 8th graders has increased steadily in the last decade C. homosexual youth are the least likely to engage in the use of alcohol and marijuana D. inhalant abuse is more prevalent among students in the 8th grade than the 12th grade E. marijuana use among 12th graders has decreased steadily since 1980

Copyright 2007 by the American Academy of Pediatrics

page 351

2007 PREP SA on CD-ROM Critique: 169 Preferred Response: D

The overall use of drugs of abuse (alcohol, tobacco, illicit drugs) among high school students in the United States was at its highest during the late 1970s. Although substance abuse declined during the ensuing decade, the trend was reversed by increases in usage through much of the 1990s. More recently, from 2001 to 2005, there has been a gradual decline in the use of most drugs of abuse. Nevertheless, substance abuse remains a major public health concern. Two periodic surveys track national trends in use of alcohol, tobacco, and other drugs by high school students and have Internet sites that report their data: Monitoring the Future (MTF) and the Youth Risk Behavior Survey. Data are reported as the prevalence of substance use and percentage of sample reporting use or percentage having used during a given period (eg, lifetime [ever used], use in past year [annual], or use in the past 30 days [30 day prevalence]). During the 2005 MTF survey, 50% of 12th graders and more than one in five of 8th graders reported they had tried (lifetime prevalence) an illicit drug. (Item C169A) lists the percentage of 8th and 12th graders who reported use in the past 30 days of alcohol, cigarettes (tobacco), any illicit drug, marijuana, and amphetamines. Inhalant abuse, unlike most substance abuse, is more prevalent among students in the 8th than the 12th grade (Item C169B). Alcohol remained the most widely used drug among high school students. Of the 12th graders surveyed, 47% reported alcohol use in the past 30 days, reflecting a continued slow decline from 54% in 1991. The prevalence of binge drinking (having consumed five or more drinks in a row at least once in the prior 2 weeks), however, has shown little change and was reported by 28% of 12th graders. Cigarette smoking trends for 8th and 12th graders have shown a decline in use during the past decade. Item (C169C) shows the decline in 30-day and daily prevalence reported by 8th graders during the 1996 and 2005 MTF surveys. The annual prevalence of marijuana use among 12th graders declined during the 1980s, but was followed by increases in the 1990s. More recently, there has been a gradual decrease in the annual prevalence, but little change in the 30-day prevalence of marijuana use (Item C169D). Individual risk factors associated with adolescent substance abuse include low self-esteem, poor coping skills, depression, anxiety, alienation from conventional norms, and impulsivity. Homosexual adolescents have rates of substance abuse (including alcohol and marijuana) that are significantly higher than heterosexual peers. The use of performance-enhancing substances increases the likelihood of use of other illicit drugs. References: Johnston LD, OMalley PM, Bachman JG, Schulenberg JE. Monitoring the Future: National Results on Adolescent Drug Use: Overview of Key Findings, 2005. Bethesda, Md: National Institute on Drug Abuse; 2006. Available at: http://www.monitoringthefuture.org/pubs/monographs/overview2005.pdf Kulig JW, American Academy of Pediatrics Committee on Substance Abuse. Tobacco, alcohol, and other drugs: the role of the pediatrician in prevention, identification, and management of substance abuse. Pediatrics. 2005;115:816-821. Available at: http://pediatrics.aappublications.org/cgi/content/full/115/3/816 Monitoring the Future Web Site. Available at: www.monitoringthefuture.org National Survey on Drug Use and Health. Available at: http://www.drugabusestatistics.samhsa.gov/nsduh.htm Youth Risk Behavior SurveillanceUnited States, 2003. MMWR Morbid Mortal Wkly Rep Surv Summ. 2004;53:No. SS-2. Available at: http://www.cdc.gov/mmwr/PDF/SS/SS5302.pdf

Copyright 2007 by the American Academy of Pediatrics

page 352

2007 PREP SA on CD-ROM Question: 170

You are evaluating a 13-year-old boy who has a body mass index of 32 kg/m2 and a blood pressure of 140/80 mm Hg. Initial laboratory studies reveal fasting serum triglyceride levels of 180 mg/dL (2.0 mmol/L) (normal, 40 to 100 mg/dL [0.45 to 1.1 mmol/L]). You diagnose metabolic syndrome. Of the following, the MOST appropriate evaluation in addition to fasting blood glucose is

A. erythrocyte sedimentation rate B. 5-hour oral glucose tolerance test C. Holter monitoring D. liver transaminases E. serum alpha-fetoprotein

Copyright 2007 by the American Academy of Pediatrics

page 353

2007 PREP SA on CD-ROM Critique: 170 Preferred Response: D

The concept of metabolic syndrome as a unifying term for people who have obesity, hypertension, insulin resistance, and dyslipidemia composed of low high-density lipoprotein and elevated triglyceride concentrations is presently being questioned. Nonetheless, 20% to 40% of individuals who are obese and exhibit insulin resistance have nonalcoholic fatty liver disease (nonalcoholic steatohepatitis). Elevated transaminases associated with a mild hepatic inflammatory process in such patients can progress over time to frank cirrhosis. The best treatment for children who have this disorder is weight loss, but trials are underway to test the effect of drugs that decrease insulin resistance, such as metformin and thiazolidinediones such as rosiglitazone or pioglitazone. The latter two drugs are not approved by the United States Food and Drug Administration for children. The components of metabolic syndrome are common in obese children, but the risk for type 2 diabetes is quite low. A 5-hour glucose tolerance test never is indicated for the diagnosis of diabetes; the glucose response 2 hours after oral glucose administration is sufficient to make the diagnosis. This test is indicated if the fasting blood glucose is between 100 mg/dL and 126 mg/dL (5.5 and 7 mmol/L). A fasting plasma glucose of 126 mg/dL (7 mmol/L) or greater is diagnostic of diabetes using American Diabetes Association criteria. Holter monitoring, nonspecific measures of inflammation such as erythrocyte sedimentation rate and measurement of serum alpha-fetoprotein, a tumor marker, are not indicated. References: Cruz ML, Shaibi GQ, Weigensberg MJ, Spruijt-Metz D, Ball GD, Goran MI. Pediatric obesity and insulin resistance: chronic disease risk and implications for treatment and prevention beyond body weight modification. Annu Rev Nutr. 2005;25:435-468. Available at: http://www.ncbi.nlm.nih.gov/entrez/query.fcgi?orig_db=PubMed&db=PubMed&cmd=Search&ter m=%22Annual+review+of+nutrition%22[Jour]+AND+435[page]+AND+2005[pdat] Mandato S, Lucariello M, Licenziati MR, et al. Metabolic, hormonal, oxidative, and inflammatory factors in pediatric obesity-related liver disease. J Pediatr. 2005;147:62-66. Abstract available at: http://www.ncbi.nlm.nih.gov/entrez/query.fcgi?cmd=Retrieve&db=pubmed&dopt=Abstract&list_ui ds=16027697&query_hl=75&itool=pubmed_docsum Schwimmer JB, McGreal N, Deutsch R, Finegold MJ, Lavine JE. Influence of gender, race, and ethnicity on suspected fatty liver in obese adolescents. Pediatrics. 2005;115:e561-e565. Available at: http://pediatrics.aappublications.org/cgi/content/full/115/5/e561

Copyright 2007 by the American Academy of Pediatrics

page 354

2007 PREP SA on CD-ROM Question: 171

You are evaluating a 6-year-old boy who has Duchenne muscular dystrophy. He is doing well in a regular classroom and will be attending second grade in a different school next year. On physical examination, you note a healthy-appearing boy who has pseudohypertrophy of the calf muscles and uses a Gower maneuver (Item Q171A) to rise from the floor. In gathering information to help this child's transition to a new school, you are MOST likely to ask about

A. augmented communication resources B. recent pulmonary function testing C. signs of sleep apnea D. the number of floors in the school E. wheelchair use

Copyright 2007 by the American Academy of Pediatrics

page 355

2007 PREP SA on CD-ROM Critique: 171 Preferred Response: D

The time of diagnosis and medical, social, and educational transitions are common stress points for parents in the life of their child who has a developmental disability. The need for increasing medical equipment and assistance, such as braces, a wheelchair, or a gastrostomy tube, may cause significant stress. Similarly, entering kindergarten, middle school, junior high school, and high school each present challenges. A child likely needs re-evaluation of school services provided at those times and may require significant testing or accommodations for differing physical facilities. For the ambulatory boy in the vignette, the most likely difficulty he will encounter is numerous stairs in a multilevel school that may be tiring for him to climb and descend. He is healthy and is unlikely to have problems with sleep apnea or need pulmonary function tests until his disease has progressed further. Likewise, at his present level of function, he does not need a wheelchair or communication device. However, most children who have Duchenne muscular dystrophy need educational support for related learning disabilities that may manifest over time. References: Perrin JM. Chronic illness in childhood. In: Behrman RE, Kliegman RM, Jenson HB, eds. Nelson Textbook of Pediatrics. 17th ed. Philadelphia, Pa: WB Saunders Co; 2004:135-138 Plauch Johnson C, Kastner TA, and the Committee/Section on Children With Disabilities. Helping families raise children with special health care needs at home. Pediatrics. 2005;115:507511. Available at: http://pediatrics.aappublications.org/cgi/content/full/115/2/507

Copyright 2007 by the American Academy of Pediatrics

page 356

2007 PREP SA on CD-ROM Question: 172

An 11-day-old infant presents to the clinic with a history of a temperature of 100.8F (38.2C) and a 1-day history of poor feeding. Findings on physical examination are normal. You initiate a sepsis evaluation that includes a lumbar puncture. The cerebrospinal fluid results demonstrate a white blood cell count of 6x103/mcL (6x109/L), with 68% neutrophils, 2% bands, and 30% lymphocytes. The protein concentration is 200 mg/dL (2 g/L), and the glucose value is 36 mg/dL (2.0 mmol/L). The abnormal findings prompt you to order magnetic resonance imaging, which demonstrates abnormal frontal lobes bilaterally that includes some degree of infarction but also abscesses and cerebritis (Item Q172A). Of the following, the MOST likely pathogen is

A. Citrobacter koseri B. Escherichia coli C. Klebsiella pneumoniae D. Listeria monocytogenes E. Streptococcus agalactiae

Copyright 2007 by the American Academy of Pediatrics

page 357

2007 PREP SA on CD-ROM Critique: 172 Preferred Response: A

Organisms from the genus Citrobacter cause a chronic form of bacterial meningitis in the neonate that leads to brain abscess formation in approximately 80% of infected infants. The abscesses tend to be large multiloculated areas that can be difficult to treat. The exact mechanism for the invasiveness of Citrobacter and its propensity to cause brain abscesses is unknown. Citrobacter are gram-negative enteric rods closely related to Salmonella that include at least 11 species, the most common of which are C freundii and C koseri. The organisms can be passed vertically from mother to infant at delivery, but colonization from nosocomial sources (eg, hand contamination, contaminated infant formula) has been described. Most organisms from the genus Citrobacter are resistant to ampicillin but susceptible to the later-generation cephalosporins and aminoglycosides. Treatment of brain abscesses (Item C172A) usually involves long-term antimicrobial therapy (4 to 6 weeks after sterilization of the cerebrospinal fluid) with a combined regimen that includes a third- or fourth-generation cephalosporin and an aminoglycoside. The abscess area frequently must be drained, and follow-up cranial imaging is used to help assess the duration of therapy. Patients who have multiloculated abscess usually have significant developmental problems. Although Escherichia coli, Klebsiella pneumoniae, Listeria monocytogenes, or Streptococcus agalactiae (group B Streptococcus) can cause a brain abscess, Citrobacter is most likely to be responsible for the indolent course described for the neonate in the vignette that includes brain abscess formation. Similar to Citrobacter, Enterobacter sakazakii causes neonatal meningitis with brain abscess formation, but it is encountered less frequently than Citrobacter. References: Boyce TG, Gruber WC, Fisher RG. Citrobacter. In: Feigin RD, Cherry JD, Demmler GJ, Kaplan SL, eds. Textbook of Pediatric Infectious Diseases. 5th ed. Philadelphia, Pa: WB Saunders Co; 2004:1423-1426 Goodkin HP, Harper MB, Pomeroy SL. Intracerebral abscess in children: historical trends at Childrens Hospital Boston. Pediatrics. 2004;113:1765-1770. Available at: http://pediatrics.aappublications.org/cgi/content/full/113/6/1765

Copyright 2007 by the American Academy of Pediatrics

page 358

2007 PREP SA on CD-ROM Question: 173

You are evaluating a 10-year-old boy who has intermittent urinary incontinence. Voiding cystourethrography detects a urethral stricture. Of the following, the MOST likely cause of this boy's urethral stricture is

A. carcinoma B. chronic infection C. congenital narrowing D. intermittent urolithiasis E. trauma

Copyright 2007 by the American Academy of Pediatrics

page 359

2007 PREP SA on CD-ROM Critique: 173 Preferred Response: E

Urethral stricture disease involves blockage or narrowing of the urethra that causes reduced urine flow during voiding. Most urethral strictures occur in males. The most common causes in children are previous trauma (eg, posterior urethral injury occurring as a consequence of pelvic fracture) or instrumentation of the urethra, infection (eg, gonorrhea), congenital anomaly, idiopathic, and as a complication of balanitis (balanitis xerotica obliterans). Children who have urethral strictures can develop a variety of signs and symptoms, including straining to urinate, a decrease in the size and force of the urine stream, persistent sense of bladder fullness, urine dribbling, and frequency and urgency of urination. Overt physical examination findings are generally absent. Diagnosis is confirmed by cystoscopy. Alternatively, voiding cystourethrography can be performed to determine the extent of the stricture or ultrasonography may be performed. Treatment of a urethral stricture depends on the length, location, and persistence of the stricture. In general, initial urethral strictures that are short (<1.5 cm) can be treated with either endoscopic incision or dilation. Although this form of treatment has only a modest success rate, it is minimally invasive and associated with few procedure-related complications. Recurrent strictures or strictures longer than 2 cm usually are not amenable to endoscopic incision or dilation, and this form of therapy proves to be a "temporizing" measure without much long-term success. Severe urethral stricture in males may damage the bladder and cause hydronephrosis. Carcinoma of the urethra is rare in children, and the boy described in the vignette has no history of chronic infection. Moreover, chronic gonorrhea is uncommon at this age. The boys age effectively eliminates congenital narrowing as a cause; symptoms usually develop in early childhood. Chronic intermittent urolithiasis is a very rare cause of urethral stricture, especially in patients who have no history of chronic abdominal or flank pain. References: Farhat W, McLorie G. Urethral syndromes in children. Pediatr Rev. 2001;22:17-21. Available at: http://pedsinreview.aappublications.org/cgi/content/full/22/1/17 Koraitim MM. Post-traumatic posterior urethral strictures: preoperative decision making. Urology. 2004;64:228-231. Abstract available at: http://www.ncbi.nlm.nih.gov/entrez/query.fcgi?orig_db=PubMed&db=PubMed&cmd=Search&ter m=%22Urology%22[Jour]+AND+228[page]+AND+2004[pdat] Sugimoto M, Kakehi Y, Yamashita M, Matsuki T, Inui M, Taketa S. Ten cases of congenital urethral stricture in childhood with enuresis. Int J Urol. 2005;12:558-562. Abstract available at: http://www.ncbi.nlm.nih.gov/entrez/query.fcgi?orig_db=PubMed&db=PubMed&cmd=Search&ter m=%22International+journal+of+urology+:+official+journal+of+the+Japanese+Urological+Associat ion%22[Jour]+AND+558[page]+AND+2005[pdat]

Copyright 2007 by the American Academy of Pediatrics

page 360

2007 PREP SA on CD-ROM Question: 174

A 3-year-old boy who has myelomeningocele and a history of recurrent urinary tract infections presents with a 1-day history of a temperature to 102F (38.9C) and cloudy urine. Laboratory test results include a peripheral white blood cell count of 15x103/mcL (15x109/L), with 60% neutrophils, 30% lymphocytes, and 10% monocytes. Urine obtained by catheterization is cloudy; has a strong odor; and is positive for nitrites, leukocyte esterase, and blood. Microscopic analysis shows too numerous-to-count white blood cells and 50 to 100 red blood cells, and gramnegative bacilli are seen on Gram stain. One day later, the urine culture is positive for Pseudomonas aeruginosa. Of the following, the MOST appropriate antibiotic for treatment of this patient is

A. ampicillin B. ceftazidime C. cefuroxime D. trimethoprim-sulfamethoxazole E. vancomycin

Copyright 2007 by the American Academy of Pediatrics

page 361

2007 PREP SA on CD-ROM Critique: 174 Preferred Response: B

Pseudomonas sp are gram-negative bacilli that are primarily waterborne and soilborne organisms, have relatively low virulence, and are associated with a wide variety of infections. Of the pseudomonal species, P aeruginosa is the most common. It is classified as an opportunistic pathogen infrequently causing disease in healthy hosts, but it is a major cause of infection in patients who have underlying conditions. Osteomyelitis or cellulitis associated with a puncture wound and urinary tract infections are the most common diseases caused by this organism in the otherwise healthy host. Both of these conditions can be treated with a single antibiotic agent such as a fluoroquinolone (ciprofloxacin) for oral therapy or piperacillin, piperacillin/tazobactam, ceftazidime, a fluoroquinolone, or an aminoglycoside parenterally. Combination therapy of piperacillin or ceftazidime plus an aminoglycoside is also appropriate. Serious P aeruginosa infections are more likely in patients who have underlying conditions, such as bacteremia, sepsis, and pneumonia (especially in patients who have cystic fibrosis). For these patients, therapy usually consists of two anti-Pseudomonas agents, such as a betalactam antibiotic (piperacillin, piperacillin/tazobactam, or ceftazidime) plus an aminoglycoside. Imipenem and ciprofloxacin also may be used successfully as single agents. The treatment of pneumonia usually includes the beta-lactam antibiotics ticarcillin, ticarcillin/clavulanic acid, piperacillin, piperacillin/tazobactam, or ceftazidime, usually in combination with an aminoglycoside. Combination therapy often is used to ensure treatment of resistant strains and to prevent selection of resistant mutants. The carbapenem class of antibiotics (imipenem or meropenem) and the monobactam antibiotic aztreonam generally are reserved for the treatment of serious infections caused by organisms resistant to the other betalactam antibiotics or for patients who have moderate renal disease and are at risk for aminoglycoside-related nephrotoxicity. Because pseudomonal species are found widely in the environment, it is important to distinguish between true infection and colonization of a surface because inappropriate use of antibiotics does not treat colonization and predisposes the patient to developing a resistant organism. In both situations, cultures are positive, but it is important to look at other factors related to the patients clinical condition. A positive culture is much more likely to represent colonization if a patients clinical condition is unchanged or improving and there is no evidence of infection on physical examination or laboratory testing. Two examples of probable colonization are: 1) a positive endotracheal tube culture in patient who is improving on a ventilator with fewer than 25 white blood cells on Gram stain and no change in amount or color of secretions, and 2) a patient who has a positive urine culture but no change in urine color or odor, urinalysis negative for nitrites and leukocyte esterase, and no findings on physical examination. The patient in the vignette has an underlying condition that predisposes him to urinary tract infections. Physical examination and laboratory findings indicate that he has a urinary tract infection caused by P aeruginosa. Ceftazidime is the only agent listed that has activity against this organism. References: Berthelot P, Grattard F, Mahul P, et al. Prospective study of nosocomial colonization and infection due to Pseudomonas aeruginosa in mechanically ventilated patients. Intensive Care Med. 2001;27:503-512. Abstract available at: http://www.ncbi.nlm.nih.gov/entrez/query.fcgi?orig_db=PubMed&db=PubMed&cmd=Search&ter m=%22Intensive+care+medicine%22[Jour]+AND+503[page]+AND+2001[pdat] Giamarellou H, Antoniadou A. Antipseudomonal antibiotics. Med Clin North Am. 2001;85:19-42. Abstract available at: http://www.ncbi.nlm.nih.gov/entrez/query.fcgi?orig_db=PubMed&db=PubMed&cmd=Search&ter m=%22The+Medical+clinics+of+North+America%22[Jour]+AND+19[page]+AND+2001[pdat] Prince AS. Pseudomonas aeruginosa. In: Long SS, Pickering LK, Prober CG, eds. Principles and Practice of Pediatric Infectious Diseases. 2nd ed. New York, NY: Churchill Livingstone;

Copyright 2007 by the American Academy of Pediatrics

page 362

2007 PREP SA on CD-ROM

2003:857-860

Copyright 2007 by the American Academy of Pediatrics

page 363

2007 PREP SA on CD-ROM Question: 175

A 7-year-old boy presents with a 3-year history of seasonal rhinorrhea and nasal congestion. He states that "as soon as the weather warms up," he experiences daily nasal symptoms that are often severe enough to limit his outdoor activity. On physical examination, you note infraorbital swelling, a transverse nasal crease, boggy turbinates, and clear rhinorrhea. A nasal smear demonstrates numerous eosinophils. You decide to treat him with a nasal spray. Of the following, the MOST effective long-term treatment for this boy is a nasal spray containing a(n)

A. anticholinergic B. corticosteroid C. decongestant D. mast cell stabilizer E. saline solution

Copyright 2007 by the American Academy of Pediatrics

page 364

2007 PREP SA on CD-ROM Critique: 175 Preferred Response: B

The boy described in the vignette presents with classic symptoms of allergic rhinitis. The approach to medical management of allergic rhinitis should take into account the patients primary symptoms, past experiences with medications, and any coexisting medical problems. Nasal corticosteroids have become a first-line therapy for allergic rhinitis, with studies demonstrating improvement in nasal pruritus, sneezing, nasal obstruction, and rhinorrhea. Further, nasal steroids typically are well tolerated and have minimal adverse effects. Anticholinergic nasal sprays are approved for nonallergic rhinitis and are suited best for treatment of vasomotor rhinitis and atrophic rhinitis, conditions more commonly seen in adults. Nasal decongestants often are purchased without a prescription and markedly reduce nasal congestion. However, recurrent or prolonged use of nasal decongestants can result in rhinitis medicamentosa, a condition of rebound nasal congestion. Nasal cromolyn is approved for allergic rhinitis, although it must be taken several times a day and has been shown to be inferior to nasal steroids for treatment of allergic rhinitis. Nasal saline should be considered a routine adjunct for nasal irrigation, but it is generally considered inferior to nasal corticosteroids for the treatment of allergic rhinitis. The use of other medications to treat allergic rhinitis is based on patient symptoms. Sneezing, nasal pruritus, and ocular itching often can be managed with an oral antihistamine. An ophthalmic antihistamine or vasoconstricting eye drop can be added when ocular symptoms do not improve with oral antihistamines alone. Nasal antihistamines also have been approved for allergic rhinitis. References: Atkins D, Leung DYM. Principles of treatment of allergic disease. In: Behrman RE, Kliegman RM, Jenson HB, eds. Nelson Textbook of Pediatrics. 17th ed. Philadelphia, Pa: WB Saunders Co 2004:752-758 Mahr TA, Sheth K. Update on allergic rhinitis. Pediatr Rev. 2005;26:284-289. Available at: http://pedsinreview.aappublications.org/cgi/content/full/26/8/284

Copyright 2007 by the American Academy of Pediatrics

page 365

2007 PREP SA on CD-ROM Question: 176

An 11-year-old highly competitive gymnast presents to your office with complaints of increasing right wrist pain over the past 2 to 3 months. She recalls no specific trauma to the wrist or arm. On physical examination, you note no swelling of the distal forearm or wrist. She has normal range of motion at the wrist. There is moderate-to-marked tenderness of the distal radius, but no tenderness more distally over the wrist joint. A radiograph of the forearm shows slight widening of the distal radial physis. Of the following, you are MOST likely to explain to the child and family that

A. minor pains such as this are common in athletes and should not cause alarm B. she must stop her training immediately C. there is no evidence for trauma to her bones D. this is a common injury in young gymnasts that can be overcome with an altered training
regimen

E. this most likely represents a wrist sprain

Copyright 2007 by the American Academy of Pediatrics

page 366

2007 PREP SA on CD-ROM Critique: 176 Preferred Response: D

The combination of history, physical examination, and radiologic findings reported for the girl in the vignette are most consistent with stress injury to the distal radial physis. This overuse injury may lead to subsequent derangement in radial-ulnar growth. Until the injury has healed fully, the girl should alter her training to remove stress on her distal radius by splinting her wrist, but continued aerobic training such as running still can be permitted. Chronic wrist pain has been reported in up to 79% of young gymnasts. In one recent report, more than 50% of the gymnasts studied had evidence of stress injury to the distal radial physis, but only 7% had frank widening of the growth plate on radiography. The consequence of this injury, if left untreated, is distal radius physeal arrest with secondary ulna-radial length difference. The distal radial physis is located just proximal to the wrist within the distal radius. The presence of point tenderness over the distal radius and the lack of tenderness over the wrist are not consistent with a sprained wrist. The abnormal radiograph showing widening of the physis confirms the diagnosis. References: American Academy of Pediatrics Committee on Sports Medicine and Fitness. Intensive training and sports specialization in young athletes. Pediatrics. 2000;106:154-157. Available at: http://pediatrics.aappublications.org/cgi/content/full/106/1/154 Caine D, Howe W, Ross W, Bergman G. Does repetitive physical loading inhibit radial growth in female gymnasts? Clin J Sport Med. 1997;7:302-308. Abstract available at: http://www.ncbi.nlm.nih.gov/entrez/query.fcgi?orig_db=PubMed&db=PubMed&cmd=Search&ter m=7[volume]+AND+302[page]+AND+1997[pdat]+AND+Caine+D[author] DiFiori JP, Puffer JC, Aish B, Dorey F. Wrist pain, distal radial physeal injury, and ulnar variance in young gymnasts: does a relationship exist? Am J Sports Med. 2002;30:879-885. Abstract available at: http://www.ncbi.nlm.nih.gov/entrez/query.fcgi?orig_db=PubMed&db=PubMed&cmd=Search&ter m=Am+J+Sports+Med[Jour]+AND+879[page]+AND+2002[pdat] Kolt GS, Kirkby RJ. Epidemiology of injury in elite and subelite female gymnasts: a comparison of retrospective and prospective findings. Br J Sports Med. 1999;33:312-318. Abstract available at: http://www.ncbi.nlm.nih.gov/entrez/query.fcgi?orig_db=PubMed&db=PubMed&cmd=Search&ter m=Br+J+Sports+Med[Jour]+AND+312[page]+AND+1999[pdat]

Copyright 2007 by the American Academy of Pediatrics

page 367

2007 PREP SA on CD-ROM Question: 177

You diagnose tinea capitis in a 7-year-old girl. She is otherwise healthy and has no known allergies to medications. You plan to prescribe oral griseofulvin. Of the following, it is MOST appropriate to prescribe the drug

A. after obtaining a baseline complete blood count B. after obtaining baseline liver function tests C. with clinical follow-up only at 4 to 6 weeks D. with serial complete blood counts during therapy E. with serial liver function tests during therapy

Copyright 2007 by the American Academy of Pediatrics

page 368

2007 PREP SA on CD-ROM Critique: 177 Preferred Response: C

Griseofulvin remains the preferred treatment for tinea capitis. The desired dose is 20 mg/kg per day for at least 6 weeks. Although many rare severe adverse effects, including agranulocytosis and aplastic anemia, have been reported, griseofulvin remains a safe drug for use in children, and laboratory monitoring at baseline or during therapy is not required in an otherwise healthy child. Clinical follow-up at 4 to 6 weeks is adequate surveillance for the adverse reactions. Newer antifungal agents have been introduced, but none have the longstanding safety profile of griseofulvin. References: Bennett ML, Fleischer AB, Loveless JW, Feldman SR. Oral griseofulvin remains the treatment of choice for tinea capitis in children. Pediatr Dermatol. 2000:17:304-309. Available at: http://www.blackwell-synergy.com/doi/abs/10.1046/j.15251470.2000.01784.x?prevSearch=allfield%3A%28%22Oral+griseofulvin+remains+the+treatment+ of+choice+for+tinea+capitis+in+children%22%29 Friedlander S. Fungal infections: superficial fungal infections. In: Schachner LA, Hansen RC, eds. Pediatric Dermatology. 3rd ed. St. Louis, Mo: Mosby; 2003:1093-1106 Weston WL, Lane AT, Morelli JG. Fungal and yeast infections of the skin: hair infections (tinea capitis). In: Color Textbook of Pediatric Dermatology. 3rd ed. St. Louis, Mo: Mosby; 2002:63-67

Copyright 2007 by the American Academy of Pediatrics

page 369

2007 PREP SA on CD-ROM Question: 178

A 16-year-old boy comes to your office with a 6-month history of abdominal cramping. He states that the cramps immediately precede a bowel movement and that passage of stool results in pain relief. There is no clear association with any type of food group. The patient's bowel movements are variable, ranging from hard stools every other day to loose stools several times a day. Weight and height are normal, as are physical examination findings and measurements of stool guaiac, complete blood count, erythrocyte sedimentation rate, albumin, aspartate aminotransferase, alanine aminotransferase, immunoglobulin A, and tissue transglutaminase. Stool studies are negative for Giardia sp, Clostridium difficile, and enteric pathogens. Of the following, the MOST appropriate next step is

A. colonoscopy and biopsy B. fiber supplementation C. observation D. oral tegaserod E. referral to a psychiatrist

Copyright 2007 by the American Academy of Pediatrics

page 370

2007 PREP SA on CD-ROM Critique: 178 Preferred Response: B

Irritable bowel syndrome (IBS) is a functional gastrointestinal disorder that typically occurs in teenagers and young adults. According to the Rome 2 criteria, IBS is defined as abdominal discomfort or pain that is relieved with defecation and is associated with either a change in frequency or consistency of stool. Typically, as described for the boy in the vignette, the affected patient has a sudden abdominal cramp, develops an urge to "run to the bathroom," and passes a mushy stool containing mucus. Although diarrhea is common, rectal bleeding does not occur. Infectious and inflammatory causes of diarrhea should be excluded by physical examination (including stool for occult blood test) and by stool studies and laboratory testing (including complete blood count, erythrocyte sedimentation rate, and measurement of immunoglobulin A and tissue transglutaminase antibody). The precise causes of IBS are poorly understood, but heredity, altered colonic motility, alterations in colonic microflora, and excessive gas production have been postulated to play a role. Psychological factors may also worsen symptoms and predispose to health-seeking behavior, but stress is not believed to be the principal cause of the symptoms in most patients. IBS has a different symptom complex from functional abdominal pain in the school-age child, which is typically periumbilical and not associated with disordered defecation. However, approximately 20% to 25% of children who have functional abdominal pain develop irritable bowel symptoms within 5 years after their initial evaluation. Because of a paucity of randomized trials, treatment of IBS is highly individualized and patient-specific. Treatment often begins with a trial of a fiber supplement, which has been reported to help up to 50% of patients. Other treatments include antispasmodics (eg, hyoscyamine), antidiarrheals (loperamide), cholestyramine, and probiotics. The 5HT4 agonist tegaserod has been shown to be effective in adults who have constipation-predominant IBS, but is not first-line treatment and is not indicated in patients who have diarrhea. Psychotherapy, cognitive behavioral therapy, hypnosis, and other complementary therapies may be of benefit, but generally are not first-line therapy. Because IBS can be debilitating and is treated easily, observation alone generally is not recommended. Patients who have persistent or refractory symptoms may need to undergo flexible sigmoidoscopy or colonoscopy at some point to rule out microscopic colonic inflammation (microscopic colitis). Celiac disease should be excluded by serology or endoscopy. However, for a patient who has classic IBS and no "red flags," endoscopic evaluation can be postponed until after a trial of therapy. References: Hadley SK, Gaarder SM. Treatment of irritable bowel syndrome. Am Fam Physician. 2005;72:2501-2506. Available at: http://www.aafp.org/afp/20051215/2501.html Talley NJ, Spiller R. Irritable bowel syndrome: a little understood organic bowel disease? Lancet. 2002;360:555-564. Available at: http://www.sciencedirect.com/science?_ob=ArticleURL&_aset=V-WA-A-W-A-MsSAYZW-UUA-UAACCBABYDE-AACWYEVZDE-EEZVDZDUV-A-U&_rdoc=1&_fmt=summary&_udi=B6T1B46HNW1W10&_coverDate=08%2F17%2F2002&_cdi=4886&_orig=search&_st=13&_sort=d&view=c&_acct =C000052235&_version=1&_urlVersion=0&_userid=1300165&md5=1d7e48bc1cdde305fff9bbbe 5787207d Walker LS, Guite JW, Duke M, Barnard JA, Greene JW. Recurrent abdominal pain: a potential precursor of irritable bowel syndrome in adolescents and young adults. J Pediatr. 1998;132:1010-1015. Abstract available at: http://www.ncbi.nlm.nih.gov/entrez/query.fcgi?cmd=Retrieve&db=pubmed&dopt=Abstract&list_ui ds=9627595&query_hl=6&itool=pubmed_docsum

Copyright 2007 by the American Academy of Pediatrics

page 371

2007 PREP SA on CD-ROM Question: 179

An infant is born following a pregnancy complicated by no prenatal care and reduced fundal height for gestation on examination during labor. Fetal heart rate tracings are nonreassuring. Physical examination of the infant reveals a birthweight of 1,800 g, flattened facies (Item Q179A), low-set ears, respiratory distress, a large flank mass on the left, and joint contractures. Renal ultrasonography documents a single left multicystic and dysplastic kidney; the right kidney is absent. Of the following, the BEST explanation for these findings is

A. Alport disease B. congenital nephrotic syndrome C. congenital Wilms tumor D. oligohydramnios sequence E. Turner syndrome

Copyright 2007 by the American Academy of Pediatrics

page 372

2007 PREP SA on CD-ROM Critique: 179 Preferred Response: D

The infant described in the vignette is growth-restricted, has a flank mass consistent with an enlarged kidney, and has the characteristic phenotype of the fetal oligohydramnios sequence. Bilateral renal aplasia (true Potter syndrome), which occurs in 1 per 3,000 births, or severe dysplasia, such as that seen in multicystic dysplastic or hereditary polycystic kidney disease and resulting in oligohydramnios (Potter sequence), both jeopardize fetal and neonatal wellbeing. Renal aplasia/dysplasia in the fetus results in oligohydramnios that alters amniotic fluid dynamics and interrupts normal development of the fetal lung, especially during the canalicular stage of lung development from 16 to 24 weeks gestation. Pulmonary hypoplasia results and may prove fatal in the neonatal period despite efforts at assisted ventilation. An additional complication of this aberrant renal development is reduced intrauterine volume, resulting in fetal constraint. In such instances, the growth restriction is generalized, the facies is flattened (Item C179A), the ears appear low-set, the arms and legs may be malpositioned with clubbing of the feet (Item C179B) and joint contractures, and breech presentation is not uncommon. Alport disease is an inherited condition (X-linked in 85% and autosomal recessive in 15%) that involves the kidneys, cochlea, and eyes, leading to sensorineural deafness and nephritis. The defect affects type IV collagen, and clinical stigmata do not present in the newborn period. Congenital nephrotic syndrome is characterized by edema, ascites or hydrops, and associated proteinuria, hyperlipidemia, and hypoalbuminemia. It is most common among families that have Finnish ancestry. Wilms tumor, also known as a mesoblastic nephroma, may be congenital and may be associated with aniridia, hemihypertrophy, or Beckwith-Wiedemann syndrome. Findings on the physical examination include hypertension, hematuria, and a flank mass. Turner syndrome is due to an absence of part or all of one of the X chromosomes. Renal anomalies seen in this condition include renal aplasia or hypoplasia, rotated or horseshoe kidneys, or renal duplication. The clinical phenotype includes growth restriction, lymphedema of the dorsal aspects of the hands and feet, cystic hygroma, and cardiovascular defects such as coarctation of the aorta and other left-sided cardiac outflow tract lesions. References: Davis ID, Avner ED. Isolated glomerular diseases with recurrent gross hematuria: Alport syndrome. In: Behrman RE, Kliegman RM, Henson HB, eds. Nelson Textbook of Pediatrics. 17th ed. Philadelphia, Pa: Saunders; 2004:1738-1739 Fras JL, Davenport ML, Committee on Genetics and Section on Endocrinology. Health supervision for children with Turner syndrome. Pediatrics. 2003;111:692-702. Available at: http://pediatrics.aappublications.org/cgi/content/full/111/3/692 Jaffe N, Huff V. Neoplasms of the kidney. In: Behrman RE, Kliegman RM, Henson HB, eds. Nelson Textbook of Pediatrics. 17th ed. Philadelphia, Pa: Saunders; 2004:1711-1714 Jones KL. Oligohydramnios sequence. In: Smiths Recognizable Patterns of Human Malformation. 6th ed. Philadelphia, Pa: Elsevier Saunders; 2006:726-727 Judge NE, Morrison SC. Perinatal ultrasound. In: Martin RJ, Fanaroff AA, Walsh MC, eds. Fanaroff and Martins Neonatal-Perinatal Medicine: Diseases of the Fetus and Infant. 8th ed. Philadelphia, Pa: Mosby-Elsevier; 2006:141-166 Rapaport R. Hypofunction of the ovaries: hypergonadotropic hypogonadism in the female (primary hypogonadism). In: Behrman RE, Kliegman RM, Henson HB, eds. Nelson Textbook of Pediatrics. 17th ed. Philadelphia, Pa: Saunders; 2004:1931-1934 Vogt BA, Avner ED. Nephrotic syndrome: congenital nephrotic syndrome. In: Behrman RE, Kliegman RM, Henson HB, eds. Nelson Textbook of Pediatrics. 17th ed. Philadelphia, Pa:

Copyright 2007 by the American Academy of Pediatrics

page 373

2007 PREP SA on CD-ROM

Saunders; 2004:1757

Copyright 2007 by the American Academy of Pediatrics

page 374

2007 PREP SA on CD-ROM Question: 180

A 16-month-old girl presents with the acute onset of difficulty breathing after attending a birthday party. She is in moderate distress, with a respiratory rate of 56 breaths/min. She has slightly diminished breath sounds on the right. You suspect foreign body aspiration. A plain anteroposterior chest radiograph appears normal. Of the following, the MOST appropriate radiographic study to confirm your suspicion of foreign body aspiration in this girl is

A. airway fluoroscopy B. barium swallow C. inspiratory and expiratory chest radiographs D. magnetic resonance imaging of the chest E. ventilation perfusion scan

Copyright 2007 by the American Academy of Pediatrics

page 375

2007 PREP SA on CD-ROM Critique: 180 Preferred Response: A

Young children are at high risk for foreign body aspiration, which causes a significant number of pediatric deaths each year. A history of choking in these cases is not always available, so it is important for the clinician to maintain a high level of suspicion for aspiration in young children who have acute respiratory distress. Physical findings associated with foreign body aspiration include asymmetric breath sounds and wheezing, cough, tachypnea, and retractions, but some affected children have normal and equal breath sounds bilaterally. Plain radiography of the chest should be the initial diagnostic test, keeping in mind that many foreign bodies are food particles and, therefore, radiolucent. The chest radiograph findings may be normal, as described for the child in the vignette. Abnormal findings may include atelectasis, consolidation, mediastinal shift, or air trapping. If clinical suspicion for foreign body aspiration is high, further radiologic testing is indicated, even with normal chest film results. Airway fluoroscopy is an appropriate next test for the child in the vignette. Fluoroscopy enables the clinician to evaluate diaphragm and mediastinum movement and is a noninvasive and safe procedure. High-resolution computed tomography scan is another tool that may aid in the diagnosis of foreign body aspiration, but may require sedation. This may be difficult in a patient who has acute respiratory distress. Older children may cooperate for inspiratory and expiratory radiographs (Item C180A), which may accentuate an area of air trapping, but these are impractical for young children. Lateral decubitus films may be useful in young patients. Barium swallow results may be abnormal following ingestion of a foreign body, but they will not confirm or rule out a diagnosis of aspiration. Magnetic resonance imaging and ventilation perfusion scans are not useful in the diagnosis of foreign body aspiration. References: Long FR. Imaging evolution of airway disorders in children. Radiol Clin North Am. 2005;43:371389. Available at: http://www.radiologic.theclinics.com/article/PIIS0033838904002209/abstract Rovin JD, Rodgers BM. Pediatric foreign body aspiration. Pediatr Rev. 2000;21:86-90. Available at: http://pedsinreview.aappublications.org/cgi/content/full/21/3/86

Copyright 2007 by the American Academy of Pediatrics

page 376

2007 PREP SA on CD-ROM Question: 181

A 13-month-old boy presents to your office after his mother called for an urgent appointment because he had a bad cough and noisy breathing. He has been previously healthy and is fully immunized. On physical examination, his temperature is 100.9F (38.3C), pulse rate is 142 beats/min, respiratory rate is 36 breaths/min (crying), and pulse oximetry reading is 98% on room air. The mother states that he went to sleep normally with only mild symptoms of an upper respiratory tract infection and awakened at 4 am with noisy breathing. He tolerated sips of juice this morning and has had no vomiting, diarrhea, or high fever. After the boy settles down from having his vital signs measured, he has a "barking cough," and on auscultation, you notice stridor with every breath. Mild suprasternal retractions are visible on examination of the chest. He prefers to sit up and looks slightly anxious. Of the following, the treatment MOST likely to provide improvement is

A. ceftriaxone intramuscularly B. dexamethasone orally or intramuscularly C. humidified oxygen by face mask D. nebulized albuterol E. nebulized budesonide

Copyright 2007 by the American Academy of Pediatrics

page 377

2007 PREP SA on CD-ROM Critique: 181 Preferred Response: B

The boy described in the vignette most likely has croup, the most common cause of upper airway obstruction in young children. The disease usually is due to a viral infection (parainfluenza types 1 and 2). Croup typically has a nocturnal presentation in a child 6 months to 2 years of age, with either no prodrome or an upper respiratory tract infection. Noninfectious (spasmodic) croup may be seen in children who have atopy or a history of asthma. The differential diagnosis of acute upper airway obstruction includes foreign body ingestion/aspiration, bacterial tracheitis, and epiglottitis. Because of Haemophilus influenzae type b immunization, epiglottitis is rare today. Severe tonsillitis or parapharyngeal abscesses also may cause acute upper airway obstruction, but these conditions generally afflict older children and adolescents. Signs and symptoms of upper airway obstruction include preference for an upright posture (sometimes tripoding, with the hand placed on the examination table), breathing through an open mouth, dysphagia or inability to swallow secretions, and stridor. It is common for viral laryngotracheobronchitis to cause wheezing or signs of lower airway obstruction, as well. Assessment for respiratory distress includes evaluating for retractions, accessory muscle use, hypoxia, and fatigue. Stridor at rest implies potential upper airway compromise due to subglottic and vocal cord edema. Therapy for airway obstruction is avoidance of agitation (venipuncture for laboratory studies is not indicated or helpful) and early administration of oral or injectable steroids. Nebulized budesonide has not been shown to be superior to parenteral dexamethasone. Provision of humidified oxygen is indicated as a triage intervention for all children in respiratory distress, but it does not represent definitive management. Use of mist tents or nebulized saline also is not definitive therapy. Nebulized albuterol does not constrict edematous airway tissue and is not therapeutic in croup, unless the condition is associated with wheezing or other evidence of bronchoconstriction. Nebulized racemic epinephrine may have a short-term benefit but is not definitive treatment. Antibiotics are not indicated in viral croup. Viral croup usually can be distinguished from bacterial tracheitis based on lower temperature, abrupt onset of symptoms, and the presence of upper respiratory tract symptoms for a brief period prior to the development of croup. If a child appears toxic, has severe respiratory distress, or fails therapy for croup, other diagnoses should be considered, such as epiglottitis, bacterial tracheitis, severe croup, or foreign body. These conditions are airway emergencies. Most children who have mild-to-moderate croup are managed successfully as outpatients. Children who have stridor at rest, impending respiratory failure, or underlying disease should be observed in the hospital. References: Cohen LF. In brief: stridor and upper airway obstruction in children. Pediatr Rev. 2000;21:4-5. Available at: http://pedsinreview.aappublications.org/cgi/content/full/21/1/4 Malhotra A, Krilov LR. Viral croup. Pediatr Rev. 2001;22:5-12. Available at: http://pedsinreview.aappublications.org/cgi/content/full/22/1/5 Russell K, Wiebe N, Saenz A, et al. Glucocorticoids for croup. The Cochrane Database of Systematic Reviews. 2004;1:CD001955. Available at: http://www.mrw.interscience.wiley.com/cochrane/clsysrev/articles/CD001955/frame.html

Copyright 2007 by the American Academy of Pediatrics

page 378

2007 PREP SA on CD-ROM Question: 182

A father brings his 2-year-old daughter to you because he has noticed "funny eye movements" over the past 2 weeks. Upon physical examination, you find that the child has pendular nystagmus (Item Q182A), her head is tilted to the right, and she nods her head. Other findings on the physical examination are normal. Of the following, the MOST likely diagnosis is

A. congenital nystagmus B. optic glioma C. phenytoin intoxication D. retinoblastoma E. spasmus nutans

Copyright 2007 by the American Academy of Pediatrics

page 379

2007 PREP SA on CD-ROM Critique: 182 Preferred Response: E

Nystagmus is an involuntary, rhythmic oscillation of both eyes (rarely one) in which at least one phase is slow. Nystagmus reflects a deficit in gaze-holding mechanisms, analogous to brakes that no longer stop an automobile appropriately. Jerk nystagmus is characterized by a slow phase in one direction, usually back to central position, followed by a fast, compensatory movement in the opposite direction, back to the far lateral side. A few beats of jerk nystagmus in the horizontal plane are physiologic, or normal, when a child gazes far laterally. Sustained nystagmus, especially if associated with other neurologic findings, is abnormal. The differential diagnosis includes drug toxicity, vestibular dysfunction, tumor, or infarction. Chiari malformations produce downbeating or vertical jerk nystagmus. Pendular nystagmus is defined as slow movements in both phases, with the eyes moving to and fro. Pendular nystagmus can be a sign of brainstem infarction, spinocerebellar degeneration, multiple sclerosis, or rarely seizure. Spasmus nutans (Item C182A) is a benign, transient disorder without known cause that is characterized by pendular nystagmus, intermittent head tilt, and nodding or titubating of the head, as described for the girl in the vignette. The head tilt can be mistaken as muscular torticollis. The onset is between ages 1 and 3 years, most often in the first 12 postnatal months, with spontaneous resolution within 1 to 2 years. A glioma in the optic pathway can cause jerk or pendular nystagmus, but it also produces loss of visual acuity or field, and over time, optic atrophy. Phenytoin intoxication produces jerk nystagmus. Congenital nystagmus can have pendular to jerk movements that often are not noted until several months or years after birth, and it frequently is characterized by some loss of visual acuity. This disorder can be inherited in variable pattern. Retinoblastoma at presentation does not produce nystagmus, but instead leukocoria (Item C182B) with loss of the red reflex. References: Hoyt CS, Good WV. Ophthalmic problems in childhood. In: Berg BO, ed. Child Neurology: A Clinical Manual. 2nd ed. Philadelphia, Pa: JB Lippincott Co; 1994:241-255 Olitsky SE, Nelson LB. Disorders of eye movement and alignment. In: Behrman RE, Kliegman RM, Jenson HB, eds. Nelson Textbook of Pediatrics. 17th ed. Philadelphia, Pa: WB Saunders Co; 2004:2092-2096

Copyright 2007 by the American Academy of Pediatrics

page 380

2007 PREP SA on CD-ROM Question: 183

You are evaluating a 6-hour-old male infant who was born after a term pregnancy and normal delivery and weighs 4 kg. The infant is comfortable but exhibits mild tachypnea, with a respiratory rate of 50 breaths/min. His heart rate is regular at 150 beats/min. His oxygen saturation is 60% in all extremities and does not increase significantly with the administration of oxygen by face mask. His lungs are clear, and there are no murmurs, gallops, or rub. You suspect transposition of the great arteries. Of the following, the BEST management strategy is

A. diuretic therapy intravenously for pulmonary edema B. increasing the left-to-right (aorta-to-pulmonary artery) shunt at the ductus arteriosus C. increasing the right-to-left shunt at the foramen ovale D. increasing the right-to-left (pulmonary artery-to-aorta) shunt at the ductus arteriosus E. intubation and mechanical ventilation with an Fio2 of 1.0

Copyright 2007 by the American Academy of Pediatrics

page 381

2007 PREP SA on CD-ROM Critique: 183 Preferred Response: B

Understanding blood flow patterns in the patient who has transposition of the great arteries (TGA) requires a clear understanding of shunting patterns in the healthy fetus. Maintenance of the normal patterns can be essential to the management of the neonate who has TGA. In the normal heart, the right atrium and right ventricle deliver desaturated blood to the organ of oxygenation. In the fetus, this organ is the placenta, and its fetal blood supply is via the umbilical artery that arises from the fetal descending aorta. The ductus arteriosus provides a fetal shunting pathway that allows the right side of the fetal heart to deliver desaturated blood to the organ of oxygenation by shunting the blood away from the high-resistance pulmonary arteries and into the descending aorta. This direction of flow occurs in part because the fetal pulmonary vascular resistance is slightly higher than the fetal systemic vascular resistance because the developing lungs are filled with fluid and the placenta is a low-resistance circuit. At birth, when the lungs expand with air and the placenta is removed from the circulation, the pulmonary vascular resistance falls and the systemic vascular resistance increases. This leads to a reversal of flow across the ductus arteriosus (from the systemic into the pulmonary circuit). Over the subsequent hours and days, the ductus arteriosus begins the process of spontaneous closure. The foramen ovale is an important fetal shunt that prenatally allows the relatively oxygenated blood returning from the placenta to cross from the right atrium into the left atrium. In so doing, the blood with the highest oxygen content is directed to the coronary and cerebral circulations. Patency of the foramen in utero results from a slightly higher pressure in the right atrium than the left due to very little (approximately 10% of the combined fetal cardiac output) blood returning to the left atrium from the lungs. The pressure difference pushes the flap of the foramen into the left atrium, creating the hole and allowing right-to-left blood flow. At birth, when the lungs are expanded and the entire cardiac output is directed into the lungs, pressure in the left atrium increases and rises slightly above that in the right atrium, forcing the flap of the foramen against the atrial septum and closing the hole. TGA is the second most common form of cyanotic heart disease, but it is the most common to present in the first day (or hours) after birth. Affected patients have atrioventricular concordance and ventriculoarterial discordance (Item C183A); that is, the left atrium connects to the left ventricle and the right atrium connects to the right ventricle, but the left ventricle connects to the pulmonary artery and the right ventricle connects to the aorta. Thus, desaturated blood returning from the body enters the right heart, only to be pumped back to the aorta. Patients can exhibit profound cyanosis if there is no reliable method to mix left-sided blood into the right side (atrial septal defect or ventricular septal defect). The volume of pulmonary blood flow in TGA is generally normal, but because of the anatomy, oxygenated blood from the left heart repeatedly is delivered to the lungs. To achieve effective pulmonary blood flow and deliver desaturated blood to the lungs, the ductus arteriosus must remain patent, allowing desaturated blue blood from the high-resistance systemic circuit to cross from the aorta into the pulmonary artery. This blood is oxygenated in the lungs (hence, the term effective pulmonary blood flow) and returns to the left atrium. As the total amount of blood entering the lungs (from the left heart and via the ductus arteriosus) increases, so does the volume of blood in the left atrium. The atrial septum is stretched to accommodate the increased volume, and the foramen ovale frequently opens. Because pressure in the left atrium now exceeds that in the right atrium, the direction of shunting across the foramen is left-toright, allowing oxygenated blood to enter the right heart, which in TGA, pumps to the aorta. Thus, the combination of the ductus arteriosus and the foramen ovale can be used to increase oxygenated blood flow to the body in a patient who has TGA. Patency of the ductus arteriosus can be maintained by prostaglandins such as PGE-1, administered as a continuous intravenous infusion. If the patency of the foramen ovale is not sufficient to maintain adequate oxygen saturation in the neonate, a balloon atrial septostomy, a catheter procedure to enlarge the atrial communication, can be performed. Diuretics are not indicated for the infant in the vignette because there is normal pulmonary blood flow into well-functioning lungs. Increasing right-to-left shunting at the foramen ovale delivers more blue blood into the left heart, rather than the desired delivery of more oxygenated

Copyright 2007 by the American Academy of Pediatrics

page 382

2007 PREP SA on CD-ROM

blood to the aorta. Similarly, increasing right-to-left shunting at the ductus arteriosus can occur only if the pulmonary vascular resistance exceeds the systemic vascular resistance, which further diminishes effective pulmonary blood flow and reduces blood returned to the left atrium. Intubation and mechanical ventilation are not indicated because the patient does not have respiratory distress or failure. Because the lungs work normally and the cause for the cyanosis is anatomic, high Fi2 would not change oxygen saturation significantly. References: Friedman AH, Fahey JT. The transition from fetal to neonatal circulation: normal responses and implications for infants with heart disease. Semin Perinatol. 1993;17:106-121. Abstract available at: http://www.ncbi.nlm.nih.gov/entrez/query.fcgi?cmd=Retrieve&db=pubmed&dopt=Abstract&list_ui ds=8327901&query_hl=12&itool=pubmed_DocSum Teitel D. Right to left shunts. In: Rudolph CD, Rudolph AM, Hostetter MK, Lister G, Siegel NJ, eds. Rudolphs Pediatrics. 21st ed. New York, NY: McGraw Hill; 2003:1814-1826

Copyright 2007 by the American Academy of Pediatrics

page 383

2007 PREP SA on CD-ROM Question: 184

You have just diagnosed Ebstein anomaly in a newborn. Of the following, the MOST likely prenatal exposure to result in this abnormality is

A. alcohol B. coumarin C. lithium D. phenytoin E. retinoic acid

Copyright 2007 by the American Academy of Pediatrics

page 384

2007 PREP SA on CD-ROM Critique: 184 Preferred Response: C

Several retrospective studies have provided evidence that lithium is a human teratogen, with malformation rates as high as 12% in exposed infants. Among the most common defects are cardiac anomalies, including Ebstein anomaly, which is a rare malformation characterized by a malformed tricuspid valve and secondary abnormalities of the right ventricle and atrium. Atrial septal defect is another commonly reported abnormality. More recent studies have provided conflicting data about the degree of risk associated with maternal lithium use, leading some to suggest that it can be used cautiously in pregnancy among patients for whom other medications are not efficacious in controlling bipolar disorder. In general, even among those who would consider the use of lithium during pregnancy, it is reserved for patients at substantial risk of relapse if lithium is discontinued. It still is discontinued in these patients in very early pregnancy during embryogenesis and restarted if deterioration occurs. If lithium is administered during the second or third trimesters, care must be taken to avoid high serum concentrations; a neonatal concentration of lithium equivalent to the maternal level leads to sedation in the newborn period. Thyroid function disturbances also have been reported in infants of mothers treated with lithium. Patients must be counseled in detail about the relative risks and benefits of this approach. Lactating women should be advised that breastfeeding also might produce significant lithium concentrations in the infant. Fetal alcohol syndrome is characterized by prenatal-onset growth restriction and facial dysmorphisms. Although cardiac defects occur, they usually are septal defects. Fetal coumarin exposure can result in nasal hypoplasia, stippling of the epiphyses, and low birthweight. Phenytoin syndrome includes growth deficiency, a dysmorphic face (Item C184A), and hypoplasia of the distal phalanges (Item C184B). Cardiac anomalies are less frequent and do not include Ebstein anomaly. Retinoic acid embryopathy is a severe syndrome characterized by central nervous system defects, microtia, and conotruncal malformations. References: Cohen LS, Friedman JM, Jefferson JW, Johnson EM, Weiner ML. A reevaluation of risk of in utero exposure to lithium. JAMA. 1994;271:146-150. Abstract available at: http://www.ncbi.nlm.nih.gov/entrez/query.fcgi?orig_db=PubMed&db=PubMed&cmd=Search&ter m=%22JAMA+:+the+journal+of+the+American+Medical+Association%22[Jour]+AND+1994[pdat] +AND+Cohen+LS[author] Jones KL. Dysmorphology. In: Behrman RE, Kliegman RM, Jenson HB, eds. Nelson Textbook of Pediatrics. 17th ed. Philadelphia Pa: Saunders; 2004:616-623 Stoll BJ, Kliegman RM. Metabolic disturbances. In: Behrman RE, Kliegman RM, Jenson HB, eds. Nelson Textbook of Pediatrics. 17th ed. Philadelphia PA: Saunders; 2004:609-613

Copyright 2007 by the American Academy of Pediatrics

page 385

2007 PREP SA on CD-ROM Question: 185

A 15-year-old agitated and delusional boy presents to the emergency department. Physical examination reveals tachycardia, sweating, and pupils that are dilated and react slowly, but there is no nystagmus, and the eyelids and conjunctivae are normal. Of the following, the substance that MOST likely was abused by this boy is

A. alprazolam B. heroin C. marijuana D. methamphetamine E. phencyclidine

Copyright 2007 by the American Academy of Pediatrics

page 386

2007 PREP SA on CD-ROM Critique: 185 Preferred Response: D

The symptoms and signs exhibited by the 15-year-old boy described in the vignette are most likely the result of methamphetamine abuse. Expected short-term signs and symptoms due to the direct effects of methamphetamines on adrenergic receptors include tachycardia, hypertension, diaphoresis, agitation, and fever. The typical eye signs are dilated pupils that are slow to react to light, normal eyelids and conjunctivae, and no nystagmus. Among the other symptoms are nausea, vomiting, psychosis, hypervigilence, violent behavior, strokes, and seizures. High doses have been associated with ventricular irritability and life-threatening arrhythmias. Frequent use of methamphetamines can cause insomnia and weight loss. Cerebrovascular damage, symptoms similar to Parkinson disease, homicidal and suicidal thoughts, and psychosis can result from chronic use. Psychotic symptoms (eg, paranoia, visual and auditory hallucinations, delusions, and rages of violent behavior) can persist for months to years after discontinuing use. Although there are no physiologic manifestations of methamphetamine addiction/withdrawal, abusers can develop drug tolerance and experience withdrawal symptoms. Such symptoms, sometimes termed a withdrawal syndrome, may include depression, fatigue, anxiety, aggression, paranoia, problems with concentration and memory, and craving for the drug. Intermittent cravings for amphetamines may occur months to years after discontinuing use and may be triggered by situations. The anticipated physical signs of alprazolam (a benzodiazepine) abuse are sleepiness and sedation. Pupils are of normal size and may be slow to react. Under the influence of heroin, the pupils are constricted, and agitation is not typically observed. With marijuana use, the pupils, although slow to react, usually are of normal size, and conjunctival injection is common. Tachycardia and sweating are not present. Although phencyclidine use is associated with agitation and combativeness, the pupils are normal-sized, not constricted, and other eye abnormalities may be observed, including retracted upper eyelid, swollen lid, and vertical and horizontal nystagmus. References: Jenkins RR. Substance abuse: amphetamines. In: Behrman RE, Kliegman RM, Jenson HB, eds. Nelson Textbook of Pediatrics. 17th ed. Philadelphia, Pa: WB Saunders Co; 2004:660 MacKenzie RG, Heischober B. Methamphetamine. Pediatr Rev. 1997;18:305-309. Available at: http://pedsinreview.aappublications.org/cgi/content/full/18/9/305 Neinstein LS, Heischober BS. Miscellaneous drugs: stimulants, inhalants, opiates, depressants, and anabolic steroids. In: Neinstein LS, ed. Adolescent Health Care A Practical Guide. 4th ed. Philadelphia, Pa: Lippincott, Williams & Wilkins; 2002:1337-1369

Copyright 2007 by the American Academy of Pediatrics

page 387

2007 PREP SA on CD-ROM Question: 186

A 14-year-old-boy has been followed yearly by another physician. When you see him for the first time, he brings records that you use to construct a growth curve (Item Q186A). His parents state that they are not worried about his growth because his 20-year-old brother was a slow grower and still seems to be growing a little. Physical examination reveals 6-mL testes and Sexual Maturity Rating 2 pubic hair. There are no other signs of puberty. His bone age on radiography is 12 years. Of the following, the MOST appropriate suggestion for the family is that

A. a period of watchful waiting for 6 months is reasonable because he likely has delayed puberty B. a short course of injected testosterone might help boost his growth and initiate puberty, which
is delayed

C. eating a high-calorie, high-protein meal once a day may initiate his puberty D. laboratory studies to assess his endocrine status should be obtained immediately E. little can be offered to improve his growth because he probably is approaching the end of his
growth phase

Copyright 2007 by the American Academy of Pediatrics

page 388

2007 PREP SA on CD-ROM Critique: 186 Preferred Response: D

Children who have constitutionally delayed maturation and are slow to progress through puberty but otherwise normal have appropriate increments in linear growth that parallel the normal growth curve after the age of about 2 years. They then appear to fall from the normal growth curve when they do not have a pubertal growth spurt at the usual age. In some, prolonged prepubertal growth attenuation, lasting 1 to 3 years, may lead to a perfectly reasoned medical decision to pursue further investigation. Children who have genetic short stature usually find their own growth curves by 1 to 2 years of age, and subsequently grow along those curves until they reach adult height, which although decreased, is commensurate with family heights. Children who have congenital growth hormone deficiency grow slowly in length from the age of about 9 months, but usually have normal weight and even appear a bit chubby. Acquired growth hormone deficiency leads to growth attenuation at a later age but usually is associated with weight maintenance. Children who have endocrine causes of growth attenuation (hypothyroidism, growth hormone deficiency, adrenocortical excess) generally are not underweight. Growth attenuation (failure to grow over the past several years), as documented in the growth curve constructed for the boy described in the vignette, is a worrisome sign of hidden illness that demands immediate investigation. The investigation should look for hidden metabolic or gastrointestinal disease, including celiac disease, inflammatory bowel disease, hypothyroidism, growth hormone deficiency, and, if there is concern based on physical appearance (changes may be subtle), Cushing disease. Laboratory studies for the boy in the vignette should assess his endocrine status. Delayed puberty rarely is associated with prolonged growth attenuation, but such attenuation requires evaluation; watchful waiting is not appropriate. Children who are underweight for height commonly have metabolic/nutritional or gastrointestinal disorders. Those who maintain weight or are slightly heavy for height more commonly have hypothyroidism, Cushing disease, or growth hormone deficiency. The child in the vignette should not be offered adjunctive therapies until a clear diagnosis has been made. Boys grow until their bones are fully fused at a bone age of about 18 years; a boy whose bone age is 12 years still has his pubertal growth spurt ahead of him. References: Israel EJ, Levitsky LL, Anupindi SA, Pitman MB. Case Records of the Massachusetts General Hospital. Weekly clinicopathological exercises. Case 3-2005. A 14-year-old boy with recent slowing of growth and delayed puberty. N Engl J Med. 2005;352:393-403 Kemp S, Gngr N. Growth failure. eMedicine Specialties: Pediatrics: Endocrinology. 2005. Available at: http://www.emedicine.com/ped/topic902.htm Reiter EO. Short stature/tall stature. In: Burg FD, Ingelfinger JR, Polin RA, Gershon AA, eds. Gellis & Kagans Current Pediatric Therapy. Philadelphia, Pa: WB Saunders; 2003:679-682

Copyright 2007 by the American Academy of Pediatrics

page 389

2007 PREP SA on CD-ROM Question: 187

You see a 9-month-old girl for a health supervision visit. She is in her infant carrier with a propped bottle. The mother explains that the infant does not sleep through the night, which is disrupting the mother's ability to function well at work. The infant appears well, with length and head circumferences at the 25th percentile and weight at the 95th percentile. During the examination, the infant cries. The mother hands you a bottle and asks if she may leave to check on a referral for her older child. Of the following, you are MOST likely to suggest to the mother

A. alteration of mother's work schedule B. dental referral for the infant C. establishment of a consistent bedtime routine for the infant D. nutrition consultation for the infant E. social work evaluation

Copyright 2007 by the American Academy of Pediatrics

page 390

2007 PREP SA on CD-ROM Critique: 187 Preferred Response: E

The relationship between an infant and parent is complex, and there are normal variations in how parent and child communicate, with great differences based on infant and child temperament and parental responsibilities and distractions. An infants ability to feed and a parents ability to provide adequate nutrition for the infant are often reflective of the relationship between them. Common feeding practices that suggest a disordered relationship include propping the bottle and feeding to quiet the infant. The mother described in the vignette is working, sleep-deprived, and distracted during the childs examination. When her child cries, her response is to offer a bottle instead of looking for a source of discomfort for the infant. In this case, she is so distracted that she does not even try to feed the infant herself, asking the clinician to feed her. Social work assistance in evaluating the family situation may allow the family to take advantage of community services that would help the mother to attend to her child with fewer distractions. Alteration of mothers work schedule probably will not address all of the issues in this situation. Children who drink from a propped bottle are at risk for dental caries, but this information could be shared with the mother without a dental visit. The mother could benefit from parent education and counseling to help manage the infants sleeping schedule, but this is only part of the problem. Similarly, the infants weight likely is related to feeding practice and can be addressed with parenting education. References: Block RW, Krebs NF, and the Committee on Child Abuse and Neglect and the Committee on Nutrition. Failure to thrive as a manifestation of child neglect. Pediatrics. 2005;116:1234-1237. Available at: http://pediatrics.aappublications.org/cgi/content/full/116/5/1234 Fraser JJ Jr, McAbee GN, and Committee on Medical Liability. Dealing with the parent whose judgment is impaired by alcohol or drugs: legal and ethical considerations. Pediatrics. 2004;114: 869-873. Available at: http://pediatrics.aappublications.org/cgi/content/full/114/3/869 Kairys SW, Johnson CF, and Committee on Child Abuse and Neglect. The psychological maltreatment of childrentechnical report. Pediatrics. 2002;109:e68. Available at: http://pediatrics.aappublications.org/cgi/content/full/109/4/e68

Copyright 2007 by the American Academy of Pediatrics

page 391

2007 PREP SA on CD-ROM Question: 188

A 4-month-old boy presents to the clinic with a 2-day history of a temperature of 100.6F (38.1C) and vomiting. The infant was born at 26 weeks' gestation, and his birthweight was 960 g. He remained in the hospital for 3 months after birth because of multiple complications, including bilateral grade 4 intraventricular hemorrhages necessitating the placement of a ventriculoperitoneal shunt 6 weeks ago. Physical examination reveals an afebrile infant who has a shunt bubble on the right temporo-occipital region and a gastrostomy button. You explain that his shunt bubble requires aspiration to evaluate the cerebrospinal fluid for infection. Of the following, the organism that is MOST likely to be isolated in the aspirate is

A. Enterococcus sp B. Escherichia coli C. nontypeable Haemophilus influenzae D. Staphylococcus epidermidis E. Streptococcus pneumoniae

Copyright 2007 by the American Academy of Pediatrics

page 392

2007 PREP SA on CD-ROM Critique: 188 Preferred Response: D

Although the pathogens that commonly cause community-acquired bacterial meningitis (Enterococcus sp, Escherichia coli, Haemophilus influenzae, or Streptococcus pneumoniae) may be responsible for the illness in the patient described in the vignette, the most common organism responsible for ventriculoperitoneal shunt (VPS) infections is Staphylococcus epidermidis. This organism is isolated in more than 50% of VPS infections, with S aureus accounting for an additional 20%. Treatment of VPS infections includes specific antimicrobial therapy directed at the causative organism and, in most instances, removal or replacement of the VPS. S epidermidis is just one of the many species of coagulase-negative staphylococci that colonize humans. Only S epidermidis and S saprophyticus have demonstrated consistent pathogenicity for humans. Nosocomial infections with S epidermidis often are associated with implanted hardware. Venous, arterial, and peritoneal catheters; cerebrospinal fluid and vascular shunts; and other prosthetic materials (eg, joints, valves) frequently are infected with this organism because they tend to produce a protective biofilm that surrounds the organism and leads to enhanced adhesion to foreign surfaces. In addition, immunocompromised and neonatal intensive care unit patients are at increased risk for serious infections with S epidermidis. Most S epidermidis isolates are methicillin-resistant, but the organism remains susceptible to vancomycin. Therefore, the empiric antimicrobial agent of choice is vancomycin with or without rifampin or gentamicin (used for synergy in some difficult cases) until susceptibility data are available. Because of the biofilm production, foreign bodies that are infected often must be removed for adequate treatment. References: Flynn PM, Barrett FF. Infection associated with medical devices. In: Behrman RE, Kliegman RM, Jenson HB, eds. Nelson Textbook of Pediatrics. 17th ed. Philadelphia, Pa: WB Saunders Co; 2004:858-860 Kiang M, Cheng TL. In brief: Staphylococcus epidermidis. Pediatr Rev. 2003;24:430-431. Available at: http://pedsinreview.aappublications.org/cgi/content/full/24/12/430 Todd JK. Staphylococcus. In: Behrman RE, Kliegman RM, Jenson HB, eds. Nelson Textbook of Pediatrics. 17th ed. Philadelphia, Pa: WB Saunders Co; 2004:861-866

Copyright 2007 by the American Academy of Pediatrics

page 393

2007 PREP SA on CD-ROM Question: 189

You are seeing a 10-year-old boy and his 13-year-old sister for the first time. When you review the medical records provided by their mother, you find normal medical histories, vital signs, and physical examination results for the children. However, the family history indicates that two of the children's uncles are receiving hemodialysis and are deaf and one grandfather died of kidney disease. You obtain a screening urinalysis (UA) in both children. The boy's UA reveals moderate blood, negative protein, and 20 to 30 red blood cells/high-power field (RBC/hpf); the girl's UA reveals trace blood with 5 to 10 RBC/hpf. Of the following, the MOST accurate statement regarding the prognosis for these children is that

A. the boy will develop end-stage renal disease (ESRD); the girl will not develop ESRD B. the chances of developing ESRD are equal in the boy and girl C. the boy will develop ESRD with hearing deficits; the girl will not develop ESRD D. the boy will develop ESRD and esophageal leiomyomatosis; the girl will develop only hearing
deficits

E. the boy will develop ESRD and giant cell thrombocytosis; the girl will develop only ESRD

Copyright 2007 by the American Academy of Pediatrics

page 394

2007 PREP SA on CD-ROM Critique: 189 Preferred Response: C

Alport syndrome (hereditary nephritis) is an uncommon disease, affecting about 1 in 5,000 persons. There are several modes of inheritance, the most common being X-linked. Although there is a male-to-female ratio of 1:1, men are affected earlier and more severely than women. Other modes of inheritance include autosomal dominant, autosomal recessive, and heterogeneous. Females who have the defective gene on one of their two X chromosomes usually are asymptomatic, although they may exhibit some degree of minor renal insufficiency; most females who have Alport syndrome have some blood in the urine. Males who have the defective gene usually develop renal failure between the second and third decades of life. In most cases, the mutation lies in abnormal collagen type IV (COL4A) protein that is an essential part of the glomerular basement membrane. Other conditions that may be inherited and present with hematuria, such as benign familial hematuria and thin membrane disease, are associated with significantly more favorable renal outcomes than is seen with Alport syndrome. The primary characteristics of Alport syndrome are progressive renal failure, sensorineural hearing deficits, and abnormalities of the lens of the eye (lenticonus). Uncommon associated conditions include immunologic abnormality of skin, disorders of platelets, abnormalities of white blood cells, and smooth muscle tumors. Alport syndrome can be diagnosed by several methods. Most commonly, it is diagnosed initially by testing the urine (for blood) of persons who have an affected family member. The urine result may be confirmed by performing a kidney biopsy and assessing for the presence or absence of the COL4A protein. In certain cases, testing for the abnormal gene in Alport syndrome may be performed. The strong family history of renal disease with associated hearing deficits reported for the boy in the vignette is very suggestive of Alport syndrome. Based on the usual mode of inheritance, it can be assumed that the boy will develop end-stage renal disease, but his sister may develop only minor kidney disease or simply be a carrier of the gene. The family history suggests that the boy likely also will develop some degree of hearing deficits. Unusual associated anomalies, including esophageal leiomyomatosis and giant cell thrombocytopenia, are seen only occasionally in Alport syndrome and usually are associated with autosomal modes of inheritance. The family history indicates that this boy has inherited the disease through an X-linked pattern. References: Kashtan CE. Familial hematurias: what we know and what we don't. Pediatr Nephrol. 2005;20:1027-1035. Abstract available at: http://www.ncbi.nlm.nih.gov/entrez/query.fcgi?orig_db=PubMed&db=PubMed&cmd=Search&ter m=%22Pediatric+nephrology+(Berlin,+Germany)%22[Jour]+AND+1027[page]+AND+2005[pdat] Gross O, Netzer KO, Lambrecht R, Seibold S, Weber M. Meta-analysis of genotype-phenotype correlation in X-linked Alport syndrome: impact on clinical counselling. Nephrol Dial Transpl. 2002;17:1218-1227. Available at: http://ndt.oxfordjournals.org/cgi/content/full/17/7/1218 Longo I, Porcedda P, Mari F, et al. COL4A3/COL4A4 mutations: from familial hematuria to autosomal-dominant or recessive Alport syndrome. Kidney Int. 2002;61:1947-1956. Abstract available at: http://www.ncbi.nlm.nih.gov/entrez/query.fcgi?cmd=Retrieve&db=pubmed&dopt=Abstract&list_ui ds=12028435&query_hl=59&itool=pubmed_docsum

Copyright 2007 by the American Academy of Pediatrics

page 395

2007 PREP SA on CD-ROM Question: 190

A 16-year-old previously healthy boy presents with a 2-week history of intermittent elevated temperatures to 102F (39C), headache, malaise, fatigue, myalgias, and a progressively worsening nonproductive cough. During history-taking, he reports that recently he explored several caves while hiking in a forest preserve in Ohio. Physical examination reveals a tiredappearing adolescent who has a temperature of 102.4F (39.1C), a dry cough, and diffuse intermittent rhonchi on chest auscultation. Laboratory findings include a white blood cell count of 12x103/mcL (12x109/L), with 60% neutrophils, 2% band forms, and 38% lymphocytes. Chest radiograph (Item Q190A) shows patchy left upper and left lower lobe opacities and hilar adenopathy. Of the following, the MOST likely pathogen causing this patient's condition is

A. Aspergillus fumigatus B. Coccidioides immitis C. Histoplasma capsulatum D. Rhizopus sp E. Sporothrix schenckii

Copyright 2007 by the American Academy of Pediatrics

page 396

2007 PREP SA on CD-ROM Critique: 190 Preferred Response: C

Histoplasmosis is a common granulomatous infection caused by the dimorphic fungus Histoplasma capsulatum. The clinical spectrum of disease ranges from asymptomatic in 50% of cases to generalized fatal dissemination in less than 1% of cases. The infection occurs worldwide but is most common in the Ohio, Mississippi, and Missouri River valleys in the United States. Soil contaminated by animal droppings, especially from birds and bats, plays a major role in disseminating the organism. H capsulatum frequently is isolated from soil near chicken houses, bird roosts, caves infested with bats, and silos inhabited by pigeons. Active disturbance of the environment is necessary for dissemination of the spores. Acute pulmonary histoplasmosis is the most common form of disease. Asymptomatic primary infection occurs in about 50% of adults and children exposed to H capsulatum spores. Symptomatic disease presents as an influenzalike illness, with abrupt onset of fever, malaise, headache, myalgia, and nonproductive cough, as reported for the boy in the vignette. Chest radiographs reveal patchy pneumonic infiltrates and hilar adenopathy. Aspergillus is a fungus that is ubiquitous in the environment and is found in soil, hay, compost piles, decaying vegetation, water, house dust, bedding, food, medications, surgical dressings, and potted plants. The clinical features of aspergillosis depend on the host response to the disease, but differ from those exhibited by the boy described in the vignette. The lungs and paranasal sinuses are the most frequent sites of infection, but disseminated infection may affect several body organs. Disease may be due to hypersensitivity to the organism or invasion of tissues by the fungus. Allergic pulmonary aspergillosis (due to hypersensitivity) is seen in patients who have asthma; they present with wheezing, fever, dyspnea, and productive cough. Invasive disease (Item C190A) almost exclusively occurs in patients who have underlying conditions, immunocompromised patients who have neutropenia, after medication use that causes neutrophil dysfunction, or after cytotoxic chemotherapy or immunosuppressive therapy. The condition is characterized by cough, hemoptysis, obstructive airway signs, and other pulmonary symptoms. Hematogenous dissemination is seen commonly in the severely immunocompromised host, usually in association with invasive pulmonary aspergillosis and neutropenia, with the brain, gastrointestinal tract, heart, liver, and kidneys being the most frequently involved sites. The signs and symptoms are related to impaired function of the involved organ. Sporotrichosis is caused by the dimorphic fungus Sporothrix schenckii. It is a subacute or chronic fungal infection that can involve both the superficial and deep tissues. The organism is found primarily in warm, temperate zones worldwide. In the United States, most cases occur in states bordering the Mississippi and Missouri Rivers. Cases generally follow intradermal inoculation of spores after contact with contaminated thorns, splinters, reeds, and grasses; floral nursery and tree farm personnel are at the highest risk. Lymphocutaneous sporotrichosis is the most common presentation. It initially appears as a small, painless, movable subcutaneous nodule that enlarges to become a fluctuant mass and progresses to ulceration, with the appearance of addition painless subcutaneous nodules along the lymphatic channels. The classic clinical picture is an ulcer on the finger or wrist with an associated chain of nodules along draining lymphatics. Joints or bone may become infected via dissemination through the bloodstream from a cutaneous focus. Bone lesions are often multiple, lytic, and destructive, and the tibia is involved most commonly. In joint lesions, the synovium is thickened, and the cartilage degenerates. The knee is the most commonly affected joint. Although Coccidioides immitis may produce the symptoms reported for the patient in the vignette, this organism is found in the western desert states of the United States, not in the midwest. Rhizopus sp are opportunistic fungi that cause disease in severely immunocompromised individuals. References: American Academy of Pediatrics. Aspergillosis. In: Pickering LK, ed. Red Book: 2006 Report of the Committee on Infectious Diseases. 27th ed. Elk Grove Village, Ill: American Academy of

Copyright 2007 by the American Academy of Pediatrics

page 397

2007 PREP SA on CD-ROM

Pediatrics; 2006:219-222 American Academy of Pediatrics. Histoplasmosis. In: Pickering LK, ed. Red Book: 2006 Report of the Committee on Infectious Diseases. 27th ed. Elk Grove Village, Ill: American Academy of Pediatrics; 2006:371-374 American Academy of Pediatrics. Sporotrichosis. In: Pickering LK, ed. Red Book: 2006 Report of the Committee on Infectious Diseases. 27th ed. Elk Grove Village, Ill: American Academy of Pediatrics; 2006:595-597 Hughes WT. Aspergillosis. In: McMillan JA, DeAngelis CD, Feigin RD, Warshaw JB, eds. Oskis Pediatrics Principles and Practice. 3rd ed. Philadelphia, Pa: Lippincott Williams & Wilkins; 1999:1155-1156 Leggiadro RJ. Histoplasmosis. In: McMillan JA, DeAngelis CD, Feigin RD, Warshaw JB, eds. Oskis Pediatrics Principles and Practice. 3rd ed. Philadelphia, Pa: Lippincott Williams & Wilkins; 1999:1163-1164 McGinnis MR. Sporotrichosis. In: McMillan JA, DeAngelis CD, Feigin RD, Warshaw JB, eds. Oskis Pediatrics Principles and Practice. 3rd ed. Philadelphia, Pa: Lippincott Williams & Wilkins; 1999:1164-1165

Copyright 2007 by the American Academy of Pediatrics

page 398

2007 PREP SA on CD-ROM Question: 191

The parents of an 8-year-old girl who has asthma would like to know more about possible triggers. The girl has had mild persistent asthma since age 5 years and frequently has increased symptoms in the fall that coincide with starting school. During the fall, she experiences increased rhinorrhea, sneezing, and ocular pruritus. These nasal and ocular symptoms have occurred each fall for the past 3 years but generally do not occur at other times of the year. Of the following, the MOST likely trigger for the patient's asthma exacerbations is

A. allergic rhinitis B. exercise C. gastroesophageal reflux D. psychogenic cough E. sinusitis

Copyright 2007 by the American Academy of Pediatrics

page 399

2007 PREP SA on CD-ROM Critique: 191 Preferred Response: A

The girl described in the vignette most likely is experiencing asthma exacerbations in the fall due to seasonal allergic rhinitis. Viral illnesses and allergic rhinitis are two common asthma triggers for school-age children. Further, approximately 80% of children who have asthma develop allergic rhinitis and demonstrate positive skin tests to aeroallergens. Exercise may result in asthma symptoms, but the relative absence of symptoms during other times of year for the girl in the vignette makes this less likely. Gastroesophageal reflux (GER) can cause cough and exacerbate asthma. Eliciting the complaint of heartburn or regurgitation can be difficult in a child, but the relative seasonality of symptoms for this girl makes GER unlikely. Stress related to starting school may result in a psychogenic cough or school absence, but psychogenic cough is a diagnosis of exclusion. Also, the lack of identifiable stressors at school (eg, bullying) or home (eg, divorce) and the ocular pruritus and sneezing make psychogenic cough unlikely. Sinusitis may be a complication of uncontrolled allergic rhinitis, but the absence of purulent rhinorrhea, sinus tenderness, or fever makes sinusitis less likely than seasonal allergic rhinitis. References: Liu AH, Spahn JD, Leung DYM. Childhood asthma. In: Behrman RE, Kliegman RM, Jenson HB, eds. Nelson Textbook of Pediatrics. 17th ed. Philadelphia, Pa: WB Saunders Co 2004:760-774 Chan EY, Dundas I, Bridge PD, Healy MJ, McKenzie SA. Skin-prick testing as a diagnostic aid for childhood asthma. Pediatr Pulmonol. 2005;39:558-562. Abstract available at: http://www.ncbi.nlm.nih.gov/entrez/query.fcgi?orig_db=PubMed&db=PubMed&cmd=Search&ter m=Pediatr+Pulmonol[Jour]+AND+558[page]+AND+2005[pdat]

Copyright 2007 by the American Academy of Pediatrics

page 400

2007 PREP SA on CD-ROM Question: 192

A 16-year-old male was involved in a brawl with other teens. He was struck a number of times with fists and clubs, and he admits to punching some of his adversaries as well. He presents 6 hours later with complaint of pain in his wrist. Physical examination reveals moderate swelling and tenderness on the dorsum of the hand in the region of the base of the thumb and index finger. A radiograph of the hand reveals a fracture. Of the following, the fracture that is associated with the POOREST prognosis for healing is a

A. first metacarpal fracture B. fracture to the proximal phalanx of the thumb C. lunate fracture D. Salter II fracture of the distal radius E. scaphoid fracture

Copyright 2007 by the American Academy of Pediatrics

page 401

2007 PREP SA on CD-ROM Critique: 192 Preferred Response: E

Carpal bone fractures in children are rare due to the largely cartilaginous nature of these bones in early childhood. Most of the carpal bones are ossified by early adolescence, resulting in fracture patterns beyond that age being similar to adult injuries. The scaphoid (Item C192A) is the most commonly fractured carpal bone in children, usually as a result of a fall onto outstretched hands. The classic physical finding is tenderness in the anatomic snuffbox just distal to the radius where the scaphoid articulates with the radius. A high index of suspicion must be maintained when evaluating a hand or wrist injury in a child due to a significant risk of nonunion of the fractured scaphoid if it is not immobilized properly in a plaster cast. Unacceptable functional defects can result, requiring surgical interventions to repair the damage. The rate of scaphoid nonunion in children who are properly immobilized may be as low as 1%, significantly lower than the 5% to 12% rate found in adults. Carpal lunate fractures are exceedingly rare injuries in children, with only isolated case reports in the literature. There are no data indicating a high rate of complications. Fractures to the first metacarpal, proximal phalanx, and distal radius are relatively common. Except in cases of severe disruption of the anatomy or open fractures, uneventful healing is the expected outcome. References: Goddard N. Carpal fractures in children. Clin Orthop Relat Res. 2005;432:73-76. Abstract available at: http://www.ncbi.nlm.nih.gov/entrez/query.fcgi?orig_db=PubMed&db=PubMed&cmd=Search&ter m=432[volume]+AND+73[page]+AND+2005[pdat]+AND+Goddard+N[author] Toh S, Miura H, Arai K, Uasumura M, Wada M, Tsubo K. Scaphoid fractures in children: problems and treatment. J Pediatr Orthop. 2003;23:216-221. Abstract available at: http://www.ncbi.nlm.nih.gov/entrez/query.fcgi?orig_db=PubMed&db=PubMed&cmd=Search&ter m=J+Pediatr+Orthop[Jour]+AND+216[page]+AND+2003[pdat] Henderson B, Letts M. Operative management of pediatric scaphoid fracture nonunion. J Pediatr Orthop. 2003;23:402-406. Abstract available at: http://www.ncbi.nlm.nih.gov/entrez/query.fcgi?orig_db=PubMed&db=PubMed&cmd=Search&ter m=J+Pediatr+Orthop[Jour]+AND+402[page]+AND+2003[pdat] Dias JJ, Wildin CJ, Bhowal B, Thompson JR. Should acute scaphoid fractures be fixed? A randomized controlled trial. J Bone Joint Surg. 2005;87:2160-2168. Abstract available at: http://www.ncbi.nlm.nih.gov/entrez/query.fcgi?orig_db=PubMed&db=PubMed&cmd=Search&ter m=J+Bone+Joint+Surg[Jour]+AND+2160[page]+AND+2005[pdat]

Copyright 2007 by the American Academy of Pediatrics

page 402

2007 PREP SA on CD-ROM Question: 193

A 7-year-old girl is hospitalized after the acute onset of fever, rapid development of hypotension, diffuse erythema of the skin (Item Q193A), rapidly accelerating renal failure, and multisystem organ involvement. Toxic shock syndrome (TSS) is diagnosed. Of the following, the MOST likely finding associated with TSS caused by toxin-producing Streptococcus pyogenes is

A. foreign body at the site of infection B. necrotizing fasciitis C. recurrent episodes of S pyogenes infection D. severe myalgias E. widespread blistering of the skin

Copyright 2007 by the American Academy of Pediatrics

page 403

2007 PREP SA on CD-ROM Critique: 193 Preferred Response: B

Toxic shock syndrome (TSS) is a multisystem disease caused by toxin-producing strains of Staphylococcus aureus or Streptococcus pyogenes. Both forms are characterized by shock and widespread macular erythema. Soft-tissue infections often are seen in association with streptococcal TSS. Necrotizing fasciitis is a particularly serious complication of S pyogenes TSS that constitutes a surgical emergency. Profuse watery diarrhea, vomiting, conjunctival injection, and severe myalgias are associated with S aureus-mediated TSS, but are less common in the streptococcal form of the disease. If a focus of infection is identified, the presence of a foreign body at this site is common with S aureus-mediated TSS, but not with S pyogenes-mediated TSS. Neither type of TSS is associated with widespread blistering of the skin. Recurrent episodes of S pyogenes infection are not a common outcome following TSS. References: American Academy of Pediatrics. Toxic shock syndrome. In Pickering LK, ed. Red Book: 2006 Report of the Committee on Infectious Diseases. 27th ed. Elk Grove Village, Ill: American Academy of Pediatrics; 2006:660-665 Darmstadt G. Bacterial infections: streptococcal and staphylococcal infections. In: Schachner LA, Hansen RC, eds. Pediatric Dermatology. 3rd ed. St. Louis, Mo: Mosby; 2003:989-1011 Gerber MA. Group A Streptococcus. In: Behrman RE, Kliegman RM, Jenson HB, eds. Nelson Textbook of Pediatrics. 17th ed. Philadelphia, Pa: WB Saunders Co; 2004:874-878

Copyright 2007 by the American Academy of Pediatrics

page 404

2007 PREP SA on CD-ROM Question: 194

A 2-month-old exclusively breast-fed infant presents to your office because his mother thinks that he is irritable. His mother reports that the infant has been passing loose stools and cries when he has a bowel movement. He is generally happy at other times. Physical examination demonstrates a healthy, afebrile, vigorous infant who has normal skin color. Cardiac, pulmonary, and abdominal examination findings are all normal. Anal inspection demonstrates no fissures. A stool specimen has reddish flecks, and the guaiac test is positive. Of the following, the BEST next step is to

A. begin therapy with oral amoxicillin B. institute a trial of lansoprazole C. obtain an upper gastrointestinal radiography series D. remove milk products from the maternal diet E. send stool for Clostridium difficile toxin testing

Copyright 2007 by the American Academy of Pediatrics

page 405

2007 PREP SA on CD-ROM Critique: 194 Preferred Response: D

The infant described in the vignette is passing small amounts of blood in the stool, but appears otherwise healthy without any sign of infection. Although bacterial colitis is a possibility, the most likely diagnosis is allergic colitis. The first-line treatment of this condition is dietary restriction of milk protein from the mother's diet. Oral amoxicillin is not indicated unless there is evidence of colitis caused by an amoxicillin-sensitive pathogen. Lansoprazole, a proton pump inhibitor, is for treatment of gastroesophageal reflux. An upper gastrointestinal barium radiograph is used to diagnose malrotation, pyloric stenosis, and inflammatory bowel disease, but this presentation is not consistent with those conditions. Clostridium difficile is present in the stool of 25% of healthy term infants, but it rarely, if ever, causes colitis in this age group. Infantile allergic proctocolitis typically presents in infants younger than 3 months of age and is characterized by loose stool that contains small amounts of blood and mucus. If performed, endoscopic examination demonstrates an erythematous colon with lymphoid nodules, and biopsy shows an inflammatory infiltrate comprised primarily of eosinophils (Item C194A). The most common offending antigens triggering the inflammatory response are milk and soy, but other antigens, including wheat, eggs, corn, fish, and nuts, also may play a role. Allergic colitis may occur in both breastfed and bottle-fed infants; in the case of breastfed infants, it is believed the immunogenic peptides come from the maternal diet and pass into the mothers milk. Although generally believed to be a benign condition that resolves by 1 year of age, rare cases of severe anemia and colonic stricture have been reported with allergic colitis. Therefore, the initial recommended management involves restriction of milk or soy from the infants and mother's diets. In most infants, the rectal bleeding resolves within 3 weeks after dietary restriction, although small amounts of microscopic bleeding may persist. By 1 year of age, most infants can be rechallenged successfully with milk or soy. References: Chang J-W, Wu T-C, Wang K-S, Huang I-F, Huang B, Yu I-T. Colon mucosal pathology in infants under three months of age with diarrhea disorders. J Pediatr Gastroenterol Nutr. 2002;35:387-390. Available at: http://www.jpgn.org/pt/re/jpgn/abstract.00005176-20020900000031.htm;jsessionid=G1XZJHD0FWF6q3h0vtvs5tQTH2PFpGn5Z7s5YBXrxTQtRcWslPJl!194 1873617!-949856145!8091!1?index=1&database=ppvovft&results=1&count=10&searchid=1&nav=search Odze RD, Wershil BK, Leichtner AM, Antonioli DA. Allergic colitis in infants. J Pediatr. 1995;126:163-170. Abstract available at: http://www.ncbi.nlm.nih.gov/entrez/query.fcgi?cmd=Retrieve&db=pubmed&dopt=Abstract&list_ui ds=7844660&query_hl=10&itool=pubmed_docsum

Copyright 2007 by the American Academy of Pediatrics

page 406

2007 PREP SA on CD-ROM Question: 195

You are called to evaluate a male infant at 50 hours of age because he has not voided. He was born at term and has breastfed poorly, but has passed stool. He appears uncomfortable on physical examination, with a large abdomen and seemingly palpable bladder. There is no respiratory distress. The external genitalia are normal, and both testes descended. Of the following, the MOST appropriate initial step in this infant's evaluation is

A. consultation with a urologist B. intravenous pyelography C. nuclear renal scan D. passing of a urinary catheter E. renal ultrasonography

Copyright 2007 by the American Academy of Pediatrics

page 407

2007 PREP SA on CD-ROM Critique: 195 Preferred Response: D

The causes of acute renal failure (ARF) in the newborn include: dehydration, sepsis, congestive heart failure, toxic effects of certain drugs (angiotensin-converting enzyme inhibitors, indomethacin, amphotericin), acute tubular necrosis, congenital renal dysplasia, renal vein thrombosis, and obstructive uropathies (posterior urethral valves, neurogenic bladder, bilateral ureteropelvic junction or ureterovesical junction obstruction). The infant described in the vignette, who has not passed urine 50 hours after birth and who has a palpably full bladder, should be evaluated thoroughly. Anuria in a newborn for more than 24 hours warrants an evaluation that includes a thorough history (including obstetric history and drug exposure), review of any prenatal ultrasonography images, assessment for evidence of perinatal asphyxia or sepsis, and a family history of renal disease. The physical examination should address the genitalia, abdomen, and flanks as well as any signs of edema or oligohydramnios sequence. Laboratory evaluation should attend to serum electrolytes, urea nitrogen and creatinine, urinalysis, urine culture, and urinary electrolytes and creatinine. To obtain urine and complete the evaluation of the patient, a urinary catheter must be placed. Doing so will answer two questions: Are the urethra and bladder outlet patent or obstructed? Have the kidneys produced any urine? Once urine is obtained, the laboratory evaluation can be completed and a fractional excretion of sodium (FENa) calculated to assist in distinguishing prerenal ARF (FENa <1.0) from intrinsic ARF (FENa >1.0). If no urine can be obtained by catheterization, ultrasonography of the kidneys, ureters, and bladder should be performed. Neither nuclear renal scan nor intravenous pyelography should be performed until urine flow is established. A urologist may be consulted, but bladder catheterization should be performed first. References: Drukker A, Guignard J-P. Renal aspects of the term and preterm infant: a selective update. Curr Opin Pediatr. 2002;14:175-182. Available at: http://www.copediatrics.com/pt/re/copeds/abstract.00008480-20020400000006.htm;jsessionid=Ej9NXjYM7YHFOQLxSd9nIX3QlfWf1CTVttYNXrcN5328kqV1ogrB!1861031787!-949856144!9001!-1 Gouyon JB, Guignard JP. Management of acute renal failure in newborns. Pediatr Nephrol. 2000;14:1037-1044. Abstract available at: http://www.ncbi.nlm.nih.gov/entrez/query.fcgi?cmd=Retrieve&db=pubmed&dopt=Abstract&list_ui ds=10975322&query_hl=42&itool=pubmed_docsum Haycock GB. Management of acute and chronic renal failure in the newborn. Semin Neonatol. 2003;8:325-334. Abstract available at: http://www.ncbi.nlm.nih.gov/entrez/query.fcgi?cmd=Retrieve&db=pubmed&dopt=Abstract&list_ui ds=15001136&query_hl=44&itool=pubmed_docsum Vogt VA, Dell KM, Davis ID. The kidney and urinary tract. In: Martin RJ, Fanaroff AA, Walsh MC, eds. Fanaroff and Martins Neonatal-Perinatal Medicine: Diseases of the Fetus and Infant. 8th ed. Philadelphia, Pa: Mosby-Elsevier; 2006:1659-1684

Copyright 2007 by the American Academy of Pediatrics

page 408

2007 PREP SA on CD-ROM Question: 196

An African-American mother brings her previously healthy 2-year-old son to the emergency department because he looked pale when he woke up this morning. She also reports fever and decreased activity over the last few days. He was diagnosed with a urinary tract infection and given trimethoprim-sulfamethoxazole 3 days ago. On physical examination, he is markedly pale and appears jaundiced. His heart rate is 170 beats/min, and his respiratory rate is 30 breaths/min. The rest of his physical examination findings are unremarkable. Laboratory evaluation reveals a hemoglobin concentration of 5 g/dL (50 g/L). Of the following, the MOST likely cause of this boy's anemia is

A. aplastic anemia B. glucose-6-phosphate dehydrogenase deficiency C. hereditary spherocytosis D. sickle cell hemolytic crisis E. transient erythroblastopenia of childhood

Copyright 2007 by the American Academy of Pediatrics

page 409

2007 PREP SA on CD-ROM Critique: 196 Preferred Response: B

The sudden onset of pallor and jaundice in a healthy child is suggestive of acute hemolytic anemia, and the history of recent use of trimethoprim-sulfamethoxazole, as reported for the boy in the vignette, should raise the suspicion of glucose-6-phosphate dehydrogenase (G6PD) deficiency. G6PD is an enzyme used for glucose metabolism. Deficiency of the enzyme results in a decrease in reduced glutathione, which is important for the removal of oxygen radicals. When oxidant drugs are administered to those who have G6PD deficiency, the oxygen radicals cannot be removed adequately, and the red cell membrane is damaged. Trimethoprimsulfamethoxazole is an oxidant drug commonly prescribed for children. Other oxidant drugs include chloramphenicol, nitrofurantoin, primaquine, and acetylsalicylic acid. Moderate-to-severe illness may be another cause of oxidant stress without medication use and may be an inciting factor for acute hemolysis in those who have G6PD deficiency. There are many forms of G6PD deficiency, which is X-linked in inheritance. The A-minus type is seen in African-American males. In this type, the activity of the G6PD enzyme diminishes with the age of the red blood cell. Affected patients are asymptomatic until they take an oxidant drug or become moderately ill, at which time they develop pallor, jaundice, and fatigue. Hemoglobin and hematocrit concentrations fall dramatically, and reticulocytosis occurs. The young reticulocytes produced by patients who have A-minus type G6PD deficiency contain an adequate amount of G6PD, so resolution occurs spontaneously. The Mediterranean type of G6PD deficiency also is characterized by severe hemolysis with exposure to oxidant stress. However, enzyme deficiency in this type is seen in red blood cells of all ages, so reticulocytosis does not result in spontaneous recovery. Aplastic anemia and transient erythroblastopenia of childhood are disorders of red cell production, so jaundice is not a typical feature. Children who have hereditary spherocytosis either present with neonatal hyperbilirubinemia or later with splenomegaly and mild-to-moderate anemia. Children who have sickle cell disease are likely to develop hemolytic crisis with illness, but symptoms of vaso-occlusive crisis usually occur in the first postnatal year, making this an unusual diagnosis in a previously healthy 2-year-old. References: Segel GB, Hirsh MG, Feig SA. Managing anemia in pediatric office practice: part 1. Pediatr Rev. 2002;23:75-84. Available at: http://pedsinreview.aappublications.org/cgi/content/full/23/3/75 Segel GB, Hirsch MG, Feig SA. Managing anemia in pediatric office practice: part 2. Pediatr Rev. 2002;23:111-121. Available at: http://pedsinreview.aappublications.org/cgi/content/full/23/4/111

Copyright 2007 by the American Academy of Pediatrics

page 410

2007 PREP SA on CD-ROM Question: 197

A 2-year-old boy is pulled from the pool by the lifeguard after slipping into the water while his mother was caring for another child poolside. Of the following, the factor MOST predictive of a favorable prognosis is

A. no seizure activity B. no vomiting occurred C. palpable pulse on arrival at the hospital after cardiopulmonary resuscitation en route D. palpable pulse when emergency medical services arrive at scene E. estimated submersion time of less than 3 minutes

Copyright 2007 by the American Academy of Pediatrics

page 411

2007 PREP SA on CD-ROM Critique: 197 Preferred Response: D

Near-drowning events among children generally have one of two outcomes: survival with virtually no neurologic sequelae or death soon after arrival at the hospital. The single most important predictor of outcome for a near-drowning in a pediatric patient is whether cardiac arrest has occurred. Those who survive cardiac arrest do so only if cardiopulmonary resuscitation is required for fewer than 10 minutes. Thus, patients who have palpable pulses at the scene are most likely to survive and have a good prognosis. Other factors in the prognosis include presence of coma, hyperglycemia, and pupillary response. Persistence of coma or lack of purposeful movements by 24 hours after the submersion event correlates with poor neurologic outcome. The presence of seizure activity may signify central nervous system injury from hypoxia or may be the cause of the submersion injury. Vomiting is common in patients who survive neardrowning and may be associated with aspiration and lung injury. Length of submersion is important as a prognostic sign but is rarely known definitively. Thus, absence of seizure activity, presence of vomiting, or submersion time of less than 3 minutes are not predictive of a favorable prognosis. Submersion injury associated with marked hypothermia (core temperature less than 82.4F [28C]) may help protect the central nervous system and myocardium. Resuscitation in such patients should continue without interruption during rewarming to a core temperature to 89.6F (32C). References: Fiore M, Heidemann S. Near drowning. eMedicine Specialties: Pediatrics: Critical Care. 2004. Available at: http://www.emedicine.com/ped/topic2570.htm Glaze DG. The comatose child. In: McMIllan JA, DeAngelis CD, Feigin RD, Warshaw JB, eds. Oskis Pediatrics: Principles and Practice. 3rd ed. Philadelphia, Pa: Lippincott, Williams & Wilkins; 1999:1952-1956 Quan L. Near-drowning. Pediatr Rev. 1999;20:255-260. Available at: http://pedsinreview.aappublications.org/cgi/content/full/20/8/255

Copyright 2007 by the American Academy of Pediatrics

page 412

2007 PREP SA on CD-ROM Question: 198

A 4-month-old is brought to clinic by his parents for evaluation of bilateral droopy eyes. His mother believes this has developed just over the last week. The child recently started taking cereal in addition to breastfeeding and has been constipated. Physical examination reveals droopy eyelids (Item Q198A) and 1+ deep tendon reflexes diffusely. Of the following, the MOST likely diagnosis is

A. botulism B. blepharophimosis syndrome C. congenital ptosis D. muscular dystrophy E. myasthenia gravis

Copyright 2007 by the American Academy of Pediatrics

page 413

2007 PREP SA on CD-ROM Critique: 198 Preferred Response: A

Ptosis may be congenital or acquired. Congenital ptosis (Item C198A) is associated with a wide variety of lid dysfunctions, may be unilateral or bilateral, and may have a neural or muscular origin. Ptosis may arise from a congenital or new third nerve origin, with severe lid weakness and often associated with superior rectus weakness. Ptosis resulting from sympathetic dysfunction usually presents unilaterally, with no more than 2- to 3-mm lid weakness. The responsible lesion arises anywhere along the course of the sympathetic pathway from the hypothalamus to the orbit. Such ptosis usually is accompanied by miosis and anhidrosis (Horner syndrome) and can be congenital or newly acquired. Neuromuscular causes for ptosis that have a new-onset neuropathic or myopathic basis include myasthenia gravis, mitochondrial disease (chronic progressive external ophthalmoplegia), and toxin exposures, such as botulism, diphtheria, tick paralysis, insecticides, and vincristine. The bilateral ptosis, combined with constipation and diminished reflexes, all developing during the introduction of solid foods described for the infant in the vignette is classic for botulism. Congenital ptosis would have been present since birth, and myasthenia produces variable ptosis, not seen in this baby. Most muscular dystrophies, except myotonic dystrophy, do not produce ptosis. Blepharophimosis syndrome is an autosomal dominant condition that has high penetrance, producing bilateral and severe ptosis as well as other abnormalities, such as constricted lids, increased distance between the medial canthi, absent epicanthal folds, flat nasal bridge, and low-set ears. References: Hoyt CS, Good WV. Ophthalmic problems in childhood. In: Berg BO, ed. Child Neurology: A Clinical Manual. 2nd ed. Philadelphia, Pa: JB Lippincott Co; 1994:241-255 Olitsky SE, Nelson LB. Abnormalities of the lids. In: Behrman RE, Kliegman RM, Jenson HB, eds. Nelson Textbook of Pediatrics. 17th ed. Philadelphia, Pa: WB Saunders Co; 2004:20972098

Copyright 2007 by the American Academy of Pediatrics

page 414

2007 PREP SA on CD-ROM Question: 199

A 3-year-old girl is brought to your office for re-evaluation of a fever that began 6 days ago. Her mother tells you that her daughter's temperature has been as high as 102.2F (39C). Her physical examination was unremarkable when you examined her 3 days ago, but today you note injected sclera; cracked, red lips (Item Q199A); a strawberry appearance of her tongue; and a swollen, nontender, cervical node. You tell her mother that you believe this is Kawasaki disease. Of the following, the MOST appropriate statement to make to the mother is that

A. an exercise stress test should be performed as a baseline study B. aspirin therapy will be used until the fever subsides C. cardiac involvement may include abnormalities of the coronary arteries or the myocardium D. echocardiography should be performed to evaluate for the presence of coronary aneurysms E. immediate treatment with intravenous immune globulin will eliminate the chance of coronary
involvement

Copyright 2007 by the American Academy of Pediatrics

page 415

2007 PREP SA on CD-ROM Critique: 199 Preferred Response: C

Kawasaki disease (KD) is believed to be a multisystem illness characterized by vasculitis of small- and medium-size blood vessels, including the coronary arteries. Although the precise cause remains unclear, recent investigations suggest that at least some of the cases are related to an infectious organism (New Haven coronavirus). The median age of patients affected by KD is 2.3 years, and nearly 80% of cases occur in children younger than 4 years of age. More cases are reported in the winter and spring than during the summer and fall, and males are affected approximately 1.5 times more frequently than females. KD can be seen in patients of all ethnicities, races, and cultures, but it is most prevalent in those of Asian descent. KD is diagnosed using a set of diagnostic criteria that include the presence of fever for at least 5 days and at least four of the five following clinical findings: Changes in the extremities (edema, erythema, desquamation [Item C199A]) Polymorphous exanthem (Item C199B) that usually is truncal Conjunctival injection (Item C199C) Erythema or fissuring of the lips (Item C199D), oral cavity, and tongue Cervical lymphadenopathy In addition, the diagnosis of KD requires that the constellation of findings with which the patient presents cannot be explained by any other known disease process. Some children may have an incomplete form of the disease but still are at risk for its complications. KD has an acute phase that lasts 1 to 2 weeks after the onset of fever, which is when the diagnostic criteria typically are present. The subacute phase follows from 2 to 8 weeks after onset. Patients may demonstrate desquamation of the fingers and toes. It is during this phase when coronary aneurysms may develop, particularly in those who have not been treated with intravenous gamma globulin during the acute phase. The convalescent phase lasts for months following the illness. Patients who have KD may have involvement of the cardiovascular system that can include myocarditis, valvulitis, or arteritis. The latter is diagnosed by the presence of edematous coronary walls on echocardiography, usually between days 5 and 8 of the illness. Coronary artery aneurysms can develop after the acute phase and typically in the subacute phase. The incidence of coronary aneurysms may be as high as 22% to 25% in untreated patients compared with an incidence of 3% to 5% in those who receive intravenous immune globulin therapy in the acute phase. Exercise stress testing is not part of the initial evaluation and cannot be performed reliably in a child as young as the one in the vignette; it usually is reserved for older, more cooperative children toward the age of 7 to 8 years. High-dose aspirin therapy (80 to 100 mg/kg per day) is administered until the patient is afebrile for 48 hours, at which time the dose is decreased to 3 to 5 mg/kg per day for 6 to 8 weeks or until platelet concentrations normalize. Echocardiography usually is performed at the time of diagnosis to assess for the presence of a subclinical myocarditis, to evaluate for coronary arteritis, and to serve as a baseline for future studies. Aneurysms of the coronary arteries are not seen in the acute phase and, therefore, their absence on echocardiography should reassure neither the parents nor the pediatrician. References: Esper F, Shapiro ED, Weibel C, Ferguson D, Landry ML, Kahn JS. Association between a novel human coronavirus and Kawasaki disease. J Infect Dis. 2005;191:499-502. Available at: http://www.journals.uchicago.edu/JID/journal/issues/v191n4/33623/33623.html Newburger JW, Takahashi M, Gerber MA, et al. Diagnosis, treatment, and long-term management of Kawasaki disease: a statement for health professionals from the Committee on Rheumatic Fever, Endocarditis, and Kawasaki Disease, Council on Cardiovascular Disease in
Copyright 2007 by the American Academy of Pediatrics page 416

2007 PREP SA on CD-ROM

the Young, American Heart Association. Pediatrics. 2004;114:1708-1733. Available at: http://pediatrics.aappublications.org/cgi/content/full/114/6/1708

Copyright 2007 by the American Academy of Pediatrics

page 417

2007 PREP SA on CD-ROM Question: 200

A newborn female has an open neural tube defect, low-set ears, ventricular septal defect, and rib and vertebral column malformations. Of the following, the MOST likely maternal condition that was present during this pregnancy is

A. alcoholism B. diabetes mellitus C. hypothyroidism D. iodine deficiency E. syphilis

Copyright 2007 by the American Academy of Pediatrics

page 418

2007 PREP SA on CD-ROM Critique: 200 Preferred Response: B

Maternal diabetes mellitus can have profound effects on the development and health of the fetus. The severity of maternal disease and the subsequent fetal effects can vary considerably. Because the maternal diabetic state can be present from the time of conception, early prenatal effects can result in malformations, growth deficiency, and stillbirth. There is a threefold increase in malformations among offspring of diabetic mothers; the incidence is correlated with the severity and level of control of the maternal illness. The most common defects involve the heart, central nervous system, kidneys, and skeleton, as described for the infant in the vignette. Of the cardiac defects, ventricular septal defect, transposition of the great vessels, and dextrocardia are most common. Central nervous system defects can range from anencephaly or holoprosencephaly to spina bifida and hydrocephalus. Malformations of the lower spine also occur and are termed the caudal regression syndrome. The spine may be segmented defectively or terminate in the sacral or lumbar region, resulting in abnormal neurologic function below the level of the defect. Rib defects also may be seen. Infants of diabetic mothers also may present in the newborn period with macrosomia due to hyperinsulinemia and excessive glucose availability. The macrosomia affects both linear growth and weight. Alternatively, if the diabetic mother has substantial vascular disease, fetal growth can be impaired, resulting in growth restriction. Other complications in infants of diabetic mothers include hyperbilirubinemia, hypoglycemia, vascular thromboses, respiratory distress, and birth injury due to macrosomia. Fetal alcohol syndrome is characterized by prenatal growth deficiency, microcephaly, and cardiac defects. Neural tube and vertebral column defects are not common features. Maternal hypothyroidism has little effect on the fetus, which produces its own thyroid hormone; women who have untreated hypothyroidism have been reported to give birth to healthy offspring. Maternal iodine deficiency can cause fetal deficiency of the mineral, which results in goiter, signs of cretinism, retarded bone growth, constipation, umbilical hernia, and mottling in the newborn. Prompt treatment with iodine is necessary to prevent mental retardation. Maternal syphilis can affect the fetal skin, mucous membranes, liver, central nervous system, and bones. Cardiac anomalies and open neural tube defects are not common features. References: Jones KL. Dysmorphology. In: Behrman RE, Kliegman RM, Jenson HB, eds. Nelson Textbook of Pediatrics. 17th ed. Philadelphia Pa: Saunders; 2004:616-623 Riskin A, Haney PM. Infant of a diabetic mother. UpToDate. 2006;14.1. Available at: http://www.utdol.com/utd/content/topic.do?topicKey=neonatol/7268&type=A&selectedTitle=1~12 Stoll BJ, Kliegman RM. The endocrine system: infants of diabetic mothers. In: Behrman RE, Kliegman RM, Jenson HB, eds. Nelson Textbook of Pediatrics. 17th ed. Philadelphia Pa: Saunders; 2004:613-614

Copyright 2007 by the American Academy of Pediatrics

page 419

2007 PREP SA on CD-ROM Question: 201

The mother of a 14-year-old boy arranges to meet with you privately before the boy's annual health supervision visit. She is concerned because he is quiet, has no athletic interests, and has only a few friends. He is content to spend the weekend shopping, cooking, reading, and listening to music. Although he doesn't like school, he is an honor student. The mother also tells you her husband's youngest brother recently disclosed his homosexuality and wants to introduce his male partner to the extended family. Of the following, while counseling the mother, you are MOST likely to include a statement that

A. compared with heterosexual peers, gay high school students are more likely to abuse
substances

B. self-awareness of sexual orientation is established by age 5 years C. sexual orientation is culturally determined D. sexual play with same-sex friends is a clear marker for homosexuality E. she should explain to her son that he is free to choose his sexual orientation

Copyright 2007 by the American Academy of Pediatrics

page 420

2007 PREP SA on CD-ROM Critique: 201 Preferred Response: A

Pediatricians should be aware that some adolescents may have concerns about their sexual orientation or that of siblings, friends, parents, or other relatives. Likewise, parents may have questions or concerns about the sexual orientation of their children. The term sexual orientation refers to an individuals pattern of physical and emotional arousal toward other persons. Homosexual individuals are attracted to persons of the same sex. Gender identity is the knowledge of oneself as being male or female. Sexual orientation is not synonymous with sexual activity or sexual behavior. Homosexual individuals may engage in sexual activities/behaviors with persons of the same or opposite sex. Sexual orientation, which is biologically based, is not determined by any one factor, but by a combination of genetic, hormonal, and environmental influences. Theories that adverse life events (eg, abnormal parenting, sexual abuse) influence sexual orientation have not been substantiated. Homosexual youth are at a higher risk of school drop-out, homelessness, and substance abuse. They also are more likely to be threatened at school. Homosexual males are more likely to attempt suicide than heterosexual males. Although sexual orientation is believed to be established in early childhood, a time when homosexual individuals experience a sense of being different, a self-awareness of homosexual identity develops during the adolescent years. Cultural taboos or social intolerance may inhibit homosexual disclosure and behavior, but same-sex experimentation in early adolescence is not uncommon and is not a marker for homosexuality. Sexual behavior/activity is a choice; sexual orientation is inherent and is not a choice. Sexuality education should be an integrated component of comprehensive health care and the longitudinal relationship that pediatricians develop with children, adolescents, and their families. During the early adolescent years, both adolescents and their parents will have questions about puberty and sexuality that offer counseling opportunities. Important points for pediatricians who are counseling parents about their adolescents emerging sexuality are listed in Item C201A. Parents should be encouraged to allow their fifth and sixth grade children to participate in school sexual health education programs that provide information about puberty and balance abstinence messages with decision making, including contraception and sexually transmitted disease prevention strategies. Successful programs provide opportunities for young adolescents to practice communication and negotiation skills and include information about access to services. References: Committee on Psychosocial Aspects of Child and Family Health and Committee on Adolescence. Sexuality education for children and adolescents. Pediatrics. 2001;108:498-502. Available at: http://pediatrics.aappublications.org/cgi/content/full/108/2/498 Frankowski BL and Committee on Adolescence. Sexual orientation and adolescents. Pediatrics. 2004;113:1827-1832. Available at: http://pediatrics.aappublications.org/cgi/content/full/113/6/1827 Gutgesell ME, Payne N. Issues of adolescent psychological development in the 21st century. Pediatr Review. 2004;25:79-85. Available at: http://pedsinreview.aappublications.org/cgi/content/full/25/3/79 Sieving RE, Oliphant JA, Blum RW. Adolescent sexual behavior and sexual health. Pediatr Rev. 2002;23:407-416. Available at: http://pedsinreview.aappublications.org/cgi/content/full/23/12/407

Copyright 2007 by the American Academy of Pediatrics

page 421

2007 PREP SA on CD-ROM Question: 202

You are evaluating a short 14-year-old boy who is underweight for height. His growth curve is shown in Item Q202A. He says he is feeling well. His mother worries that he does not have as much stamina as he did at age 12 years. Of the following, the laboratory study that is MOST likely to be useful in assessing the reason for poor growth is measurement of

A. free thyroxine B. insulin-like growth factor 1 C. insulin-like growth factor binding protein 3 D. tissue transglutaminase antibody E. urine free cortisol

Copyright 2007 by the American Academy of Pediatrics

page 422

2007 PREP SA on CD-ROM Critique: 202 Preferred Response: D

Most children who have short stature and slow growth, such as the boy described in the vignette, do not have endocrine disorders. Healthy children may have a slow linear growth velocity during middle childhood and early adolescent ages if they have constitutional growth delay. However, severe attenuation of weight gain preceding attenuation of linear growth almost always is a sign of a severe underlying disorder. The most common pathologic causes of slow growth are an underlying metabolic, renal, or gastrointestinal disorder. Laboratory tests directed at identifying these disorders can help distinguish between constitutional delay and other conditions. Celiac disease is a common cause of short stature associated with underweight. If immunoglobulin A concentrations are normal, measurement of tissue transglutaminase antibody is the best screening study for celiac disease. Endocrine causes of growth attenuation, including hypothyroidism, growth hormone deficiency, and Cushing syndrome, are not associated with decreased weight for height. Measures of insulin-like growth factor-1 (IGF-1) and insulin-like growth factor binding protein 3 are good screening studies for growth hormone deficiency, although hepatic IGF-1 production also is stimulated by insulin and may be low in underweight children. A 24-hour urine free cortisol measurement is an excellent screening study for Cushing syndrome, but this disorder is associated with at least mild obesity. References: Kemp S, Gngr N. Growth failure. eMedicine Specialties: Pediatrics: Endocrinology. 2005. Available at: http://www.emedicine.com/ped/topic902.htm Reiter EO. Short stature/tall stature. In: Burg FD, Ingelfinger JR, Polin RA, Gershon AA, eds. Gellis & Kagans Current Pediatric Therapy. Philadelphia, Pa: WB Saunders; 2003:679-682 Israel EJ, Levitsky LL, Anupindi SA, Pitman MB. Case Records of the Massachusetts General Hospital. Weekly clinicopathological exercises. Case 3-2005. A 14-year-old boy with recent slowing of growth and delayed puberty. N Engl J Med. 2005;352:393-403 van Rijn JC, Grote FK, Oostdijk W, Wit JM. Short stature and the probability of coeliac disease, in the absence of gastrointestinal symptoms. Arch Dis Child. 2004;89:882-883. Available at: http://adc.bmjjournals.com/cgi/content/full/89/9/882

Copyright 2007 by the American Academy of Pediatrics

page 423

2007 PREP SA on CD-ROM Question: 203

You are evaluating a 2-year-old boy because he bangs his head on his bed before he goes to sleep. His mother has no other concerns about his behavior. There have been no recent changes in the family or stresses that she can identify. Physical examination findings are normal except for excoriated skin on the boy's forehead and minimal ecchymosis. Of the following, the MOST likely diagnosis is

A. autism B. brain injury C. childhood depression D. global developmental delay E. normal variant behavior

Copyright 2007 by the American Academy of Pediatrics

page 424

2007 PREP SA on CD-ROM Critique: 203 Preferred Response: E

Head banging is one of several common rhythmic movements that include body rocking and thumb sucking. As many as 20% of children may bang their heads. It is more common in boys (3:1) and occurs most commonly between 6 months and 4 years of age. The cause of head banging is unknown, but it usually resolves without intervention. In fact, parental attention to the head banging may prolong the practice. Head banging may cause abrasions or mild bruising, as noted in the vignette; children who have significant injury require further evaluation. Although head banging is common in typically developing children, it is more frequent in children who have developmental disorders such as mental retardation and autism as well as in children who have hearing or vision impairment. Because the mother describes no other concerns regarding her childs development or behavior, the boy in the vignette likely has head banging behavior that will resolve without intervention. His mother should be counseled to ignore the behavior or distract her son with other activities. Children who are depressed may perform rhythmic behaviors, but they typically have other manifestations, such as change from their previous routines of play and eating patterns. Children who have global developmental delay have cognitive delays as well as delays in adaptive behavior, such as difficulty with eating independently and cooperating with dressing. There is no history of head injury in the child in the vignette, so head banging as a manifestation of brain injury is unlikely, and neuroimaging is unnecessary. Children who have autism typically have difficulty with communication (eg, language delays, lack of pointing and gaze-following), impaired social skills (lack of pretend play), and rigid or repetitive behavior and play patterns that are not described in the vignette. References: American Psychiatric Association. Stereotypic movement disorder. Diagnostic and Statistical Manual of Mental Disorders. 4th ed revised. Washington, DC: American Psychiatric Association; 2000:131-133 Boyce WT, Shonkoff JP. Compulsive behaviors. In: Rudolph CD, Rudolph AM, Hostet6ter MK, Lister G, Siegel NJ, eds. Rudolphs Pediatrics. 21st ed. New York, NY: McGraw-Hill; 2003:459462

Copyright 2007 by the American Academy of Pediatrics

page 425

2007 PREP SA on CD-ROM Question: 204

You are asked to review a case for morbidity and mortality conference. The infant was born at term to a 19-year-old gravida 1, para 1 woman by normal spontaneous vaginal delivery. The mother was known to be group B Streptococcus-negative, but she did have genital warts. The Apgar scores were 9 at 1 minute and 10 at 5 minutes. On the seventh postnatal day, the infant developed a temperature of 103F (39.4C) and was brought to the emergency department. At this time, the infant was in shock and required mechanical ventilation. Physical examination revealed scleral icterus and hepatosplenomegaly but no skin lesions. A lumbar puncture could not be performed. Laboratory results include: White blood cell count of 2.34x10 3/mcL (2.34x109/L), with 32% lymphocytes, 41% neutrophils, 8% bands, 15% monocytes, 3% eosinophils, and 1% basophils Hemoglobin of 7.1 g/dL (71 g/L) Hematocrit of 21% (0.21) Platelet count of 40x10 3/mcL (40x109/L) Prothrombin time of 41.2 seconds Activated partial thromboplastin time of >106 seconds Aspartate aminotransferase concentration of 3,086 U/L Alanine aminotransferase concentration of 456 U/L Total bilirubin of 4.4 mg/dL (75.2 mcmol/L) The chest radiograph demonstrated diffuse interstitial infiltrates bilaterally (Item Q204A). The patient did poorly over the next 3 days and died despite aggressive management in a pediatric intensive care unit. Of the following, the MOST likely cause of this patient's death is

A. adenovirus B. Escherichia coli C. group B Streptococcus D. herpes simplex virus E. Listeria monocytogenes

Copyright 2007 by the American Academy of Pediatrics

page 426

2007 PREP SA on CD-ROM Critique: 204 Preferred Response: D

Neonates presenting with shocklike symptoms during the first 7 to 10 days after birth are a diagnostic challenge. The differential diagnosis includes five major categories: 1) bacterial sepsis, 2) inborn errors of metabolism, 3) ductal-dependent complex congenital heart disease, 4) nonaccidental trauma, and 5) viral sepsis. Clinicians must make rapid treatment decisions based on history and physical examination. The fever, hepatosplenomegaly, leukopenia, and abnormal findings on chest radiography reported for the patient in the vignette suggest infection as the most likely cause of the infants illness. Group B Streptococcus, Escherichia coli, and Listeria monocytogenes are causes of neonatal septic shock, but the interstitial pulmonary involvement and hepatitis reported in this case make a virus more likely. Although adenoviral infection can have this type of presentation, the most likely viral pathogen is herpes simplex. Neonatal infections with the herpes simplex virus (HSV) may be due to either HSV type 1 (25%) or HSV type 2 (75%). Approximately 80% of mothers of affected infants have no signs or symptoms of HSV infection prior to delivery. The virus commonly is transmitted to the infant from the mother because of the large amount of virus present and the prolonged shedding. If the primary infection affects only the cervix, the mother is usually asymptomatic. The infants become colonized with HSV at delivery and usually present with active disease between 5 and 10 days after birth. Neonatal disease may present as: 1) disease localized to the skin, eye, or mouth (SEM); 2) disseminated disease involving the liver, lungs, adrenal glands, central nervous system, and skin; and 3) localized central nervous system disease. In disseminated disease, approximately 20% of neonates never develop skin lesions. Therefore, the lack of skin lesions does not eliminate HSV as a potential pathogen for a sick infant. The diagnosis of neonatal HSV can be confirmed with: 1) viral cultures of skin lesions, the nasopharynx, or cerebrospinal fluid; 2) direct fluorescent antibody staining of vesicle scrapings; or 3) DNA detection by polymerase chain reaction test of the cerebrospinal fluid. The Tzanck preparation can be helpful but only indicates the presence of a DNA virus and is not specific for HSV. All neonatal HSV infections should be treated with intravenous acyclovir. The dosage is 60 mg/kg per day in three divided doses given intravenously for 14 days for SEM disease and for at least 21 days for disseminated or central nervous system disease. References: Kimberlin DW, Lin CY, Jacobs RF, et al. Natural history of neonatal herpes simplex virus infections in the acyclovir era. Pediatrics. 2001;108:223-229. Available at: http://pediatrics.aappublications.org/cgi/content/full/108/2/223 Waggoner-Fountain LA, Grossman LB. Herpes simplex virus. Pediatr Rev. 2004;25:86-93. Available at: http://pedsinreview.aappublications.org/cgi/content/full/25/3/86

Copyright 2007 by the American Academy of Pediatrics

page 427

2007 PREP SA on CD-ROM Question: 205

A 10-year-old girl presents to the emergency department with a 1-day history of brown urine. She reports no dysuria, urgency, frequency, or abdominal or flank pain. Her vital signs include: temperature, 98.8F (37.1C); blood pressure, 165/97 mm Hg; heart rate, 84 beats/min; and respiratory rate, 20 breaths/min. On physical examination, moderate periorbital edema is evident, but there are no other abnormalities. Urinalysis reveals moderate blood and 4+ protein. The serum complement 3 (C3) concentration is low, and the C4 concentration is normal. Of the following, the MOST likely cause of this girl's hematuria is

A. focal segmental glomerulosclerosis B. immunoglobulin A nephropathy C. lupus nephritis D. membranoproliferative glomerulonephritis E. postinfectious acute glomerulonephritis

Copyright 2007 by the American Academy of Pediatrics

page 428

2007 PREP SA on CD-ROM Critique: 205 Preferred Response: E

The girl in the vignette has painless gross hematuria without fever. Fever would suggest possible infection, and abdominal pain would hint at infection, stone, or renal malformation (tumor, cyst). She has an elevated blood pressure and periorbital edema, which may be due to hypoalbuminemia or fluid retention. Her urinalysis shows blood and protein. The most significant clue to the cause of her renal disease is her low serum complement 3 (C3) concentration and normal C4 concentration. She has strong evidence of nephritis (gross hematuria, hypertension, periorbital edema), decreased C3, and normal C4 values, which are consistent with postinfectious acute glomerulonephritis (PIAGN). If both the C3 and C4 values were low, the nephritis more likely would be membranoproliferative GN or lupus nephritis. Children who have either focal segmental glomerulosclerosis or immunoglobulin A nephropathy may develop gross hematuria, hypertension, edema, and nephritis, but because of the pathogenesis of these diseases, the serum complement values would be normal. PIAGN is a common but generally self-limited renal disease that usually occurs in childhood. Among the several known pathogenetic organisms associated with PIAGN, the most common is group A beta-hemolytic Streptococcus. Most children who have PIAGN recover complete renal function and exhibit normalization of C3 levels by 6 weeks, although some continue to exhibit hematuria and/or proteinuria for prolonged periods after initial presentation. Some also have hypertension, primarily due to salt and water retention, for up to 3 months. Occasionally, the disease may assume a rapidly progressive course, resulting in acute renal failure and the need for treatment with high-dose intravenous corticosteroids and possibly intravenous cyclophosphamide or dialysis if renal failure persists. In these patients, the renal outcome is guarded. References: Berrios X, Lagomarsino E, Solar E, Sandoval G, Guzman B, Riedel I. Post-streptococcal acute glomerulonephritis in Chile--20 years of experience. Pediatr Nephrol. 2004;19:306-312. Abstract available at: http://www.ncbi.nlm.nih.gov/entrez/query.fcgi?cmd=Retrieve&db=pubmed&dopt=Abstract&list_ui ds=14689289&query_hl=61&itool=pubmed_docsum Kasahara T, Hayakawa H, Okubo S, et al. Prognosis of acute poststreptococcal glomerulonephritis (APSGN) is excellent in children, when adequately diagnosed. Pediatr Int. 2001;43:364-367. Abstract available at: http://www.ncbi.nlm.nih.gov/entrez/query.fcgi?orig_db=PubMed&db=PubMed&cmd=Search&ter m=%22Pediatrics+international+:+official+journal+of+the+Japan+Pediatric+Society%22[Jour]+AN D+364[page]+AND+2001[pdat] Pinto SW, Sesso R, Vasconcelos E, Watanabe YJ, Pansute AM. Follow-up of patients with epidemic poststreptococcal glomerulonephritis. Am J Kidney Dis. 2001;38:249-255. Abstract available at: http://www.ncbi.nlm.nih.gov/entrez/query.fcgi?orig_db=PubMed&db=PubMed&cmd=Search&ter m=am+j+kidney+dis[Jour]+AND+249[page]+AND+2001[pdat]

Copyright 2007 by the American Academy of Pediatrics

page 429

2007 PREP SA on CD-ROM Question: 206

A 5-month-old female infant presents with a 1-day history of fever to 102F (38.9C), emesis, and multiple episodes of greenish diarrhea with streaks of blood. Her mother states that the infant is less active, sleepier, and has no interest in feeding. Physical examination reveals a listless infant who has a sunken anterior fontanelle, dry mucous membranes with decreased skin turgor, and skin irritation in the diaper area. Of the following, the MOST likely cause of this patient's gastroenteritis is

A. astrovirus B. Escherichia coli C. Norwalk virus D. rotavirus E. Salmonella sp

Copyright 2007 by the American Academy of Pediatrics

page 430

2007 PREP SA on CD-ROM Critique: 206 Preferred Response: D

Diarrheal diseases in children younger than 5 years of age remain one of the leading causes of morbidity and mortality in developing countries and are important causes of morbidity in developed countries. In the United States, children 5 years of age and younger have between 35 and 40 million episodes of diarrhea annually, resulting in about 170,000 hospitalizations and 300 deaths due to complications. Item C206A shows the most common viral and bacterial diarrheal pathogens in children by age group. Most of the infectious organisms that cause diarrhea are spread via the fecal-oral route. Organisms such as Shigella, Giardia, and Campylobacter also may be transmitted by person-toperson contact because of their low infective inoculum dose. Foodborne diseases affect all age groups. Common causes include: E coli 0157:H7 (EHEC) associated with ingestion of undercooked meat, Salmonella and Campylobacter associated with contaminated poultry products and unpasteurized milk, Yersinia enterocolitica associated with contaminated pork products, and Norwalk virus associated with the consumption of raw seafood. Water also has been documented as a vehicle of transmission for Giardia lamblia, Campylobacter, Cryptosporidium, and Norwalk virus. Child care centers serve as important reservoirs for transmission of infectious diarrheal agents, with the peak incidence of diarrhea occurring in children younger than 3 years of age who are not toilet trained. Rotavirus (seen most commonly in children younger than 12 months of age), Shigella, and Giardia have been associated with outbreaks in child care centers. The child described in the vignette has diarrhea caused by rotavirus, the most common cause of viral diarrheal disease in infants and toddlers. Norwalk virus and E coli are not common causes of diarrhea in young infants. Astrovirus is a cause of diarrhea in children younger than 4 years of age, but most of the infections are asymptomatic. Salmonella sp also may cause diarrhea in infants, but it is much less common than a viral etiology, especially in developed countries. References: American Academy of Pediatrics. Escherichia coli diarrhea (including hemolytic-uremic syndrome). In: Pickering LK, ed. Red Book: 2006 Report of the Committee on Infectious Diseases. 27th ed. Elk Grove Village, Ill: American Academy of Pediatrics; 2006:291-296 American Academy of Pediatrics. Rotavirus infections. In: Pickering LK, ed. Red Book: 2006 Report of the Committee on Infectious Diseases. 27th ed. Elk Grove Village, Ill: American Academy of Pediatrics; 2006:572-574 Ramaswamy K, Jacobson K. Infectious diarrhea in children. Gastroenterol Clin North Am. 2001;30:611-624. Abstract available at: http://www.ncbi.nlm.nih.gov/entrez/query.fcgi?orig_db=PubMed&db=PubMed&cmd=Search&ter m=%22Gastroenterology+clinics+of+North+America%22[Jour]+AND+611[page]+AND+2001[pda t] Thielman NM, Guerrant RL. Clinical practice. Acute infectious diarrhea. N Engl J Med. 2004;350:38-47

Copyright 2007 by the American Academy of Pediatrics

page 431

2007 PREP SA on CD-ROM Question: 207

A mother brings in her 2-year-old boy for food avoidance education after he was seen in the emergency department last week for anaphylaxis following peanut ingestion. This is the third anaphylactic episode after peanut ingestion since he was diagnosed with a peanut food allergy at age 1 year. At that time, he was evaluated by a pediatric allergist and determined to have positive skin tests and an elevated serum immunoglobulin (Ig) E to peanut antigen, consistent with an IgE-mediated allergy. The mother is frustrated and would like advice regarding peanut food allergy. Of the following, the BEST advice at this time is to

A. perform an oral challenge to determine the maximum peanut amount tolerated by the patient B. recommend that the patient avoid hot-pressed peanut oil C. recommend strict avoidance of all legumes (eg, peanuts, beans, soy) D. recommend strict avoidance of peanuts and tree nuts E. start daily antihistamine prophylaxis for accidental peanut ingestion

Copyright 2007 by the American Academy of Pediatrics

page 432

2007 PREP SA on CD-ROM Critique: 207 Preferred Response: D

Peanut (Arachis hypogaea) allergy is one of the most common food allergies in the United States. The initial approach to peanut food allergy is strict avoidance of all peanut food products and rapid administration of intramuscular epinephrine if a severe allergic reaction occurs. Although tree nuts are not related to the legume family (ie, peanuts, soy, beans), patients who have peanut food allergies should be counseled to avoid tree nuts for two reasons. First, approximately 25% to 50% of patients who are allergic to peanuts also are allergic to tree nuts. Second, tree nut foods may contain trace amounts of peanut because these foods frequently are processed in the same factory. Interestingly, children who have peanut food allergy rarely (approximately 5%) react to soy. Also, sunflower oil is tolerated in peanut-allergic patients. However, patients should be counseled to avoid unrefined (eg, cold-pressed, crude) and some gourmet peanut oils that may contain significant amounts of residual peanut protein. Studies have demonstrated that refined peanut oil (eg, hot-pressed) does not contain significant residual proteins that bind immunoglobulin (Ig) E from sera in peanut-allergic patients. Oral challenges typically are reserved to rule out unlikely foods that may cause a reaction or to determine if a person has outgrown a food allergy based on either history (eg, no longer experiencing a reaction after accidental ingestion), skin testing (eg, negative skin testing result), or serum IgE testing (eg, a peanut IgE <0.35 IU/mL has a strong negative predictive value). An oral challenge has been used only for research purposes to determine the maximum peanut tolerated in known allergic individuals; its use is strongly discouraged in any other scenario. Daily antihistamine therapy may lessen pruritus associated with an allergic reaction, but it is not recommended to prevent a reaction. In the past, peanut food allergy was believed to be lifelong, but newer studies suggest that up to 20% of previously sensitized individuals may outgrow their peanut food allergy. Allergies to tree nuts and shellfish are usually lifelong. References: Sampson HA, Leung DYM. Adverse reactions to foods. In: Behrman RE, Kliegman RM, Jenson HB, eds. Nelson Textbook of Pediatrics. 17th ed. Philadelphia, Pa: WB Saunders Co 2004:789792 Sampson HA. Clinical practice. Peanut allergy. N Engl J Med. 2002;346:1294-1299

Copyright 2007 by the American Academy of Pediatrics

page 433

2007 PREP SA on CD-ROM Question: 208

A 4-year-old child who has Duchenne muscular dystrophy requires operative intervention under general anesthesia for inguinal hernia repair. Of the following, the operative complication that is MORE likely to occur in this child than in a healthy child is

A. atlantoaxial rotary subluxation B. corneal damage C. malignant hyperthermia D. seizure E. surgical site infection

Copyright 2007 by the American Academy of Pediatrics

page 434

2007 PREP SA on CD-ROM Critique: 208 Preferred Response: C

Children who have neuromuscular disorders, including muscular dystrophy, myotonic dystrophy, and channelopathies such as hyperkalemic periodic paralysis, are at increased risk for complications during the perioperative period. These complications include apnea due to their underlying weakness, cardiac arrhythmias, and malignant hyperthermia (MH). The agents most commonly associated with these complications are depolarizing neuromuscular blocking agents such as succinylcholine and inhaled anesthetic agents such as halothane. MH is characterized by muscle rigidity and acute muscle necrosis. Affected patients develop a hypermetabolic state leading to metabolic acidosis, hyperthermia, cardiac arrhythmia, markedly elevated creatine kinase concentrations, and myoglobinuria. This autosomal dominant condition is caused by excessive release of intracellular calcium by the sarcoplasmic reticulum. Several genetic mutations have been described that result in MH, but the common feature is a lower threshold for calcium release. The standard treatment is the muscle relaxer dantrolene, which decreases calcium release by the sarcoplasmic reticulum. Preoperative use of dantrolene is suggested for patients at high risk for MH. Atlantoaxial rotary subluxation, corneal damage, seizure, and surgical site infection are no more common in children who have Duchenne muscular dystrophy than in the general population. References: Bertorini TE. Perisurgical management of patients with neuromuscular disorders. Neurol Clin. 2004;22:293.313. Abstract available at: http://www.ncbi.nlm.nih.gov/entrez/query.fcgi?orig_db=PubMed&db=PubMed&cmd=Search&ter m=Neurol+Clin[Jour]+AND+2004[pdat]+AND+Bertorini+TE[author] Klingler W, Lehmann-Horn F, Jurkat-Rott K. Complications of anaesthesia in neuromuscular disorders. Neuromusc Disord. 2005;15:195-206. Abstract available at: http://www.ncbi.nlm.nih.gov/entrez/query.fcgi?orig_db=PubMed&db=PubMed&cmd=Search&ter m=Neuromusc+Dis[Jour]+AND+195[page]+AND+2005[pdat]

Copyright 2007 by the American Academy of Pediatrics

page 435

2007 PREP SA on CD-ROM Question: 209

Several weeks ago, a boy presented with fever, malaise, headache, and a skin lesion. The lesion began as a red papule and slowly enlarged to form a large annular ring with a flat erythematous border (Item Q209A). Today he complains of recurrent joint pains that are particularly severe in his knees. Of the following, the MOST likely diagnosis is

A. Epstein-Barr virus infection B. human parvovirus B19 infection C. Lyme disease D. juvenile rheumatoid arthritis E. Rocky Mountain spotted fever

Copyright 2007 by the American Academy of Pediatrics

page 436

2007 PREP SA on CD-ROM Critique: 209 Preferred Response: C

The earliest manifestation of Lyme disease is the unique skin eruption, erythema chronicum migrans (ECM). ECM appears 1 to 3 weeks after the bite of a tick that is carrying the spirochete Borrelia burgdorferi. A red papule erupts at the site of the tick bite and slowly progresses over several weeks to form a large, annular, erythematous lesion (Item C209A) that exceeds 5 cm in diameter, as described for the boy in the vignette. The lesion has a flat red border and central clearing. Untreated ECM appears to resolve spontaneously, but multiple lesions may appear 3 to 5 weeks after the tick bite as a manifestation of early disseminated disease. Other manifestations of this stage of infection include cranial nerve palsies, meningitis, conjunctivitis, arthritis, headaches, fatigue, myalgias, and low-grade fever. These symptoms reflect spirochetemia. Late disease is characterized most commonly by recurrent arthritis that usually is pauciarticular and affects the large joints, particularly the knees. Arthritis may occur without a history of earlier stages of illness. Recurrent rash and joint symptoms have resulted in confusion of Lyme disease with juvenile rheumatoid arthritis. Early Lyme disease (eg, when erythema chronicum migrans is present) is diagnosed clinically. Although serologic testing is available, immunoglobulin (Ig) M-specific antibody peaks 3 to 6 weeks and IgG antibody weeks to months after infection. Because erythema chronicum migrans appears 1 to 2 weeks after a tick bite, results of serologic testing at this time may be negative. Diagnosis may be difficult in patients who have later stages of the disease and should be based on clinical findings and the results of serologic testing. Infectious mononucleosis caused by Epstein-Barr virus infection) may be accompanied by a rash that is pink and morbilliform. If a patient is treated with amoxicillin, a bright red, generalized morbilliform eruption (Item C209B) often occurs. Cutaneous findings in human parvovirus B19 infection are the classic erythematous slapped-cheek appearance (Item C209C) followed by a lacylike rash (Item C209D) on proximal extremities. Systemic juvenile rheumatoid arthritis is characterized by the appearance of a salmon-pink macular eruption (Item C209E) that transiently appears in association with fever spikes. Cutaneous manifestations of Rocky Mountain spotted fever typically consist of a maculopapular, petechial rash (Item C209F) that usually begins on the wrists and ankles and spreads within hours to the trunk. Palms and soles typically are involved. References: American Academy of Pediatrics. Lyme disease (Borrelia burgdorferi infection). In Pickering LK, ed. Red Book: 2006 Report of the Committee on Infectious Diseases. 27th ed. Elk Grove Village, Ill: American Academy of Pediatrics; 2006:428-433 Weston WL, Lane AT, Morelli JG. Bacterial infections (pyodermas) and spirochetal infections of the skin: spirochetal diseases. In: Color Textbook of Pediatric Dermatology. 3rd ed. St. Louis, Mo: Mosby; 2002:56-57 Bodemer C. Epstein-Barr virus (EBV) infections. In: Schachner LA, Hansen RC, eds. Pediatric Dermatology. 3rd ed. St. Louis, Mo: Mosby; 2003:1071-1076 Miller ML, Cassidy JT. Juvenile rheumatoid arthritis. In: Behrman RE, Kliegman RM, Jenson HB, eds. Nelson Textbook of Pediatrics. 17th ed. Philadelphia, Pa: WB Saunders Co; 2004:799-805 American Academy of Pediatrics. Epstein-Barr virus infections (infectious mononucleosis). In Pickering LK, ed. Red Book: 2006 Report of the Committee on Infectious Diseases. 27th ed. Elk Grove Village, Ill: American Academy of Pediatrics; 2006:286-288 American Academy of Pediatrics. Rocky Mountain spotted fever. In Pickering LK, ed. Red Book: 2006 Report of the Committee on Infectious Diseases. 27th ed. Elk Grove Village, IL: American Academy of Pediatrics; 2006:570-572

Copyright 2007 by the American Academy of Pediatrics

page 437

2007 PREP SA on CD-ROM

Galen W. Rickettsial infection. In: Schachner LA, Hansen RC, eds. Pediatric Dermatology. 3rd ed. St. Louis, Mo: Mosby; 2003:1025-1030 Weston WL, Lane AT, Morelli JG. Viral infections: viral exanthems: the morbilliform eruptions. In: Color Textbook of Pediatric Dermatology. 3rd ed. St. Louis, Mo: Mosby; 2002:89-100

Copyright 2007 by the American Academy of Pediatrics

page 438

2007 PREP SA on CD-ROM Question: 210

The family of a diabetic patient in your practice requests evaluation for celiac disease. They have heard from other families of children who have diabetes that patients who have type 1 diabetes are at increased risk for this condition. Of the following, a TRUE statement regarding celiac disease screening is that

A. empiric gluten withdrawal is the diagnostic test of choice B. initial screening should include serum immunoglobulin A (IgA) and tissue transglutaminase
antibody

C. patients who have selective IgA deficiency have a lower rate of celiac disease than the
general population

D. the most specific antibody test for celiac screening is the antigliadin IgG antibody E. the prevalence of celiac disease in children who have type 1 diabetes mellitus is greater than
10%

Copyright 2007 by the American Academy of Pediatrics

page 439

2007 PREP SA on CD-ROM Critique: 210 Preferred Response: B

The patient described in the vignette has type 1 diabetes mellitus, and it is now recognized that such patients are at increased risk of celiac disease (nontropical sprue, gluten-sensitive enteropathy). The prevalence of celiac disease in children who have diabetes is approximately 3% to 4% in those screened at diabetes onset, and the overall prevalence is approximately 6%. Early diagnosis of celiac disease (sprue) has the potential to reduce later complications of untreated sprue, such as growth failure, osteopenia, and intestinal lymphoma. For these reasons, screening for celiac disease should be considered for all patients who have type 1 diabetes mellitus. Other risk factors for celiac disease, and in which screening should be considered, include Down syndrome, chronic lymphocytic (Hashimoto) thyroiditis, selective immunoglobulin (Ig)A deficiency, and a first-degree relative who has celiac disease. The classic signs and symptoms of celiac disease are age-dependent. Toddlers present with abdominal distention, diarrhea, irritability, and failure to thrive. In contrast, older children and adolescents more commonly present with abdominal pain, anemia, growth attenuation, and arthritis. Currently, however, many high-risk but asymptomatic children are detected by serologic screening. Because celiac disease is a lifelong condition that requires permanent dietary restriction, empiric withdrawal of gluten from the diet without a definitive diagnosis is not appropriate. The initial serologic screening test suggested by the North American Society for Pediatric Gastroenterology and Nutrition is the measurement of serum tissue transglutaminase IgA (TTG) antibody, in conjunction with a measurement of total serum IgA. The antibody screening test has a sensitivity of greater than 95%, although it may have false-negative results in patients who are IgA-deficient. The IgA antiendomysial antibody test also has excellent sensitivity and specificity, but it is more expensive and labor-intensive. Tests for antibodies to gliadin (IgG and IgA) and reticulin are also available, but because they have a large number of false-positive results (ie, poor specificity), they should be used only under selected circumstances. Currently, celiac disease cannot be diagnosed definitively by serology alone. Therefore, a patient who has a positive TTG antibody test result should have the diagnosis verified through endoscopy and duodenal biopsy. In celiac disease, the duodenal biopsy demonstrates varying degrees of inflammation and villous atrophy. Patients whose duodenal biopsy results are consistent with celiac disease should be placed on a lifelong gluten-free diet. In most patients, compliance can be documented by return of the tissue transglutaminase antibody levels to normal within 6 to 12 months, and follow-up biopsy is usually not necessary. References: Barera G, Bonfanti R, Viscardi M, et al. Occurrence of celiac disease after onset of type 1 diabetes: a 6 year prospective longitudinal study. Pediatrics. 2002;109:833-838. Available at: http://pediatrics.aappublications.org/cgi/content/full/109/5/833 Farrell RJ, Kelly CP. Celiac sprue. N Engl J Med. 2002;346:180-188. Available at: http://content.nejm.org/cgi/content/extract/346/3/180 Hill ID, Dirks MH, Liptak GS, et al. Guideline for the diagnosis and treatment of celiac disease in children: recommendations of the North American Society for Pediatric Gastroenterology, Hepatology and Nutrition. J Pediatr Gastroenterol Nutr. 2005;40:1-19. Available at: http://www.jpgn.org/pt/re/jpgn/abstract.00005176-20050100000001.htm;jsessionid=G1XZJHD0FWF6q3h0vtvs5tQTH2PFpGn5Z7s5YBXrxTQtRcWslPJl!194 1873617!-949856145!8091!1?index=1&database=ppvovft&results=1&count=10&searchid=2&nav=search

Copyright 2007 by the American Academy of Pediatrics

page 440

2007 PREP SA on CD-ROM Question: 211

A newborn in your neonatal intensive care unit has had intermittent seizures for 72 hours. You have been unable to control the seizures with phenobarbital, hydantoin, and lorazepam. Electrolyte, calcium, magnesium, and glucose concentrations are normal. The infant subsequently becomes apneic, comatose, and unresponsive. Of the following, the BEST laboratory test to determine the cause of coma in this infant is

A. analysis of whole blood lead concentration B. evaluation of urine for reducing substances C. measurement of serum amino acids, organic acids, lactate, and ammonia D. measurement of serum cortisol, thyroxine, and thyroid-stimulating hormone E. measurement of very long-chain fatty acids

Copyright 2007 by the American Academy of Pediatrics

page 441

2007 PREP SA on CD-ROM Critique: 211 Preferred Response: C

Seizures in the neonatal period often are due to hypoxic-ischemic encephalopathy or intracranial pathology such as malformation or hemorrhage. Most are controlled readily with a single or, possibly, two anticonvulsant therapies. The patient described in the vignette has refractory seizures that have not responded to three anticonvulsants, and the infants condition progresses to coma. This presentation is not uncommon in metabolic diseases, where seizures may be focal or multifocal clonic jerks or generalized and tonic. Such events typically occur after the first 48 postnatal hours. Because transient abnormalities in glucose and electrolytes are not present in this patient, inborn errors of metabolism need to be ruled out. A categorical approach to the laboratory evaluation of the comatose newborn or young infant is helpful. Toxins and drugs can be evaluated by history and blood, urine, or meconium toxicology screens. Intracranial pathology can be evaluated by ultrasonography or computed tomography scan. Hypoxia-ischemia mechanisms can be discerned by history, blood gas analysis, evidence of multiple organ system dysfunction, the neurologic examination, and the timing of the onset of seizures (generally in the first 24 hours of postnatal life). Infection may be systemic or focal (central nervous system) and can be evaluated by cultures, antigen testing, and use of specific viral polymerase chain reaction tests of the blood, urine, and cerebrospinal fluid. The diagnosis of inborn errors of metabolism requires consideration of broad categories of metabolism: problems with amino acid metabolism that produce elevated serum concentrations of certain amino acids (eg, phenylketonuria with excessive serum phenylalanine and urinary excretion of phenylalanine and its metabolites); carbohydrate metabolism that may result in hypoglycemia, organic acidemias, hyperammonemia, and acidurias; and problems in fatty acid metabolism that result in hypoglycemia with corresponding hypoketosis and often elevated serum transaminases and ammonia concentrations. For these reasons, the evaluation should include measurement of serum electrolytes, glucose, lactic acid, ammonia, and amino acids as well as organic acids from the serum and urine. Lead toxicity is a consideration for coma in the older infant, toddler, or child exposed to environmental lead. Measurement of urinary reducing substances may be helpful in galactosemia, but is insufficient when evaluating encephalopathy and coma in the newborn or young infant in whom an inborn error of metabolism is being ruled out. Serum cortisol, thyroxine, and thyroid-stimulating hormone may be important in evaluating congenital hypotonia, persistent hypoglycemia, or suspected panhypopituitarism. Total and free carnitine and acylcarnitine profile results are abnormal in infants who have fatty acid oxidation defects. Affected infants present with hypoglycemia, cardiomyopathy, and urinary hypoketosis. References: Burton BK. Inborn errors of metabolism in infancy: a guide to diagnosis. Pediatrics. 1998;102:e69. Available at: http://pediatrics.aappublications.org/cgi/content/full/102/6/e69 Enns GM, Packman S. Diagnosing inborn errors of metabolism in the newborn: clinical features. NeoReviews. 2001;2:e183-e191. Available at: http://neoreviews.aappublications.org/cgi/content/full/2/8/e183 Lemley KV, Hintz SR, Enns GM. Continuous renal replacement therapy in the initial management of neonatal hyperammonemia due to urea cycle defects. NeoReviews. 2000;1:e173-e179. Available at: http://neoreviews.aappublications.org/cgi/content/full/1/9/e173 Menkes JH, Wilcox WR. Inherited metabolic diseases of the nervous system. In: Menkes JH, Sarnat HB, Maria BL, eds. Child Neurology. 7th ed. Philadelphia, Pa: Lippincott Williams & Wilkins; 2000:29-142 Zinn AB. Inborn errors of metabolism. In: Martin RJ, Fanaroff AA, Walsh MC, eds. Fanaroff and Martins Neonatal-Perinatal Medicine: Diseases of the Fetus and Infant. 8th ed. Philadelphia, Pa:

Copyright 2007 by the American Academy of Pediatrics

page 442

2007 PREP SA on CD-ROM

Mosby-Elsevier; 2006:1597-1658

Copyright 2007 by the American Academy of Pediatrics

page 443

2007 PREP SA on CD-ROM Question: 212

An 18-month-old girl is brought to your clinic because her mother feels she is pale. She has no relevant findings on past medical history and eats a regular diet. She is alert and interactive but has significant pallor. The remainder of the physical examination results are normal. A complete blood count reveals a normal white blood cell count and platelet count. The hemoglobin concentration is 4.5 g/dL (45 g/L) and mean cell volume is 74 fL. Of the following, the MOST likely diagnosis is

A. acute lymphoblastic leukemia B. Diamond-Blackfan anemia C. glucose-6-phosphate dehydrogenase deficiency D. iron deficiency anemia E. transient erythroblastopenia of childhood

Copyright 2007 by the American Academy of Pediatrics

page 444

2007 PREP SA on CD-ROM Critique: 212 Preferred Response: E

The previously healthy child described in the vignette presents with severe, normocytic anemia and otherwise normal findings, features that are most consistent with transient erythroblastopenia of childhood (TEC). TEC is a disease of unknown cause that results in transient failure of the erythroid cell line. A history of preceding viral illness may be elicited. The average age of affected children is 2 years; the condition rarely occurs in the first year after birth. Findings on the physical examination usually are normal, except for marked pallor and other signs of anemia, such as tachycardia, tachypnea, and fatigue. Laboratory findings include normocytic anemia and reticulocytopenia. Red cell adenosine deaminase activity is normal, and i antigen expression can be normal or increased. Treatment consists of erythrocyte transfusion for children who have hemoglobin values less than 5 g/dL (50 g/L). Those who have hemoglobin levels higher than 5 g/dL (50 g/L) may recover without transfusion. The prognosis is excellent. Diamond-Blackfan anemia is caused by insensitivity to erythropoietin and usually presents in infants younger than 1 year of age with pallor and fatigue. Other features, such as facial abnormalities, thumb abnormalities, and congenital heart disease, are present in up to 25% of affected patients. Laboratory findings include macrocytic anemia and reticulocytopenia. Red cell adenosine deaminase activity and i antigen expression are increased. Iron deficiency causes a microcytic, hypochromic anemia. Children who have acute lymphoblastic leukemia rarely present with isolated anemia, and other findings, such as hepatosplenomegaly, lymphadenopathy, neutropenia, neutrophilia, or thrombocytopenia, are expected. Glucose-6-phosphate dehydrogenase deficiency causes the acute onset of pallor with jaundice, which is not described in the child in the vignette. References: Segel GB, Hirsh MG, Feig SA. Managing anemia in pediatric office practice: part 1. Pediatr Rev. 2002;23:75-84. http://pedsinreview.aappublications.org/cgi/content/full/23/4/111 Segel GB, Hirsch MG, Feig SA. Managing anemia in pediatric office practice: part 2. Pediatr Rev. 2002;23:111-121. Available at: http://pedsinreview.aappublications.org/cgi/content/full/23/4/111

Copyright 2007 by the American Academy of Pediatrics

page 445

2007 PREP SA on CD-ROM Question: 213

A 12-year-old girl presents to the emergency department with abdominal pain. Her parents report that she awakened with a temperature of 101F (38.4C) this morning, complained of abdominal pain, and has vomited twice. There is no diarrhea, and there are no sick contacts. She reports nausea and no interest in food or drink. She was previously entirely healthy. Urinalysis reveals a specific gravity of 1.015 and is otherwise normal. Of the following, the finding that MOST indicates the need for immediate surgical intervention is

A. abdominal distention B. hyperactive bowel sounds C. pain in the right lower quadrant D. rigidity of the abdominal wall E. voluntary guarding

Copyright 2007 by the American Academy of Pediatrics

page 446

2007 PREP SA on CD-ROM Critique: 213 Preferred Response: D

The differential diagnosis of abdominal pain in children is an almost daily challenge for the pediatrician. Young children are especially challenging because they cannot verbally report many of the symptoms elicited in history-taking or even respond to questions regarding palpation of the abdomen. The approach to the abdominal examination should elicit as much patient cooperation as possible and prevent crying and movement, which may confuse findings. Examination findings such as tenderness may occur in children who have both nonsurgical abdominal complaints and peritoneal inflammation. The presence of bowel sounds is usually reassuring. Exquisite tenderness elicited by walking, buttoning pants, and climbing on the examination table strongly suggests a surgical abdomen. Distention with tenderness, rigidity of the abdominal wall to palpation, and rebound tenderness are more specific signs of peritoneal irritation and, therefore, a surgical abdomen. Simple abdominal distention may occur with constipation, gaseous distention, or pregnancy. Hyperactive bowel sounds usually suggest increased peristalsis against a bowel obstruction or gastroenteritis and are not generally present in peritonitis. Pain in the right lower quadrant is suggestive when other signs of peritonitis are present, but it also may occur with ovarian cyst or torsion, gastroenteritis, constipation, pneumonia, and other disease processes. Voluntary guarding is common on examination, even in children and adults who have no abdominal pain. References: Ashcraft KW. Consultation with the specialist: acute abdominal pain. Pediatr Rev. 2000;21:363367. Available at: http://pedsinreview.aappublications.org/cgi/content/full/21/11/363 Pegoli W. Appendicitis. In: McMIllan JA, DeAngelis CD, Feigin RD, Warshaw JB, eds. Oskis Pediatrics: Principles and Practice. 3rd ed. Philadelphia, Pa: Lippincott, Williams & Wilkins; 1999:1702-1703

Copyright 2007 by the American Academy of Pediatrics

page 447

2007 PREP SA on CD-ROM Question: 214

During clinic rounds with medical students, you are seeing a 2-year-old girl who has otitis media. One of the students asks about the potential complications of otitis media. Of the following, the statement you are MOST likely to make is that

A. cholesteatoma is more common when chronic otitis media with effusion is treated with
tympanostomy tubes rather than no tubes

B. ear drainage is an uncommon complication after myringotomy and tympanostomy tube


insertion

C. intracranial lateral sinus thrombosis occurs in 8% of ear infections, especially with amoxicillinresistant bacteria

D. sensorineural hearing loss is prevented best by placement of tympanostomy tubes E. warning symptoms and signs of intracranial complication include vomiting and blurred vision

Copyright 2007 by the American Academy of Pediatrics

page 448

2007 PREP SA on CD-ROM Critique: 214 Preferred Response: E

Since the advent of antibiotics, intracranial complications of otitis media have become increasingly uncommon, but the incidence of infratemporal complications of the middle ear, mastoid air space, and adjacent structures remains unchanged. Hearing loss is the most common complication. Although tympanostomy tubes often are placed for chronic otitis media, whether they prevent hearing loss remains controversial. Other complications of otitis media include perforation of the tympanic membrane, otorrhea, and mastoiditis. Chronic infection may lead to cholesteatoma, and tympanostomy prevents this complication. Ear drainage is common after tympanostomy and myringotomy. Intracranial suppurative complications of otitis media, such as meningitis, lateral sinus thrombosis, epidural and subdural abscess or empyema, and otitic hydrocephalus resembling pseudotumor cerebri, now occur in far less than 1% of children. The affected child may present with fever, irritability, malaise, and signs of increased intracranial pressure, such as headache, vomiting, diplopia, or papilledema. A focal seizure also may occur. References: Hendley JO. Clinical practice: otitis media. N Engl J Med. 2002;347:1169-1174. Available at: http://content.nejm.org/cgi/content/extract/347/15/1169 Paradise JL. Otitis media. In: Behrman RE, Kliegman RM, Jenson HB, eds. Nelson Textbook of Pediatrics. 17th ed. Philadelphia, Pa: WB Saunders Co; 2004:2138-2148

Copyright 2007 by the American Academy of Pediatrics

page 449

2007 PREP SA on CD-ROM Question: 215

You receive a call from a teenage patient, who tells you that she is having palpitations and feels somewhat lightheaded. You refer her to your local emergency department, where no symptoms are discerned and physical examination findings are normal. The emergency department sends a copy of the patient's electrocardiogram by facsimile to your office for you to review (Item Q215A). Of the following, the BEST interpretation of the girl's electrocardiogram is

A. premature atrial contractions B. premature ventricular contractions C. supraventricular tachycardia D. ventricular tachycardia E. Wolff-Parkinson-White syndrome

Copyright 2007 by the American Academy of Pediatrics

page 450

2007 PREP SA on CD-ROM Critique: 215 Preferred Response: B

The myocardium is an electrical generator that propagates electrical impulses throughout its tissue. In the healthy heart, the sinus node, situated in the right atrium, initiates the cardiac impulse with which depolarization causes the atrium to contract. This is represented as a P wave on the surface electrocardiogram (ECG). The impulse travels to the atrioventricular node (AVN), where it is slightly delayed before it is passed along to the His Purkinje system. The PR interval represents the time of delay of the AVN. When the impulse reaches ventricular myocardium, ventricular contraction occurs, which appears as the QRS complex on surface ECG. The process completes as the ventricles repolarize and the T wave is seen on surface ECG. When the atria depolarize early and from tissue outside of the sinus node, a premature atrial contraction is seen on surface ECG. When such atrial activity is not associated with a ventricular contraction, it is referred to as a blocked P wave. When ventricular cells depolarize spontaneously, early, and in isolation, a widened QRS complex appears on surface ECG. This is referred to as a premature ventricular contraction (Item C215A) and is not related to a preceding P wave. More than two of these ventricular beats occurring in a row at a rate that is faster than the sinus rate represents ventricular tachycardia (Item C215B). This rhythm can be deleterious and may lead to cardiovascular collapse if it is sustained. Supraventricular tachycardia (Item C215C) describes an abnormal heart rhythm that originates in the atrium. Most often it is a narrow QRS complex rhythm, often with rates between 240 and 300 beats/min. Wolff-Parkinson-White syndrome (Item C215D) is characterized by a complex of findings on the surface ECG, including a shortened PR interval and ventricular pre-excitation (early depolarization of the QRS complex) with a delta wave. The ECG for the girl in the vignette reveals a sinus rhythm, with a single wide QRS complex that occurs in the absence of a P wave and is early compared with the preceding heart rate. This is interpreted as premature ventricular contractions. References: Kaltman J, Shah M. Evaluation of the child with an arrhythmia. Pediatr Clin North Am. 2004;51:1537-1551. Abstract available at: http://www.ncbi.nlm.nih.gov/entrez/query.fcgi?orig_db=PubMed&db=PubMed&cmd=Search&ter m=%22Pediatric+clinics+of+North+America%22[Jour]+AND+1537[page]+AND+2004[pdat] Liebman J. Electrocardiography. In: Moller JH, Hoffman JIE, eds. Pediatric Cardiovascular Medicine. Philadelphia, Pa: Churchill Livingston; 2000:111-141

Copyright 2007 by the American Academy of Pediatrics

page 451

2007 PREP SA on CD-ROM Question: 216

A newborn female has a cardiac murmur. Before the cardiologist arrives to evaluate her, she has a seizure. Results of laboratory testing include a serum calcium concentration of 5.0 mg/dL (1.25 mmol/L). Subsequently, echocardiography reveals an aortic arch anomaly. Of the following, the MOST appropriate test to obtain to aid in the diagnosis of this infant is

A. brainstem auditory evoked potentials B. electroencephalography C. fluorescent in situ hybridization analysis of chromosome 22 D. peripheral blood chromosome analysis E. thyroid function testing

Copyright 2007 by the American Academy of Pediatrics

page 452

2007 PREP SA on CD-ROM Critique: 216 Preferred Response: C

The infant described in the vignette has a number of features of 22q11 deletion syndrome, which includes most infants affected by DiGeorge syndrome (80% to 90%) and velocardiofacial syndrome (95%). DiGeorge syndrome results from abnormal cervical neural crest migration into the derivatives of the third and fourth pharyngeal arches and pouches during early embryogenesis. The pattern of malformations includes hypoplasia or aplasia of the thymus and parathyroid glands and structural abnormalities of the great vessels. The defects lead to the classic clinical manifestations of deficient cellular immunity due to T-cell dysfunction, causing increased susceptibility to infection; hypocalcemia due to absent parathyroid hormone, resulting in hypocalcemic seizures; and aortic arch abnormalities. Some affected infants also have dysmorphic facial features, which can include short palpebral fissures, short philtrum, micrognathia, and ear abnormalities. Most patients who have DiGeorge syndrome are found to have partial monosomy for the proximal long arm of chromosome 22 due to microdeletion of 22q11.2. This microdeletion can be detected by fluorescent in situ hybridization (FISH) using a molecular probe specific for the region. Although the deletion occasionally is evident on routine chromosome analysis, the FISH study is much more sensitive. Most cases represent de novo deletion events, although familial cases have been described. The 22q11.2 deletion is associated with a variety of other phenotypes, including the velocardiofacial syndrome, conotruncal anomaly-face syndrome, and others. Velocardiofacial syndrome denotes patients who have the features of DiGeorge syndrome plus craniofacial (unusual palpebral fissures, broad nasal root, narrow upper lip) and palatal (cleft uvula, submucous cleft palate, cleft palate) abnormalities. Of interest, all of these phenotypes can be evident in a single family in which a heritable chromosome 22 deletion is present. It has been suggested that this constellation of related syndromes that have the common etiologic factor of a chromosome 22q11.2 deletion should be termed "22q11 deletion syndrome." Because deafness is not a major feature of DiGeorge syndrome, obtaining brainstem auditory evoked potentials is not indicated at this time and will not aid in diagnosis. The cause of the seizure in the infant described in the vignette is hypocalcemia, and electroencephalography would not provide additional useful information for management, which should include administration of calcium. Thyroid function usually is preserved in DiGeorge syndrome. References: American Academy of Pediatrics Committee on Genetics. Molecular genetic testing in pediatric practice: a subject review. Pediatrics. 2000;106:1494-1497. Available at: http://pediatrics.aappublications.org/cgi/content/full/106/6/1494 Beck AE, Hudgins L. Congenital cardiac malformations in the neonate: isolated or syndromic? NeoReviews. 2003;4:e105. Available at: http://neoreviews.aappublications.org/cgi/content/full/4/4/e105 Jones KL. Deletion 22q11.2 syndrome. In: Smith's Recognizable Patterns of Human Malformation. 6th ed. Philadelphia, Pa: Elsevier Saunders Co; 2006:298-301 Lin RJ, Cherry AM, Bangs CD, Hoyme HE. FISHing for answers: the use of molecular cytogenetic techniques in neonatology. NeoReviews. 2003;4:e94. Available at: http://neoreviews.aappublications.org/cgi/content/full/4/4/e94

Copyright 2007 by the American Academy of Pediatrics

page 453

2007 PREP SA on CD-ROM Question: 217

An 18-year-old girl presents for a health supervision visit prior to leaving for college. She has a letter from the college she plans to attend instructing entering freshman who choose to live in the dormitories to obtain a meningococcal vaccination. You review vaccine information and obtain consent for the immunization. Of the following, an ACCURATE statement about recommendations for the use of the meningococcal MCV4 vaccine is that it

A. is administered subcutaneously as a single dose B. protects against serogroup B C. requires revaccination every 3 to 5 years D. should be administered routinely to 11- to 12-year-olds E. should be administered to high-risk children older than age 2 years

Copyright 2007 by the American Academy of Pediatrics

page 454

2007 PREP SA on CD-ROM Critique: 217 Preferred Response: D

In 2005, the American Academy of Pediatrics (AAP) issued recommendations for use of meningococcal vaccines. Currently, there are two available meningococcal vaccines: MSPSV4, (Menomune-A/C/Y/W-135; Sanofi Pasteur, Swiftwater, Pa.) licensed for use in children older than 2 years, and MCV4, (Menactra; Sanofi Pasteur), licensed for use in ages 11 to 55 years. Both vaccines contain capsular polysaccharides from Neisseria meningitidis serogroups A, C, Y, and W-135. MSPSV4 contains 50 mg of each serogroup, and MCV4 contains 4 mg of each serogroup conjugated to 48 mg of diphtheria toxoid. Conjugation changes the expression of the immune response from T cell-independent to T cell-dependent, providing an improved primary response and a strong response at re-exposure. Thus, the conjugated vaccine MCV4 provides potential clinical advantages over MSPSV4. In addition to the longer-lasting immunity, MCV4 reduces nasopharyngeal carriage, which should create a herd immunity effect. Neither vaccine provides protection against serogroup B, a more common cause of disease in infants than adolescents in the United States. The AAP recommendations (2005) for meningococcal vaccine use include the administration of MCV4 for: Routine immunization at the 11- to 12-year-old health supervision visit Immunization of those who did not previously receive MCV4 at two points: high school entry (age 15 years) and college entry for those who will live in dormitories Children 11 years or older who are at increased risk, including those who have terminal complement deficiency, who have anatomic or functional asplenia, or who travel to countries on the Centers for Disease Control and Prevention (CDC) advisory list for the vaccine The possible reimmunization of adolescents 3 to 5 years after immunization with the MPSV4 vaccine if they remain at increased risk for the disease MCV4 is administered intramuscularly; MPSV4 is administered subcutaneously. There are no recommendations to revaccinate children every 3 to 5 years; the duration of protection with MCV4 currently is not known but is expected to be longer than 3 years. Information from MCV4 evaluation studies will guide future revaccination policies. Although clinical trials of MCV4 are ongoing in young children, MPSV4 is recommended for use in high-risk children from 2 to 10 years of age. Updates on meningococcal vaccination recommendations can be found on the AAP and CDC Web sites. References: Centers for Disease Control and Prevention. Prevention and control of meningococcal disease: recommendations of the Advisory Committee on Immunization Practices (ACIP). Morbid Mortal Wkly Rep MMWR. 2005;54(RR-7):1-21. Available at: http://www.cdc.gov/mmwr/pdf/rr/rr5407.pdf Committee on Infectious Diseases. Prevention and control of meningococcal disease: recommendations for use of meningococcal vaccines in pediatric patients. Pediatrics. 2005;116:496-505. Available at: http://pediatrics.aappublications.org/cgi/content/full/116/2/496

Copyright 2007 by the American Academy of Pediatrics

page 455

2007 PREP SA on CD-ROM Question: 218

A 15-year-old girl who is hirsute has had menses six times in the past year. She is overweight (body mass index of 35 kg/M2) and has a blood pressure of 110/75 mm Hg. Her cholesterol concentration is 170 mg/dL (4.4 mmol/L), with a high-density lipoprotein cholesterol concentration of 55 mg/dL (1.4 mmol/L) and fasting triglyceride value of 74 mg/dL (0.84 mmol/L). Her fasting blood glucose value is 108 mg/dL (6.0 mmol/L), and a 2-hour post-oral glucose blood glucose value is 115 mg/dL (6.4 mmol/L). Of the following, the MOST likely diagnosis for this patient is

A. idiopathic hirsutism B. impaired glucose tolerance C. Cushing syndrome D. metabolic syndrome E. polycystic ovary syndrome

Copyright 2007 by the American Academy of Pediatrics

page 456

2007 PREP SA on CD-ROM Critique: 218 Preferred Response: E

The diagnosis of metabolic syndrome currently is undergoing re-evaluation. There are presently five consensus criteria for this diagnosis in adults and no formal consensus criteria for the diagnosis in childhood. Adult criteria include: variable combinations of obesity, increased waist circumference, hypertension, elevated fasting triglycerides, and low high-density lipoprotein cholesterol as well as impaired glucose tolerance or diabetes and evidence of insulin resistance. Although polycystic ovary syndrome (PCOS) may be seen in women who have this constellation of findings, it is not part of the diagnostic criteria in adults. Age- and sex-specific criteria for these different components have been adapted by various investigators for use in children and adolescents. The diagnosis of PCOS is clinical, based on two of the following three findings: evidence of hyperandrogenism or measured elevated androgen levels, irregular menses or amenorrhea, and the presence of polycystic ovaries on ultrasonography, in the absence of other defined causes, such as congenital adrenal hyperplasia or other androgenic disorders. Generally, girls who have PCOS also have hyperinsulinism and are obese. Many affected girls have components of the metabolic syndrome, including hypertension, obesity, high fasting triglyceride concentrations, low high-density lipoprotein values, and glucose intolerance or frank type 2 diabetes. The girl described in the vignette is overweight, but has normal blood pressure and lipid values. Although her fasting blood glucose is slightly high, she has normal glucose tolerance. Therefore, she does not meet the adult criteria for metabolic syndrome. Because she has irregular menses, she cannot be said to have idiopathic hirsutism. Although Cushing syndrome can be suspected in a girl who has oligomenorrhea, obesity, and hirsutism, it is a very uncommon disorder in adolescents. Statistically, PCOS is the most common disorder associated with the findings reported for the girl. A 24-hour urine free cortisol, overnight dexamethasone-suppressed serum cortisol, or an evening salivary cortisol measurement in the normal range could rule out Cushing syndrome. References: Coviello AD, Legro RS, Dunaif A. Adolescent girls with polycystic ovary syndrome have an increased risk of the metabolic syndrome associated with increasing androgen levels independent of obesity and insulin resistance. J Clin Endocrinol Metab. 2006;91:492-497. Abstract available at: http://www.ncbi.nlm.nih.gov/entrez/query.fcgi?cmd=Retrieve&db=pubmed&dopt=Abstract&list_ui ds=16249280&query_hl=79&itool=pubmed_docsum Janssen I, Katzmarzyk PT, Srinivasan SR, et al. Combined influence of body mass index and waist circumference on coronary artery disease risk factors among children and adolescents. Pediatrics. 2005;115:1623-1630. Available at: http://pediatrics.aappublications.org/cgi/content/full/115/6/1623 Kahn R, Buse J, Ferrannini E, Stern M; American Diabetes Association; European Association for the Study of Diabetes. The metabolic syndrome: time for a critical appraisal: joint statement from the American Diabetes Association and the European Association for the Study of Diabetes. Diabetes Care. 2005;28:2289-2304. Available at: http://care.diabetesjournals.org/cgi/content/full/28/9/2289 Palmert MR, Gordon CM, Kartashov AI, Legro RS, Emans SJ, Dunaif A. Screening for abnormal glucose tolerance in adolescents with polycystic ovary syndrome. J Clin Endocrinol Metab. 2002;87:1017-1023. Available at: http://jcem.endojournals.org/cgi/content/full/87/3/1017

Copyright 2007 by the American Academy of Pediatrics

page 457

2007 PREP SA on CD-ROM Question: 219

You are seeing a 6-year-old girl whose mother is concerned about the girl masturbating. For the past several months, the girl has touched her genitals while watching television with her mother. The parents have been divorced for 2 years, and the girl spends weekends with her father. You question her mother further. Of the following, the response that MOST increases your suspicion for behavior that is out of the norm is

A. a history of urinary tract infection as an infant B. play-acting intercourse C. the practice of taking bubble baths D. shyness with her mother in the bathroom E. wearing of tight jeans

Copyright 2007 by the American Academy of Pediatrics

page 458

2007 PREP SA on CD-ROM Critique: 219 Preferred Response: B

Masturbation begins in infancy and is a nearly universal activity. Parents may be encouraged to counsel their children on keeping the behavior private, but dramatically negative reactions should be avoided. Physical conditions such as vulvovaginitis or penile irritation as well as urethral irritation caused by wearing tight jeans may trigger increased masturbation; the behavior usually decreases when the disorder is treated. The child described in the vignette may be bored or may have vulvovaginitis related to bubble baths or urethral irritation causing her masturbatory behavior. She should be evaluated matter of factly, and private behavior should be discussed. Children of this age are becoming more aware of differences between sexes and may become shy with parents in the bathroom. A history of a single urinary tract infection in infancy is not concerning and is unrelated to the girls behavior. In contrast, behavior that is indicative of sexual exploitation, such as excessive or compulsive masturbation or mimicking intercourse deserves further evaluation. A history of any of these activities in the child in the vignette warrants involvement of the appropriate social services to ensure her safety. References: Dixon SD. Gender and sexuality: normal development to problems and concerns. In: Rudolph CD, Rudolph AM, Hostetter MK, Lister G, Siegel NJ, eds. Rudolphs Pediatrics. 21st ed. New York, NY: McGraw-Hill; 2003:470-476. Meyer TL, Cheng TL. In brief: unveiling the secrecy behind masturbation. Pediatr Rev. 2002;23:148-149. Available at: http://pedsinreview.aappublications.org/cgi/content/full/23/4/148

Copyright 2007 by the American Academy of Pediatrics

page 459

2007 PREP SA on CD-ROM Question: 220

Just before clinic starts, you find your two visiting medical students talking about a movie they saw over the weekend in which some of the characters suffered from the "black death" or bubonic plague. They involve you in the conversation and ask you what you know about this disease of antiquity. They are astounded to hear that plague still exists in the world today. You decide to use this time to make a teaching point and query them about the appropriate antimicrobial therapy for plague. Of the following, the MOST appropriate antimicrobial agent is

A. ampicillin B. aztreonam C. ceftriaxone D. gentamicin E. meropenem

Copyright 2007 by the American Academy of Pediatrics

page 460

2007 PREP SA on CD-ROM Critique: 220 Preferred Response: D

The most appropriate antimicrobial agents to treat patients who have plague (Yersinia pestis) are streptomycin and gentamicin. Because streptomycin has become difficult to obtain in recent years, gentamicin has become the drug of choice. Alternative agents include doxycycline, tetracycline, and chloramphenicol. The usual duration of therapy is 7 to 10 days. Ampicillin, aztreonam, ceftriaxone, and meropenem are not effective. Plague commonly presents with fever and painful, swollen regional lymph nodes that are termed buboes. The buboes usually are found in the inguinal region but also can be present in the axillary or cervical areas. Besides bubonic plague, patients may present with the septicemic form (hypotension, respiratory distress, intravascular coagulopathy), the pneumonic form, or meningeal plague. Plague is a zoonotic disease that can be transmitted by contact with sick or dying animals or by the bite of their fleas. Disease in the United States is found predominately in New Mexico, Arizona, Colorado, and California. References: American Academy of Pediatrics. Plague. In: Pickering LK, ed. Red Book: 2006 Report of the Committee on Infectious Diseases. 27th ed. Elk Grove Village, Ill: American Academy of Pediatrics; 2006:523-525 Goldstein MD. Plague (Yersinia pestis). In: Feigin RD, Cherry JD, Demmler GJ, Kaplan SL, eds. Textbook of Pediatric Infectious Diseases. 5th ed. Philadelphia, Pa: WB Saunders Co; 2004:1487-1492 Patt HA, Feigin RD. Diagnosis and management of suspected cases of bioterrorism: a pediatric perspective. Pediatrics. 2002;109:685-692. Available at: http://pediatrics.aappublications.org/cgi/content/full/109/4/685

Copyright 2007 by the American Academy of Pediatrics

page 461

2007 PREP SA on CD-ROM Question: 221

A 14-year-old boy presents to the emergency department with a 2-week history of bilateral leg edema and a 3-day history of abdominal swelling. His vital signs are: temperature, 98.4F (36.9C); blood pressure, 125/67 mm Hg; heart rate, 84 beats/min; and respiratory rate, 20 breaths/min. Physical examination shows moderate ascites and 2+ leg edema. His urinalysis reveals negative blood and 4+ protein. Serum complement concentrations are ordered and found to be normal. Of the following, the MOST likely cause of his edema and proteinuria is

A. immunoglobulin A nephropathy B. lupus nephritis C. membranous nephropathy D. membranoproliferative glomerulonephritis E. postinfectious acute glomerulonephritis

Copyright 2007 by the American Academy of Pediatrics

page 462

2007 PREP SA on CD-ROM Critique: 221 Preferred Response: C

Nephrotic syndrome (NS) is a constellation of symptoms that include proteinuria, hypoalbuminemia, edema, and hyperlipidemia. The most important aspect is proteinuria; the other symptoms simply are outcomes of excessive urinary protein leak. Proteinuria is not the only cause of hypoalbuminemia; other causes include advanced liver disease, severe malnutrition, and gastrointestinal losses. If NS is suspected, a urinalysis may provide clues to the cause. Among the many causes of NS in children, the most common are minimal-change NS (MCNS), focal segmental glomerulosclerosis (FSGS), membranous nephropathy (MN), any cause of acute glomerulonephritis (AGN), membranoproliferative GN (MPGN), lupus GN, and immunoglobulin A nephropathy (IgAN). The practitioners goal is to determine the cause of NS to devise the best management plan. Although most glomerular diseases are not inherited, a family history of renal disease may suggest FSGS, LN, or IgAN. Urinalysis may be helpful, with the presence of blood making MCNS, FSGS, and MN less likely, although any of these conditions may feature protein and blood in the urine. Measuring serum electrolytes is essential to determine overall renal function in any child who has NS, but it is unlikely to provide any clues to the cause. An important clue may be obtained by assessing serum levels of complement 3 and 4. Normal serum complement values, as reported for the boy in the vignette, generally eliminate postinfectious AGN, MPGN, and Tupus nephritis due to Henoch-Schnlein purpura as the cause of NS. The older age of the boy described in the vignette, absence of hematuria, and normal serum complement values suggest MN as the most likely cause of his renal disease. Another common cause could be FSGS. MCNS is most common in toddlers and early school-age children; MN, MPGN, and IgAN generally present in older (>10 years of age) children. References: Appel GB, Cook HT, Hageman G, et al. Membranoproliferative glomerulonephritis type II (dense deposit disease): an update. J Am Soc Nephrol. 2005;16:1392-1403. Abstract available at: http://www.ncbi.nlm.nih.gov/entrez/query.fcgi?orig_db=PubMed&db=PubMed&cmd=Search&ter m=%22Journal+of+the+American+Society+of+Nephrology+:+JASN%22[Jour]+AND+1392[page] +AND+2005[pdat] Perfumo F, Martini A. Lupus nephritis in children. Lupus. 2005;14:83-88. Abstract available at: http://www.ncbi.nlm.nih.gov/entrez/query.fcgi?orig_db=PubMed&db=PubMed&cmd=Search&ter m=%22Lupus%22[Jour]+AND+83[page]+AND+2005[pdat] Rana K, Isbel N, Buzza M, et al. Clinical, histopathologic, and genetic studies in nine families with focal segmental glomerulosclerosis. Am J Kidney Dis. 2003;41:1170-1178. Abstract available at: http://www.ncbi.nlm.nih.gov/entrez/query.fcgi?orig_db=PubMed&db=PubMed&cmd=Search&ter m=%22American+journal+of+kidney+diseases+:+the+official+journal+of+the+National+Kidney+F oundation%22[Jour]+AND+1170[page]+AND+2003[pdat]

Copyright 2007 by the American Academy of Pediatrics

page 463

2007 PREP SA on CD-ROM Question: 222

A 3-year-old boy who has spina bifida has a history of recurrent urinary tract infections (UTIs). He is currently being treated for a UTI. Monitoring laboratory evaluation shows a blood urea nitrogen of 23 mg/dL (8.2 mmol/L) and a creatinine concentration of 1.1 mg/dL (97.2 mcmol/L). Of the following, the class of antibiotics that is MOST commonly associated with renal toxicity is

A. aminoglycosides B. azalides C. carbapenems D. cephalosporins E. penicillins

Copyright 2007 by the American Academy of Pediatrics

page 464

2007 PREP SA on CD-ROM Critique: 222 Preferred Response: A

Multiple antibiotic classes can cause renal toxicity, but the class that is the most likely to cause nephrotoxicity is the aminoglycosides (gentamicin, tobramycin, amikacin, streptomycin, and kanamycin). The renal toxicity of these agents is related to their concentrative uptake by the proximal renal tubular cells and their capacity to interact with critical intracellular targets. Risk factors associated with the development of such nephrotoxicity include prolonged course of therapy, supertherapeutic doses, concurrent use of other nephrotoxic medications, dehydration, and underlying liver disease. The development of nephrotoxicity is characterized by the gradual onset of partial-to-complete, reversible, nonoliguric renal failure. The patient may exhibit hypertension, and laboratory evaluation demonstrates elevations in blood urea nitrogen and creatinine values as well as elevated protein concentrations on urinalysis. Nephrotoxicity rarely is associated with the other antibiotics listed. References: Beauchamp D, Labrecque G. Aminoglycoside nephrotoxicity: do time and frequency of administration matter? Curr Opin Crit Care. 2001;7:401-408. Abstract available at: http://www.ncbi.nlm.nih.gov/entrez/query.fcgi?orig_db=PubMed&db=PubMed&cmd=Search&ter m=%22Current+opinion+in+critical+care%22[Jour]+AND+401[page]+AND+2001[pdat] Kaloyanides GJ. Antibiotic-related nephrotoxicity. Nephrol Dial Transplant. 1994;9(suppl 4):130134. Abstract available at: http://www.ncbi.nlm.nih.gov/entrez/query.fcgi?orig_db=PubMed&db=PubMed&cmd=Search&ter m=%22Nephrology,+dialysis,+transplantation+:+official+publication+of+the+European+Dialysis+a nd+Transplant+Association++European+Renal+Association%22[Jour]+AND+130[page]+AND+1994[pdat] Mingeot-Leclercq MP, Tulkens PM. Aminoglycosides: nephrotoxicity. Antimicrob Agents Chemother. 1999;43:1003-1012. Available at: http://aac.asm.org/cgi/content/full/43/5/1003?view=long&pmid=10223907 Nagai J, Takano M. Molecular aspects of renal handling of aminoglycosides and strategies for preventing nephrotoxicity. Drug Metab Pharmacokinet. 2004;19:159-170. Available at: http://www.jstage.jst.go.jp/article/dmpk/19/3/159/_pdf

Copyright 2007 by the American Academy of Pediatrics

page 465

2007 PREP SA on CD-ROM Question: 223

A 9-year-old girl is stung on the left leg by a hornet while playing outside. Within 10 minutes, she experiences diffuse pruritus, and a large area of erythema develops at the sting site. Five minutes later, extensive raised welts appear on her trunk and extremities. Her parents rush her to the emergency department, where intravenous antihistamines and steroids are administered. During this episode, she denies difficulty breathing or swallowing. No tongue or uvula edema is evident during examination, and her symptoms gradually resolve over 4 hours. Of the following, the NEXT recommended step in her management is to

A. admit the girl to the hospital for 24 hours B. discharge the girl to home C. perform skin testing to hornet and other stinging insects within the next 72 hours D. prescribe a daily oral antihistamine for the girl to take when playing outside E. start allergy shots to hornet

Copyright 2007 by the American Academy of Pediatrics

page 466

2007 PREP SA on CD-ROM Critique: 223 Preferred Response: B

The reaction described for the girl in the vignette is consistent with an immunoglobulin (Ig)Emediated systemic reaction limited to cutaneous symptoms. Children younger than 16 years of age who experience reactions limited to urticaria or angioedema have an overall risk for future systemic reactions of approximately 5% to 10%, a rate similar to individuals experiencing large local reactions. After appropriate initial management, the girl in the vignette can be discharged home with appropriate counseling. She does not meet current guidelines for further testing or allergy shots. Hospital admission rarely is required for such patients unless symptoms persist despite administration of epinephrine. Certain sting reactions warrant skin testing and potentially allergen immunotherapy. Skin testing is recommended for individuals younger than 16 years experiencing systemic reactions that involve more than cutaneous symptoms and in individuals older than 16 years who have any type of systemic reaction, even cutaneous symptoms only. Individuals in these groups have a 30% to 60% chance for a systemic reaction when stung again by the same type of insect. Allergen immunotherapy is beneficial in these high-risk patients, lowering the risk of future systemic reaction to less than 5%. When indicated, skin testing should be delayed for 2 to 4 weeks after the reaction; testing prior to this period may result in false-negative results. Antihistamines may improve local symptoms or pruritus, but they do not have a role in daily prophylaxis for insect sting reactions. References: Sicherer SH, Leung DYM. Insect allergy. In: Behrman RE, Kliegman RM, Jenson HB, eds. Nelson Textbook of Pediatrics. 17th ed. Philadelphia, Pa: WB Saunders Co 2004:786-788 Golden DB, Kagey-Sobotka A, Norman PS, Hamilton RG, Lichtenstein LM. Outcomes of allergy to insect stings in children, with and without venom immunotherapy. N Engl J Med. 2004;351:668674. Abstract available at: http://www.ncbi.nlm.nih.gov/entrez/query.fcgi?orig_db=PubMed&db=PubMed&cmd=Search&ter m=N+Engl+J+Med[Jour]+AND+668[page]+AND+2004[pdat] Moffitt JE, Golden DBK, Reisman RE, et al. Stinging insect hypersensitivity: a practice parameter update. J Allergy Clin Immunol. 2004;114:869-886

Copyright 2007 by the American Academy of Pediatrics

page 467

2007 PREP SA on CD-ROM Question: 224

An anxious 16-year-old girl presents to the emergency department complaining of a sudden onset of feeling that she is unable to breathe. She also complains of tingling around her lips and fingertips. Upon further questioning, you learn that she has had frequent similar episodes in the past. She has no history of asthma, and she has no chest pain or fever. Physical examination reveals a respiratory rate of 30 to 40 breaths/min, oxygen saturation on room air of 99%, no intercostal retractions, and no fever. No wheezing or crackles are evident on chest auscultation; breath sounds are good bilaterally. The balance of findings on the physical examination, including a complete neurologic evaluation, are normal. Of the following, the symptom that BEST distinguishes the correct diagnosis from compensatory hyperventilation is

A. chronicity of the complaint B. lack of chest pain C. lack of fever D. oxygen saturation E. respiratory rate

Copyright 2007 by the American Academy of Pediatrics

page 468

2007 PREP SA on CD-ROM Critique: 224 Preferred Response: A

The chronicity of the complaints reported by the girl in the vignette, particularly in the absence of any signs or symptoms of organic disorders, suggests hyperventilation syndrome rather than compensatory hyperventilation. A rapid respiratory rate may be a compensation for organic disease such as pneumonia, asthma, diabetic ketoacidosis, increased intracranial pressure, congestive heart failure, or toxin ingestion or it may be a manifestation of dysfunctional breathing. Hyperventilation syndrome is a form of dysfunctional breathing that is defined as breathing in excess of metabolic requirements that leads to an acute decrease in arterial carbon dioxide values (hypocapnia). Symptoms of hypocapnia, such as paresthesias of the face and hands, generally appear when the P2 declines to 20 mm Hg. Other symptoms include dyspnea, chest pain or tightness, muscle spasm, dizziness, and palpitations. The cause of hyperventilation syndrome is unclear; in fact, some authors doubt its existence. The cause is probably multifactorial. Early literature often used hyperventilation syndrome and panic attack interchangeably, but it has become clear that they are not necessarily the same. One recent study showed that patients who have hyperventilation syndrome experience an accentuated increase in ventilation after a change in body position from supine to standing, suggesting increased sensitivity of the baroreceptor response. Other studies have shown a strong association between hyperventilation syndrome and asthma, suggesting that some affected patients may be experiencing mild asthma attacks. The acute evaluation of a patient presenting with isolated tachypnea should focus on ruling out acute organic causes. A history and physical examination help to rule out obvious problems such as congestive heart failure, asthma, pneumonia, or increased intracranial pressure. Additional suggested measures to rule out an organic cause in the acute setting include chest radiography, electrocardiography, arterial blood gas determination, and measurement of serum electrolytes and hemoglobin. A trial of bronchodilators also can be ordered. Prior recommendations to ask the patient to breathe into a paper bag in an attempt to reverse the hypocapnia have been discouraged by recent authors, who point out that this practice could be dangerous in the patient who has hyperventilation due to hypoxia. Patients with hyperventilation syndrome due to anxiety often show spontaneous resolution of their symptoms after assurance that there is no organic cause for the symptoms. Selected patients who have unremitting symptoms may require a ventilation perfusion scan to rule out the presence of a pulmonary embolus or more detailed pulmonary evaluation. Organic problems such as congestive heart failure and pneumonia may be present in the absence of fever or chest pain. Blood oxygen saturation is normal in patients who have diabetic ketoacidosis and increased intracranial pressure. Thus, fever, chest pain, and oxygen saturation do not reliably distinguish compensatory hyperventilation from hyperventilation syndrome. However, the recurrent nature of this patients complaints would not be expected in compensatory hyperventilation due to any of these organic etiologies because spontaneous recovery from any of them is unlikely without specific therapy. Impending respiratory failure should be suspected in patients who present with tachypnea, diminished level of consciousness, poor skeletal muscle tone, and cyanosis. References: American Heart Association. Airway, ventilation, and management of respiratory distress and failure. In: PALS Provider Manual. Dallas, Tex: American Heart Association; 2002:81-126 Gardner WN. The pathophysiology of hyperventilation disorders. Chest. 1996;109:516-534. Available at: http://www.chestjournal.org/cgi/reprint/109/2/516 Keeley D, Osman L. Dysfunctional breathing and asthma: it is important to tell the difference [editorial]. Br Med J. 2001;322:1075-1076. Available at: http://bmj.bmjjournals.com/cgi/content/full/322/7294/1075

Copyright 2007 by the American Academy of Pediatrics

page 469

2007 PREP SA on CD-ROM

Laffey JG, Kavanagh BP. Medical progress: hypocapnia. N Engl J Med. 2002;347:43-53 Malmberg LP, Tamminen K, Sovijrvi AR. Orthostatic increase of respiratory gas exchange in hyperventilation syndrome. Thorax. 2000;55:295-301. Available at: http://thorax.bmjjournals.com/cgi/content/full/55/4/295 Saisch SGN, Wessely S, Gardner WN. Patients with acute hyperventilation presenting to an inner-city emergency department. Chest. 1996;110:952-957. Available at: http://www.chestjournal.org/cgi/reprint/110/4/952

Copyright 2007 by the American Academy of Pediatrics

page 470

2007 PREP SA on CD-ROM Question: 225

The mother of a 4-month-old infant is planning a winter trip to the tropics with her infant and asks about the use of sunscreens and the safe amount of sun exposure for the infant. Of the following, the MOST appropriate advice for the infant is to

A. apply waterproof sunscreen with a UVB SPF of 30 or greater at least every 30 minutes B. avoid mid-day sun and apply sunscreen with a UVB SPF of 15 or greater C. avoid all but incidental sun exposure because of decreased sweating and the risk of heat
stroke

D. comply strictly with the use of physical sun blocks such as zinc oxide and titanium dioxide
paste

E. use only special sun-protective clothing

Copyright 2007 by the American Academy of Pediatrics

page 471

2007 PREP SA on CD-ROM Critique: 225 Preferred Response: C

The proper advice for management of the 4-month-old infant described in the vignette is to avoid all but incidental sun exposure; infants younger than 6 months of age should be kept out of direct sunlight. Because they are not mobile and cannot remove themselves from uncomfortable light and heat, they should be moved under a tree, umbrella, or stroller canopy, although an umbrella or canopy in the vicinity of reflective surfaces (eg, the beach) may reduce ultraviolet radiation exposure by only 50%. Many infants have impaired functional sweating, and exposure to the heat of the sun may increase the risk of heatstroke. Sunburn may occur readily because an infant's skin has less melanin than at any other time in life. For infants and toddlers older than 6 months, mid-day sun avoidance remains the best method of sun protection. Special sun-protective clothing as well as sunscreen agents that have solar protection factor (SPF) of 30 or greater are recommended as adjuncts to other sunprotective practices. When engaged in swimming or water sports, sunscreens should be reapplied regularly throughout the period of sun exposure. Physical sun blocks containing zinc oxide and titanium dioxide are becoming more cosmetically acceptable and provide excellent sun protection. More than 80% of lifetime sun exposure is received before 18 years of age. Sun safety habits should be instilled early in life, and all children and adolescents should use protective measures. Sunburn is the most obvious sign of sun damage, and cellular damage to the skin may occur even with the use of sunscreens. For all ages, sun avoidance between 10 and 4 is preferable. References: American Academy of Pediatrics. Committee on Environmental Health. Ultraviolet light: a hazard to children. Pediatrics. 1999;104:328-333. Available at: http://pediatrics.aappublications.org/cgi/content/full/104/2/328 Raimer S, Raimer BG. Physical injury: solar radiation. In: Schachner LA, Hansen RC, eds. Pediatric Dermatology. 3rd ed. St. Louis, Mo: Mosby; 2003:1227-1234 Weston WL, Lane AT, Morelli JG. Sun sensitivity: sunburn. In: Color Textbook of Pediatric Dermatology. 3rd ed. St. Louis, Mo: Mosby; 2002:144-147

Copyright 2007 by the American Academy of Pediatrics

page 472

2007 PREP SA on CD-ROM Question: 226

A 5-month-old infant presents with a history of vomiting between 10 and 20 times a day. She is growing and developing normally. There is no blood in the vomitus, no respiratory symptoms, and no history of apnea. The parents are frustrated from cleaning up after the baby vomits and want something done. Physical examination and upper gastrointestinal radiograph evaluation results are normal. Of the following, the MOST accurate statement about this patient is that she

A. is at increased risk of sudden infant death syndrome B. is likely to develop an esophageal stricture in later life C. probably will outgrow the condition by 1 year of age D. should be referred for a head computed tomography scan E. should undergo endoscopy to rule out eosinophilic esophagitis

Copyright 2007 by the American Academy of Pediatrics

page 473

2007 PREP SA on CD-ROM Critique: 226 Preferred Response: C

The infant described in the vignette has frequent vomiting, but she has no signs suggesting pathologic reflux, such as recurrent pneumonia, hematemesis, or failure to thrive. Therefore, it is likely that she will outgrow her reflux by 1 year of age. Gastroesophageal reflux rarely has been reported to cause esophageal strictures, but this complication generally is seen in children who have other medical conditions (eg, hiatal hernia, tracheoesophageal fistula, cerebral palsy) and not in healthy term infants. There is no strong evidence in the medical literature to suggest that infants who have benign reflux are at risk of sudden infant death syndrome. Although intracranial lesions always should be considered in a child who is vomiting, nothing in this child's presentation warrants computed tomography scan of the head at this time. Endoscopy to rule out eosinophilic esophagitis is not necessary unless there is a strong history of other atopic disease. Infantile gastroesophageal reflux, defined as the regurgitation of gastric contents out of the mouth, occurs in approximately 50% of term infants and peaks between 4 and 6 months of age. According to one study, approximately 20% of infants vomit five times a day or more between 4 and 6 months of age, but by 12 months of age, the reflux resolves in 90% to 95% of infants. Therefore, observation and education are appropriate interventions for healthy babies who have no signs of pathologic reflux. A small subset of infants may have reflux-associated pathology, including esophagitis, recurrent pneumonia, or reactive airway disease. For these infants, additional evaluation (with esophageal pH study or endoscopy) and therapy (with histamine receptor antagonists, proton pump inhibitors, or prokinetics) may be of benefit until they outgrow the condition. In contrast, although reflux is less common in older children (prevalence of approximately 3%), older children are less likely to outgrow the condition and are more likely to require long-term therapy. References: Nelson SP, Chen EH, Syniar GM, Christoffel KK. Prevalence of symptoms of GE reflux during infancy. A pediatric practice-based survey. Arch Pediatr Adolesc Med. 1997;151:569-572. Abstract available at: http://www.ncbi.nlm.nih.gov/entrez/query.fcgi?cmd=Retrieve&db=pubmed&dopt=Abstract&list_ui ds=9193240&query_hl=18&itool=pubmed_docsum Nelson SP, Chen EH, Syniar GM, Christoffel KK. Prevalence of symptoms of gastroesophageal reflux during childhood: a pediatric practice-based survey. Arch Pediatr Adolesc Med. 2000;154:150-154. Abstract available at: http://www.ncbi.nlm.nih.gov/entrez/query.fcgi?cmd=Retrieve&db=pubmed&dopt=Abstract&list_ui ds=10665601&query_hl=20&itool=pubmed_docsum Rudolph CD, Mazur LJ, Liptak GS, et al. Guidelines for evaluation and treatment of gastroesophageal reflux in infants and children: recommendations of the North American Society for Pediatric Gastroenterology and Nutrition. J Pediatr Gastroenterol Nutr. 2001;32(suppl 2):S1S31. Available at: http://www.jpgn.org/pt/re/jpgn/abstract.00005176-20010000200001.htm;jsessionid=EjrOla6kqCUYONz2cAq2M3Da4Bz2hAxBakJXhuVwuU1Aecpv0uEz!1070481199!-949856145!9001!-1

Copyright 2007 by the American Academy of Pediatrics

page 474

2007 PREP SA on CD-ROM Question: 227

Retinopathy of prematurity (ROP) is the leading neonatal cause of blindness in the United States. Of the following, the population at GREATEST risk for severe ROP comprises infants

A. admitted to a neonatal intensive care unit for any cause B. born at less than 37 weeks' gestation who require parenteral nutrition C. born at less than 32 weeks' gestation who require oxygen therapy D. born at 28 weeks' gestation regardless of the need for assisted ventilation E. who have a family history of myopia

Copyright 2007 by the American Academy of Pediatrics

page 475

2007 PREP SA on CD-ROM Critique: 227 Preferred Response: D

Retinopathy of prematurity (ROP) is the leading neonatal cause of blindness in the United States. Up to 400 infants each year are blinded by this condition. It affects 20% to 50% of all neonatal intensive care unit (NICU) patients weighing less than 1,500 g, with particular predilection for the smallest and most immature newborns. Blindness is greatly increased in infants who have stage 3 or 4 disease. The theories of contributing causal factors include extreme prematurity, very low birthweight, exposure to supplemental environmental oxygen, possible oxidizing agent exposure (including iron), and the generation of inflammatory cytokines. Perturbation of programmed angiogenesis in the developing retina ex utero and corresponding metabolic changes result in certain vascularized and ischemic zones of the retina. Vascular endothelial growth factor is altered, and a proliferative vasculopathy that has immature, leaky, and tortuous tufts of new vessels jeopardizes perfusion of the retina and imparts biomechanical traction upon it. Results of these processes include impaired peripheral vision in the peripheral zones of vascularity and potential retinal detachment with corresponding blindness or visual field deficits. The greatest risk for ROP is in infants born at less than 29 weeks gestation who weigh less than 1,200 g. Because retinal vascularity generally is completed by 37 to 44 weeks gestation, prematurity itself confers risk, but the risk is shifted toward the extreme end of prematurity. Oxygen therapy is a contributing risk factor, but imposes the greatest risk in the smallest and most immature infants. Myopia is more common in very low-birthweight preterm infants than in term infants, but a family history of myopia has no bearing on the risk of ROP. In view of these facts and findings about ROP, the American Academy of Pediatrics, the American Academy of Ophthalmology, and the American Association for Pediatric Ophthalmology and Strabismus jointly published guidelines for ROP screening examination of preterm infants. Candidates for screening include infants whose birthweights are less than 1,500 g, whose gestational ages are less than 32 weeks, and selected infants weighing 1,500 to 2,000 g at birth who have an unstable clinical course if judged to be at risk by the attending pediatrician or neonatologist. The examination should be performed by a qualified and experienced ophthalmologist, and the results should be reported using the International Classification of Retinopathy of Prematurity. The first examination should be performed between 4 and 6 weeks of postnatal age or at a corresponding postconceptual age of 31 to 34 weeks, whichever is later. References: Olitsky SE, Nelson LB. Disorders of the retina and vitreous. In: Behrman RE, Kliegman RM, Jenson HB, eds. Nelson Textbook of Pediatrics. 17th ed. Philadelphia, Pa: Saunders; 2004:21132118 Phelps DL. Retinopathy of prematurity: practical clinical approach. NeoReviews. 2001;2:e174e179. Available at: http://neoreviews.aappublications.org/cgi/content/full/2/7/e174 Phelps DL. The eye (part 3): retinopathy of prematurity. In: Fanaroff AA, Martin RJ, eds. Neonatal-Perinatal Medicine: Diseases of the Fetus and Infant. 7th ed. Baltimore, Md: Mosby, Inc; 2002:1595-1602 Phelps DL and the ETROP Study Group. The early treatment for retinopathy of prematurity study: better outcomes, changing strategy [commentary]. Pediatrics. 2004;114:490-491. Available at: http://pediatrics.aappublications.org/cgi/content/full/114/2/490 Section on Ophthalmology American Academy of Pediatrics, American Academy of Ophthalmology, American Association for Pediatric Ophthalmology and Strabismus. Screening examination of premature infants for retinopathy of prematurity. Pediatrics. 2006;117:572-576. Available at: http://pediatrics.aappublications.org/cgi/content/full/117/2/572

Copyright 2007 by the American Academy of Pediatrics

page 476

2007 PREP SA on CD-ROM Question: 228

You are seeing a 6-month-old boy for a health supervision visit. On physical examination, you note bilateral, nontender scrotal swelling (Item Q228A). The scrotum transilluminates (Item Q228B). The remainder of the physical examination findings are normal. Of the following, the MOST likely diagnosis is

A. hydrocele B. inguinal hernia C. orchitis D. testicular germ cell tumor E. varicocele

Copyright 2007 by the American Academy of Pediatrics

page 477

2007 PREP SA on CD-ROM Critique: 228 Preferred Response: A

Scrotal swelling is a common complaint in the pediatric population. Many of the causes are a result of abnormal testicular descent. When the testes descend into the scrotum from the abdomen, they bring with them a portion of the peritoneum, known as the processus vaginalis, which normally obliterates (Item C228A). The tunica vaginalis subsequently forms around the testes in the scrotum. Failure of the processus vaginalis to obliterate proximally results in fluid accumulation around the testes, which forms a hydrocele. Physical findings include painless, bilateral swelling of the scrotum and transillumination, as seen in the infant described in the vignette. Most hydroceles resolve within 1 year; persistence beyond 1 year of age warrants surgical evaluation. Failure of distal obliteration of the processus vaginalis causes an inguinal hernia, in which abdominal contents descend into the inguinal canal. This is characterized by an intermittent bulge (Item C228B) in the inguinal canal, which is made worse with straining or crying. A varicocele is caused by dilatation of the pampiniform venous plexus and rarely is seen in infancy. A characteristic bag of worms appearance (Item C228C) is noted when the patient stands upright. Orchitis usually is associated with a viral infection in postpubescent boys. Features include tender swelling of the testis, with erythema of the overlying scrotum (Item C228D). Testicular germ cell tumors may be seen in infancy, but the typical physical finding is a painless scrotal mass. References: Elder JS. Acute and chronic scrotal swelling. In: Kliegman RM, Greenbaum LA, Lye PS, eds. Practice Strategies in Pediatric Diagnosis and Therapy. 2nd ed. Philadelphia, Pa: Elsevier Saunders; 2004:465-474 Schneck FX, Bellinger MF. Abnormalities of the testes and scrotum and their surgical management. In: Walsh PC, Retik AB, Vaughan ED Jr, Wein AJ, eds. Campbells Urology. 8th ed. Philadelphia, Pa: Elsevier; 2002:2353-2356

Copyright 2007 by the American Academy of Pediatrics

page 478

2007 PREP SA on CD-ROM Question: 229

A 14-year-old boy who had a sore throat and fever 2 weeks ago presents to the emergency department still dressed in his football gear from the practice field, where he complained of acute abdominal pain. He exhibits tachypnea, tachycardia, and mild hypotension and complains of intense pain in the left upper quadrant. Of the following, the MOST definitive study to diagnose this child's condition is

A. abdominal computed tomography scan B. abdominal ultrasonography C. complete blood count D. diagnostic peritoneal lavage E. partial thromboplastin time

Copyright 2007 by the American Academy of Pediatrics

page 479

2007 PREP SA on CD-ROM Critique: 229 Preferred Response: A

Splenic rupture in children almost always is due to blunt trauma, although susceptibility may be enhanced in children who have underlying disease, such as Epstein-Barr viral infection with splenomegaly. Because trauma to the upper abdomen may be suspected in children who have blunt trauma due to sports, motor vehicle accident, or assault, the approach to the child in whom splenic injury is suspected should be the same as for all trauma victims. Attention to airway, breathing, and circulation should be the initial step in evaluation. For the child who has suspected splenic rupture, hemodynamic stability should be ensured. Peritoneal lavage almost never is indicated in children because computed tomography scan of the abdomen with contrast provides more specific and sensitive information about the extent of abdominal injury. Limited evidence suggests that ultrasonography is sensitive and specific in accurately diagnosing blunt abdominal trauma, but, as with other ultrasonographic procedures, results may be related to the skill of the operator. Although many children do not require surgical intervention for splenic laceration, splenic rupture or laceration associated with hemodynamic instability may require such measures. Baseline complete blood count and coagulation studies (eg, partial thromboplastin time) should be obtained, but they will not establish the diagnosis of splenic injury. In addition, an intravenous line should be placed, and the patient should not be fed until surgical clearance or several hours of observation indicate no need for surgery. Support with intravenous fluids and blood products is indicated for the patient described in the vignette, who has hypotension and tachycardia. Children who require splenic resection are at increased risk for postsplenectomy sepsis, although the occurrence of this complication may be waning because of improved immunization against encapsulated organisms. References: Haller JA Jr. Blunt trauma to the abdomen. Pediatr Rev. 1996;17:29-31 Sills RH. Spleen and lymph nodes. In: McMillan JA, DeAngelis CD, Feigin RD, Warshaw JB, eds. Oskis Pediatrics: Principles and Practices. 3rd ed. Philadelphia, Pa: Lippincott, Williams & Wilkins; 1999:1465-1472 Sharma OP, Oswanski MF, Singer D, Raj SS, Daoud YA. Assessment of nonoperative management of blunt spleen and liver trauma. Am Surg. 2005;71:379-386. Abstract available at: http://www.ncbi.nlm.nih.gov/entrez/query.fcgi?cmd=Retrieve&db=pubmed&dopt=Abstract&list_ui ds=15986966&query_hl=1&itool=pubmed_docsum Stengel D, Bauwens K, Sehouli J, et al. Emergency ultrasound-based algorithms for diagnosing blunt abdominal trauma. The Cochrane Database of Systematic Reviews. 2005;2:CD004446. Available at: http://www.mrw.interscience.wiley.com/cochrane/clsysrev/articles/CD004446/frame.html

Copyright 2007 by the American Academy of Pediatrics

page 480

2007 PREP SA on CD-ROM Question: 230

A 4-year-old girl remains intubated, mechanically ventilated, and completely unresponsive in the intensive care unit following a massive subarachnoid hemorrhage of unknown cause. She is hemodynamically stable and not receiving any sedating medications. She has a temperature of 98.1F (36.7C) and has no evidence of infection. The girl has not exhibited any brainstem or cerebral function for more than 24 hours. Of the following, the statement you are MOST likely to make while on rounds with the resident team is that

A. a nuclear medicine blood flow study is the best method to determine whether the girl is brain
dead

B. electroencephalography must be performed to ascertain whether this girl is dead C. even though this child is brain dead, her stable cardiac status precludes a declaration of death D. no further testing is indicated because this child is brain dead E. organ donation is contraindicated in this child once she is declared dead

Copyright 2007 by the American Academy of Pediatrics

page 481

2007 PREP SA on CD-ROM Critique: 230 Preferred Response: A

Brain death is equivalent to cardiorespiratory death, and a child is legally dead at the time the criteria for brain death are fulfilled. The determination of brain death is most helpful to the process of organ procurement for transplantation. Clinical criteria for brain death include a known, irreversible cause, along with complete absence of cerebral and brainstem function for more than 12 hours (for children older than 1 year) determined by at least two physicians. The patient must be normothermic and normotensive, and there must be no toxins, drugs, or metabolic disorders present that could obscure neurologic examination findings. Absence of brainstem function is confirmed by physical findings plus an apnea test, with arterial P2 greater than 60 mm Hg rising by at least 20 mm Hg over 8 to 10 minutes while the patient receives 100% oxygen. Ancillary neurodiagnostic studies may be helpful in diagnosing brain death. Electroencephalography with electrocerebral silence is consistent with but not pathognomonic for brain death. A cerebral radionuclide scan or conventional angiography showing complete absence of blood flow in the vertebral and carotid arteries is pathognomonic for brain death. Such flow studies may be helpful when facial injury precludes examination or there is difficulty establishing a known cause, as in the vignette. Without further evaluation, the 4-year-old child described here cannot be declared brain dead yet by current criteria because the cause of her subarachnoid hemorrhage is unknown. A nuclear medicine blood flow study can document brain death by demonstrating no blood flow in the intracranial major arteries. Her stable cardiac status does not preclude a declaration of brain death, although further testing is indicated. Electroencephalography alone, even if devoid of any activity, does not definitively establish brain death. Once declared brain dead, this child likely can be a candidate for organ donation. References: Determination of brain death. Ad Hoc Committee on Brain Death. J Pediatr. 1987;110:15-19 Frankel LR, Mathers LH. Withdrawal or withholding of life support, brain death, and organ procurement. In: Behrman RE, Kliegman RM, Jenson HB, eds. Nelson Textbook of Pediatrics. 17th ed. Philadelphia, Pa: WB Saunders Co; 2004:340-341 Report of Special Task Force. Guidelines for the determination of brain death in children. American Academy of Pediatrics Task Force on Brain Death in Children Pediatrics. 1987;80:298300

Copyright 2007 by the American Academy of Pediatrics

page 482

2007 PREP SA on CD-ROM Question: 231

You are evaluating a 12-year-old boy who recently moved to your community. His family history reveals that his father, who is 36 years old, suffers from extremely high blood cholesterol levels (>500 mg/dL [12.9 mmol/L]). The boy's mother states that her husband's brother has the "same problem." Findings on the boy's physical examination are normal. Of the following, the BEST next step for your patient is to

A. begin therapy with a lipid-lowering medication B. institute a low-fat diet plan and follow up in 3 months C. measure cholesterol and triglyceride concentrations D. obtain a blood sample for genetic testing E. order a baseline electrocardiogram

Copyright 2007 by the American Academy of Pediatrics

page 483

2007 PREP SA on CD-ROM Critique: 231 Preferred Response: C

There is good scientific evidence that the process of atherosclerosis begins in childhood. Children who have high cholesterol concentrations have a greater risk of having elevated cholesterol values as adults than does the general population. In addition, several genetic syndromes affect the normal metabolic processes of cholesterol-rich lipoproteins. These familial diseases are characterized by early onset of frequently severe atherosclerotic changes in the coronary artery system. Among such syndromes are familial hypercholesterolemia, familial combined hyperlipidemia, familial hypertriglyceridemia, familial dysbetalipoproteinemia, and familial decreased high-density lipoprotein. Although the enzyme or protein abnormality varies in each of these rare entities, the end result is similar, with affected children being at high risk for the development of coronary heart disease at a young age. Population approaches to lowering cholesterol concentrations in children and adolescents have focused on the adoption of diets that are lower in saturated fat, total fat, and cholesterol. However, in cases such as the at-risk boy described in the vignette, an individualized approach is required in an attempt to facilitate early diagnosis and management. Therefore, children and adolescents who have a family history of premature cardiovascular disease or at least one parent who has a high blood cholesterol reading should be considered for selective screening. Children who have a family history of any of the genetic hyperlipidemias also should undergo early screening of cholesterol and triglycerides. Initiation of therapy with lipid-lowering medications is not appropriate until a diagnosis has been made. Similarly, genetic testing, which often is expensive and not available for all disorders, may not be of value as a first step. Electrocardiography does not have a diagnostic or prognostic role in the asymptomatic patient. Instituting a low-fat diet plan may not be wrong, but it does not aid in reaching a diagnosis. References: Cohen MS. Fetal and childhood onset of adult cardiovascular diseases. Pediatr Clin North Am. 2004;51:1697-1719. Abstract available in: http://www.ncbi.nlm.nih.gov/entrez/query.fcgi?orig_db=PubMed&db=PubMed&cmd=Search&ter m=%22Pediatric+clinics+of+North+America%22[Jour]+AND+1697[page]+AND+2004[pdat] Kwiterovich RC. Disorders of lipid and lipoprotein metabolism. In: Rudolph CD, Rudolph AM, Hostetter MK, Lister G, Siegel NJ, eds. Rudolphs Pediatrics. 21st ed. New York, NY: McGraw Hill; 2003:693-711 Lauer RM, Snetselaar L, Muhonen LF. Hyperlipidemia in children and adolescents. In: Moller JH, Hoffman JIE, eds. Pediatric Cardiovascular Medicine. Philadelphia, Pa: Churchill Livingston; 2000:793-803

Copyright 2007 by the American Academy of Pediatrics

page 484

2007 PREP SA on CD-ROM Question: 232

A 4-month-old child is admitted to the hospital for evaluation of failure to thrive and generalized seizures. On physical examination, the child appears wasted and has a protuberant abdomen and marked hepatomegaly. Laboratory evaluation reveals fasting hypoglycemia, lactic acidosis, hyperuricemia, and hyperlipidemia. The boy's parents are first cousins. Of the following, the BEST long-term management of this disorder is

A. oral dietary supplementation with long-chain fatty acids B. oral dietary supplementation with protein C. regular intravenous administration of 10% dextrose in water D. regular intravenous administration of glucagon E. regular oral administration of cornstarch

Copyright 2007 by the American Academy of Pediatrics

page 485

2007 PREP SA on CD-ROM Critique: 232 Preferred Response: E

The child described in the vignette has glycogen storage disease type I (GSD I) or von Gierke disease. GSD I is an autosomal recessive disorder resulting from deficiency of glucose-6phosphatase. Consequently, the final step in the catabolism of glycogen to glucose is impeded. Excess accumulation of the substrate glucose-6-phosphate results in increased lactate production. Hepatomegaly, which is sometimes evident at birth, is progressive, and linear growth is delayed. Infants who have GSD I typically present when they begin to sleep through the night. The prolonged fasting associated with the increased sleep pattern results in hypoglycemia, and affected children may present with irritability, pallor, and seizures. In addition to hypoglycemia, metabolic derangements include lactic acidosis, hyperuricemia, hyperlipidemia, and ketonuria, as reported for the boy in the vignette. A number of renal complications can occur, and some patients develop renal failure. Diagnosis of GSD I is based on the near-to-complete absence of the enzyme glucose-6phosphatase in liver tissue. Open-liver biopsy frequently is preferred to needle biopsy to ensure attainment of an appropriate specimen and to control bleeding, which may be excessive. Once the diagnosis is established, molecular genetic testing is available. Treatment is twofold: avoidance of fasting and frequent administration of carbohydrates. Supplementation with long-chain fatty acids and protein is not necessary because affected individuals do not have difficulty metabolizing them. In infants, nasogastric feedings may be useful at night. Uncooked cornstarch provides for the slow release of glucose and is a mainstay of treatment in infants and children. Glucagon administration is not helpful due to the inability to break down glycogen effectively. Intravenous 10% dextrose and water is cumbersome and difficult over the long term. References: Chen Y-T. Defects in metabolism of carbohydrates: glycogen storage diseases. In: Behrman RE, Kliegman RM, Jenson HB, eds. Nelson Textbook of Pediatrics. 17th ed. Philadelphia Pa: Saunders; 2004:469-475 Craigen WJ, Darras BT. Overview of disorders of glycogen metabolism. UpToDate. 2006;14.1. Available at: http://www.utdol.com/utd/content/topic.do?topicKey=dis_chld/8136&type=A&selectedTitle=2~11 Rake JP, Visser G, Labrune P, et al. Guidelines for management of glycogen storage disease type I: European Study on Glycogen Storage Disease Type I (ESGSD I). Eur J Pediatr. 2002;161(suppl 1):S112-S119. Abstract available at: http://www.ncbi.nlm.nih.gov/entrez/query.fcgi?cmd=Retrieve&db=pubmed&dopt=Abstract&list_ui ds=12373584&query_hl=8&itool=pubmed_docsum

Copyright 2007 by the American Academy of Pediatrics

page 486

2007 PREP SA on CD-ROM Question: 233

A 17-year-old boy presents for a sports physical. He has a learning disability and is shy. His height is at the 75th percentile, and his body mass index is at the 85th percentile. Physical examination findings include minimal facial hair, bilateral gynecomastia (breast >4 cm in diameter), and small testes (testicular volume of 6 mL). Of the following, the MOST likely cause of this patient's gynecomastia is

A. constitutional delay of puberty B. incomplete androgen insensitivity syndrome C. Klinefelter syndrome D. obesity E. pubertal gynecomastia

Copyright 2007 by the American Academy of Pediatrics

page 487

2007 PREP SA on CD-ROM Critique: 233 Preferred Response: C

Pubertal gynecomastia (Item C233A) is a benign and self-limited increase in glandular and stromal tissue that is believed to result from a temporary alteration in the ratio of estrogen to testosterone. Approximately two thirds of adolescent males develop pubertal gynecomastia. The mean age of onset is 13 years, and 50% of affected boys are at Sexual Maturity Rating stage 3 when breast enlargement begins. Although the condition initially is unilateral, eventually more than 75% of affected boys develop bilateral gynecomastia. A mobile, firm, rubbery nodule is palpated beneath the areola in boys who have pubertal gynecomastia. This breast tissue may extend beyond the areolar border, but it is rare for glandular tissue to exceed 4 cm in diameter. In addition, patients who have pubertal gynecomastia have normal testicular size. Pubertal gynecomastia usually resolves within 12 to 18 months; in only 8% of boys does it persist beyond 2 years. Adolescent males, such as the one described in the vignette, who are tall and have gynecomastia and small testes probably have Klinefelter syndrome (impaired androgen production), not pubertal gynecomastia. Klinefelter syndrome, the clinical manifestation of an extra X chromosome in males, occurs in 1 in 1,000 live male births. Other causes of nonpubertal gynecomastia include conditions that increase serum estrogen concentration, increase the bioavailability of estrogen, decrease serum androgen concentrations, or alter estrogen and androgen receptors. A history and physical examination guide the search for associated clinical conditions that include drugs (eg, spironolactone, ketoconazole, digitalis, anabolic steroids, cimetidine), liver and kidney disease, hyperthyroidism, androgen insensitivity syndromes, and neoplasms. Constitutional delay of puberty is not a cause of gynecomastia and is associated with short stature. Androgen insensitivity is an infrequent cause of gynecomastia that usually presents as a cause of primary amenorrhea in phenotypic females who have a 46,XY karyotype. Testes in affected individuals may be located in the labioscrotal folds. Obesity alone does not explain the clinical finding of gynecomastia for the boy in the vignette, who has a body mass index of 85% (at risk for obesity) and small testes. Although obese males may have pubertal gynecomastia and other conditions associated with nonpubertal gynecomastia, it is not uncommon for adipose tissue to be confused with breast tissue. When adipose tissue is present, palpation in the supine position often reveals a subareolar depression surrounded by firm tissue (the doughnut sign). References: Neinstein LS, Joffe A. Gynecomastia. In: Neinstein LS, ed. Adolescent Health Care a Practical Guide. 4th ed. Philadelphia, Pa: Lippincott Williams & Wilkins; 2002:264-270 Rapaport R. Gynecomastia. In: Behrman RE, Kliegman RM, Jenson HB, eds. Nelson Textbook of Pediatrics. 17th ed. Philadelphia, Pa: 2004:1930-1931

Copyright 2007 by the American Academy of Pediatrics

page 488

2007 PREP SA on CD-ROM Question: 234

You are evaluating a 14-year-old boy who has a body mass index of 40 kg/m2. His mother and 25-year-old sister have type 2 diabetes. A fasting blood glucose concentration for the boy is 110 mg/dL (6.1 mmol/L). Of the following, the MOST appropriate next step to screen for diabetes is to

A. measure glycosylated hemoglobin B. measure serum insulin and C-peptide concentrations C. perform a 2-hour oral glucose tolerance test D. measure blood glucose 1 hour after a high-carbohydrate breakfast E. repeat a fasting blood glucose measurement

Copyright 2007 by the American Academy of Pediatrics

page 489

2007 PREP SA on CD-ROM Critique: 234 Preferred Response: C

The boy described in the vignette is at high risk for type 2 diabetes because of his elevated body mass index and strong family history. His elevated fasting blood glucose value is worrisome but not diagnostic of diabetes. Criteria for the diagnosis of diabetes are two random blood glucose values greater than 200 mg/dL (11.1 mmol/L) or one random value of 200 mg/dL (11.1 mmol/L) with symptoms, a fasting plasma glucose greater than 126 mg/dL (6.7 mmol/L), or a 2-hour postglucose tolerance test glucose value of greater than 200 mg/dL (11.1 mmol/L). Therefore, a 2hour oral glucose tolerance test should be performed for the boy in the vignette to determine whether he meets the criteria for diabetes. Although glycosylated hemoglobin values are elevated in diabetes that is not perfectly controlled, measurement of this compound is not a good screening test for diabetes. Elevated insulin and C-peptide concentrations are found in any individual who has insulin resistance. Levels may help differentiate type 1 (low insulin and C-peptide) from type 2 (high insulin and Cpeptide) diabetes, but are not diagnostic of diabetes itself. Many overweight children have elevated insulin levels because they are insulin-resistant. There are no standards for the diagnosis of diabetes by measurement of blood glucose after a poorly defined high-carbohydrate breakfast. Similarly, a repeat fasting glucose is likely to give an indeterminate value; only a 2-hour oral glucose tolerance test is definitive for the diagnosis of diabetes. References: American Diabetes Association. Type 2 diabetes in children and adolescents. Pediatrics. 2000;105:671-680. Available at: http://pediatrics.aappublications.org/cgi/content/full/105/3/671 Kaufman FR. Type 2 diabetes in children and youth. Endocrinol Metab Clin North Am. 2005;34:659-676. Abstract available at: http://www.ncbi.nlm.nih.gov/entrez/query.fcgi?orig_db=PubMed&db=PubMed&cmd=Search&ter m=%22Endocrinology+and+metabolism+clinics+of+North+America%22[Jour]+AND+659[page]+ AND+2005[pdat] Lipton RB, Drum M, Burnet D, et al. Obesity at the onset of diabetes in an ethnically diverse population of children: what does it mean for epidemiologists and clinicians? Pediatrics. 2005;115:e553-e560. Available at: http://pediatrics.aappublications.org/cgi/content/full/115/5/e553

Copyright 2007 by the American Academy of Pediatrics

page 490

2007 PREP SA on CD-ROM Question: 235

You are evaluating a 5-year-old boy who has cerebral palsy and mental retardation, is fed through a gastrostomy tube, and is dependent for all his care. He will be attending a full-day program at the school in which he previously was enrolled. His parents are divorced, and his mother is his primary caretaker. She will begin working while he is in school. He has a 10-yearold brother with whom he shares a room and who alerts his mother when his brother needs help at night. Of the following, the concern you are MOST likely to address is

A. family stress B. need for nursing services during the night C. need for the mother to be available during school hours D. potential for child abuse in school E. vulnerability to communicable diseases

Copyright 2007 by the American Academy of Pediatrics

page 491

2007 PREP SA on CD-ROM Critique: 235 Preferred Response: A

Chronic illness has a significant impact on an affected child and his or her family. Family members struggle to balance the needs of individuals with the increased time and energy devoted to the care of a child who has the disability. The increased emotional and economic burden on the family can cause intolerable stresses that lead to divorce, depression, and other psychosocial difficulties. Although much of a familys energy is devoted to the child who has a chronic illness, other healthy children in the family are aware of the stresses in the family, and their needs and concerns also must be addressed. The pediatrician can be an invaluable resource to families who need assistance accessing the appropriate community resources to handle the emotional and financial stresses of caring for a child who has a disability or chronic illness. Children who have disabilities are at increased risk of abuse and neglect in educational and child care settings, but the child described in the vignette is in a stable educational environment with caregivers who likely know him well. His mother needs to be available in case of an emergency during school hours, but she should be supported if she chooses to work. The child is unlikely to qualify for home nursing services. He should receive routine and additional immunizations, such as influenza, depending on his risk for serious sequelae of infection. References: Perrin JM. Chronic illness in childhood. In: Behrman RE, Kliegman RM, Jenson HB, eds. Nelson Textbook of Pediatrics. 17th ed. Philadelphia, Pa: WB Saunders Co; 2004:135-138 Michaud LJ. Therapy services and anticipatory guidance for the child with a disability. In: Rudolph CD, Rudolph AM, Hostetter MK, Lister G, Siegel NJ, eds. Rudolphs Pediatrics. 21st ed. New York, NY: McGraw-Hill; 2003:544-553

Copyright 2007 by the American Academy of Pediatrics

page 492

2007 PREP SA on CD-ROM Question: 236

You are discussing diarrheal diseases with a group of medical students interested in international health. You advise them that there are more than 2,000 serovars of Salmonella. Of the following, the serovar that has the MOST public health implications is

A. Heidelberg B. Newport C. Paratyphi D. Typhi E. Typhimurium

Copyright 2007 by the American Academy of Pediatrics

page 493

2007 PREP SA on CD-ROM Critique: 236 Preferred Response: D

The serovar of Salmonella that has the most public health implications is Typhi, the causative agent for typhoid fever. Because humans are the only reservoir for Salmonella serovar Typhi, contact with an infected person is necessary for infection to occur. Salmonella serovar Paratyphi may cause an enteric fever, but it is recovered less commonly than Salmonella serovar Typhi. Salmonella serovar Heidelberg, Newport, and Typhimurium can become extraintestinal and cause bacteremia or other clinical illness, but they do not have the public health implications of Salmonella serovar Typhi. In a patient who has symptoms suggestive of Salmonella infection, the diagnosis may be made by culture of the stool or normally sterile body fluids (eg, blood, cerebrospinal fluid). Other methods to detect Salmonella serovar Typhi have included rapid serum tests using enzyme immunoassays and latex agglutination, but these have met with little success. DNA detection through the use of polymerase chain reaction testing is both sensitive and specific but is not widely available. The Widal test (febrile agglutinins; antibody detection of O and H antigens from Salmonella serovar Typhi) is not reliable and should not be used. References: American Academy of Pediatrics. Salmonella infections. In: Pickering LK, ed. Red Book: 2006 Report of the Committee on Infectious Diseases. 27th ed. Elk Grove Village, Ill: American Academy of Pediatrics; 2006:579-584 Cleary TG. Salmonella. In: Behrman RE, Kliegman RM, Jenson HB, eds. Nelson Textbook of Pediatrics. 17th ed. Philadelphia, Pa: WB Saunders Co; 2004:912-918

Copyright 2007 by the American Academy of Pediatrics

page 494

2007 PREP SA on CD-ROM Question: 237

A 12-year-old African-American girl presents to your office with a 2-day history of gross hematuria. She describes the urine as brown. She states that she has had an upper respiratory tract infection for about 3 days. She denies dysuria, urgency, or frequency. Her vital signs and physical examination findings are normal. Urinalysis reveals: specific gravity, 1.025; pH, 6.5; large blood; no protein; too numerous-to-count red blood cells; and 0 to 2 white blood cells. Serum electrolyte concentrations are normal. Of the following, the MOST likely cause of her gross hematuria is

A. focal segmental glomerulosclerosis B. immunoglobulin A nephropathy C. lupus nephritis D. membranoproliferative glomerulonephritis E. papillary necrosis

Copyright 2007 by the American Academy of Pediatrics

page 495

2007 PREP SA on CD-ROM Critique: 237 Preferred Response: B

Immunoglobulin A nephropathy (IgAN) or Berger disease is the most common cause of primary glomerulonephritis in adults in the world. In contrast, only about 5% to 10% of children who have glomerular disease exhibit IgAN. Although once considered to be a random disease with no genetic component, the prevailing opinion now is that there may be a strong familial component to IgAN. The pathogenesis of IgAN remains elusive. It is not a classically defined autoimmune disease, although the disease develops from deposition of IgA-containing immune complexes in the kidney. It is tempting to assume that the pathogenesis is related to mucosal IgA because the disease often follows an upper respiratory tract infection, but deposited IgA is predominantly polymeric IgA1. The association of some cases of IgAN with syndromes that affect the respiratory or gastrointestinal (GI) tracts supports the relationship between IgAN and the immune system. Moreover, it has been shown that the gross hematuria worsens during or after upper respiratory tract or GI infections. Serum IgA concentrations may be elevated in about 50% of patients who have IgAN, making this finding relatively nonspecific and not diagnostic. The natural history of the disease varies. About one third of patients have a benign course, one third experience slow progression to renal failure, and one third have a more progressive course in which renal failure develops within 20 years. IgAN is more common in whites and Asians than other ethnic groups and is observed frequently in American Indians. IgAN is more common in males than females. The girl in the vignette has the classic presentation of IgAN: a brief history of painless, gross hematuria following an upper respiratory tract infection. Her urinalysis demonstrates too numerous-to-count red blood cells but no protein, and her renal function is normal. Children who have focal segmental glomerulosclerosis may develop gross hematuria, but this is not common. Patients who have lupus nephritis may develop either microscopic or gross hematuria, but they often have associated symptoms (eg, rashes, arthritis, anorexia). Children who have membranoproliferative glomerulonephritis may develop gross hematuria, but they often present with proteinuria, edema, and hypertension. Finally, children who have papillary necrosis usually have sickle cell disease or trait, and the gross hematuria generally is associated with pain, typically abdominal or flank pain. The follow-up of a patient who has IgAN includes at least quarterly assessment of urine protein by urinalysis and determination of a random protein-to-creatinine ratio (P/C). A rising urine P/C is highly predictive of progressive renal disease. Frequent measurement of serum electrolyte values is vital, especially for patients who have evidence of significant proteinuria or hypertension. Treatment typically begins with agents to reduce proteinuria: angiotensinconverting enzyme inhibitors or angiotensin receptor blockers. Oral corticosteroids may be effective in some cases but generally do not alter the final outcome significantly. Use of calcineurin inhibitors such as cyclosporine or mycophenolate mofetil has been shown to slow the progression of IgAN in adults, but larger trials are necessary to determine their efficacy fully. References: Mitsioni A. IgA nephropathy in children. Nephrol Dial Transpl. 2001;16(suppl 6):123-125. Available at: http://ndt.oxfordjournals.org/cgi/reprint/16/suppl_6/123 Utsunomiya Y, Koda T, Kado T, et al. Incidence of pediatric IgA nephropathy. Pediatr Nephrol. 2003;18:511-515. Abstract available at: http://www.ncbi.nlm.nih.gov/entrez/query.fcgi?orig_db=PubMed&db=PubMed&cmd=Search&ter m=%22Pediatric+nephrology+(Berlin,+Germany)%22[Jour]+AND+511[page]+AND+2003[pdat] Yoshikawa N, Tanaka R, Iijima K. Pathophysiology and treatment of IgA nephropathy in children. Pediatr Nephrol. 2001;16:446-457. Abstract available at: http://www.ncbi.nlm.nih.gov/entrez/query.fcgi?orig_db=PubMed&db=PubMed&cmd=Search&ter m=%22Pediatric+nephrology+(Berlin,+Germany)%22[Jour]+AND+446[page]+AND+2001[pdat]

Copyright 2007 by the American Academy of Pediatrics

page 496

2007 PREP SA on CD-ROM Question: 238

You are seeing a 3-year-old boy for follow-up 2 days after cystoscopy to remove a piece of a toy that he inserted into his urethra. His mother states that he has been doing well at home other than occasionally complaining of pain when he urinates. Physical examination reveals only mild redness at the urethral meatus. Urinalysis is positive for leukocyte esterase, and microscopic examination of the urine shows 10 to 20 white blood cells. Gram stain of the urine is negative except for a few white blood cells. Urine culture is negative at 24 hours. Of the following, the MOST likely cause of this patient's pyuria is

A. cystitis B. Kawasaki disease C. pyelonephritis D. urethral instrumentation E. urethral stone

Copyright 2007 by the American Academy of Pediatrics

page 497

2007 PREP SA on CD-ROM Critique: 238 Preferred Response: D

Urethritis is the response of the urethra to inflammation of any cause, both infectious and noninfectious. The most common clinical presentation is dysuria, urinary frequency, and urethral discharge or itching. Gram stain of the urethral secretions or of a first voided urine specimen may show polymorphonuclear leukocytes. Urethral instrumentation or surgery and the presence of a foreign body are two common causes of non-sexually transmitted urethritis. In these cases, the signs and symptoms are usually due to irritation and mild inflammation and generally resolve spontaneously once the offending agent is removed. Treatment is symptomatic. Secondary infection should be ruled out with urinalysis and urine culture. If an infection is present, antibiotic therapy should be directed toward the organism that is isolated. The absence of fever, systemic symptoms, and red cells on urinalysis as well as the negative urine culture in the patient described in the vignette make cystitis, pyelonephritis, and urethral stone unlikely. Although sterile pyuria may occur in Kawasaki disease, the childs symptoms are not consistent with this diagnosis. References: Krieger JN. Urethritis: etiology, diagnosis, treatment, and complications. In: Gillenwater JY, Grayhack JT, Howards SS, Mitchell ME, eds. Adult and Pediatric Urology. 4th ed. Philadelphia, Pa: Lippincott Williams & Wilkins; 2002:1849-1881

Copyright 2007 by the American Academy of Pediatrics

page 498

2007 PREP SA on CD-ROM Question: 239

A 12-month-old male infant presents for an ear re-evaluation 1 month after being treated for his fourth episode of otitis media. His parents describe a normal birth history and normal development. The child is breastfed and does not attend child care. His immunizations are up to date through 6 months of age, including three doses of the conjugated pneumococcal vaccine. There is no history of sinusitis, pneumonia, sepsis, meningitis, or urinary tract infections. After the boy's last otitis media infection, your colleague measured the child's serum immunoglobulin (Ig) concentrations, and results included a low IgG of 150 mg/dL (1.5 g/L), a normal IgM of 80 mg/dL (0.8 g/L), and a normal IgA of 40 mg/dL (0.4 g/L). Of the following, the next BEST laboratory test to evaluate this infant's antibody function is

A. B- and T-cell flow cytometry B. delayed-type hypersensitivity testing C. isohemagglutinins D. nitroblue tetrazolium test E. serum protein electrophoresis

Copyright 2007 by the American Academy of Pediatrics

page 499

2007 PREP SA on CD-ROM Critique: 239 Preferred Response: C

The infant described in the vignette most likely has transient hypogammaglobulinemia of infancy, but the recurrent infections raise the possibility of an underlying immunodeficiency. Assessment of antibody function is prudent for a child who presents with recurrent sinopulmonary infections and a low serum immunoglobulin (Ig) G concentration. The assessment may be either by measurement of serum isohemmaglutinins (ie, antibodies against blood antigens) in patients who do not have type AB blood or comparison of pre- and postantibody responses to protein (eg, tetanus, diphtheria, measles, mumps, hepatitis B) and polysaccharide (eg, 23-valent pneumococcal) vaccines. Isohemmaglutinin testing is readily available because it is used in blood typing. Flow cytometry sorts cells by their shape, granularity, and cell surface markers. Low-to-absent B lymphocytes are seen with X-linked (Bruton) agammaglobulinemia, but further studies would be needed to assess the functionality of the B lymphocyte. Delayed-type hypersensitivity testing is a useful screening tool for T-lymphocyte function. This test involves intradermal application of an antigen (eg, tetanus, mumps, Candida, trichophyton), with interpretation 48 to 72 hours later. The nitroblue tetrazolium test is a reduction assay that measures the superoxide burst of neutrophils. The inability to create superoxide anions, as seen with chronic granulomatous disease, results in ineffective bacterial phagocytosis, primarily for catalase-positive organisms (eg, Staphylococcus aureus, Aspergillus, Nocardia, Burkholderia cepacia). Serum protein electrophoresis (SPEP) separates proteins based on their size, shape, and electrical charge. A SPEP performed in the patient described in the vignette would show a small gamma band, reflecting the patients low serum IgG level, but it does not assess qualitative antibody function. References: Buckley RH. Primary defects in antibody production. In: Behrman RE, Kliegman RM, Jenson HB, eds. Nelson Textbook of Pediatrics. 17th ed. Philadelpha, Pa: WB Saunders Co 2004:689-692 Kidon MI, Handzel ZT, Schwartz R, Altboum I, Stein M, Zan-Bar I. Symptomatic hypogammaglobulinemia in infancy and childhood - clinical outcome and in vitro immune responses. BMC Fam Pract. 2004;5:23. Available at: http://www.biomedcentral.com/14712296/5/23

Copyright 2007 by the American Academy of Pediatrics

page 500

2007 PREP SA on CD-ROM Question: 240

A father carries his 5-year-old child into the emergency department after a motor vehicle crash in which his vehicle rolled over several times. The accident occurred 1 minute away. The boy was restrained by lap and shoulder belts. He did lose consciousness. In his father's arms, the child moans softly but does not open his eyes. Of the following, the MOST dangerous consequence of transporting this injured child to the emergency department in this manner is

A. delay in instituting intravenous fluid resuscitation B. delay in providing supplemental oxygen C. exacerbation of a cervical spine injury D. hospital personnel not having adequate time to prepare for his arrival E. vascular injury to the extremities

Copyright 2007 by the American Academy of Pediatrics

page 501

2007 PREP SA on CD-ROM Critique: 240 Preferred Response: C

The child described in the vignette has lost consciousness as a result of head trauma inflicted during a motor vehicle collision. He should be assumed to have a cervical spine injury until proven otherwise. Full immobilization with a cervical collar and a long spine board are indicated during transportation to a medical facility. Failure to immobilize his neck may cause or worsen a cervical spine injury. Motor vehicle accidents are the leading cause of spinal trauma in children, followed by falls. The incidence of spinal cord injury was approximately 1.5% in two studies that used national trauma registry data. Approximately one third of children who have spinal trauma have associated spinal cord damage. Unlike adults, in whom recovery from spinal cord damage is rare, children show a high rate of partial neurologic recovery from such injuries. The initial evaluation of an accident victim should focus on the ABCs (airway, breathing, and circulation), followed by a brief neurologic examination to assess for disability or neurologic deficits. As long as the spine is protected, exclusion of a spinal cord injury may be safely deferred until systemic instability has been addressed. Evaluation for cervical spine injury should be undertaken in the conscious patient who exhibits tenderness to palpation over the spine and all patients who have altered levels of consciousness. Lateral, anteroposterior, and odontoid radiographs identify up to 92% of all cervical spine fractures. The lateral films should include the base of the skull, all seven cervical vertebrae, and the first thoracic vertebra. If complete visualization of this region is not possible with the lateral view, a swimmers view of the lower cervical and upper thoracic area should be obtained. If there is still difficulty in visualizing this area, computed tomography scan of the neck should be ordered. In the presence of a significant fracture in a comatose patient or a neurologic deficit in a conscious patient, magnetic resonance imaging should be obtained to provide the highest quality images of the spinal cord. Pharmacologic therapy with methylprednisolone after acute cervical spinal cord injury has been recommended in the past after initial studies appeared to show benefit. Later studies have not confirmed the benefit of methylprednisolone. This is a controversial area, with no clear consensus on the standard of care. The current Advanced Trauma Life Support Manual continues to recommend its use. The transfer of the child in the vignette via private automobile should not cause a delay in the administration of either intravenous fluids or supplemental oxygen. It is unlikely that a vascular injury would be worsened by this mode of transport. Finally, it is helpful but not essential to notify emergency department personnel of an arriving trauma victim. References: American College of Surgeons. Spine and spinal cord trauma. In: Advanced Trauma Life Support Program for Doctors. 6th ed. Chicago, Ill: American College of Surgeons; 1997:215230 Carreon LY, Glassman SD, Campbell MJ. Pediatric spine fractures: a review of 137 hospital admissions. J Spinal Disord Tech. 2004;17:477-482. Abstract available at: http://www.ncbi.nlm.nih.gov/entrez/query.fcgi?orig_db=PubMed&db=PubMed&cmd=Search&ter m=J+Spinal+Disord+Tech[Jour]+AND+477[page]+AND+2004[pdat] Martin BW, Dykes E, Lecky FE. Patterns and risks in spinal trauma. Arch Dis Child. 2004;89:860865. Available at: http://adc.bmjjournals.com/cgi/content/full/89/9/860 Pharmacological therapy after acute cervical spinal cord injury. Neurosurgery. 2002;50(suppl):S63-S72. Abstract available at: http://www.ncbi.nlm.nih.gov/entrez/query.fcgi?cmd=Retrieve&db=pubmed&dopt=Abstract&list_ui ds=12431289&query_hl=38&itool=pubmed_docsum Wang MY, Hoh DJ, Leary SP, Griffith P, McComb JG. High rates of neurological improvement
Copyright 2007 by the American Academy of Pediatrics page 502

2007 PREP SA on CD-ROM

following severe traumatic pediatric spinal cord injury. Spine. 2004;29:1493-1497. Abstract available at: http://www.ncbi.nlm.nih.gov/entrez/query.fcgi?orig_db=PubMed&db=PubMed&cmd=Search&ter m=Spine[Jour]+AND+1493[page]+AND+2004[pdat]

Copyright 2007 by the American Academy of Pediatrics

page 503

2007 PREP SA on CD-ROM Question: 241

A 1-week-old infant presents for his first newborn evaluation. He had been discharged apparently well and thriving at 48 hours of age. He now exhibits grouped vesicles (Item Q241A) on an erythematous base that were not present at birth. Wright stain of scrapings from the floor of the vesicles reveals multinucleated giant cells (Item Q241B) and balloon cells. Of the following, the MOST likely diagnosis is

A. bullous impetigo B. congenital varicella C. herpes simplex virus infection D. incontinentia pigmenti E. recessive dystrophic epidermolysis bullosa

Copyright 2007 by the American Academy of Pediatrics

page 504

2007 PREP SA on CD-ROM Critique: 241 Preferred Response: C

Grouped vesicles (Item C241A) on an erythematous base, as described for the infant in the vignette, suggests neonatal herpes simplex virus (HSV) infection. Lesions may erupt anywhere, but vesicles on the scalp or buttocks are particularly common. Monitoring electrodes may produce sufficient skin trauma to expedite invasion by HSV and induce skin lesions. Vesicles may be present at birth, but onset after delivery is more likely. Up to 30% of infants who have neonatal HSV do not exhibit skin lesions. Mucous membrane involvement is common. HSV in newborns may present as: 1) disseminated disease involving multiple organs; 2) localized central nervous system disease; or 3) disease localized to the skin, eyes, and mouth. In the absence of skin lesions, it is difficult to diagnose neonatal HSV infection. In neonates, HSV infection should be considered in the differential diagnosis of fever, irritability, and abnormal cerebrospinal fluid findings. Neonatal HSV infections are very serious and associated with high mortality and morbidity rates. Initial symptoms of HSV infection may present between birth and 1 month of age. Bullous impetigo (Item C241B) is a common skin manifestation of Staphylococcus aureus infection. As the name suggests, lesions are bullae (or crusted round erosions if the bullae rupture), not vesicles. Gram stain of the contents of a bulla reveals clusters of gram-positive cocci, and a bacterial culture can confirm the presence of Staphylococcus aureus. Congenital varicella is rare, but may mimic HSV infection in the newborn. Affected infants exhibit vesicles in association with abnormalities involving the eye, central nervous system, gastrointestinal system, or limbs. A linear arrangement of vesicles that have minimal erythema at the base is characteristic of incontinentia pigmenti (Item C241C). Rapid development of the warty hyperkeratotic stage distinguishes incontinentia pigmenti from HSV infection. Hemorrhagic bullae appear at sites of even minimal skin trauma in newborns who have recessive dystrophic epidermolysis bullosa (RDEB). Removal of the blister roof leaves a raw, bleeding base that heals with scar formation. Early loss of finger nails as a result of blistering and scarring is characteristic of RDEB. References: American Academy of Pediatrics. Herpes simplex. In Pickering LK, ed. Red Book: 2006 Report of the Committee on Infectious Diseases. 27th ed. Elk Grove Village, Ill: American Academy of Pediatrics; 2006:361-371 Kohl S. Herpes simplex virus. In: Behrman RE, Kliegman RM, Jenson HB, eds. Nelson Textbook of Pediatrics. 17th ed. Philadelphia, Pa: WB Saunders Co; 2004:1051-1056 Weston WL, Lane AT, Morelli JG. Genodermatoses: epidermolysis bullosa, ectodermal dysplasias, incontinentia pigmenti. In: Color Textbook of Pediatric Dermatology. 3rd ed. St. Louis, Mo: Mosby; 2002:274-283 Weston WL, Lane AT, Morelli JG. Skin diseases in newborns: transient skin disease. In: Color Textbook of Pediatric Dermatology. 3rd ed. St. Louis, Mo: Mosby; 2002:299-308

Copyright 2007 by the American Academy of Pediatrics

page 505

2007 PREP SA on CD-ROM Question: 242

The father of three children in your practice recently was diagnosed with Crohn disease. His wife does not have Crohn disease. He asks you if his children, ages 10, 12, and 16 years, are at increased risk for developing the same illness. Of the following, you are MOST likely to advise the father that

A. although his children are at increased risk of developing Crohn disease, their risk of developing
ulcerative colitis is decreased

B. Crohn disease in childhood usually presents in children younger than age 5 years C. each of his children has at least a 20% chance of developing Crohn disease during his or her
lifetime

D. most patients who have Crohn disease can be diagnosed by genetic testing E. smoking is associated with an increased risk of developing Crohn disease

Copyright 2007 by the American Academy of Pediatrics

page 506

2007 PREP SA on CD-ROM Critique: 242 Preferred Response: E

Crohn disease is an illness characterized by regional intestinal inflammation, most commonly involving the terminal ileum or colon. However, the upper small bowel and stomach also may be involved. Presenting features of Crohn disease include abdominal pain, diarrhea, rectal bleeding, growth failure, or perianal inflammation. Diagnosis typically is established by a combination of radiologic imaging and endoscopy. Similar to coronary artery disease or diabetes, inflammatory bowel diseases (Crohn disease and ulcerative colitis) are complex polygenic conditions that result from interactions between genetic predisposition and the environment. People of northern European and Jewish descent are at increased risk of Crohn disease. Family history is a known risk factor for both inflammatory bowel diseases, and probands who have one type (eg, Crohn) may have a relative who has the other type (eg, ulcerative colitis). Thus, if a father has Crohn disease, the risk of his children developing either Crohn disease or ulcerative colitis is increased over the general population. Between 5% and 20% of patients who have Crohn disease have a similarly affected first-degree relative. Crohn disease typically presents in adolescence and young adulthood and is rare in children younger than age 5 years. The genetics and epidemiology of Crohn disease are under active investigation. In 2001, mutations in the NOD2 gene were identified in approximately 25% of patients who had Crohn disease. This gene is found primarily in human macrophages and is involved in the response of the immune system to bacterial lipid antigens (innate immunity). It is, therefore, theorized that the intestinal inflammation in the disease occurs in patients genetically predisposed to immune activation by common bacterial antigens. Currently, although genetic testing for NOD2 mutations is available, there is no genetic test that detects all types of Crohn disease. Environmental triggers may play a role in initiating the disease. The best established environmental risk factor for Crohn disease is smoking, with many epidemiologic studies suggesting a twofold relative risk in smokers. References: Hyams JS. Inflammatory bowel disease. Pediatr Rev. 2005;26:314-320 . Available at: http://pedsinreview.aappublications.org/cgi/content/full/26/9/314 Griffiths A, Hugot J-P. Crohn disease. In: Walker WA, Goulet O, Kleinman RE, Sherman PM, Shneider BL, Sanderson IR, eds. Pediatric Gastrointestinal Disease. 4th ed. Hamilton, Ontario, Canada: BC Decker; 2005:789-824

Copyright 2007 by the American Academy of Pediatrics

page 507

2007 PREP SA on CD-ROM Question: 243

You are admitting a 35-week gestation newborn to the neonatal intensive care unit for respiratory distress at 4 hours of age. She requires assisted ventilation. The resident working with you asks what test of pulmonary function is preferred in your initial assessment of this newborn. Of the following, the BEST response is

A. arterial blood gas testing B. capillary blood gas testing C. end-tidal carbon dioxide monitoring D. pulse oximetry monitoring E. transcutaneous Pao2 monitoring

Copyright 2007 by the American Academy of Pediatrics

page 508

2007 PREP SA on CD-ROM Critique: 243 Preferred Response: A

A newborn who has respiratory distress and needs assisted ventilation, such as the patient described in the vignette, requires monitoring of the adequacy of both oxygenation (Pa2) and ventilation (Pa2). Numerous monitoring methods have emerged over the years, but the preferred method for assessing overall pulmonary function is arterial blood gas testing. Standard methods exist for sampling arterial, venous, or arterialized capillary blood to measure blood gases. Such measures (and normal newborn ranges) include the pH (7.35 to 7.45), Pa2 (35 to 45 mm Hg), and Pa2 (50 to 80 mm Hg). Standard nomograms employing these measurements can calculate the serum bicarbonate and the base excess (or deficit). Using a co-oximeter, whole blood hemoglobin saturation also can be measured. Obviously, such blood gas samples must be obtained via an invasive arterial or venous catheter, arterial puncture, venipuncture, or heel-stick. The best representative of adequate pulmonary function is the arterial blood gas. The capillary blood gas sample is the least reliable manner to measure physiologic and biochemical stability of the acutely ill newborn who has respiratory distress. The limits imposed by this technique include adequacy of tissue perfusion at the site of collection (heel), which is addressed by heel warming to 39 to 42F for several minutes; circulatory volume; and vasoconstriction due to circulatory shock or hypothermia. Capillary blood also rarely can be acquired without disturbing the infant. Hence, even the best sample of arterialized capillary blood may yield spuriously elevated Pa2. The P2 measurement from the capillary bed is not the same as arterial Pa2 and only approximates it at best. In the presence of a dynamically changing respiratory status in an ill newborn, the capillary blood gas can be used only to estimate gross changes in Pa2 or pH. Oxygenation is monitored best by arterial blood gas sampling or continuous pulse oximetry. Pulse oximetry is a measure of the arterial hemoglobin saturation acquired by infrared lightemitting diode technology across an extremity (finger, toe, hand) that is read by a detector on the same extremity. The absorption of light by saturated hemoglobin contributes to the calculated and averaged reading that is typically displayed in a digital readout on a bedside monitor. Hemoglobin saturation and its correlation with arterial Pa2 may be affected by temperature, acid-base balance, adequacy of 2,3-diphosphoglycerol content of the erythrocytes, and the presence of fetal hemoglobin. Normal values in the newborn are 88% to 96% saturation. The normal hemoglobin-oxygen saturation curve is widely available in standard textbooks. It must be correlated with an arterial blood gas sample at some point in time to know best how adequately the pulse oximetry value represents the true arterial oxygenation of the patient. This methodology does not provide any assessment of ventilation (no measure of Pa2). Transcutaneous Pa2 monitoring is a limited and somewhat dated technology that relies on heated skin allowing for capillary blood oxygen to be measured across a membrane by a silver electrode. The technology requires frequent machine calibration, changing the electrode site every 4 hours to avoid skin damage, and correlation with arterial blood gas Pa2. It does not provide a measure of ventilation. End-tidal P2 may be measured using an adapter on the connecting apparatus for the endotracheal tube to the ventilator. The gas typically is sampled through a side port, and averaged values are read on a monitor. It is best used to follow trends in P2; real-time correlation with arterial blood gas Pa2 varies. It is subject to error with the accumulation of moisture in the circuit over time. It does not provide a measure of oxygenation. References: Durand DJ, Phillips B, Boloker J. Blood gases: technical aspects and interpretation. In: Goldsmith JP, Karotkin EH, eds. Assisted Ventilation of the Neonate. 4th edition. Philadelphia, Pa: Saunders; 2003:279-292

Copyright 2007 by the American Academy of Pediatrics

page 509

2007 PREP SA on CD-ROM

Hay WW Jr. History of pulse oximetry in neonatal medicine. NeoReviews. 2005;6:e533-e538. Available at: http://neoreviews.aappublications.org/cgi/content/full/6/12/e533 Henning R, South M. Respiratory failure. In: Taussig LM, Landau LI, eds. Pediatric Respiratory Medicine. St. Louis, Mo: Mosby; 1999:404-430 Wood BR. Physiologic principles. In: Goldsmith JP, Karotkin EH, eds. Assisted Ventilation of the Neonate. 4th ed. Philadelphia, Pa: Saunders; 2003:15-40

Copyright 2007 by the American Academy of Pediatrics

page 510

2007 PREP SA on CD-ROM Question: 244

A 2-year-old girl is brought to your clinic because her mother noticed blood in her underwear that morning. She has otherwise been doing well, and she recently has been toilet trained. Findings on the physical examination, including the hymen and external genitalia, are normal. There is a small amount of purulent, bloody discharge at the vaginal introitus. Of the following, the MOST likely cause of her bleeding is

A. penetrating trauma B. precocious puberty C. sarcoma botryoides D. urethral prolapse E. vaginal foreign body

Copyright 2007 by the American Academy of Pediatrics

page 511

2007 PREP SA on CD-ROM Critique: 244 Preferred Response: E

The differential diagnosis of vaginal bleeding in preadolescent girls varies with age. The presence of maternal estrogens can cause scant, often bloody, vaginal discharge in neonates in the first few weeks after birth. In toddlers and older children, a foreign body in the vagina (frequently toilet paper) is a common cause of vaginal bleeding. Foul-smelling, bloody discharge is evident on physical examination, and the foreign body may not be seen initially. Vulvovaginitis, which may be nonspecific or associated with group A streptococcal infection, also may result in vaginal bleeding. Other less common causes that may not be evident on physical examination include endodermal carcinoma, clear cell adenocarcinoma, and capillary venous malformation. Evidence of penetrating trauma may not always be present in the case of sexual abuse, so a thorough history should be obtained and a high index of suspicion should be maintained despite normal examination findings. Precocious puberty can cause vaginal bleeding, but other signs such as thelarche would be expected. Urethral prolapse is often confused with vaginal bleeding. Physical examination findings include a doughnut-shaped mass (Item C244A) at the vaginal introitus and blood in the underwear. Sarcoma botryoides is a polypoid form of rhabdomyosarcoma that may be apparent on physical examination as a grapelike mass at the introitus. References: Lang ME, Darwish A, Long AM. Vaginal bleeding in the prepubertal child. Can Med Assoc J. 2005;172:1289-1290. Available at: http://www.cmaj.ca/cgi/content/full/172/10/1289 Sanfilippo JS. Bleeding. In: Behrman RE, Kliegman RM, Jenson HB, eds. Nelson Textbook of Pediatrics. 17th ed. Philadelphia, Pa: WB Saunders Co; 2004:1833

Copyright 2007 by the American Academy of Pediatrics

page 512

2007 PREP SA on CD-ROM Question: 245

During a health supervision visit of a 6-year-old child, you ask the mother if there are any guns in the home. She states that her husband is a hunter, but he keeps his shotgun in his pickup truck. Of the following, the BEST anticipatory guidance with regard to firearm safety is to tell the mother to

A. enroll herself and her child in gun safety classes B. ensure that she specifically asks if other guns are in the home C. ensure that there are gun safety locks on the shotgun D. insist that the gun be stored in a locked gun cabinet or safe with ammunition locked separately E. teach the child to use the gun properly at the earliest possible age

Copyright 2007 by the American Academy of Pediatrics

page 513

2007 PREP SA on CD-ROM Critique: 245 Preferred Response: D

Potential for injury in a household with children that also contains firearms is a significant concern. According to the American Academy of Pediatrics policy statement on firearm injuries that was reaffirmed in 2004, unintentional shootings account for 24% of firearm-related deaths in children younger than 5 years of age, 26% in children 5 through 9 years of age, 21% in children 10 through 14 years of age, and 5% in adolescents 15 through 19 years of age. It is imperative that pediatricians address the presence of guns in the home at health supervision visits and that anticipatory guidance include discussion of best safety practices. Clearly, for young children, such as the boy described in the vignette, locking unloaded firearms in a cabinet, with ammunition stored separately in another locked location, is best. Due to the potential for nonaccidental firearm injury among adolescents, surveillance for signs of depression and behavioral concerns, in addition to maintaining the previously noted gun storage safety practices, is paramount. Suicide is the leading cause of death in adolescence and has a higher incidence when both guns and alcohol are accessible. Recent studies suggest that the mother may be unfamiliar with the ownership of and storage practices for firearms in the household. All parents should be encouraged to ask other family members if they own firearms and where they are located. Gun safety locks are helpful but not as effective as storing the gun safely in a locked gun cabinet away from ammunition. Gun safety classes have not been shown to reduce the risk of firearm injury for children. Indeed, they may increase the risk, perhaps because of decreased parental vigilance after the child has completed the course. References: American Academy of Pediatrics Committee on Injury and Poison Prevention. Firearm-related injuries affecting the pediatric population. Pediatrics. 2000;105:888-895. Available at: http://pediatrics.aappublications.org/cgi/content/full/105/4/888 Azrael D, Miller M, Hemenway D. Are household firearms stored safely? It depends on whom you ask. Pediatrics. 2000;106:e31. Available at: http://pediatrics.aappublications.org/cgi/content/full/106/3/e31 Brent RL, Weitzman M. The pediatricians role and responsibility in educating parents about environmental risks. Pediatrics. 2004;113:1167-1172. Available at: http://pediatrics.aappublications.org/cgi/content/full/113/4/S1/1167

Copyright 2007 by the American Academy of Pediatrics

page 514

2007 PREP SA on CD-ROM Question: 246

You are called to see a hospitalized 9-year-old girl who suddenly has become dystonic, with her neck hyperextended, and is unable to move her eyes, now superiorly deviated. The nurses relate that this girl has non-Hodgkin lymphoma and has been receiving highly emetogenic chemotherapy. Of the following, the drug MOST likely to have caused this girl's symptoms and signs is

A. aprepitant B. diphenhydramine C. lorazepam D. metoclopramide E. ondansetron

Copyright 2007 by the American Academy of Pediatrics

page 515

2007 PREP SA on CD-ROM Critique: 246 Preferred Response: D

Metabolic disorders, degenerative diseases, and infections can produce movement disorders, but drug exposures also should be considered as causes of ataxia, tremor, and dystonia. Ataxia can be triggered by anticonvulsant toxicity from phenytoin, phenobarbital, and even carbamazepine. Alcohol and thallium also can lead to ataxia. Drugs that can cause tremor include amphetamines, valproic acid, neuroleptics such as phenothiazines, tricyclic antidepressants, caffeine, and theophylline. Stimulant medications can unmask Tourette syndrome, but by themselves do not produce tics. Certain drugs are capable of producing an acute dystonic reaction in children. Therapeutic doses of phenytoin or carbamazepine rarely cause progressive dystonia in children who have epilepsy and an underlying structural abnormality of the brain. Children may have an idiosyncratic reaction to neuroleptic drugs, characterized by acute dystonic posturing that may be confused with encephalopathy. Intravenous diphenhydramine, 1 to 2 mg/kg per dose, may reverse the drug-related dystonia rapidly. Severe rigidity combined with high fever and delirium also may occur as part of the neuroleptic malignant syndrome a few days after the initiation of neuroleptic drugs. The 9-year-old girl described in the vignette is experiencing a classic acute dystonic reaction, with posturing and even oculogyric crisis. Metoclopramide is in the family of neuroleptic drugs, although used most often for nausea or gastrointestinal motility and most likely is the cause of the dystonic reaction. The antiemetics lorazepam and ondansetron do not induce movement disorders, and diphenhydramine is used to reverse dystonia from metoclopramide. Aprepitant is a new substance P/neurokinin-1 antagonist used for chemotherapy-induced nausea and vomiting. It can cause weakness or dizziness, but does not cause movement disorders. References: Johnston MV. Movement disorders. In: Behrman RE, Kliegman RM, Jenson HB, eds. Nelson Textbook of Pediatrics. 17th ed. Philadelphia, Pa: WB Saunders Co; 2004:2019-2023 Pranzatelli MR. Movement disorders in childhood. Pediatr Rev. 1996;17:388-394

Copyright 2007 by the American Academy of Pediatrics

page 516

2007 PREP SA on CD-ROM Question: 247

You are called to evaluate an 18-month-old infant who was found playing with a medication bottle. The top was open, and many of the pills were on the floor. The prescription was for a tricyclic antidepressant used by the patient's older brother. On physical examination, the boy appears clumsy but awake and flushed. He has a heart rate of 150 beats/min, a respiratory rate of 28 breaths/min, and dilated pupils. He is breathing easily and has strong pulses. Of the following, the MOST important next step in this patient's management is

A. administration of syrup of ipecac B. electrocardiography and continuous cardiac monitoring C. lidocaine therapy to prevent ventricular arrhythmias D. measurement of serum drug levels for tricyclic antidepressants E. measurement of serum electrolyte levels

Copyright 2007 by the American Academy of Pediatrics

page 517

2007 PREP SA on CD-ROM Critique: 247 Preferred Response: B

Tricyclic antidepressants (TCAs) are a commonly prescribed group of drugs for several disorders, although the introduction of the newer classes of antidepressants, serotonin reuptake inhibitors, may reduce the use of TCAs. Nonetheless, toxicity from TCAs remains a major cause of morbidity and mortality in children. With their widespread use in older children, adolescents, and adults, toddlers continue to be at risk for accidental ingestion of TCAs. When TCAs are ingested in toxic amounts, they primarily affect the central nervous and cardiovascular systems. Central nervous system signs and symptoms of TCA toxicity include irritability, euphoria, seizures, and unresponsiveness. There also may be autonomic nervous system symptoms, such as mydriasis, dry skin, dry mouth, urinary retention, and tachycardia. Among the direct effects on the cardiac system are a delay in signal conduction through the bundle of His, depression of myocardial contractile function, and prolongation of the QRS and the QT intervals. These latter cardiac effects may potentiate arrhythmia formation. Electrocardiography (ECG) with subsequent cardiac monitoring is essential in the evaluation of the child suspected of having a TCA toxic ingestion. Monitoring the ECG complex should be continued for several hours because cardiac dysrhythmias may occur late after the ingestion. The voltage intervals should be measured, with particular attention to the QRS duration and the QT interval. A QRS duration of greater than 100 msec is associated with the development of seizures; a QRS duration of more than 160 msec is associated with ventricular dysrhythmias that may be particularly difficult to treat. Measuring serum drug concentrations of TCAs is not helpful in the prognosis or management of TCA ingestion. Serum electrolytes may be measured, but the results do not carry predictive value in the management of TCA toxicity. Providing syrup of ipecac is not indicated in patients who have ingested TCAs. Lidocaine therapy is not indicated in the absence of ventricular arrhythmias. References: Kerr GW, McGuffie AC, Wilkie S. Tricyclic antidepressant overdose: a review. Emerg Med J. 2001;18:236-241. Available at: http://emj.bmjjournals.com/cgi/content/full/18/4/236 Michael JB, Sztajnkrycer MD. Deadly pediatric poisons: nine common agents that kill at low doses. Emerg Med Clin North Am. 2004;22:1019-1050. Abstract available at: http://www.ncbi.nlm.nih.gov/entrez/query.fcgi?db=pubmed&cmd=Retrieve&dopt=AbstractPlus&li st_uids=15474780 Prez-Fontn J, Lister G. The acutely ill infant and child. In: Rudolph CD, Rudolph AM, Hostetter MK, Lister G, Siegel NJ, eds. Rudolphs Pediatrics. 21st ed. New York, NY: McGraw-Hill; 2003:271-400

Copyright 2007 by the American Academy of Pediatrics

page 518

2007 PREP SA on CD-ROM Question: 248

In examining a 4-year-old girl who is new to your practice, you discover that she has rudimentary thumbs and is well below the 5th percentile for both weight and height. You also observe irregular hyperpigmentation on the trunk and anogenital areas. Of the following, the MOST likely hematologic disorder associated with these findings is

A. acute lymphoblastic leukemia B. Bloom syndrome C. Diamond-Blackfan anemia D. Fanconi anemia E. thrombocytopenia and absent radii (TAR) syndrome

Copyright 2007 by the American Academy of Pediatrics

page 519

2007 PREP SA on CD-ROM Critique: 248 Preferred Response: D

Approximately 80% of patients who have Fanconi anemia exhibit congenital anomalies. Recognizing these anomalies is important because the aplastic anemia usually is not evident until 4 to 14 years of age. Pancytopenia at birth is unusual. Among the unusual features described are hyperpigmentation (60% of patients); short stature (57%); upper limb anomalies (48%) that include absent, hypoplastic, supernumerary, or bifid thumbs and aplasia of the first metacarpal or the radius; hypogonadism (40%); and microcephaly (27%). Almost 20% of affected patients manifest a variety of other skeletal anomalies. The only potential cure for this autosomal recessive disorder is bone marrow transplantation, and early diagnosis allows for better donor identification and patient preparation. It is also important to make the diagnosis to enable parents to choose prenatal diagnosis for future pregnancies. Several complementation groups have been identified in Fanconi anemia, with type A accounting for almost 66% of cases. If the specific complementation group can be identified in the index case, DNA analysis can be used for prenatal diagnosis. If a specific DNA defect is not identified, increased chromosomal breakage in peripheral blood lymphocytes in the presence of cross-linking agents can be used. Thrombocytopenia and absent radii (TAR) syndrome is also known as congenital amegakaryocytic thrombocytopenia. The classic finding of absent radii and present thumbs differentiates it from Fanconi anemia (Item C248A). Anomalies of hands, shoulders, and lower limbs also may be seen. TAR presents with isolated thrombocytopenia in the neonate, but 45% of patients ultimately develop aplastic anemia. Inheritance is complex but is usually autosomal recessive. Diamond-Blackfan anemia, or congenital hypoplastic anemia, is associated with physical abnormalities in approximately 25% of patients. Anomalies include dysmorphic facial features, micrognathia, flattening of the thenar eminences, and triphalangeal thumbs. Bloom syndrome is a rare chromosomal breakage syndrome in which several skeletal anomalies may be found; the major hematologic risk is development of leukemia and other malignancies. Only mild anemia is typical. Acquired disorders also may be associated with both hematologic and skeletal abnormalities. Children who have newly diagnosed acute lymphoblastic leukemia often present with bone pain and generalized radiologic signs, but they do not have congenital abnormalities. References: DAndrea AD. The constitutional pancytopenias. In: Behrman RE, Kliegman RM, Jenson HB, eds. Nelson Textbook of Pediatrics. 17th ed. Philadelphia Pa: Saunders; 2004:1642-1644 Wong W, Glader B. Approach to the newborn who has thrombocytopenia. NeoReviews. 2004;5:e444. Available at: http://neoreviews.aappublications.org/cgi/content/full/5/10/e444

Copyright 2007 by the American Academy of Pediatrics

page 520

2007 PREP SA on CD-ROM Question: 249

A 14-year-old girl presents with complaints of lower abdominal cramps that have occurred on days 1 and 2 of her menstrual cycle for the past year. The cramps usually are associated with bloating and loose bowel movements. The symptoms have caused school absences on the first day of the past three periods. Use of acetaminophen has provided no relief. Menarche was at age 12 years. Her menstrual cycle is regular; menses last 5 days and are characterized by moderate flow. Of the following, the MOST appropriate management of these symptoms is the use of

A. acetaminophen with codeine B. fluoxetine C. montelukast D. naproxen sodium E. tramadol hydrochloride

Copyright 2007 by the American Academy of Pediatrics

page 521

2007 PREP SA on CD-ROM Critique: 249 Preferred Response: D

Dysmenorrhea is the most common menstrual disorder among adolescents. Most studies report that more than 50% of menstruating 12- to 17-year-old girls experience dysmenorrhea with each menstrual cycle. Primary dysmenorrhea is defined as crampy lower abdominal and pelvic pain that occurs with menses and is not associated with pelvic pathology. It is the most common form of dysmenorrhea and is caused by prostaglandins that are produced during ovulatory cycles that usually begin 6 to 12 months after menarche. Prostaglandins increase resting myometrial tone and stimulate uterine contractions that lead to myometrial ischemia and pain. Systemic circulation of prostaglandins causes headaches, nausea, vomiting, and diarrhea. Numerous studies have documented the effectiveness of nonsteroidal anti-inflammatory drugs (NSAIDs) that inhibit prostaglandins for the treatment of dysmenorrhea; the most commonly recommended of which are ibuprofen and naproxen sodium. When initiated early for cramps and used consistently, they relieve symptoms in most adolescent patients who have dysmenorrhea. For adolescents who have primary dysmenorrhea and fail to improve with adequate NSAID therapy, a contraceptive containing estrogen and progesterone may be beneficial. The lack of responsiveness to acetaminophen experienced by the girl in the vignette is expected; acetaminophen is a weak prostaglandin inhibitor. The use of acetaminophen with codeine may provide pain relief, but fails to address the underlying mechanism of pain in dysmenorrhea and may be associated with undesired adverse effects. Fluoxetine, a selective serotonin reuptake inhibitor, has been beneficial for the treatment of premenstrual syndrome and menstrual dysphoric disorder in women. Montelukast, a leukotriene receptor antagonist, has an anti-inflammatory effect that is useful for the treatment of asthma, but not dysmenorrhea. Tramadol hydrochloride is an analgesic that does not inhibit prostaglandins and, therefore, is not useful in the management of dysmenorrhea. References: Braverman PK, Neinstein LS. Dysmenorrhea and premenstrual syndrome. In: Neinstein LS, ed. Adolescent Health Care A Practical Guide.. 4th ed. Philadelphia, Pa: Lippincott Williams & Wilkins; 2002:952-965 Davis AR, Westhoff CL. Primary dysmenorrhea in adolescent girls and treatment with oral contraceptives. J Pediatr Adolesc Gynecol. 2001;14:3-8. Abstract available at: http://www.ncbi.nlm.nih.gov/entrez/query.fcgi?cmd=Retrieve&db=pubmed&dopt=Abstract&list_ui ds=11358700&query_hl=40&itool=pubmed_docsum Jenkins RR. Menstrual problems. In: Behrman RE, Kliegman RM, Jenson HB, eds. Nelson Textbook of Pediatrics. 17th ed. Philadelphia, Pa: Saunders; 2004:663-667 Laufer MR, Goldstein DP. Gynecologic pain: dysmenorrhea, acute and chronic pelvic pain, endometriosis, and the premenstrual syndrome. In: Emans SJH, Laufer MR, Goldstein DP, eds. Pediatric and Adolescent Gyencology.. 5th ed. Philadelphia, Pa: Lippincott, Williams & Wilkins; 2004:417-476

Copyright 2007 by the American Academy of Pediatrics

page 522

2007 PREP SA on CD-ROM Question: 250

During a health supervision visit with a 12-year-old girl, you palpate a firm thyroid gland (Item Q250A). Measurement of free thyroxine is 2.0 ng/dL (25.7 pmol/L) (normal, 0.6 to 1.6 ng/dL [7.7 to 20.6 pmol/L]), thyroid-stimulating hormone is less than 0.01 mcIU/mL (0.01 mcIU/L) (normal, 0.5 to 5.0 mcIU/mL [0.5 to 5.0 mcIU/L]), and triiodothyronine is 468 pg/dL (7.2 pmol/L) (normal, 60 to 180 pg/dL [0.92 to 2.8 pmol/L]). She has fine, smooth skin and finger and tongue tremor. Of the following, the MOST likely additional finding on physical examination is

A. hepatomegaly B. lymphadenopathy C. positive Chvostek sign D. pretibial myxedema E. thyroid bruit

Copyright 2007 by the American Academy of Pediatrics

page 523

2007 PREP SA on CD-ROM Critique: 250 Preferred Response: E

The laboratory results reported for the girl in the vignette are indicative of hyperthyroidism. Physical findings of hyperthyroidism can include a thyroid bruit because of the blood coursing through a hyperactive and simulated thyroid gland and, in extremely rare instances in children, pretibial myxedema. The latter finding is more common in adults, but still very rare. Other symptoms and signs include widened palpebral fissures or actual ophthalmopathy (less common in children than adults), nervousness and irritability, sweatiness, myopathy, difficulty concentrating in school and sleeping, daytime tiredness, weight loss, diarrhea, and heat intolerance. Hyperthyroidism should not cause hypocalcemia or intensify the Chvostek sign. Hepatomegaly is not a sign of hyperthyroidism, although occasional individuals who experience a thyroid storm may have hepatic disease, including jaundice. Lymphadenopathy is not a finding in thyrotoxicosis. References: Gold JG, Sadeghi-Nejad A. Hyperthyroidism. eMedicine Specialties: Pediatrics: Endocrinology. 2006. Available at: http://www.emedicine.com/ped/topic1099.htm LaFranchi S. Hyperthyroidism in childhood and adolescence. UptoDate. 2006; 14.1. Available at: http://uptodateonline.com/utd/content/topic.do?file=pediendo/5570&type=A&selectedTitle=3~131 Levitsky LL. Graves disease. eMedicine Specialities: Pediatrics: Endocrinology. 2004. Available at: http://www.emedicine.com/ped/topic899.htm

Copyright 2007 by the American Academy of Pediatrics

page 524

2007 PREP SA on CD-ROM Question: 251

You are seeing a new patient who is 11 years old and has Duchenne muscular dystrophy. His uncle died of the disorder 2 years ago. His mother asks to see you privately and tells you that her son does not know his diagnosis; she asks that you refer to his "walking problem." On physical examination, you note that the boy is in a wheelchair and is unable stand without support. You ask to speak with the mother after evaluating the boy. Of the following, you are MOST likely to tell her that

A. he likely knows his diagnosis B. you agree to her plan C. you are recommending psychological evaluation for the boy D. you are referring the family to child protective services E. you are required to tell the boy his diagnosis

Copyright 2007 by the American Academy of Pediatrics

page 525

2007 PREP SA on CD-ROM Critique: 251 Preferred Response: A

Family members of a child who has a serious illness grieve for their childs illness and the loss of normal childhood. Denial is common, and physicians can help families accept a diagnosis while maintaining hope for improvement or recovery. It is important to ask about the parents understanding of the diagnosis. Although family responses vary, based on history and culture, most parents require time to process and review new information, especially when they are being given an unexpected and unwanted diagnosis. The boy in the vignette likely knows his diagnosis because he has an uncle who recently died with the same disorder. His mother is trying to protect him from the diagnosis and herself from having to discuss the diagnosis and prognosis with him. A primary care physician can facilitate discussions regarding diagnosis and prognosis in cooperation with the family. Appropriate supportive personnel differ for specific situations, but social workers, counselors, community support groups, and clergy are examples of people who may be supportive to the family during this process. Although there is no requirement to tell the boy his diagnosis, doing so would be beneficial. Therefore, working with the mother to achieve this goal is desirable. Counseling may be appropriate for both child and family members, but there is no specific need for psychological evaluation. Likewise, there is no need to refer the boy to child protective services unless he is not receiving appropriate care. Rather than agreeing to the mothers plan, the clinician should help her discuss the diagnosis with her son. References: Perrin JM. Chronic illness in childhood. In: Behrman RE, Kliegman RM, Jenson HB, eds. Nelson Textbook of Pediatrics. 17th ed. Philadelphia, Pa: WB Saunders Co; 2004:135-138

Copyright 2007 by the American Academy of Pediatrics

page 526

2007 PREP SA on CD-ROM Question: 252

A 5-year-old girl presents after having a brief generalized seizure. Her mother reports that the child has had a 3-day history of fever, tenesmus, and bloody diarrhea. On physical examination, you find a mildly toxic-appearing child who has a temperature of 104F (40C) and diffuse abdominal tenderness. The rectal examination produces significant pain. Stool from her rectum is guaiac-positive. You tell her mother that you believe the diarrhea has an infectious cause. Of the following, the MOST likely pathogen is

A. Cryptosporidium sp B. rotavirus C. Salmonella sp D. Shigella sp E. Yersinia sp

Copyright 2007 by the American Academy of Pediatrics

page 527

2007 PREP SA on CD-ROM Critique: 252 Preferred Response: D

Patients who have symptoms of infections due to Cryptosporidium sp or rotavirus usually do not appear toxic, have fever, or have blood in their stool. Infections with Salmonella or Yersinia sp can produce these symptoms, but the degree of toxicity and tenesmus described for the patient in the vignette suggests an infection caused by Shigella sp. Four species of Shigella have been identified: S sonnei, S flexneri, S boydii, and S dysenteriae. In industrialized countries, S sonnei and S flexneri are the most common pathogens. The virulence trait shared by all species of Shigella is the ability to invade intestine, which generally is more marked than with Salmonella infection. The pathologic changes of infection occur primarily in the colon, with mucosal edema, ulcerations, bleeding tissues, and exudates present upon inspection. Humans are the only natural host for this organism, and the primary mode of transmission is fecal-oral contamination. Neurologic manifestations (eg, seizures) are the most common extraintestinal manifestation of illness and occur in as many as 40% of patients. Such neurologic manifestations do not seem to be accounted for by the presence of Shiga toxin alone; the exact mechanism by which they occur is not well understood. As with other forms of gastroenteritis, clinicians should address fluid and electrolyte problems prior to considering antimicrobial therapy for patients who have Shigella infections. Unlike infections with other pathogens (eg, Salmonella), most experts suggest providing antimicrobial therapy for patients who are ill with infections due to Shigella. Resistance patterns vary by region, and the choice of antimicrobial agent should be guided by the resistance pattern of the infection organism. Children who attend child care must have negative stool cultures prior to their return. References: American Academy of Pediatrics. Shigella infections. In: Pickering LK, ed. Red Book: 2006 Report of the Committee on Infectious Diseases. 27th ed. Elk Grove Village, Ill: American Academy of Pediatrics; 2006:589-591 Cleary TG. Shigella. In: Behrman RE, Kliegman RM, Jenson HB, eds. Nelson Textbook of Pediatrics. 17th ed. Philadelphia, Pa: WB Saunders Co; 2004:919-920

Copyright 2007 by the American Academy of Pediatrics

page 528

2007 PREP SA on CD-ROM Question: 253

A 3-month-old boy is admitted to the hospital for evaluation of failure to thrive. His birthweight was at the 50th percentile and length at the 75th percentile. Currently, his weight is below the 5th percentile and length is at the 25th percentile. His vital signs and physical examination results are otherwise normal. He appears well hydrated. Measurement of serum electrolytes reveals: sodium, 139 mEq/L (139 mmol/L); potassium, 4.7 mEq/L (4.7 mmol/L); chloride, 114 mEq/L (114 mmol/L); bicarbonate 12 mEq/L (12 mmol/L); blood urea nitrogen, 8 mg/dL (2.9 mmol/L); and creatinine, 0.3 mg/dL (26.5 mcmol/L). A consulting nephrologist recommends measurement of urine pH (which is 7.5) and urine ammonium (which is 12,000 mcM/L) (normal, >60,000 mcM/L). Of the following, the MOST likely cause of this child's acidosis is

A. inborn error of metabolism B. lactic acidosis C. type I (distal renal tubular) acidosis D. type II (proximal renal tubular) acidosis E. type IV renal tubular acidosis

Copyright 2007 by the American Academy of Pediatrics

page 529

2007 PREP SA on CD-ROM Critique: 253 Preferred Response: C

The infant described in the vignette presents with failure to thrive, and measurement of his serum electrolytes reveals a normal anion gap and hyperchloremic metabolic acidosis. The key element in his presentation is the urine pH, which is 7.5, indicating that it is highly alkaline. In proximal renal tubular acidosis (RTA), as the urine leaves the proximal tubule, the relative inability to reabsorb bicarbonate results in a very alkaline pH of the tubular fluid. However, the distal tubule in patients who have proximal RTA is healthy, and it extrudes high quantities of acid, dropping the tubular pH to a lower value (=6.0). In contrast, because the distal nephron represents the last real opportunity for the kidney to regulate acid-base balance, children who have distal RTA cannot adequately compensate for the lack of acid extrusion, resulting in very alkaline urine. The urine ammonium value in the child in the vignette is very low at 12,000 mcM/L (normal >60,000 mcM/L), which is consistent with reduced acid (as part of ammonium) secretion and strongly predictive of distal RTA. RTA is a common cause of metabolic acidosis in childhood. There are two primary types of RTA: proximal or type II and distal or type I. Type IV RTA, also called hyperkalemic RTA, is uncommon and will be discussed only briefly. A review of basic renal physiology in the maintenance of acid-base balance is necessary to understand RTA. The primary responsibility of the proximal tubule in the preservation of normal acid-base homeostasis is to reabsorb filtered bicarbonate. It achieves this goal by reabsorbing water and carbon dioxide from the renal (apical membrane) tubule into the cell and then converting them to hydrogen and bicarbonate within the cell. Hydrogen is extruded back into the tubule, and bicarbonate is reabsorbed at the other end (basolateral membrane) of the cell. Thus, the reabsorption of base (bicarbonate) is linked with the expulsion of acid (hydrogen). In contrast, the primary role of the distal tubule is to secrete acid, either as free hydrogen ion, as part of ammonium (ammonia plus hydrogen), or linked to phosphorous. The distal nephron does play a role in bicarbonate reabsorption, but its primary role is to secrete acid. Because children who have proximal (type II) RTA have difficulty reabsorbing bicarbonate, the serum bicarbonate concentration falls, resulting in acidosis. In distal (type I) RTA, the distal tubule struggles to secrete acid. This excess acid must be buffered in the blood, principally by bicarbonate, which also results in a decline in the serum bicarbonate or acidosis. In distal RTA, there may be concomitant proximal tubule disturbances. The level of the serum bicarbonate in either proximal or distal RTA may vary, depending on the extent of the deficit. To maintain electroneutrality in RTA, the loss of bicarbonate is compensated by the retention of chloride, also a negatively charged ion. Thus, the anion gap ([sodium]-[chloride + bicarbonate]) is maintained at 10 to 16. In other types of acidosis, such as lactic acidosis, the excess accumulation of acid does not induce an increase in chloride absorption and, thus, the anion gap is elevated. Given the normal anion gap, this child cannot have either an inborn error of metabolism or lactic acidosis, which typically results in a high (>16) anion gap. Additionally, the normal serum potassium value effectively eliminates type IV RTA as a cause of the acidosis because the lack of aldosterone or renal resistance to aldosterone in this condition induces acidosis and hyperkalemia. References: Adedoyin O, Gottlieb B, Frank R, et al. Evaluation of failure to thrive: diagnostic yield of testing for renal tubular acidosis. Pediatrics. 2003;112:e463-e466. Available at: http://pediatrics.aappublications.org/cgi/content/full/112/6/e463 Roth KS, Chan JC. Renal tubular acidosis: a new look at an old problem. Clin Pediatr (Phila). 2001;40:533-543. Abstract available at: http://www.ncbi.nlm.nih.gov/entrez/query.fcgi?orig_db=PubMed&db=PubMed&cmd=Search&ter m=%22Clinical+pediatrics%22[Jour]+AND+533[page]+AND+2001[pdat] Watanabe T. Proximal renal tubular dysfunction in primary distal renal tubular acidosis. Pediatr Nephrol. 2005;20:86-88. Abstract available at:

Copyright 2007 by the American Academy of Pediatrics

page 530

2007 PREP SA on CD-ROM

http://www.ncbi.nlm.nih.gov/entrez/query.fcgi?cmd=Retrieve&db=pubmed&dopt=Abstract&list_ui ds=15549407&query_hl=72&itool=pubmed_docsum

Copyright 2007 by the American Academy of Pediatrics

page 531

2007 PREP SA on CD-ROM Question: 254

You are asked to evaluate two children, ages 3 and 9 years, on the pediatric hematologyoncology inpatient unit who have developed fever, cough, increased work of breathing, and nodular lesions on their chest radiographs. The children are isolated in private rooms and have different nurses and doctors caring for them. The children's rooms are located adjacent to an area where a new playroom is being constructed. Of the following, the MOST likely pathogen causing these patients' pneumonia is

A. Aspergillus sp B. Candida parapsilosis C. Legionella pneumophila D. Pseudomonas aeruginosa E. respiratory syncytial virus

Copyright 2007 by the American Academy of Pediatrics

page 532

2007 PREP SA on CD-ROM Critique: 254 Preferred Response: A

Transmission-based precautions in the hospital are designed for patients documented or suspected to be colonized or infected with pathogens spread by the airborne, droplet, or contact routes. Such patients require additional precautions beyond standard precautions to interrupt transmission or spread of disease. Airborne transmission occurs by dissemination of airborne droplet nuclei (small-particle residue of evaporated droplets containing microorganisms that remain suspended in the air for long periods), dust particles containing the infectious agent, or fungal spores. Microorganisms spread by the airborne route can be dispersed widely by air currents (eg, ventilation systems, particularly during times of construction) and may be inhaled by or deposited on a susceptible host within the same room or a long distance from the source patient, depending on environmental factors. Therefore, special air handling and ventilation are required to prevent airborne transmission. Spores of filamentous fungi, such as Aspergillus, represent a considerable environmental hazard, especially to severely immunosuppressed patients, such as the children described in the vignette. Other microorganisms transmitted by airborne droplet nuclei include Mycobacterium tuberculosis, measles virus, varicella virus, and disseminated zoster. Specific recommendations in airborne precautions include: the use of private rooms; negative air-pressure ventilation with externally exhausted or HEPA-filtered air, if recirculated; and wearing of respiratory protective devices in cases of suspected or proven tuberculosis. Droplet transmission occurs when droplets containing microorganisms generated from infected persons, primarily during coughing, sneezing, and talking, are propelled a short distance and deposited on the hosts conjunctivae, nasal mucosa, or mouth. Because these relatively large droplets do not remain suspended in the air, special air handling and ventilation are not required to prevent transmission. Contact transmission is the most frequent route of transmission of nosocomial infections and can occur by direct or indirect contact with the infected or colonized surface. Candida parapsilosis, Legionella pneumophila, and Pseudomonas aeruginosa are not transmitted by the airborne route, and only standard precautions are needed to prevent spread. Respiratory syncytial virus is transmitted by direct or close contact with contaminated secretions, and limiting its spread in the hospital only requires contact precautions. References: American Academy of Pediatrics. Infection control for hospitalized children. In: Pickering LK, ed. Red Book: 2006 Report of the Committee on Infectious Diseases. 27th ed. Elk Grove Village, Ill: American Academy of Pediatrics; 2006:153-164 Edmond M. Isolation. Infect Control Hosp Epidemiol. 1997;18:58-64. Abstract available at: http://www.ncbi.nlm.nih.gov/entrez/query.fcgi?orig_db=PubMed&db=PubMed&cmd=Search&ter m=%22Infection+control+and+hospital+epidemiology+:+the+official+journal+of+the+Society+of+H ospital+Epidemiologists+of+America%22[Jour]+AND+58[page]+AND+1997[pdat] Garner JS. The Hospital Infection Control Practices Advisory Committee. Guideline for isolation precautions in hospitals. Infect Control Hosp Epidemiol. 1996;17:53-80

Copyright 2007 by the American Academy of Pediatrics

page 533

2007 PREP SA on CD-ROM Question: 255

A 10-year-old boy presents to the emergency department with a severe asthma exacerbation that requires intubation and admission to the intensive care unit. The child's parents state that this is the third hospital admission for asthma since he was diagnosed at age 6 years. The parents are worried about the severity of their son's asthma flares and would like to know the underlying risk factors for fatal asthma. His current medications include an oral antihistamine, an inhaled corticosteroid, and an as-needed beta-2 agonist. Of the following, the risk factor MOST associated with fatal and near-fatal asthma is

A. coexisting allergic rhinitis B. female sex C. high socioeconomic status D. inhaled corticosteroid use E. poor perception of symptoms

Copyright 2007 by the American Academy of Pediatrics

page 534

2007 PREP SA on CD-ROM Critique: 255 Preferred Response: E

In the past 30 years, asthma-related case-fatality rates in industrialized countries have varied from 1.5 to 8.5 per 100,000 persons. Patients who have severe disease are believed to be at greatest risk, but those who have mild disease may have near-fatal or fatal asthma attacks. Identified risk factors for near-fatal and fatal asthma attacks include marked circadian variation in lung function, male sex, psychosocial disturbances, frequent visits to the emergency department, hospitalization for asthma within the past year, poor perception of hypoxia or airway obstruction, use of two or more beta-agonist inhalers per month, low socioeconomic status coupled with a low educational level, and previous life-threatening asthma attacks. Neither increased inhaled corticosteroid use nor allergic rhinitis has been linked to near-fatal or fatal asthma attacks. References: Guill MF. Asthma update: clinical aspects and management. Pediatr Rev. 2004;25:335-344. Available at: http://pedsinreview.aappublications.org/cgi/content/full/25/10/335 Liu AH, Spahn JD, Leung DYM. Childhood asthma. In: Behrman RE, Kliegman RM, Jenson HB, eds. Nelson Textbook of Pediatrics. 17th ed. Philadelphia, Pa: WB Saunders Co 2004:760-774

Copyright 2007 by the American Academy of Pediatrics

page 535

2007 PREP SA on CD-ROM Question: 256

A 17-year-old boy is brought to the emergency department after he drove his car head-on into another motor vehicle. He was not restrained and was ejected from the vehicle. On physical examination, he has a Glasgow Coma Scale score of 5 and severe head trauma with an open scalp wound that is actively bleeding. He is breathing spontaneously, but a portion of the right side of his chest seems to move paradoxically, moving in with inspiration and out with expiration. On auscultation, he has decreased breath sounds on the affected side. Of the following, the MOST likely cause for his chest examination abnormalities is

A. flail chest B. hemothorax C. lung contusion with splinting D. open pneumothorax E. tension pneumothorax

Copyright 2007 by the American Academy of Pediatrics

page 536

2007 PREP SA on CD-ROM Critique: 256 Preferred Response: A

The boy described in the vignette has a flail chest, a condition that occurs after blunt trauma fractures two or more ribs in two or more places, interrupting the continuity of this segment of the chest wall with the rest of the thoracic cage. The injury typically results in paradoxic motion of the chest wall, as demonstrated by the patient in the vignette. Negative intrapleural pressure during the inspiratory phase of breathing causes the segment to be pulled down into the chest cavity with chest expansion, and positive intrapleural pressure during the expiratory phase causes the segment to bulge out from the thoracic cage with chest wall relaxation. Flail chest can lead to respiratory insufficiency due to splinting from pain or to airway edema from an underlying pulmonary contusion. At the scene of the injury, the patient should be placed with the injured segment down, allowing better expansion and ventilation of the unaffected side. A patient who has flail chest and is in respiratory distress should undergo endotracheal intubation and be supported with positive pressure ventilation. This protects the airway and provides optimal expansion and splinting of the affected segment. Most patients who have flail chest do not require intubation and can be managed with analgesics and aggressive pulmonary physiotherapy. Hemothorax refers to the accumulation of blood in the pleural space. An open pneumothorax is characterized by a penetrating injury to the thoracic wall that permits air to move in and out of the thoracic cavity with each breath. A tension pneumothorax (Item C256A) is caused by an air leak within the lung in the absence of a penetrating injury to the thoracic wall. This causes an increasing amount of air under increasing pressure within the pleural space, leading to collapse of the ipsilateral lung. A patient who has splinting due to lung contusion limits ipsilateral chest wall movement during both phases of respiration to decrease pain. Any of these conditions can be present in conjunction with a flail chest, but only flail chest causes paradoxic motion of a segment of the thoracic wall. References: American College of Surgeons. Thoracic trauma. In: Advanced Trauma Life Support Program for Doctors. Chicago, Ill: American College of Surgeons; 1997:125-141 Kadish HA. Thoracic trauma. In: Fleisher GR, Ludwig S, Henretig FM, eds. Textbook of Pediatric Emergency Medicine. 5th ed. Philadelphia, Pa: Lippincott Williams & Wilkins; 2006:1433-1452

Copyright 2007 by the American Academy of Pediatrics

page 537

Вам также может понравиться